模块6
更新时间:2021-06-27 21:07:55
模块 年级 问题 答案 正确答案
6 1 2006 1. 起生理性调节和激素样作用的胃肠肽不包括 (2.0分) A.促胃液素
B.抑胃肽
C.促胰液素
D.缩胆囊素
E.生长抑素
E.生长抑素
6 1 2006 2. 以下属于促进胃液分泌的内源性物质的是(2.0分) A.前列腺素
B.内因子
C.生长抑素
D.上皮生长因子
E.乙酰胆碱
E.乙酰胆碱
6 1 2006 3. 多种胃肠肽也分布于中枢神经系统中,这种双重分布的肽,被称为 (2.0分) A.脑肠肽
B.内因子
C.糖皮质激素
D.胃泌素
E.胆囊收缩素
A.脑肠肽
6 1 2006 4. 下面不属于代谢综合征的成分是(2.0分) A.血脂异常
B.超重/肥胖
C.高血糖
D.高血压
E.冠心病
E.冠心病
6 1 2006 5. 胃肠道微生态失调相关性疾病不包括:(2.0分) A.炎症性肠病(IBD)
B.肝病
C.大肠癌
D.抗生素相关性腹泻
E.肠易激综合征
E.肠易激综合征
6 1 2006 6. 炎症性肠病的特征性皮肤表现有:(2.0分) A.红斑狼疮
B.结节性红斑
C.天胞疮
D.剥脱性皮炎
E.银屑病
B.结节性红斑
6 1 2006 7. 下列不属于常用的Hp检测方法的是:(2.0分) A.Hp-DNA的聚合酶链式反应
B.粘膜组织染色
C.快速尿素酶试验
D.血清抗体检测
E.Hp呼气试验
A.Hp-DNA的聚合酶链式反应
6 1 2006 8. PCOS和脂肪肝的最重要共同发病机制:(2.0分) A.糖代谢紊乱
B.脂质代谢紊乱
C.胰岛素抵抗
D.基因多态性
E.蛋白质代谢紊乱
C.胰岛素抵抗
6 1 2006 9. Which of the followering statements is not included in UC?(2.0分) A.Crypt abscess
B.limited inflammation in Mucosa and submucosa
C.Tends to begin in the rectum and be continuous
D.Granuloma
D.Granuloma
6 1 2006 10. Which is the most important part to the diagnosis of UC?(2.0分) A.Colonoscopy show continuous ulcers and pathology show crypt abscess
B.Lower abdominal pain and fecal urgency
C.Anemia,low serum albumin
D.Bloody diarrhea
E.negtive stool cultures
A.Colonoscopy show continuous ulcers and pathology show crypt abscess
6 1 2006 11. Which of the followering statements is not the cause of toxic megacolon (2.0分) A.corticosteroid
B.Anticholinergics
C.Barium enema
D.Lower potassium level
A.corticosteroid
6 1 2006 12. The most important evidence to diagnosis toxic megacolon is(2.0分) A.signs of septic shock
B.Abdominal plain shows colonic dilation
C.Abdominal distention
D.loss of bowel sounds
B.Abdominal plain shows colonic dilation
6 1 2006 13. Which of the following is not included in the CD?(2.0分) A. "Cobblestoning"
B.Serpiginous ulcers
C.Crypt abscess
D.caseating granulomas
D.caseating granulomas
6 1 2006 14. A 35 year-old man requires surgical resection for an intestinal disorder. Three years later, he returns with diarrhea, macrocytic anemia, and gallstones. What was the underlying disorder? (2.0分) A.Giant anal condylomata.
B.Crohn's disease.
C.Fulminant ulcerative colitis.
D.Barrett's esophagus.
E.Perforated sigmoid diverticulitis.
B.Crohn's disease.
6 1 2006 15. Infliximab is used in the treatment of patients with inflammatory bowel disease (IBD). Which of the following is the predominant mechanism of action of infliximab? (2.0分) A.Release of prostacyclin.
B.IL-2 inhibition.
C.Antimetabolite interference with DNA synthesis.
D.Binding to soluble TNF.
D.Binding to soluble TNF.
6 1 2006 16. Which of the following drugs is LEAST useful for the maintenance of remission in a young woman with colonic Crohn's disease? (2.0分) A.Azathioprine
B.6MP
C.Metronidazole
D.Prednisone
E.Infliximab
D.Prednisone
6 1 2006 17. Common seen Symptoms of intestinal TB Not include : (2.0分) A.diarrhea
B.abdominal mass
C.abdominal pain
D.nausea
D.nausea
6 1 2006 18. Predisposing factors of intestinal TB include(2.0分) A.malnutrition
B.lowered resistance
C.poor sanitation
D.Fatigue
E.all of above
E.all of above
6 1 2006 19. 28岁,急性阑尾炎穿孔术后6天,腹部胀痛不适,呈持续性,伴恶心、呕吐,肛门排气消失。腹部检查见全腹膨隆。未见蠕动波,肠鸣音消失,腹部压痛,轻度反跳痛。腹立位平片示小肠及大肠均有充气及液平面,据此你诊断为(2.0分) A.急性机械性不完全性高位小肠梗阻
B.急性机械性不完全性低位小肠梗阻
C.急性不完全性结肠梗阻
D.麻痹性肠梗阻
E.急性机械性完全性小肠梗阻
D.麻痹性肠梗阻
6 1 2006 20. 我国成人肠梗阻最常见的原因是(2.0分) A.肠粘连
B.肠扭转
C.腹内疝
D.肠套叠
E.肠肿瘤
A.肠粘连
6 1 2006 21. A 55-year-old woman goes to the emergency department because of vomiting and severe abdominal cramping of 3 day's duration. Her pain is centered on the umbilicus. She denies being exposed to a viral or bacterial illness. Her medical history includes a pre(2.0分) A.Ascaris infection
B.Adhesions
C.Cancer
D.Intussusception
E.Volvulus
B.Adhesions
6 1 2006 22. 男,68岁,腹痛、腹胀1天,便秘3天,恶性无呕吐。体检:腹胀显著,散在压痛,未扪及肿块,指肠指检阴性。腹部X线平片可见一极度扩张的的双袢肠曲。首先考虑诊断是:(2.0分) A.低位小肠梗阻
B.肿瘤所致的肠梗阻
C.肠套叠
D.粪便所致结肠梗阻
E.乙状结肠扭转
E.乙状结肠扭转
6 1 2006 23. 粘连性肠梗阻最常见于:(2.0分) A.腹腔手术后
B.化脓性腹膜炎
C.先天性发育异常
D.胎粪性腹膜炎
E.腹腔创伤后
A.腹腔手术后
6 1 2006 24. 女性,39岁,半年前因肛旁脓肿切开引流,之后局部皮肤反复红肿,破溃,局部痒,应考虑为(2.0分) A.混合痔
B.肛瘘
C.肛旁疖肿
D.血栓性外痔
E.肛旁慢性肉芽肿
B.肛瘘
6 1 2006 25. 一患痔多年的病人突感肛门剧痛,排便,走路,咳嗽时疼痛加剧。检查:肛门部有一圆形肿块突出,与周围分界清楚,触痛,应诊断为(2.0分) A.外痔血栓形成
B.直肠息肉脱出
C.肛门旁皮下脓肿
D.内痔嵌顿
E.外痔合并感染
A.外痔血栓形成
6 1 2006 26. 男性,60岁,患慢性支气管炎多年,经常便秘,近来大便带血,便后觉有肿物自肛门脱出,可以还纳,考虑为(2.0分) A.直肠完全脱出
B.直肠部分脱出
C.内痔脱出
D.直肠息肉脱出
E.环痔
C.内痔脱出
6 1 2006 27. 男性,50岁,间歇性便血10余年,近1个月持续便血,量多,常呈喷射状,感头晕,肛诊齿线上粘膜呈较大的串球形隆起应诊断为(2.0分) A.直肠癌
B.内痔
C.混合痔
D.外痔
E.直肠息肉
B.内痔
6 1 2006 28. 肛裂(2.0分) A.便血量多而色黑
B.便血少而疼痛
C.便血污秽而腥臭
D.便血量多而鲜红
E.便污血而疼痛
B.便血少而疼痛
6 1 2006 29. 内痔(2.0分) A.便血污秽而腥臭
B.便血少而疼痛
C.便血量多而鲜红
D.便血量多而色黑
E.便污血而疼痛
C.便血量多而鲜红
6 1 2006 30. 直肠指检,扪到条索状,伴有轻压痛,挤压时外口有脓性分泌物流出者应考虑(2.0分) A.外痔
B.肛瘘
C.内痔
D.直肠息肉
E.直肠癌
B.肛瘘
6 1 2006 31. 肛管的括约肌功能主要依靠哪种结构的作用(2.0分) A.外括约肌的深部
B.内括约肌
C.肛管直肠环
D.肛提肌
E.外括约肌皮下部
C.肛管直肠环
6 1 2006 32. 直肠癌远处转移最常见的部位是(2.0分) A.脑
B.肺
C.肝
D.纵隔
E.脊柱
C.肝
6 1 2006 33. 直肠癌最基本的诊断方法是(2.0分) A.钡剂灌肠
B.乙状结肠镜
C.直肠镜
D.B型超声检查
E.直肠指检
E.直肠指检
6 1 2006 34. 直肠癌DuckesB期是(2.0分) A.癌肿侵犯浆肌层,无淋巴结转移
B.癌肿侵犯超过浆肌层,有淋巴结转移
C.癌肿侵犯在直肠壁内,有淋巴结转移
D.癌肿侵犯浅肌层,无淋巴结转移
E.癌肿伴有远处转移
A.癌肿侵犯浆肌层,无淋巴结转移
6 1 2006 35. 以下哪项检查适用于评估直肠癌局部侵犯深度?(2.0分) A.结肠镜
B.MRI
C.CT仿真肠镜
D.气钡双重造影
E.腹部B超
B.MRI
6 1 2006 36. 患者46岁,暗红血便及右上腹疼痛3月,电子肠镜检查发现盲肠肿瘤,病理证实为腺癌,全身检查未发现远处转移病灶,该患者应进行以下哪种手术?(2.0分) A.右半结肠切除术
B.盲肠切除术
C.结肠次全切除术
D.回肠横结肠短路手术
E.扩大右半结肠切除术
A.右半结肠切除术
6 1 2006 37. 患者乙状结肠癌切除术后病理报告为:低分化腺癌,侵犯出外膜层,伴淋巴结转移(4+/18)。术前检查时未见远处转移情况,该患者的TNM分期为:(2.0分) A.T3N2aM0
B.T2N1cM0
C.T4aN2aM0
D.T4bN2bM0
E.T3N2bM0
C.T4aN2aM0
6 1 2006 38. 粪便隐血试验阳性者提示出血量多少: (2.0分) A.每日出血量30ml以上
B.每日出血量5ml以上
C.每日出血量60ml以上
D.每日出血量10ml以上
E.每日出血量20ml以上
B.每日出血量5ml以上
6 1 2006 39. 消化性溃疡并发呕血时,首选下列哪种药物止血: (2.0分) A.心得安
B.甲氰咪胍
C.安络血
D.PPI
E.垂体后叶素
D.PPI
6 1 2006 40. 下列哪项不是上消化道出血手术治疗的指征: (2.0分) A.大出血同时伴有急性穿孔
B.年龄超过45岁
C.大出血时伴有幽门梗阻
D.过去有多次大出血,近期反复大出血
E.大出血同时伴有急性弥漫性腹膜炎
B.年龄超过45岁
6 1 2006 41. 关于上消化道出血的处理哪项有误: (2.0分) A.嘱患者保持安静
B.注意观察尿量变化
C.定期复查血象
D.一律禁食
E.防治失血性休克
D.一律禁食
6 1 2006 42. 处理上消化道大出血休克患者,首先采取哪项措施: (2.0分) A.电凝或激光治疗
B.冰盐水洗胃
C.口服止血药
D.补充血容量
E.手术治疗
D.补充血容量
6 1 2006 43. 肝硬化食管静脉曲张出血,以下哪项内科处理不恰当: (2.0分) A.应用垂体加压素
B.禁食,密切观察血压脉搏
C.低蛋白低流质饮食
D.三腔气囊压迫止血
E.出血停止后导泻、灌肠以预防肝昏迷
C.低蛋白低流质饮食
6 1 2006 44. 下列哪些有助于肝硬化食管胃底静脉曲张破裂出血的诊断: (2.0分) A.肝炎病史
B.节律性疼痛
C.低白蛋白血症
D.门静脉高压
E.脾大
B.节律性疼痛
6 1 2006 45. 患者,女,30岁,无溃疡病史,因关节酸痛,服水杨酸制剂,6小时前突然大量呕血,伴晕厥,血压80/50mmHg,心率125次/分,则提示消化道出血的量至少达到: (2.0分) A.400~500ml
B.250~300ml
C.5~10ml
D.50~100ml
E.1000ml以上
E.1000ml以上
6 1 2006 46. 肠易激综合征腹痛的规律是: (2.0分) A.疼痛进食-缓解;
B.疼痛-排便加重;
C.进食-疼痛-缓解;
D.无明显规律性;
E.疼痛-排便缓解
A.疼痛进食-缓解;
6 1 2006 47. 以下哪项不是肠易激综合征的症状? (2.0分) A.脂肪泻
B.排便频率的改变
C.腹胀
D.腹痛
E.排便性状的改变
A.脂肪泻
6 1 2006 48. 简述肠易激综合征的治疗原则 : (2.0分) A.根据主要症状对症治疗及根据严重程度分级治疗
B.一般治疗
C.建立良好的医患关系
D.重视心理治疗
E.以上均是
E.以上均是
6 1 2006 49. 根据罗马III标准,下列哪项不是IBS分型中的一种? (2.0分) A.便秘型
B.腹泻型
C.混合型
D.腹痛型
E.不定型
D.腹痛型
6 1 2006 50. The duration of Irritable Bowel Syndrome is at least: (2.0分) A.12 months
B.6 months
C.9 months
D.3 months
E.1 months
B.6 months
6 1 2007 1. PPH术后2天的男性38岁患者,突然出现低热以及直肠和下腹部疼痛。实验室检查发现WBC 14000/ul,腹部平片发现腹膜后积气。下一步最恰当的治疗措施是:(2.5分) A.剖腹探查、结肠切除
B.麻醉下直肠检查和局部清创
C.使用广谱抗生素
D.直肠镜检查以及直肠灌洗
E.经腹会阴联合直肠切除术
C.使用广谱抗生素
6 1 2007 2. 肝硬化食管静脉曲张出血,以下哪项内科处理不恰当: (2.5分) A.低蛋白低流质饮食
B.禁食,密切观察血压脉搏
C.三腔气囊压迫止血
D.应用垂体加压素
E.出血停止后导泻、灌肠以预防肝昏迷
A.低蛋白低流质饮食
6 1 2007 3. 多种胃肠肽也分布于中枢神经系统中,这种双重分布的肽,被称为 (2.5分) A.脑肠肽
B.糖皮质激素
C.胃泌素
D.内因子
E.胆囊收缩素
A.脑肠肽
6 1 2007 4. 34岁HIV阳性的男性同性恋患者,主诉排便时肛门疼痛伴鲜红血便。检查发现典型的肛管后正中肛裂,裂口无潜行、无分泌物。对于该患者最恰当的处理是:(2.5分) A.麻醉下行肛门检查和活检
B.结肠镜检查后口服止痛剂和氢化可的松
C.直肠镜检查、温水坐浴、增加膳食纤维
D.Y-V肛门整形
E.肛门内括约肌侧方切开术
C.直肠镜检查、温水坐浴、增加膳食纤维
6 1 2007 5. PCOS和脂肪肝的最重要共同发病机制:(2.5分) A.脂质代谢紊乱
B.糖代谢紊乱
C.基因多态性
D.胰岛素抵抗
E.蛋白质代谢紊乱
D.胰岛素抵抗
6 1 2007 6. 患者,女,30岁,无溃疡病史,因关节酸痛,服水杨酸制剂,6小时前突然大量呕血,伴晕厥,血压80/50mmHg,心率125次/分,则提示消化道出血的量至少达到: (2.5分) A.250~300ml
B.50~100ml
C.5~10ml
D.400~500ml
E.1000ml以上
E.1000ml以上
6 1 2007 7. 粪便隐血试验阳性者提示出血量多少: (2.5分) A.每日出血量60ml以上
B.每日出血量10ml以上
C.每日出血量30ml以上
D.每日出血量5ml以上
E.每日出血量20ml以上
D.每日出血量5ml以上
6 1 2007 8. 多发生在暴发型和重症溃疡性结肠炎患者的并发症是(2.5分) A.消化道出血
B.癌变
C.中毒性巨结肠
D.穿孔
E.多脏器功能衰竭
C.中毒性巨结肠
6 1 2007 9. 急性肠梗阻不会出现那个变化(2.5分) A.肠腔因气体和液体的积聚而膨胀
B.肠壁血运障碍
C.肠腔压力升高
D.肠壁变薄
E.肠壁代偿性肥厚
E.肠壁代偿性肥厚
6 1 2007 10. 克罗恩病内镜及病理的典型表现包括(2.5分) A.肠腔狭窄和粘膜病变
B.鹅卵石样表现和隐窝脓肿
C.环形溃疡和淋巴细胞浸润
D.纵型溃疡和非干酪样肉芽肿
E.假息肉和干酪样肉芽肿
D.纵型溃疡和非干酪样肉芽肿
6 1 2007 11. 高位小肠梗阻的临床表现特点应为:(2.5分) A.存在腹痛、呕吐、腹胀及排气排便停止
B.以腹痛、呕吐和排便、排气停止为著,但腹胀不明显
C.以呕吐和排便、排气停止为著,腹痛轻微无腹胀
D.以腹痛和呕吐为著,而腹胀,排便,排气停止不明显
E.以腹痛和腹胀为著,无呕吐,无排气排便停止
D.以腹痛和呕吐为著,而腹胀,排便,排气停止不明显
6 1 2007 12. 处理上消化道大出血休克患者,首先采取哪项措施: (2.5分) A.口服止血药
B.冰盐水洗胃
C.补充血容量
D.电凝或激光治疗
E.手术治疗
C.补充血容量
6 1 2007 13. 28岁,急性阑尾炎穿孔术后6天,腹部胀痛不适,呈持续性,伴恶心、呕吐,肛门排气消失。腹部检查见全腹膨隆。未见蠕动波,肠鸣音消失,腹部压痛,轻度反跳痛。腹立位平片示小肠及大肠均有充气及液平面,据此你诊断为(2.5分) A.急性机械性不完全性高位小肠梗阻
B.急性机械性不完全性低位小肠梗阻
C.急性不完全性结肠梗阻
D.麻痹性肠梗阻
E.急性机械性完全性小肠梗阻
D.麻痹性肠梗阻
6 1 2007 14. 我国成人肠梗阻最常见的原因是(2.5分) A.肠套叠
B.肠粘连
C.腹内疝
D.肠扭转
E.肠肿瘤
B.肠粘连
6 1 2007 15. 起生理性调节和激素样作用的胃肠肽不包括 (2.5分) A.促胃液素
B.缩胆囊素
C.抑胃肽
D.促胰液素
E.生长抑素
E.生长抑素
6 1 2007 16. 直肠癌的主要临床症状和体检不包括(2.5分) A.直肠指检可发现病变
B.排便习惯改变
C.腹胀、腹痛等不全性肠梗阻
D.便血
E.腹部可触及包块
E.腹部可触及包块
6 1 2007 17. 男,68岁,腹痛、腹胀1天,便秘3天,恶性无呕吐。体检:腹胀显著,散在压痛,未扪及肿块,指肠指检阴性。腹部X线平片可见一极度扩张的的双袢肠曲。首先考虑诊断是:(2.5分) A.肠套叠
B.粪便所致结肠梗阻
C.低位小肠梗阻
D.肿瘤所致的肠梗阻
E.乙状结肠扭转
E.乙状结肠扭转
6 1 2007 18. 38岁男性憩室炎患者在乙状结肠切除术后出现逆行性射精,最可能是手术时损伤了一下哪项:(2.5分) A.盆腔内脏神经
B.直肠下神经
C.骨盆神经丛
D.阴部神经
E.骶前神经丛
E.骶前神经丛
6 1 2007 19. 对一个既往体健的22岁男性藏毛窦患者来说,一下哪种治疗方法可以使他最快痊愈,并且术后复发率最低?(2.5分) A.注射苯酚
B.Bascom手术
C.切除以及Z字整形
D.Karydakis手术
E.广泛切除
B.Bascom手术
6 1 2007 20. 粘连性肠梗阻最常见于:(2.5分) A.胎粪性腹膜炎
B.先天性发育异常
C.化脓性腹膜炎
D.腹腔手术后
E.腹腔创伤后
D.腹腔手术后
6 1 2007 21. 男性,55岁,暗红血便2个月,脐周及下腹部隐痛不适,腹平软,无压痛及肿块,便化验潜血(+)。下列哪种检查能明确诊断(2.5分) A.结肠镜检查和活检
B.直肠指诊
C.大便培养
D.乙状结肠镜检查
E.肛门镜检查
A.结肠镜检查和活检
6 1 2007 22. 直肠癌DuckesB期是(2.5分) A.癌肿侵犯在直肠壁内,有淋巴结转移
B.癌肿侵犯浆肌层,无淋巴结转移
C.癌肿侵犯超过浆肌层,有淋巴结转移
D.癌肿侵犯浅肌层,无淋巴结转移
E.癌肿伴有远处转移
B.癌肿侵犯浆肌层,无淋巴结转移
6 1 2007 23. 患者46岁,暗红血便及右上腹疼痛3月,电子肠镜检查发现盲肠肿瘤,病理证实为腺癌,全身检查未发现远处转移病灶,该患者应进行以下哪种手术?(2.5分) A.回肠横结肠短路手术
B.结肠次全切除术
C.盲肠切除术
D.右半结肠切除术
E.扩大右半结肠切除术
D.右半结肠切除术
6 1 2007 24. 根据肛瘘瘘管的行走部位,可分为4种类型,以下那一个不正确?(2.5分) A.括约肌间
B.括约肌内型
C.经括约肌
D.括约肌上
E.括约肌外型
B.括约肌内型
6 1 2007 25. 关于上消化道出血量的估计下列哪项是对的: (2.5分) A.胃内积血500ml才会发生呕血
B.一次失血超过500ml可有急性失血的全身表现
C.每日出血量200ml以上才会出现黑便
D.每日出血量在50ml以上粪便隐血试验才呈阳性反应
E.面色苍白,脉搏>110次/分,收缩压< 80mmHg估计出血量占总血量的10%以下
B.一次失血超过500ml可有急性失血的全身表现
6 1 2007 26. 下列不属于常用的Hp检测方法的是:(2.5分) A.快速尿素酶试验
B.血清抗体检测
C.Hp-DNA的聚合酶链式反应
D.粘膜组织染色
E.Hp呼气试验
C.Hp-DNA的聚合酶链式反应
6 1 2007 27. 下列关于结直肠癌术后随访的叙述,哪一项是正确的:(2.5分) A.术前接受过全结肠镜检查的患者,术后1年内需进行肠镜复查
B.术后3年内,每3月进行胸腹部CT检查
C.术后5年内,每3月进行胸片和肝脏B超检查
D.术后3月内进行肠镜复查
E.以上都不对
A.术前接受过全结肠镜检查的患者,术后1年内需进行肠镜复查
6 1 2007 28. 以下属于促进胃液分泌的内源性物质的是 (2.5分) A.前列腺素
B.生长抑素
C.上皮生长因子,
D.内因子,
E.乙酰胆碱
E.乙酰胆碱
6 1 2007 29. 患者乙状结肠癌切除术后病理报告为:低分化腺癌,侵犯出外膜层,伴淋巴结转移(4+/18)。术前检查时未见远处转移情况,该患者的TNM分期为:(2.5分) A.T4aN2aM0
B.T4bN2bM0
C.T3N2aM0
D.T2N1cM0
E.T3N2bM0
A.T4aN2aM0
6 1 2007 30. 2岁老年男性患者,患有间歇性发作的肛门瘙痒症,在过去的几个月里肛周瘙痒和不适感加重,无便血和肛门疼痛,查体发现左侧肛缘皮炎样改变以及小溃疡形成。下列哪项是最佳的下一步治疗措施:(2.5分) A.内括约切开术
B.激素软膏
C.硝酸甘油软膏
D.局部广泛切除
E.活检
E.活检
6 1 2007 31. 肠结核常见的临床表现不包括以下哪一项(2.5分) A.腹部肿块
B.结核毒血症状
C.便秘
D.腹泻
E.里急后重
E.里急后重
6 1 2007 32. 结肠癌远处转移最常见的部位是(2.5分) A.纵隔
B.脑
C.肝
D.肺
E.脊柱
C.肝
6 1 2007 33. 消化性溃疡并发呕血时,首选下列哪种药物止血: (2.5分) A.甲氰咪胍
B.安络血
C.心得安
D.PPI
E.垂体后叶素
D.PPI
6 1 2007 34. 引起上消化道出血的最常见的疾病是: (2.5分) A.消化性溃疡
B.慢性胃炎
C.肝硬化食管、胃底静脉曲张破裂
D.胃癌
E.出血性胃炎
A.消化性溃疡
6 1 2007 35. 男,65岁,阵发性腹痛、腹胀、肛门停止排气排便2天,无呕吐。体检:腹部明显膨隆, 腹软,散在不固定压痛,可闻及高亢肠鸣音。腹部平片示结肠明显扩张,范围自盲肠至乙状结肠,盲肠直径为8cm,腹部可见少量小肠充气。以下那一个正确:(2.5分) A.高位小肠梗阻
B.麻痹性肠梗阻
C.低位小肠梗阻
D.降结肠梗阻
E.以上都不是
E.以上都不是
6 1 2007 36. 对肠结核和溃疡性结肠炎的鉴别诊断最有意义的是(2.5分) A.结肠镜检查
B.肝功能检查
C.粪常规和隐血
D.胃肠钡餐检查
E.腹部平片
A.结肠镜检查
6 1 2007 37. A 54-year-old man has had colicky abdominal pain and protracted vomiting for several days. He has developed progressive moderate abdominal distention, and has not had a bowel movement or pass gas for five days. He has high-pitched, loud bowel sounds that (2.5分) A.Acute pancreatitis
B.Strangulated obstruction of bowel
C.Carcinoma of sigmoid colon
D.acute appendicitis
E.Simple obstruction of small intestine
B.Strangulated obstruction of bowel
6 1 2007 38. 小肠型轻型克罗恩病治疗有效的5-ASA类药物是 (2.5分) A.Pentasa
B.Asacol
C.Salofalk
D.Apriso
E.Balsalazide
A.Pentasa
6 1 2007 39. A 41-year-old male is brought to the ED with acute hematemesis. Abdominal exam reveals distension, no rebound or guarding, hepatosplenomegaly, and dull fluid wave. Which of the following is the most likely source of the upper gastrointestinal bleeding (UG(2.5分) A.Perforated duodenal ulcer
B.Arteriovenous malformation (AVM)
C.Mallory-Weiss tear
D.Chronic gastritis
E.Esophageal varices
E.Esophageal varices
6 1 2007 40. 以下哪项不是肠易激综合征的症状? (2.5分) A.腹胀
B.排便频率的改变
C.腹痛
D.脂肪泻
E.排便性状的改变
D.脂肪泻
6 1 2008 1. 高位小肠梗阻的临床表现特点应为:(4.0分) A.以腹痛和呕吐为著,而腹胀,排便,排气停止不明显
B.以腹痛、呕吐和排便、排气停止为著,但腹胀不明显
C.以呕吐和排便、排气停止为著,腹痛轻微无腹胀
D.存在腹痛、呕吐、腹胀及排气排便停止
E.以腹痛和腹胀为著,无呕吐,无排气排便停止
A.以腹痛和呕吐为著,而腹胀,排便,排气停止不明显
6 1 2008 2. 绞窄性肠梗阻最容易发生的酸碱失衡类型是(3.0分) A.呼吸性碱中毒
B.代谢性酸中毒
C.代谢性碱中毒
D.呼吸性酸中毒
E.呼吸性酸中毒和代谢性碱中毒
B.代谢性酸中毒
6 1 2008 3. 我国成人肠梗阻最常见的原因是(3.0分) A.肠扭转
B.肠粘连
C.腹内疝
D.肠套叠
E.肠肿瘤
B.肠粘连
6 1 2008 4. A 54-year-old man has had colicky abdominal pain and protracted vomiting for several days. He has developed progressive moderate abdominal distention, and has not had a bowel movement or pass gas for five days. He has high-pitched, loud bowel sounds that coincide with the colicky pain, and X-ray show distended loops of small bowel and air-fluid levels. Five years ago he had an exploratory laparotomy for a blunt trauma of abdomen. Six hours after being hospitalized and placed on NG suction and IV fluids.He develops fever, leukocytosis, tenderness, and rebound tenderness. What is the most likely diagnosis? (4.0分) A.acute appendicitis
B.Acute pancreatitis
C.Strangulated obstruction of bowel
D.Carcinoma of sigmoid colon
E.Simple obstruction of small intestine
C.Strangulated obstruction of bowel
6 1 2008 5. 男,68岁,腹痛、腹胀1天,便秘3天,恶性无呕吐。体检:腹胀显著,散在压痛,未扪及肿块,指肠指检阴性。腹部X线平片可见一极度扩张的的双袢肠曲。首先考虑诊断是:(3.0分) A.低位小肠梗阻
B.粪便所致结肠梗阻
C.肠套叠
D.肿瘤所致的肠梗阻
E.乙状结肠扭转
E.乙状结肠扭转
6 1 2008 6. 起生理性调节和激素样作用的胃肠肽不包括(4.0分) A.促胃液素
B.缩胆囊素
C.促胰液素
D.抑胃肽
E.生长抑素
E.生长抑素
6 1 2008 7. 多种胃肠肽也分布于中枢神经系统中,这种双重分布的肽,被称为(3.0分) A.脑肠肽
B.糖皮质激素
C.内因子
D.胃泌素
E.胆囊收缩素
A.脑肠肽
6 1 2008 8. 下面不属于代谢综合征的成分是(4.0分) A.超重/肥胖
B.高血压
C.血脂异常
D.高血糖
E.冠心病
E.冠心病
6 1 2008 9. 炎症性肠病的特征性皮肤表现有:(3.0分) A.结节性红斑
B.红斑狼疮
C.天胞疮
D.剥脱性皮炎
E.银屑病
A.结节性红斑
6 1 2008 10. PCOS和脂肪肝的最重要共同发病机制:(3.0分) A.脂质代谢紊乱
B.糖代谢紊乱
C.胰岛素抵抗
D.基因多态性
E.蛋白质代谢紊乱
C.胰岛素抵抗
6 1 2008 11. 患者乙状结肠癌切除术后病理报告为:低分化腺癌,侵犯出外膜层,伴淋巴结转移(4+/18)。术前检查时未见远处转移情况,该患者的TNM分期为:(4.0分) A.T2N1cM0
B.T3N2aM0
C.T4aN2aM0
D.T4bN2bM0
E.T3N2bM0
C.T4aN2aM0
6 1 2008 12. 以下哪项检查适用于评估直肠癌局部侵犯深度?(3.0分) A.结肠镜
B.气钡双重造影
C.CT仿真肠镜
D.MRI
E.腹部B超
D.MRI
6 1 2008 13. 结肠癌远处转移最常见的部位是(3.0分) A.脑
B.纵隔
C.肝
D.肺
E.脊柱
C.肝
6 1 2008 14. 直肠癌的主要临床症状和体检不包括(4.0分) A.便血
B.排便习惯改变
C.腹胀、腹痛等不全性肠梗阻
D.直肠指检可发现病变
E.腹部可触及包块
E.腹部可触及包块
6 1 2008 15. 直肠癌最基本的诊断方法是(3.0分) A.直肠镜
B.乙状结肠镜
C.钡剂灌肠
D.B型超声检查
E.直肠指检
E.直肠指检
6 1 2008 16. 肠结核出现腹部肿块最常见于(4.0分) A.溃疡性肠结核并局限性腹膜炎
B.溃疡性肠结核并肠曲及周围粘连
C.增生性肠结核
D.溃疡性肠结核同时有女性生殖性结核
E.溃疡性肠结核同时有肠系膜淋巴结结核
C.增生性肠结核
6 1 2008 17. 下面哪项不是肠结核常见的临床表现(3.0分) A.便秘
B. 腹痛
C.便血
D.低热
E.腹块
C.便血
6 1 2008 18. 肠结核手术适应症不包括(3.0分) A.完全性肠梗阻 B 急性肠穿孔
B.慢性肠穿孔瘘管形成内科治疗未能闭合者
C.肠出血,经过积极抢救不能止血者
D.结核性腹膜炎
NaN
6 1 2008 19. 对于结核性腹膜炎腹部症状的描述,错误的是(3.0分) A.早期腹痛明显
B.腹痛多在脐周,下腹
C. 多为持续性隐痛或钝痛
D.可有阵发性绞痛 E 偶可表现为急腹症
A.早期腹痛明显
6 1 2008 20. 根据WHO诊断标准,可确诊为克罗恩病的是(3.0分) A.区域性肠道病变粘膜呈铺路石样改变
B.非连续性肠道病变伴纵行溃疡
C.全层性炎症性肠道病变伴肿块及狭窄
D.节段性肠道病变纵行溃疡
E. 节段性肠道病变肠壁全层性炎非干酪样肉芽肿
E. 节段性肠道病变肠壁全层性炎非干酪样肉芽肿
6 1 2008 21. 关于溃疡性结肠炎的叙述,下列哪项正确(4.0分) A.病变呈节段性分布
B.典型的表现是慢性腹泻,便血,抗生素治疗无效
C.溃疡深,易穿孔
D.常常急性起病
E.病变多位于右半结肠
B.典型的表现是慢性腹泻,便血,抗生素治疗无效
6 1 2008 22. 柳氮磺胺吡啶治疗溃疡性结肠炎的最主要机制是(3.0分) A.降低肠内酸度促进溃疡愈合
B.抑制炎症作用
C.促进肠道上皮再生
D.抑制细菌生长
E.免疫抑制作用
B.抑制炎症作用
6 1 2008 23. 34岁HIV阳性的男性同性恋患者,主诉排便时肛门疼痛伴鲜红血便。检查发现典型的肛管后正中肛裂,裂口无潜行、无分泌物。对于该患者最恰当的处理是:(4.0分) A.直肠镜检查、温水坐浴、增加膳食纤维
B.结肠镜检查后口服止痛剂和氢化可的松
C.麻醉下行肛门检查和活检
D.Y-V肛门整形
E.肛门内括约肌侧方切开术
A.直肠镜检查、温水坐浴、增加膳食纤维
6 1 2008 24. PPH术后2天的男性38岁患者,突然出现低热以及直肠和下腹部疼痛。实验室检查发现WBC 14000/ul,腹部平片发现腹膜后积气。下一步最恰当的治疗措施是:(3.0分) A.使用广谱抗生素
B.直肠镜检查以及直肠灌洗
C.麻醉下直肠检查和局部清创
D.剖腹探查、结肠切除
E.经腹会阴联合直肠切除术
A.使用广谱抗生素
6 1 2008 25. 72岁老年男性患者,患有间歇性发作的肛门瘙痒症,在过去的几个月里肛周瘙痒和不适感加重,无便血和肛门疼痛,查体发现左侧肛缘皮炎样改变以及小溃疡形成。下列哪项是最佳的下一步治疗措施:(3.0分) A.局部广泛切除
B.激素软膏
C.硝酸甘油软膏
D.内括约切开术
E.活检
E.活检
6 1 2008 26. 42岁男性,在混合痔切除术后第5天出现明显的出血症状,生命体征:血压100/60mmHg,心率110bpm,体温37.3℃,关于下一步处理方式,一下哪个选项最正确:(3.0分) A.ECT出血扫描
B.结肠镜
C.血管造影
D.住院观察
E.麻醉下局部探查
E.麻醉下局部探查
6 1 2008 27. 根据肛瘘瘘管的行走部位,可分为4种类型,以下那一个不正确?(3.0分) A.括约肌间
B.经括约肌
C.括约肌内型
D.括约肌上
E.括约肌外型
C.括约肌内型
6 1 2008 28. 肠结核预防的重点是(3.0分) A.有关结核病的预防宣传 B 加强卫生监督
B.提倡用公筷,牛奶须灭菌消毒
C.肠外结核,特别是肺结核的早期整治
D.保持排便通畅
D.保持排便通畅
6 1 2008 29. 下列哪项对溃疡性结肠炎的轻重程度判断无帮助(4.0分) A.血白细胞
B.血红蛋白
C.白蛋白
D. 大便次数
E.体温
A.血白细胞
6 1 2008 30. 女性, 26岁,反复排粘液脓血便4年,加重3天,胃纳良好,体重无明显减轻,其最可能的病因是:(3.0分) A.肠易激综合征
B.大肠癌
C. 肠结核
D.溃疡性结肠炎
E.真菌性肠炎
D.溃疡性结肠炎
6 1 2009 1. 肠结核出现腹部肿块最常见于(2.5分) A.溃疡性肠结核并局限性腹膜炎
B.溃疡性肠结核并肠曲及周围粘连
C.增生性肠结核
D.溃疡性肠结核同时有女性生殖性结核
E.溃疡性肠结核同时有肠系膜淋巴结结核
C.增生性肠结核
6 1 2009 2. 患者,女, 25岁,低热,便秘与腹泻交替3年,查体:有下腹5X5cm肿块,质中等,较固定请压痛。最具有诊断意义的检查是(2.5分) A.血沉
B.血常规
C.结肠镜
D.X线钡剂透视
E.诊断性腹腔镜穿刺
C.结肠镜
6 1 2009 3. 结核性腹水检查中,不正确的是(2.5分) A.偶见乳糜样腹水
B.比重一般超过1.018,蛋白含量在30g/L以上
C.合并肝硬化时腹水性质接近漏出液
D.白细胞计数超过500X106/L,以淋巴细胞为主
E.腹水浓缩后做结核分枝杆菌培养阳性率高
E.腹水浓缩后做结核分枝杆菌培养阳性率高
6 1 2009 4. 诊断克罗恩病最有意义的临床病理改变(2.5分) A.全壁性炎症
B.裂隙样溃疡
C. 非干酪样肉芽肿
D. 干酪样肉芽肿
E. 瘘管形成
C. 非干酪样肉芽肿
6 1 2009 5. 根据WHO诊断标准,可确诊为克罗恩病的是(2.5分) A.区域性肠道病变 粘膜呈铺路石样改变
B.非连续性肠道病变伴纵行溃疡
C.全层性炎症性肠道病变伴肿块及狭窄
D.节段性肠道病变 纵行溃疡
E.节段性肠道病变 肠壁全层性炎 非干酪样肉芽肿
E.节段性肠道病变 肠壁全层性炎 非干酪样肉芽肿
6 1 2009 6. 关于溃疡性结肠炎的叙述,下列哪项正确(2.5分) A.病变呈节段性分布
B.典型的表现是慢性腹泻,便血,抗生素治疗无效
C.溃疡深,易穿孔
D.常常急性起病
E.病变多位于右半结肠
B.典型的表现是慢性腹泻,便血,抗生素治疗无效
6 1 2009 7. 下列哪项不是柳氮磺胺吡啶的副作用(2.5分) A.恶心,呕吐
B.食欲不振
C.自身免疫性溶血
D. 粒细胞减少
E.不可逆性男性不育
E.不可逆性男性不育
6 1 2009 8. 高位小肠梗阻的临床表现特点应为:(2.5分) A.以腹痛和呕吐为著,而腹胀,排便,排气停止不明显
B.以腹痛、呕吐和排便、排气停止为著,但腹胀不明显
C.以呕吐和排便、排气停止为著,腹痛轻微无腹胀
D.存在腹痛、呕吐、腹胀及排气排便停止
E.以腹痛和腹胀为著,无呕吐,无排气排便停止
A.以腹痛和呕吐为著,而腹胀,排便,排气停止不明显
6 1 2009 9. 男,68岁,腹痛、腹胀1天,便秘3天,恶性无呕吐。体检:腹胀显著,散在压痛,未扪及肿块,指肠指检阴性。腹部X线平片可见一极度扩张的的双袢肠曲。首先考虑诊断是:(2.5分) A.低位小肠梗阻
B.粪便所致结肠梗阻
C.肠套叠
D.肿瘤所致的肠梗阻
E.乙状结肠扭转
E.乙状结肠扭转
6 1 2009 10. 绞窄性肠梗阻最容易发生的酸碱失衡类型是(2.5分) A.呼吸性碱中毒
B.代谢性酸中毒
C.代谢性碱中毒
D.呼吸性酸中毒
E.呼吸性酸中毒和代谢性碱中毒
B.代谢性酸中毒
6 1 2009 11. 28岁,急性阑尾炎穿孔术后6天,腹部胀痛不适,呈持续性,伴恶心、呕吐,肛门排气消失。腹部检查见全腹膨隆。未见蠕动波,肠鸣音消失,腹部压痛,轻度反跳痛。腹立位平片示小肠及大肠均有充气及液平面,据此你诊断为(2.5分) A.急性机械性不完全性低位小肠梗阻
B.急性机械性不完全性高位小肠梗阻
C.急性不完全性结肠梗阻
D.麻痹性肠梗阻
E.急性机械性完全性小肠梗阻
D.麻痹性肠梗阻
6 1 2009 12. 我国成人肠梗阻最常见的原因是(2.5分) A.肠扭转
B.肠粘连
C.腹内疝
D.肠套叠
E.肠肿瘤
B.肠粘连
6 1 2009 13. A 55-year-old woman goes to the emergency department because of vomiting and severe abdominal cramping of 3 day's duration. Her pain is centered on the umbilicus. She denies being exposed to a viral or bacterial illness. Her medical history includes a previous cholecystectomy and an appendectomy after which she developed an infection. Her abdomen is not tender, but hyperactive, high-pitched peristalsis with rushes coincides with palpable bowel cramping. Abdominal X-ray films taken in the upright positions demonstrate a ladder-like series of distended small bowel loops. Which of the following is the most likely explanation for these findings?(2.5分) A.Adhesions
B.Ascaris infection
C.Cancer
D.Intussusception
E.Volvulus
A.Adhesions
6 1 2009 14. 男性,77岁,粘液稀便2个月,脐周及下腹部隐痛不适,腹平软,无压痛及肿块,便化验潜血(+)。如该患者状态较差,有高血压及糖尿病。距肛门9cm处的直肠癌,能切除,应选择哪种手术为佳(2.5分) A.经腹直肠癌切除,人工肛门,远端封闭
B.拉下式直肠癌切除术
C.经腹直肠癌切除术
D.腹会阴联合直肠癌根治术
E.乙状结肠造瘘术
A.经腹直肠癌切除,人工肛门,远端封闭
6 1 2009 15. A 70-year-old woman comes to the emergency department with abdominal cramping, rectal bleeding, and tenderness of the left lower quadrant of the abdomen. Which of the following tests is most appropriate for establishing the diagnosis?(2.5分) A.Angiography
B.Colonoscopy
C.Laparoscopy
D.Tagged erythrocyte scan
B.Colonoscopy
6 1 2009 16. 50岁女性因偶发便血行肠镜检查,发现距肛5cm直肠前壁可见一直径2cm的质硬溃疡性病灶。活检病理提示:粘膜肌层增厚,固有层纤维肌性缺失,但无恶性肿瘤的证据。体检未见直肠外脱垂。以下哪项是最恰当得处理方法:(2.5分) A.补充纤维素
B.经会阴直肠切除术
C.氩离子凝固术
D.经肛切除溃疡性病灶
E.前切除+直肠固定术
A.补充纤维素
6 1 2009 17. 直肠癌(2.5分) A.便血量多而鲜红
B.便血少而疼痛
C.便血污秽而腥臭
D.便血量多而色黑
E.便污血而疼痛
C.便血污秽而腥臭
6 1 2009 18. 以下哪项检查适用于评估直肠癌局部侵犯深度?(2.5分) A.结肠镜
B.气钡双重造影
C.CT仿真肠镜
D.MRI
E.腹部B超
D.MRI
6 1 2009 19. 男性,55岁,暗红血便2个月,脐周及下腹部隐痛不适,腹平软,无压痛及肿块,便化验潜血(+)。下列哪种检查能明确诊断(2.5分) A.结肠镜检查和活检
B.乙状结肠镜检查
C.大便培养
D.直肠指诊
E.肛门镜检查
A.结肠镜检查和活检
6 1 2009 20. A 45-year-old woman with a 10-year history of Crohn's disease reports having anal pain from fissures, diarrhea, significant vaginal drainage, and perianal and perineal excoriation for the past 18 months. Colonoscopy demonstrates a rectovaginal fistula 5 cm from the dentate line, severe proctitis, and a normal sigmoid colon. She is currently managed with 6-mercaptopurine 50 mg daily, infliximab infusions every 6 weeks, and prednisone 40 mg daily. Which of the following is the most appropriate management?(2.5分) A.Continue current regimen and placement of a seton
B.Transanal rectal advancement flap with proximal diversion
C.Continue current regimen and laparoscopic diversion
D.I ntersphincteric proctectomy with end colostomy
C.Continue current regimen and laparoscopic diversion
6 1 2009 21. 直肠指检,扪到条索状,伴有轻压痛,挤压时外口有脓性分泌物流出者应考虑(2.5分) A.内痔
B.肛瘘
C.外痔
D.直肠息肉
E.直肠癌
B.肛瘘
6 1 2009 22. 38岁男性憩室炎患者在乙状结肠切除术后出现逆行性射精,最可能是手术时损伤了一下哪项:(2.5分) A.直肠下神经
B.盆腔内脏神经
C.骨盆神经丛
D.阴部神经
E.骶前神经丛
E.骶前神经丛
6 1 2009 23. 42岁男性,在混合痔切除术后第5天出现明显的出血症状,生命体征:血压100/60mmHg,心率110bpm,体温37.3℃,关于下一步处理方式,一下哪个选项最正确:(2.5分) A.ECT出血扫描
B.结肠镜
C.血管造影
D.住院观察
E.麻醉下局部探查
E.麻醉下局部探查
6 1 2009 24. 对一个既往体健的22岁男性藏毛窦患者来说,一下哪种治疗方法可以使他最快痊愈,并且术后复发率最低?(2.5分) A.注射苯酚
B.切除以及Z字整形
C.Bascom手术
D.Karydakis手术
E.广泛切除
C.Bascom手术
6 1 2009 25. 34岁HIV阳性的男性同性恋患者,主诉排便时肛门疼痛伴鲜红血便。检查发现典型的肛管后正中肛裂,裂口无潜行、无分泌物。对于该患者最恰当的处理是:(2.5分) A.直肠镜检查、温水坐浴、增加膳食纤维
B.结肠镜检查后口服止痛剂和氢化可的松
C.麻醉下行肛门检查和活检
D.Y-V肛门整形
E.肛门内括约肌侧方切开术
A.直肠镜检查、温水坐浴、增加膳食纤维
6 1 2009 26. 55岁男性,痔切除术后肛门狭窄伴后正中肛裂,经扩肛等后续治疗无效。检查发现狭窄环一整圈,位于齿线上方,使用小号的Hill-Ferguson肛门拉钩无法进一步检查。对于该患者,一下那种外科治疗方法最恰当:(2.5分) A.粘膜前徙性皮瓣
B.括约肌切开
C.House皮瓣
D.带括约肌切开的V-Y肛门整形
E.双侧皮肤前徙性皮瓣
E.双侧皮肤前徙性皮瓣
6 1 2009 27. PPH术后2天的男性38岁患者,突然出现低热以及直肠和下腹部疼痛。实验室检查发现WBC 14000/ul,腹部平片发现腹膜后积气。下一步最恰当的治疗措施是:(2.5分) A.使用广谱抗生素
B.直肠镜检查以及直肠灌洗
C.麻醉下直肠检查和局部清创
D.剖腹探查、结肠切除
E.经腹会阴联合直肠切除术
A.使用广谱抗生素
6 1 2009 28. 起生理性调节和激素样作用的胃肠肽不包括(2.5分) A.促胃液素
B.缩胆囊素
C.促胰液素
D.抑胃肽
E.生长抑素
E.生长抑素
6 1 2009 29. 以下属于促进胃液分泌的内源性物质的是 (2.5分) A.生长抑素
B.前列腺素
C.上皮生长因子,
D.内因子,
E.乙酰胆碱
E.乙酰胆碱
6 1 2009 30. 多种胃肠肽也分布于中枢神经系统中,这种双重分布的肽,被称为 (2.5分) A.脑肠肽
B.糖皮质激素
C.内因子
D.胃泌素
E.胆囊收缩素
A.脑肠肽
6 1 2009 31. Hp感染最容易并发的皮肤疾病是:(2.5分) A.区域性脱斑
B.酒糟鼻
C.荨麻疹
D.剥脱性皮炎
E.白癜风
C.荨麻疹
6 1 2009 32. 下列不属于常用的Hp检测方法的是:(2.5分) A.粘膜组织染色
B.快速尿素酶试验
C.血清抗体检测
D.Hp-DNA的聚合酶链式反应
E.Hp呼气试验
D.Hp-DNA的聚合酶链式反应
6 1 2009 33. 粪便隐血试验阳性者提示出血量多少: (2.5分) A.每日出血量10ml以上
B.每日出血量60ml以上
C.每日出血量30ml以上
D.每日出血量5ml以上
E.每日出血量20ml以上
D.每日出血量5ml以上
6 1 2009 34. 消化性溃疡并发呕血时,首选下列哪种药物止血:(2.5分) A.甲氰咪胍
B.质子泵抑制剂
C.心得安
D.安络血
E.垂体后叶素
B.质子泵抑制剂
6 1 2009 35. 关于上消化道出血的处理哪项有误: (2.5分) A.嘱患者保持安静
B.注意观察尿量变化
C.一律禁食
D.定期复查血象
E.防治失血性休克
C.一律禁食
6 1 2009 36. 引起上消化道出血的最常见的疾病是: (2.5分) A.慢性胃炎
B.消化性溃疡
C.肝硬化食管、胃底静脉曲张破裂
D.胃癌
E.出血性胃炎
B.消化性溃疡
6 1 2009 37. 关于上消化道出血量的估计下列哪项是对的: (2.5分) A.每日出血量在50ml以上粪便隐血试验才呈阳性反应
B.每日出血量200ml以上才会出现黑便
C.胃内积血500ml才会发生呕血
D.一次失血超过500ml可有急性失血的全身表现
E.面色苍白,脉搏>110次/分,收缩压< 80mmHg估计出血量占总血量的10%以下
D.一次失血超过500ml可有急性失血的全身表现
6 1 2009 38. 判断上消化道出血部位最好的检查方法是:(2.5分) A.内窥镜检查
B.钡餐检查
C.B超检查
D.选择性动脉造影
E.胃液分析
A.内窥镜检查
6 1 2009 39. 处理上消化道大出血休克患者,首先采取哪项措施: (2.5分) A.冰盐水洗胃
B.口服止血药
C.补充血容量
D.电凝或激光治疗
E.手术治疗
C.补充血容量
6 1 2009 40. 患者,女,30岁,无溃疡病史,因关节酸痛,服水杨酸制剂,6小时前突然大量呕血,伴晕厥,血压80/50mmHg,心率125次/分,则提示消化道出血的量至少达到: (2.5分) A.5~10ml
B.50~100ml
C.250~300ml
D.400~500ml
E.1000ml以上
E.1000ml以上
6 1 2010 1. 起生理性调节和激素样作用的胃肠肽不包括 (2.5分) A.促胃液素
B.缩胆囊素
C.促胰液素
D.抑胃肽
E.生长抑素
E.生长抑素
6 1 2010 2. Hp感染最容易并发的皮肤疾病是(2.5分) A.区域性脱斑
B.酒糟鼻
C.荨麻疹
D.剥脱性皮炎
E.白癜风
C.荨麻疹
6 1 2010 3. 下面不属于代谢综合征的成分是(2.5分) A.超重/肥胖
B.高血压
C.血脂异常
D.高血糖
E.冠心病
E.冠心病
6 1 2010 4. 胃肠道微生态失调相关性疾病不包括(2.5分) A.抗生素相关性腹泻
B.大肠癌
C.炎症性肠病(IBD)
D.肝病
E.肠易激综合征
E.肠易激综合征
6 1 2010 5. 炎症性肠病的特征性皮肤表现有(2.5分) A.结节性红斑
B.红斑狼疮
C.天胞疮
D.剥脱性皮炎
E.银屑病
A.结节性红斑
6 1 2010 6. 肠结核出现腹部肿块最常见于(2.5分) A.溃疡性肠结核并局限性腹膜炎
B.溃疡性肠结核并肠曲及周围粘连
C.增生性肠结核
D.溃疡性肠结核同时有女性生殖性结核
E.溃疡性肠结核同时有肠系膜淋巴结结核
C.增生性肠结核
6 1 2010 7. 关于肠结核哪项说法是错误的(2.5分) A.腹泻是溃疡性肠结核的主要表现
B.溃疡性肠结核较多发生消化道出血
C.腹块见于增生性肠结核
D.增生性肠结核多以便秘为主要表现
E.肠结核常有右下腹痛
B.溃疡性肠结核较多发生消化道出血
6 1 2010 8. 结核性腹水检查中,不正确的是(2.5分) A.偶见乳糜样腹水
B.比重一般超过1.018,蛋白含量在30g/L以上
C.合并肝硬化时腹水性质接近漏出液
D.白细胞计数超过500X106/L,以淋巴细胞为主
E.腹水浓缩后做结核分枝杆菌培养阳性率高
E.腹水浓缩后做结核分枝杆菌培养阳性率高
6 1 2010 9. 肠结核手术适应症不包括(2.5分) A.完全性肠梗阻
B.急性肠穿孔
C.慢性肠穿孔瘘管形成内科治疗未能闭合者
D.肠出血,经过积极抢救不能止血者
E.结核性腹膜炎
E.结核性腹膜炎
6 1 2010 10. 对于结核性腹膜炎腹部症状的描述,错误的是(2.5分) A.早期腹痛明显
B.腹痛多在脐周,下腹
C.多为持续性隐痛或钝痛
D.可有阵发性绞痛
E.偶可表现为急腹症
A.早期腹痛明显
6 1 2010 11. 诊断克罗恩病最有意义的临床病理改变 (2.5分) A.全壁性炎症
B.裂隙样溃疡
C.非干酪样肉芽肿
D.干酪样肉芽肿
E.瘘管形成
C.非干酪样肉芽肿
6 1 2010 12. 下列哪项对溃疡性结肠炎的轻重程度判断无帮助(2.5分) A.血白细胞
B.血红蛋白
C.白蛋白
D.大便次数
E.体温
A.血白细胞
6 1 2010 13. 女性, 26岁,反复排粘液脓血便4年,加重3天,胃纳良好,体重无明显减轻,其最可能的病因是(2.5分) A.肠易激综合征
B.大肠癌
C.肠结核
D.溃疡性结肠炎
E.真菌性肠炎
D.溃疡性结肠炎
6 1 2010 14. 关于溃疡性结肠炎的叙述,下列哪项正确(2.5分) A.病变呈节段性分布
B.典型的表现是慢性腹泻,便血,抗生素治疗无效
C.溃疡深,易穿孔
D.常常急性起病
E.病变多位于右半结肠
B.典型的表现是慢性腹泻,便血,抗生素治疗无效
6 1 2010 15. 临床上确诊为重症溃疡性结肠炎首要的治疗药物是(2.5分) A.柳氮磺胺吡啶
B.肾上腺糖皮质激素
C.免疫抑制剂
D.手术治疗
E.抗胆碱能药物
B.肾上腺糖皮质激素
6 1 2010 16. 高位小肠梗阻的临床表现特点应为(2.5分) A.以腹痛和呕吐为著,而腹胀,排便,排气停止不明显
B.以腹痛、呕吐和排便、排气停止为著,但腹胀不明显
C.以呕吐和排便、排气停止为著,腹痛轻微无腹胀
D.存在腹痛、呕吐、腹胀及排气排便停止
E.以腹痛和腹胀为著,无呕吐,无排气排便停止
A.以腹痛和呕吐为著,而腹胀,排便,排气停止不明显
6 1 2010 17. 28岁,急性阑尾炎穿孔术后6天,腹部胀痛不适,呈持续性,伴恶心、呕吐,肛门排气消失。腹部检查见全腹膨隆。未见蠕动波,肠鸣音消失,腹部压痛,轻度反跳痛。腹立位平片示小肠及大肠均有充气及液平面,据此你诊断为(2.5分) A.急性机械性不完全性低位小肠梗阻
B.急性机械性不完全性高位小肠梗阻
C.急性不完全性结肠梗阻
D.麻痹性肠梗阻
E.急性机械性完全性小肠梗阻
D.麻痹性肠梗阻
6 1 2010 18. A 55-year-old woman goes to the emergency department because of vomiting and severe abdominal cramping of 3 day's duration. Her pain is centered on the umbilicus. She denies being exposed to a viral or bacterial illness. Her medical history includes a previous cholecystectomy and an appendectomy after which she developed an infection. Her abdomen is not tender, but hyperactive, high-pitched peristalsis with rushes coincides with palpable bowel cramping. Abdominal X-ray films taken in the upright positions demonstrate a ladder-like series of distended small bowel loops. Which of the following is the most likely explanation for these findings? (2.5分) A.Adhesions
B.Ascaris infection
C.Cancer
D.Intussusception
E.Volvulus
A.Adhesions
6 1 2010 19. 男,68岁,腹痛、腹胀1天,便秘3天,恶性无呕吐。体检:腹胀显著,散在压痛,未扪及肿块,指肠指检阴性。腹部X线平片可见一极度扩张的的双袢肠曲。首先考虑诊断是(2.5分) A.低位小肠梗阻
B.粪便所致结肠梗阻
C.肠套叠
D.肿瘤所致的肠梗阻
E.乙状结肠扭转
E.乙状结肠扭转
6 1 2010 20. Which bowel obstruction is candidate for nonoperative therapy initially(2.5分) A.complete obstruction,
B.closed-loop obstruction,
C.bowel obstruction with ischemia or necrosis,
D.chronic partial obstruction
E.bowel obstruction with perforation
D.chronic partial obstruction
6 1 2010 21. 关于我国结直肠癌的流行病学特点,不正确的描述是(2.5分) A.直肠癌比结肠癌发生率高
B.低位直肠癌所占的比例高
C.青年人发病率呈上升趋势
D.农村居民的发病率高于城市居民
D.农村居民的发病率高于城市居民
6 1 2010 22. 散发性结直肠癌中,常涉及的基因变化包括(2.5分) A.APC基因丢失
B.K-ras基因突变
C.p53基因突变
D.Dcc基因突变
E.以上都是
E.以上都是
6 1 2010 23. 直肠癌主要的转移途径是(2.5分) A.直接蔓延
B.淋巴转移
C.血行转移
D.种植转移
B.淋巴转移
6 1 2010 24. 直肠癌最常见的症状是(2.5分) A.便血
B.肠梗阻
C.贫血
D.便秘
A.便血
6 1 2010 25. 结直肠癌的治疗原则是(2.5分) A.化学治疗
B.根治性手术
C.放射治疗
D.以手术切除为主的综合治疗
D.以手术切除为主的综合治疗
6 1 2010 26. 患者,男,50岁,肠镜检查发现乙状结肠肿瘤,病理示腺癌,术前CT检查提示左肝有一约1.5cm大小可疑结节,考虑转移。该患者合适治疗方案是(2.5分) A.术前靶向治疗→手术切除原发灶和肝转移灶→术后辅助化疗
B.姑息性化疗
C.手术切除原发灶和肝转移灶→术后辅助化疗
D.手术切除原发灶和肝转移灶
C.手术切除原发灶和肝转移灶→术后辅助化疗
6 1 2010 27. 36岁,男性,反复便血伴便后肛门外有物脱出2年,需要用手回纳,便血血色鲜红,大便每日1次,无腹痛,无肛门疼痛;表现为便后滴血,时呈喷射状出血,该患者最应该考虑的诊断是(2.5分) A.肛裂
B.外痔
C.内痔
D.直肠脱垂
E.肛瘘
C.内痔
6 1 2010 28. 男性52岁,内痔注射术后3天,突然出现低热以及直肠和下腹部疼痛。实验室检查发现WBC 14000/ul,中性粒细胞90%,腹部CT检查发现腹膜后积气。下一步最恰当的治疗措施是(2.5分) A.使用广谱抗生素
B.直肠镜检查以及直肠灌洗
C.麻醉下直肠检查和局部清创
D.剖腹探查、结肠切除
E.经腹会阴联合直肠切除术
A.使用广谱抗生素
6 1 2010 29. 42岁男性,在混合痔切除术后第5天出现明显的出血症状,血色鲜红伴血块,生命体征:血压100/60mmHg,心率110bpm,体温37.3℃,关于下一步处理方式,一下哪个选项最正确(2.5分) A.结肠镜检查
B.肛门镜检查
C.凝血功能检查
D.血管造影
E.ECT出血扫描
B.肛门镜检查
6 1 2010 30. 患者主诉反复肛周疼痛伴分泌物6个月,检查发现,右侧距离肛门4cm处肉芽样结节,伴压痛,挤压时有脓性分泌物流出;直肠指检,条索状物不明显,后位肛隐窝硬结,伴肛门直肠环纤维化,应考虑(2.5分) A.混合痔
B.单纯性肛瘘
C.复杂性肛瘘
D.直肠息肉
E.直肠癌
C.复杂性肛瘘
6 1 2010 31. 粪便隐血试验阳性者提示出血量多少(2.5分) A.每日出血量10ml以上
B.每日出血量60ml以上
C.每日出血量30ml以上
D.每日出血量5ml以上
E.每日出血量20ml以上
D.每日出血量5ml以上
6 1 2010 32. 下列哪项不是上消化道出血手术治疗的指征(2.5分) A.过去有多次大出血,近期反复大出血
B.年龄超过45岁
C.大出血时伴有幽门梗阻
D.大出血同时伴有急性穿孔
E.大出血同时伴有急性弥漫性腹膜炎
B.年龄超过45岁
6 1 2010 33. 引起上消化道出血的最常见的疾病是(2.5分) A.慢性胃炎
B.消化性溃疡
C.肝硬化食管、胃底静脉曲张破裂
D.胃癌
E.出血性胃炎
B.消化性溃疡
6 1 2010 34. 判断上消化道出血部位最好的检查方法是(2.5分) A.内窥镜检查
B.钡餐检查
C.B超检查
D.选择性动脉造影
E.胃液分析
A.内窥镜检查
6 1 2010 35. 肝硬化食管静脉曲张出血,以下哪项内科处理不恰当(2.5分) A.低蛋白低流质饮食
B.禁食,密切观察血压脉搏
C.应用垂体加压素
D.三腔气囊压迫止血
E.出血停止后导泻、灌肠以预防肝昏迷
A.低蛋白低流质饮食
6 1 2010 36. 患者,女,30岁,无溃疡病史,因关节酸痛,服水杨酸制剂,6小时前突然大量呕血,伴晕厥,血压80/50mmHg,心率125次/分,则提示消化道出血的量至少达到: (2.5分) A.5~10ml
B.50~100ml
C.250~300ml
D.400~500ml
E.1000ml以上
E.1000ml以上
6 1 2010 37. A 41-year-old male is brought to the ED with acute hematemesis. Abdominal exam reveals distension, no rebound or guarding, hepatosplenomegaly, and dull fluid wave. Which of the following is the most likely source of the upper gastrointestinal bleeding (UGI)? (2.5分) A.Mallory-Weiss tear
B.Perforated duodenal ulcer
C.Chronic gastritis
D.Arteriovenous malformation (AVM)
E.Esophageal varices
E.Esophageal varices
6 1 2010 38. 简述肠易激综合征的治疗原则(2.5分) A.建立良好的医患关系
B.一般治疗
C.根据主要症状对症治疗及根据严重程度分级治疗
D.重视心理治疗
E.以上均是
E.以上均是
6 1 2010 39. 请问IBS可能的病因包括: (2.5分) A.精神因素
B.肠道菌群失调
C.感染因素
D.遗传和环境因素
E.以上均是
E.以上均是
6 1 2010 40. The duration of Irritable Bowel Syndrome is at least:(2.5分) A.3 months
B.6 months
C.9 months
D.12 months
E.1 months
B.6 months
6 1 2011 1. 患者主诉排便时肛门疼痛伴鲜红血便3周。检查发现典型的肛管后正中肛裂,裂口无潜行、无分泌物。对于该患者最恰当的处理是(2.5分) A.肛门内括约肌侧方切开术
B.麻醉下行肛门检查和活检
C.Y-V肛门整形
D.直肠镜检查、温水坐浴、增加膳食纤维
E.结肠镜检查后口服止痛剂和氢化可的松
D.直肠镜检查、温水坐浴、增加膳食纤维
6 1 2011 2. 42岁女性患者,10、12、14年前分别顺产3个小孩,现患有后正中肛裂,无皮赘及肥大肛乳头,经过6周的增加膳食纤维,坐浴以及局部使用0.2%硝酸甘油后症状无显著好转,以下最恰当的治疗方式是: (2.5分) A.口服地尔硫卓
B.局部创面使用肉毒杆菌毒素
C.肛门整形
D.侧方内括约肌切开术
E.侧方外括约肌切开术
D.侧方内括约肌切开术
6 1 2011 3. 38岁男性直肠肿瘤患者在低位前切除术后出勃起功能阻碍,最可能是手术时损伤了一下哪项: (2.5分) A.腹下神经
B.盆腔内脏神经
C.骨盆神经丛
D.阴部神经
E.直肠下神经
A.腹下神经
6 1 2011 4. 根据肛瘘瘘管的行走部位,可分为4种类型,以下那一个不正确? (2.5分) A.经括约肌
B.括约肌间
C.括约肌上
D.括约肌内型
E.括约肌外型
D.括约肌内型
6 1 2011 5. 直肠指检,扪到条索状,伴有轻压痛,挤压时外口有脓性分泌物流出者应考虑(2.5分) A.内痔
B.肛瘘
C.外痔
D.直肠息肉
E.直肠癌
B.肛瘘
6 1 2011 6. 起生理性调节和激素样作用的胃肠肽不包括 (2.5分) A.促胃液素
B.缩胆囊素
C.促胰液素
D.抑胃肽
E.生长抑素
E.生长抑素
6 1 2011 7. 多种胃肠肽也分布于中枢神经系统中,这种双重分布的肽,被称为(2.5分) A.脑肠肽
B.糖皮质激素
C.内因子
D.胃泌素
E.胆囊收缩素
A.脑肠肽
6 1 2011 8. 下面不属于代谢综合征的成分是 (2.5分) A.超重/肥胖
B.高血压
C.血脂异常
D.高血糖
E.冠心病
E.冠心病
6 1 2011 9. 炎症性肠病的特征性皮肤表现有(2.5分) A.结节性红斑
B.红斑狼疮
C.天胞疮
D.剥脱性皮炎
E.银屑病
A.结节性红斑
6 1 2011 10. PCOS和脂肪肝的最重要共同发病机制(2.5分) A.脂质代谢紊乱
B.糖代谢紊乱
C.胰岛素抵抗
D.基因多态性
E.蛋白质代谢紊乱
C.胰岛素抵抗
6 1 2011 11. 高位小肠梗阻的临床表现特点应为: (2.5分) A.以腹痛和呕吐为著,而腹胀,排便,排气停止不明显
B.以腹痛、呕吐和排便、排气停止为著,但腹胀不明显
C.以呕吐和排便、排气停止为著,腹痛轻微无腹胀
D.存在腹痛、呕吐、腹胀及排气排便停止
E.以腹痛和腹胀为著,无呕吐,无排气排便停止
A.以腹痛和呕吐为著,而腹胀,排便,排气停止不明显
6 1 2011 12. 绞窄性肠梗阻最容易发生的酸碱失衡类型是(2.5分) A.呼吸性碱中毒
B.代谢性酸中毒
C.代谢性碱中毒
D.呼吸性酸中毒
E.呼吸性酸中毒和代谢性碱中毒
B.代谢性酸中毒
6 1 2011 13. 我国成人肠梗阻最常见的原因是(2.5分) A.肠扭转
B.肠粘连
C.腹内疝
D.肠套叠
E.肠肿瘤
B.肠粘连
6 1 2011 14. A 54-year-old man has had colicky abdominal pain and protracted vomiting for several days. He has developed progressive moderate abdominal distention, and has not had a bowel movement or pass gas for five days. He has high-pitched, loud bowel sounds that coincide with the colicky pain, and X-ray show distended loops of small bowel and air-fluid levels. Five years ago he had an exploratory laparotomy for a blunt trauma of abdomen. Six hours after being hospitalized and placed on NG suction and IV fluids. He develops fever, leukocytosis, tenderness, and rebound tenderness. What is the most likely diagnosis? (2.5分) A.acute appendicitis
B.Acute pancreatitis
C.Strangulated obstruction of bowel
D.Carcinoma of sigmoid colon
E.Simple obstruction of small intestine
C.Strangulated obstruction of bowel
6 1 2011 15. 男,68岁,腹痛、腹胀1天,便秘3天,恶性无呕吐。体检:腹胀显著,散在压痛,未扪及肿块,指肠指检阴性。腹部X线平片可见一极度扩张的的双袢肠曲。首先考虑诊断是: (2.5分) A.低位小肠梗阻
B.粪便所致结肠梗阻
C.肠套叠
D.肿瘤所致的肠梗阻
E.乙状结肠扭转
E.乙状结肠扭转
6 1 2011 16. 男,65岁,阵发性腹痛、腹胀、肛门停止排气排便2天,无呕吐。体检:腹部明显膨隆, 腹软,散在不固定压痛,可闻及高亢肠鸣音。腹部平片示结肠明显扩张,范围自盲肠至乙状结肠,盲肠直径为8cm,腹部可见少量小肠充气。以下那一个正确(2.5分) A.麻痹性肠梗阻
B.高位小肠梗阻
C.低位小肠梗阻
D.降结肠梗阻
E.以上都不是
E.以上都不是
6 1 2011 17. 粘连性肠梗阻最常见于(2.5分) A.先天性发育异常
B.胎粪性腹膜炎
C.化脓性腹膜炎
D.腹腔手术后
E.腹腔创伤后
D.腹腔手术后
6 1 2011 18. 对肠结核最有诊断价值的检查是(2.5分) A.X线钡餐检查发现肠腔狭窄
B.结肠镜检查示回盲部炎症
C.结肠镜下活检找到干酪性上皮样肉芽肿
D.结核菌素试验强阳性
E.粪便中查到结核杆菌
C.结肠镜下活检找到干酪性上皮样肉芽肿
6 1 2011 19. 下列结核性腹膜炎的传播途径,哪一个来源不属于血行播散(2.5分) A.粟粒性肺结核
B.睾丸结核
C.骨结核
D.关节结核
E.输卵管结核
E.输卵管结核
6 1 2011 20. 溃疡型肠结核多见的临床表现是(2.5分) A.粘液脓血便
B.鲜血便
C.里急后重
D.糊样便
E.便秘
D.糊样便
6 1 2011 21. 关于结核性腹膜炎患者的腹水,下述哪一项不正确(2.5分) A.腹水可为草黄色、淡血性或乳糜性
B.腹水比重>1.016
C.腹水蛋白质含量>30g/L
D.腹水白细胞计数>0.5*10^9/L
E.腹水结核杆菌培养大多阳性
E.腹水结核杆菌培养大多阳性
6 1 2011 22. 结核性腹膜炎最常见的并发症是(2.5分) A.急性穿孔
B.腹腔脓肿
C.肠瘘
D.肠梗阻
E.粪瘘
D.肠梗阻
6 1 2011 23. 哪项可以作为克罗恩病与溃疡性结肠炎的鉴别点(2.5分) A.便血的程度
B.发热
C.腹痛部位和程度
D.瘘管形成
E.血沉和C反应蛋白升高
D.瘘管形成
6 1 2011 24. 女性,35岁,腹泻1年。体检发现肛瘘,结肠镜示回盲部铺路石样改变,最可能的诊断是(2.5分) A.结肠癌
B.溃疡性结肠炎
C.细菌性痢疾
D.克罗恩病
E.肠结核
D.克罗恩病
6 1 2011 25. 关于溃疡性结肠炎的病理改变错误的是(2.5分) A.病变呈广泛浅小溃疡
B.病变常侵入肌层易出现穿孔及巨结肠
C.炎症反复发作可形成炎性息肉
D.病变反复发作可致结肠变形缩短
E.溃疡性结肠炎可出现癌变
B.病变常侵入肌层易出现穿孔及巨结肠
6 1 2011 26. 下列哪项不是手术治疗溃疡性结肠炎的指征(2.5分) A.反复发作的轻度溃疡性结肠炎
B.并发大出血
C.并发结肠癌变
D.肠穿孔
E.合并中毒性结肠扩张
A.反复发作的轻度溃疡性结肠炎
6 1 2011 27. 目前认为,下列哪项不是溃疡性结肠炎的主要发病因素(2.5分) A.肠道感染
B.体液免疫异常
C.细胞免疫异常
D.遗传因素
E.饮食中的亚硝胺类物质
E.饮食中的亚硝胺类物质
6 1 2011 28. 女,37岁,已婚。右下腹部不规则痛、低热、盗汗半年。4年前有"肺结核"和"子宫内膜结核"史。体检:T38.2℃,腹软,右下腹较饱满,有深压痛。粪便细菌培养阴性、未找到阿米巴。诊断首先考虑(2.5分) A.肠粘连
B.溃疡性结肠炎
C.溃疡性结肠炎
D.细菌性痢疾
E.肠结核
E.肠结核
6 1 2011 29. 女,37岁,已婚。右下腹部不规则痛、低热、盗汗半年。4年前有"肺结核"和"子宫内膜结核"史。体检:T38.2℃,腹软,右下腹较饱满,有深压痛。粪便细菌培养阴性、未找到阿米巴。哪项检查对明确诊断最有帮助(2.5分) A.右下腹部B超
B.全胃肠道钡餐检查
C.钡剂灌肠
D.电子结肠镜+病理检查
E.腹腔镜检查
D.电子结肠镜+病理检查
6 1 2011 30. 粪便隐血试验阳性者提示出血量多少(2.5分) A.每日出血量10ml以上
B.每日出血量60ml以上
C.每日出血量30ml以上
D.每日出血量5ml以上
E.每日出血量20ml以上
D.每日出血量5ml以上
6 1 2011 31. 下列哪项不是上消化道出血手术治疗的指征(2.5分) A.过去有多次大出血,近期反复大出血
B.年龄超过45岁
C.大出血时伴有幽门梗阻
D.大出血同时伴有急性穿孔
E.大出血同时伴有急性弥漫性腹膜炎
B.年龄超过45岁
6 1 2011 32. 引起上消化道出血的最常见的疾病是(2.5分) A.慢性胃炎
B.消化性溃疡
C.肝硬化食管、胃底静脉曲张破裂
D.胃癌
E.出血性胃炎
B.消化性溃疡
6 1 2011 33. 关于上消化道出血,下列哪种说法不对(2.5分) A.呕血多见于幽门以上部位出血
B.呕血者有黑便而黑便者不一定有呕血
C.应包括胆道出血
D.胃大部切除后的空肠上段出血也属于上消化道出血
E.上消化道出血均是消化系统疾病所致
E.上消化道出血均是消化系统疾病所致
6 1 2011 34. 处理上消化道大出血休克患者,首先采取哪项措施(2.5分) A.冰盐水洗胃
B.口服止血药
C.补充血容量
D.电凝或激光治疗
E.手术治疗
C.补充血容量
6 1 2011 35. 除哪项外有助于肝硬化食管胃底静脉曲张破裂出血的诊断(2.5分) A.门静脉高压
B.肝炎病史
C.节律性疼痛
D.低白蛋白血症
E.脾大
C.节律性疼痛
6 1 2011 36. Which of the following is true regarding Mallory-Weiss syndrome? (2.5分) A.Operative repair usually is necessary
B.Is the most common cause of gastrointestinal bleeding during pregnancy
C.Endoscopy is contraindicated
D.Bleeding stops spontaneously in the majority of cases
E.Accounts for 1/3 of deaths from upper gastrointestinal bleeding
D.Bleeding stops spontaneously in the majority of cases
6 1 2011 37. 肠易激综合征腹痛的规律是(2.5分) A.无明显规律性;
B.疼痛-排便加重;
C.进食-疼痛-缓解;
D.疼痛进食-缓解;
E.疼痛-排便缓解
D.疼痛进食-缓解;
6 1 2011 38. 简述肠易激综合征的治疗原则(2.5分) A.建立良好的医患关系
B.一般治疗
C.根据主要症状对症治疗及根据严重程度分级治疗
D.重视心理治疗
E.以上均是
E.以上均是
6 1 2011 39. 请问IBS可能的病因包括(2.5分) A.精神因素
B.肠道菌群失调
C.感染因素
D.遗传和环境因素
E.以上均是
E.以上均是
6 1 2011 40. The duration of Irritable Bowel Syndrome is at least(2.5分) A.3 months
B.6 months
C.9 months
D.12 months
E.1 months
B.6 months
6 1 2012 1. 以下属于促进胃液分泌的内源性物质的是:(2.5分) A.生长抑素
B.前列腺素
C.上皮生长因子,
D.内因子,
E.乙酰胆碱
E.乙酰胆碱
6 1 2012 2. Hp感染最容易并发的皮肤疾病是:(2.5分) A.区域性脱斑
B.酒糟鼻
C.荨麻疹
D.剥脱性皮炎
E.白癜风
C.荨麻疹
6 1 2012 3. 胃肠道微生态失调相关性疾病不包括:(2.5分) A.抗生素相关性腹泻
B.大肠癌
C.炎症性肠病(IBD)
D.肝病
E.肠易激综合征
E.肠易激综合征
6 1 2012 4. 下列不属于常用的Hp检测方法的是:(2.5分) A.粘膜组织染色
B.快速尿素酶试验
C.血清抗体检测
D.Hp-DNA的聚合酶链式反应
E.Hp呼气试验
D.Hp-DNA的聚合酶链式反应
6 1 2012 5. PCOS和脂肪肝的最重要共同发病机制:(2.5分) A.脂质代谢紊乱
B.糖代谢紊乱
C.胰岛素抵抗
D.基因多态性
E.蛋白质代谢紊乱
C.胰岛素抵抗
6 1 2012 6. 对肠结核最有诊断价值的检查是:(2.5分) A.X线钡餐检查发现肠腔狭窄
B.结肠镜检查示回盲部炎症
C.结肠镜下活检找到干酪性上皮样肉芽肿
D.结核菌素试验强阳性
E.粪便中查到结核杆菌
C.结肠镜下活检找到干酪性上皮样肉芽肿
6 1 2012 7. 下列结核性腹膜炎的传播途径,哪一个来源不属于血行播散:(2.5分) A.粟粒性肺结核
B.睾丸结核
C.骨结核
D.关节结核
E.输卵管结核
E.输卵管结核
6 1 2012 8. 溃疡型肠结核多见的临床表现是:(2.5分) A.粘液脓血便
B.鲜血便
C.里急后重
D.糊样便
E.便秘
D.糊样便
6 1 2012 9. 关于结核性腹膜炎患者的腹水,下述哪一项不正确:(2.5分) A.腹水可为草黄色、淡血性或乳糜性
B.腹水比重>1.016
C.腹水蛋白质含量>30g/L
D.腹水白细胞计数>0.5*10^9/L
E.腹水结核杆菌培养大多阳性
E.腹水结核杆菌培养大多阳性
6 1 2012 10. 结核性腹膜炎最常见的并发症是:(2.5分) A.急性穿孔
B.腹腔脓肿
C.肠瘘
D.肠梗阻
E.粪瘘
D.肠梗阻
6 1 2012 11. 哪项可以作为克罗恩病与溃疡性结肠炎的鉴别点:(2.5分) A.便血的程度
B.发热
C.腹痛部位和程度
D.瘘管形成
E.血沉和C反应蛋白升高
D.瘘管形成
6 1 2012 12. 女性,35岁,腹泻1年。体检发现肛瘘,结肠镜示回盲部铺路石样改变,最可能的诊断是:(2.5分) A.结肠癌
B.溃疡性结肠炎
C.细菌性痢疾
D.克罗恩病
E.肠结核
D.克罗恩病
6 1 2012 13. 关于溃疡性结肠炎的病理改变错误的是:(2.5分) A.病变呈广泛浅小溃疡
B.病变常侵入肌层易出现穿孔及巨结肠
C.炎症反复发作可形成炎性息肉
D.病变反复发作可致结肠变形缩短
E.溃疡性结肠炎可出现癌变
B.病变常侵入肌层易出现穿孔及巨结肠
6 1 2012 14. 下列哪项不是手术治疗溃疡性结肠炎的指征:(2.5分) A.反复发作的轻度溃疡性结肠炎
B.并发大出血
C.并发结肠癌变
D.肠穿孔
E.合并中毒性结肠扩张
A.反复发作的轻度溃疡性结肠炎
6 1 2012 15. 目前认为,下列哪项不是溃疡性结肠炎的主要发病因素:(2.5分) A.肠道感染
B.体液免疫异常
C.细胞免疫异常
D.遗传因素
E.饮食中的亚硝胺类物质
E.饮食中的亚硝胺类物质
6 1 2012 16. 急性肠梗阻不会出现那个变化:(2.5分) A.肠腔因气体和液体的积聚而膨胀
B.肠壁变薄
C.肠腔压力升高
D.肠壁血运障碍
E.肠壁代偿性肥厚
E.肠壁代偿性肥厚
6 1 2012 17. 28岁,急性阑尾炎穿孔术后6天,腹部胀痛不适,呈持续性,伴恶心、呕吐,肛门排气消失。腹部检查见全腹膨隆。未见蠕动波,肠鸣音消失,腹部压痛,轻度反跳痛。腹立位平片示小肠及大肠均有充气及液平面,据此你诊断为:(2.5分) A.急性机械性不完全性低位小肠梗阻
B.急性机械性不完全性高位小肠梗阻
C.急性不完全性结肠梗阻
D.麻痹性肠梗阻
E.急性机械性完全性小肠梗阻
D.麻痹性肠梗阻
6 1 2012 18. A 55-year-old woman goes to the emergency department because of vomiting and severe abdominal cramping of 3 day's duration. Her pain is centered on the umbilicus. She denies being exposed to a viral or bacterial illness. Her medical history includes a previous cholecystectomy and an appendectomy after which she developed an infection. Her abdomen is not tender, but hyperactive, high-pitched peristalsis with rushes coincides with palpable bowel cramping. Abdominal X-ray films taken in the upright positions demonstrate a ladder-like series of distended small bowel loops. Which of the following is the most likely explanation for these findings?(2.5分) A.Adhesions
B.Ascaris infection
C.Cancer
D.Intussusception
E.Volvulus
A.Adhesions
6 1 2012 19. 男,65岁,阵发性腹痛、腹胀、肛门停止排气排便2天,无呕吐。体检:腹部明显膨隆, 腹软,散在不固定压痛,可闻及高亢肠鸣音。腹部平片示结肠明显扩张,范围自盲肠至乙状结肠,盲肠直径为8cm,腹部可见少量小肠充气。以下那一个正确:(2.5分) A.麻痹性肠梗阻
B.高位小肠梗阻
C.低位小肠梗阻
D.降结肠梗阻
E.以上都不是
E.以上都不是
6 1 2012 20. 粘连性肠梗阻最常见于:(2.5分) A.先天性发育异常
B.胎粪性腹膜炎
C.化脓性腹膜炎
D.腹腔手术后
E.腹腔创伤后
D.腹腔手术后
6 1 2012 21. 男性,87岁,粘液稀便2个月,脐周及下腹部隐痛不适,体检:腹部平软,无压痛及肿块,大便检查潜血(+);肠镜检查距肛门9cm处的直肠肿块,病理:腺癌;手术前MRI评估无MRF累及,应选择哪种手术为佳?(2.5分) A.经腹直肠癌切除,人工肛门,远端封闭
B.拉下式直肠癌切除术
C.经腹直肠癌切除术
D.腹会阴联合直肠癌根治术
E.乙状结肠造瘘术
A.经腹直肠癌切除,人工肛门,远端封闭
6 1 2012 22. 以下哪项检查适用于评估直肠癌局部侵犯深度?(2.5分) A.肠镜
B.气钡双重造影
C.盆腔MRI
D.上腹部CT
C.盆腔MRI
6 1 2012 23. 结肠癌最常见的远处血运转移部位是:(2.5分) A.区域淋巴结
B.锁骨上淋巴结
C.肝
D.肺
E.脑
C.肝
6 1 2012 24. 直肠癌的主要临床症状和体检不包括:(2.5分) A.便血
B.排便习惯改变
C.腹胀、腹痛等不全性肠梗阻
D.直肠指检可发现病变
E.腹部可触及包块
E.腹部可触及包块
6 1 2012 25. 直肠癌最基本的诊断方法是:(2.5分) A.直肠镜
B.乙状结肠镜
C.钡剂灌肠
D.B型超声检查
E.直肠指检
E.直肠指检
6 1 2012 26. A 47-year-old woman presented with a 2-month history of rectal bleeding and change in bowel habits. She underwent a colonoscopy that demonstrated a 5-cm fungating, friable, and partially obstructing mass in the distal rectum, approximately 5 cm from the anal verge. The tumor was palpable on digital rectal examination on the anterior wall of rectum. Biopsy demonstrated a moderately differentiated invasive adenocarcinoma, microsatellite stable. A staging work-up, including a computed tomography scan of the chest, abdomen, and pelvis, demonstrated rectal wall thickening in the midrectum and small lymph nodes in the left perirectal fat. There was a nonspecific 3-mm right lower lobe pulmonary nodule. Rectal magnetic resonance imaging demonstrated a 3-cm mass arising from mid-distal rectum with minimal extension beyond muscularis propria into the mesorectal fat, but without invasion of mesorectal fascia. There were at least three small mesorectal lymph nodes present; the largest rounded node measured up to 8mm, and no additional pelvic lymphadenopathy was identified. Her carcinoembryonic antigen was 1.1, and all other laboratory studies were within normal limits. The optimal treatment choice is:(2.5分) A.Low anterior resection
B.Miles procedure
C.Preoperative chemotherapy
D.Preoperative chemoradiation
E.Preoperative radiation
D.Preoperative chemoradiation
6 1 2012 27. 内痔注射术后3天的患者,突然出现发热,体温38.5度,感觉直肠和下腹部疼痛,实验室检查发现WBC 14000/ul,腹部平片发现腹膜后积气。下一步最恰当的治疗措施是:(2.5分) A.CT检查
B.使用广谱抗生素
C.直肠镜检查以及直肠灌洗
D.麻醉下直肠检查和局部清创
E.剖腹探查、结肠切除
B.使用广谱抗生素
6 1 2012 28. 男性患者,75岁,间歇性发作的肛门瘙痒症6年,肛周瘙痒和不适感加重2个月,无便血和肛门疼痛,查体发现左侧肛缘皮炎样改变以及小溃疡形成。下列哪项是最佳的下一步治疗措施:(2.5分) A.活检排除恶变
B.局部广泛切除
C.激素软膏
D.硝酸甘油软膏
E.内括约切开术
A.活检排除恶变
6 1 2012 29. 42岁男性,在混合痔切除术后第5天出现明显的出血症状,生命体征:血压100/60mmHg,心率110bpm,体温37.3℃,关于下一步处理方式,一下哪个选项最正确:(2.5分) A.输血
B.麻醉下局部探查
C.ECT出血扫描
D.结肠镜
E.血管造影
B.麻醉下局部探查
6 1 2012 30. 以下对肛管部位关于齿状线的描述中,哪一个是错误的?(2.5分) A.齿状线以上的上皮是复层立方上皮(粘膜,属内胚层)
B.齿状线以上的淋巴回流方向为髂内淋巴结、肠系膜下淋巴结
C.齿状线以下的神经为躯体神经(痛觉敏锐)
D.齿状线以上静脉通过阴部内静脉(属下腔静脉系)回流
E.齿状线以下的动脉血液供应为肛门动脉
D.齿状线以上静脉通过阴部内静脉(属下腔静脉系)回流
6 1 2012 31. 反复肛周肿痛伴流脓1年余,体检:右后距离肛门4cm见肉芽样组织外口,直肠指检,扪到条索状,伴有轻压痛,挤压时外口有脓性分泌物流出者,临床应考虑:(2.5分) A.内痔
B.外痔
C.肛瘘
D.直肠息肉
E.肛管肿瘤
C.肛瘘
6 1 2012 32. 粪便隐血试验阳性者提示出血量多少: (2.5分) A.每日出血量10ml以上
B.每日出血量60ml以上
C.每日出血量30ml以上
D.每日出血量5ml以上
E.每日出血量20ml以上
D.每日出血量5ml以上
6 1 2012 33. 下列哪项不是上消化道出血手术治疗的指征:(2.5分) A.过去有多次大出血,近期反复大出血
B.年龄超过45岁
C.大出血时伴有幽门梗阻
D.大出血同时伴有急性穿孔
E.大出血同时伴有急性弥漫性腹膜炎
B.年龄超过45岁
6 1 2012 34. 引起上消化道出血的最常见的疾病是:(2.5分) A.慢性胃炎
B.消化性溃疡
C.肝硬化食管、胃底静脉曲张破裂
D.胃癌
E.出血性胃炎
B.消化性溃疡
6 1 2012 35. 关于上消化道出血,下列哪种说法不对: (2.5分) A.呕血多见于幽门以上部位出血
B.呕血者有黑便而黑便者不一定有呕血
C.应包括胆道出血
D.胃大部切除后的空肠上段出血也属于上消化道出血
E.上消化道出血均是消化系统疾病所致
E.上消化道出血均是消化系统疾病所致
6 1 2012 36. 处理上消化道大出血休克患者,首先采取哪项措施: (2.5分) A.冰盐水洗胃
B.口服止血药
C.补充血容量
D.电凝或激光治疗
E.手术治疗
C.补充血容量
6 1 2012 37. 除哪项外有助于肝硬化食管胃底静脉曲张破裂出血的诊断: (2.5分) A.门静脉高压
B.肝炎病史
C.节律性疼痛
D.低白蛋白血症
E.脾大
C.节律性疼痛
6 1 2012 38. Which of the following is true regarding Mallory-Weiss syndrome? (2.5分) A.Operative repair usually is necessary
B.Is the most common cause of gastrointestinal bleeding during pregnancy
C.Endoscopy is contraindicated
D.Bleeding stops spontaneously in the majority of cases
E.Accounts for 1/3 of deaths from upper gastrointestinal bleeding
D.Bleeding stops spontaneously in the majority of cases
6 1 2012 39. 肠易激综合征腹痛的规律是:(2.5分) A.无明显规律性;
B.疼痛-排便加重;
C.进食-疼痛-缓解;
D.疼痛进食-缓解;
E.疼痛-排便缓解
D.疼痛进食-缓解;
6 1 2012 40. 简述肠易激综合征的治疗原则: (2.5分) A.建立良好的医患关系
B.一般治疗
C.根据主要症状对症治疗及根据严重程度分级治疗
D.重视心理治疗
E.以上均是
E.以上均是
6 1 2013 1. 以下属于促进胃液分泌的内源性物质的是:(2.5分) A.生长抑素
B.前列腺素
C.上皮生长因子
D.内因子
E.乙酰胆碱
E.乙酰胆碱
6 1 2013 2. Hp感染最容易并发的皮肤疾病是:(2.5分) A.区域性脱斑
B.酒糟鼻
C.荨麻疹
D.剥脱性皮炎
E.白癜风
C.荨麻疹
6 1 2013 3. 胃肠道微生态失调相关性疾病不包括:(2.5分) A.抗生素相关性腹泻
B.大肠癌
C.炎症性肠病(IBD)
D.肝病
E.肠易激综合征
E.肠易激综合征
6 1 2013 4. 下列不属于常用的Hp检测方法的是:(2.5分) A.粘膜组织染色
B.快速尿素酶试验
C.血清抗体检测
D.Hp-DNA的聚合酶链式反应
E.Hp呼气试验
D.Hp-DNA的聚合酶链式反应
6 1 2013 5. PCOS和脂肪肝的最重要共同发病机制:(2.5分) A.脂质代谢紊乱
B.糖代谢紊乱
C.胰岛素抵抗
D.基因多态性
E.蛋白质代谢紊乱
C.胰岛素抵抗
6 1 2013 6. 对肠结核最有诊断价值的检查是:(2.5分) A.X线钡餐检查发现肠腔狭窄
B.结肠镜检查示回盲部炎症
C.结肠镜下活检找到干酪性上皮样肉芽肿
D.结核菌素试验强阳性
E.粪便中查到结核杆菌
C.结肠镜下活检找到干酪性上皮样肉芽肿
6 1 2013 7. 下列结核性腹膜炎的传播途径,哪一个来源不属于血行播散:(2.5分) A.粟粒性肺结核
B.睾丸结核
C.骨结核
D.关节结核
E.输卵管结核
E.输卵管结核
6 1 2013 8. 溃疡型肠结核多见的临床表现是:(2.5分) A.粘液脓血便
B.鲜血便
C.里急后重
D.糊样便
E.便秘
D.糊样便
6 1 2013 9. 关于结核性腹膜炎患者的腹水,下述哪一项不正确:(2.5分) A.腹水可为草黄色、淡血性或乳糜性
B.腹水比重>1.016
C.腹水蛋白质含量>30g/L
D.腹水白细胞计数>0.5*10^9/L
E.腹水结核杆菌培养大多阳性
E.腹水结核杆菌培养大多阳性
6 1 2013 10. 结核性腹膜炎最常见的并发症是:(2.5分) A.急性穿孔
B.腹腔脓肿
C.肠瘘
D.肠梗阻
E.粪瘘
D.肠梗阻
6 1 2013 11. 哪项可以作为克罗恩病与溃疡性结肠炎的鉴别点:(2.5分) A.便血的程度
B.发热
C.腹痛部位和程度
D.瘘管形成
E.血沉和C反应蛋白升高
D.瘘管形成
6 1 2013 12. 女性,35岁,腹泻1年。体检发现肛瘘,结肠镜示回盲部铺路石样改变,最可能的诊断是:(2.5分) A.结肠癌
B.溃疡性结肠炎
C.细菌性痢疾
D.克罗恩病
E.肠结核
D.克罗恩病
6 1 2013 13. 关于溃疡性结肠炎的病理改变错误的是:(2.5分) A.病变呈广泛浅小溃疡
B.病变常侵入肌层易出现穿孔及巨结肠
C.炎症反复发作可形成炎性息肉
D.病变反复发作可致结肠变形缩短
E.溃疡性结肠炎可出现癌变
B.病变常侵入肌层易出现穿孔及巨结肠
6 1 2013 14. 下列哪项不是手术治疗溃疡性结肠炎的指征:(2.5分) A.反复发作的轻度溃疡性结肠炎
B.并发大出血
C.并发结肠癌变
D.肠穿孔
E.合并中毒性结肠扩张
A.反复发作的轻度溃疡性结肠炎
6 1 2013 15. 目前认为,下列哪项不是溃疡性结肠炎的主要发病因素:(2.5分) A.肠道感染
B.体液免疫异常
C.细胞免疫异常
D.遗传因素
E.饮食中的亚硝胺类物质
E.饮食中的亚硝胺类物质
6 1 2013 16. 急性肠梗阻不会出现那个变化:(2.5分) A.肠腔因气体和液体的积聚而膨胀
B.肠壁变薄
C.肠腔压力升高
D.肠壁血运障碍
E.肠壁代偿性肥厚
E.肠壁代偿性肥厚
6 1 2013 17. 28岁,急性阑尾炎穿孔术后6天,腹部胀痛不适,呈持续性,伴恶心、呕吐,肛门排气消失。腹部检查见全腹膨隆。未见蠕动波,肠鸣音消失,腹部压痛,轻度反跳痛。腹立位平片示小肠及大肠均有充气及液平面,据此你诊断为:(2.5分) A.急性机械性不完全性低位小肠梗阻
B.急性机械性不完全性高位小肠梗阻
C.急性不完全性结肠梗阻
D.麻痹性肠梗阻
E.急性机械性完全性小肠梗阻
D.麻痹性肠梗阻
6 1 2013 18. A 55-year-old woman goes to the emergency department because of vomiting and severe abdominal cramping of 3 day's duration. Her pain is centered on the umbilicus. She denies being exposed to a viral or bacterial illness. Her medical history includes a previous cholecystectomy and an appendectomy after which she developed an infection. Her abdomen is not tender, but hyperactive, high-pitched peristalsis with rushes coincides with palpable bowel cramping. Abdominal X-ray films taken in the upright positions demonstrate a ladder-like series of distended small bowel loops. Which of the following is the most likely explanation for these findings?(2.5分) A.Adhesions
B.Ascaris infection
C.Cancer
D.Intussusception
E.Volvulus
A.Adhesions
6 1 2013 19. 男,65岁,阵发性腹痛、腹胀、肛门停止排气排便2天,无呕吐。体检:腹部明显膨隆, 腹软,散在不固定压痛,可闻及高亢肠鸣音。腹部平片示结肠明显扩张,范围自盲肠至乙状结肠,盲肠直径为8cm,腹部可见少量小肠充气。以下那一个正确:(2.5分) A.麻痹性肠梗阻
B.高位小肠梗阻
C.低位小肠梗阻
D.降结肠梗阻
E.以上都不是
E.以上都不是
6 1 2013 20. 粘连性肠梗阻最常见于:(2.5分) A.先天性发育异常
B.胎粪性腹膜炎
C.化脓性腹膜炎
D.腹腔手术后
E.腹腔创伤后
D.腹腔手术后
6 1 2013 21. 男性,57岁,粘液稀便2个月;体检:腹部平软,无压痛及肿块,大便检查潜血(+);肠镜检查距肛门9cm处的直肠肿块,病理:腺癌;手术前直肠MRI评估无MRF累及,CT检查未见远处转移;应选择哪种手术为佳?(2.5分) A.经腹直肠癌切除,人工肛门,远端封闭
B.拉下式直肠癌切除术
C.经腹直肠癌切除术
D.腹会阴联合直肠癌根治术
E.乙状结肠造瘘术
C.经腹直肠癌切除术
6 1 2013 22. 以下哪项检查适用于评估直肠癌局部侵犯深度?(2.5分) A.结肠镜
B.气钡双重造影
C.腔内超声检查
D.腹部CT增强
E.盆腔MRI增强
E.盆腔MRI增强
6 1 2013 23. 结肠癌最常见的远处血运转移部位是:(2.5分) A.区域淋巴结
B.锁骨上淋巴结
C.肝
D.肺
E.脑
C.肝
6 1 2013 24. 直肠癌的主要临床症状和体检不包括:(2.5分) A.便血
B.排便习惯改变
C.腹胀、腹痛等不全性肠梗阻
D.直肠指检可发现病变
E.腹部可触及包块
E.腹部可触及包块
6 1 2013 25. 直肠癌最基本的诊断方法是:(2.5分) A.直肠镜
B.乙状结肠镜
C.钡剂灌肠
D.B型超声检查
E.直肠指检
E.直肠指检
6 1 2013 26. A 47-year-old woman presented with a 2-month history of rectal bleeding and change in bowel habits. She underwent a colonoscopy that demonstrated a 5-cm fungating, friable, and partially obstructing mass in the distal rectum, approximately 5 cm from the anal verge. The tumor was palpable on digital rectal examination on the anterior wall of rectum. Biopsy demonstrated a moderately differentiated invasive adenocarcinoma, microsatellite stable. A staging work-up, including a computed tomography scan of the chest, abdomen, and pelvis, demonstrated rectal wall thickening in the midrectum and small lymph nodes in the left perirectal fat. There was a nonspecific 3-mm right lower lobe pulmonary nodule. Rectal magnetic resonance imaging demonstrated a 3-cm mass arising from mid-distal rectum with minimal extension beyond muscularis propria into the mesorectal fat, but without invasion of mesorectal fascia. There were at least three enlarged mesorectal lymph nodes present; the largest rounded node measured up to 8mm, and no additional pelvic lymphadenopathy was identified. Her carcinoembryonic antigen was 1.1, and all other laboratory studies were within normal limits. The optimal treatment choice is:(2.5分) A.Low anterior resection
B.Miles procedure
C.Preoperative chemotherapy
D.Preoperative chemoradiation
E.Preoperative radiation
D.Preoperative chemoradiation
6 1 2013 27. 内痔注射术后3天的患者,突然出现发热,体温38.5度,感觉直肠和下腹部疼痛,实验室检查发现WBC 14000/ul,腹部平片发现腹膜后积气。下一步最恰当的治疗措施是:(2.5分) A.进食,CT检查
B.直肠镜检查以及直肠灌洗
C.使用广谱抗生素
D.麻醉下直肠检查和局部清创
E.剖腹探查、结肠切除
C.使用广谱抗生素
6 1 2013 28. 男性患者,75岁,间歇性发作的肛门瘙痒症6年,肛周瘙痒和不适感加重2个月,无便血和肛门疼痛,查体发现左侧肛缘皮炎样改变以及小溃疡形成。下列哪项是最佳的下一步治疗措施:(2.5分) A.活检排除恶变
B.局部广泛切除
C.激素软膏
D.硝酸甘油软膏
E.内括约切开术
A.活检排除恶变
6 1 2013 29. 42岁男性,在混合痔切除术后第2天出现的休克症状,伴下腹部胀痛,无便血表现;生命体征:血压75/50mmHg,心率130bpm,体温37.3℃,关于下一步处理方式,一下哪个检查最需要:(2.5分) A.腹部CT
B.麻醉下局部探查
C.ECT出血扫描
D.结肠镜
E.血管造影
A.腹部CT
6 1 2013 30. 以下对肛管部位关于齿状线的描述中,哪一个是错误的?(2.5分) A.齿状线以上的上皮是复层立方上皮(粘膜,属内胚层)
B.齿状线以上的淋巴回流方向为髂内淋巴结、肠系膜下淋巴结
C.齿状线以下的神经为躯体神经(痛觉敏锐)
D.齿状线以上静脉通过阴部内静脉(属下腔静脉系)回流
E.齿状线以下的动脉血液供应为肛门动脉
D.齿状线以上静脉通过阴部内静脉(属下腔静脉系)回流
6 1 2013 31. 反复肛周肿痛伴流脓1年余,体检:右后距离肛门4cm见肉芽样组织外口,直肠指检,扪到条索状,伴有轻压痛,挤压时外口有脓性分泌物流出者,临床应考虑:(2.5分) A.内痔
B.外痔
C.肛瘘
D.直肠息肉
E.肛管肿瘤
C.肛瘘
6 1 2013 32. 粪便隐血试验阳性者提示出血量多少: (2.5分) A.每日出血量10ml以上
B.每日出血量60ml以上
C.每日出血量30ml以上
D.每日出血量5ml以上
E.每日出血量20ml以上
D.每日出血量5ml以上
6 1 2013 33. 下列哪项不是上消化道出血手术治疗的指征:(2.5分) A.过去有多次大出血,近期反复大出血
B.年龄超过45岁
C.大出血时伴有幽门梗阻
D.大出血同时伴有急性穿孔
E.大出血同时伴有急性弥漫性腹膜炎
B.年龄超过45岁
6 1 2013 34. 引起上消化道出血的最常见的疾病是:(2.5分) A.慢性胃炎
B.消化性溃疡
C.肝硬化食管、胃底静脉曲张破裂
D.胃癌
E.出血性胃炎
B.消化性溃疡
6 1 2013 35. 关于上消化道出血,下列哪种说法不对: (2.5分) A.呕血多见于幽门以上部位出血
B.呕血者有黑便而黑便者不一定有呕血
C.应包括胆道出血
D.胃大部切除后的空肠上段出血也属于上消化道出血
E.上消化道出血均是消化系统疾病所致
E.上消化道出血均是消化系统疾病所致
6 1 2013 36. 处理上消化道大出血休克患者,首先采取哪项措施: (2.5分) A.冰盐水洗胃
B.口服止血药
C.补充血容量
D.电凝或激光治疗
E.手术治疗
C.补充血容量
6 1 2013 37. 除哪项外有助于肝硬化食管胃底静脉曲张破裂出血的诊断: (2.5分) A.门静脉高压
B.肝炎病史
C.节律性疼痛
D.低白蛋白血症
E.脾大
C.节律性疼痛
6 1 2013 38. Which of the following is true regarding Mallory-Weiss syndrome? (2.5分) A.Operative repair usually is necessary
B.Is the most common cause of gastrointestinal bleeding during pregnancy
C.Endoscopy is contraindicated
D.Bleeding stops spontaneously in the majority of cases
E.Accounts for 1/3 of deaths from upper gastrointestinal bleeding
D.Bleeding stops spontaneously in the majority of cases
6 1 2013 39. 肠易激综合征腹痛的规律是:(2.5分) A.无明显规律性;
B.疼痛-排便加重;
C.进食-疼痛-缓解;
D.疼痛进食-缓解;
E.疼痛-排便缓解
D.疼痛进食-缓解;
6 1 2013 40. 简述肠易激综合征的治疗原则: (2.5分) A.建立良好的医患关系
B.一般治疗
C.根据主要症状对症治疗及根据严重程度分级治疗
D.重视心理治疗
E.以上均是
E.以上均是
6 1 2014 1. 以下属于促进胃液分泌的内源性物质的是:(2.5分) A.生长抑素
B.前列腺素
C.上皮生长因子,
D.内因子,
E.乙酰胆碱
E.乙酰胆碱
6 1 2014 2. Hp感染最容易并发的皮肤疾病是:(2.5分) A.区域性脱斑
B.酒糟鼻
C.荨麻疹
D.剥脱性皮炎
E.白癜风
C.荨麻疹
6 1 2014 3. 胃肠道微生态失调相关性疾病不包括:(2.5分) A.抗生素相关性腹泻
B.大肠癌
C.炎症性肠病(IBD)
D.肝病
E.肠易激综合征
E.肠易激综合征
6 1 2014 4. 炎症性肠病的特征性皮肤表现有:(2.5分) A.结节性红斑
B.红斑狼疮
C.天胞疮
D.剥脱性皮炎
E.银屑病
A.结节性红斑
6 1 2014 5. 结核性腹膜炎的主要感染途径是:(2.5分) A.经口
B.经血液循环
C.经淋巴管道
D.腹腔病变直接蔓延
E.腰椎病变直接蔓延
D.腹腔病变直接蔓延
6 1 2014 6. 最支持结核性腹膜炎诊断的是:(2.5分) A.血性腹水
B.腹水白细胞数 >500 x 106/L,以多核细胞为主
C.乳糜性腹水
D.腹水比重 <1.018,蛋白 <25g/L
E.腹水比重 >1.018,蛋白 >30g/L,腹水白细胞以单核细胞为主
E.腹水比重 >1.018,蛋白 >30g/L,腹水白细胞以单核细胞为主
6 1 2014 7. 肠结核最常见的发病部位是(2.5分) A.直肠
B.乙状结肠
C.回盲部
D.回肠末段
E.升结肠
C.回盲部
6 1 2014 8. 有关结核性腹膜炎临床表现的描述,正确的是(2.5分) A.主要表现为腹痛、腹胀,结核中毒症状少见
B.为局限的慢性化脓性腹膜炎
C.青壮年少见,男女比例约1:2
D.多数可触及腹部包块
E.多数可伴有肝脾结核
C.青壮年少见,男女比例约1:2
6 1 2014 9. 关于溃疡型肠结核的病理改变,下列哪项是错误的(2.5分) A.病变侵犯肠壁的集合淋巴管和鼓励淋巴滤泡
B.病变发展过程是:渗出病变-干酪病变-溃疡
C.有时病变可深达肌层或浆膜层
D.可累及周围腹膜或邻近肠系膜淋巴结
E.溃疡边缘与基地闭塞性动脉内膜炎少见,故易引起出血。
E.溃疡边缘与基地闭塞性动脉内膜炎少见,故易引起出血。
6 1 2014 10. 克罗恩病是(2.5分) A.病因未明的胃肠道慢性炎性肉芽肿性疾病
B.病因未明的胃肠慢性炎症性疾病
C.病因未明的末段回肠和邻近结肠慢性炎症疾病
D.病因未明的末段回肠慢性炎症性疾病
E.末段回肠和结肠慢性炎症性疾病
A.病因未明的胃肠道慢性炎性肉芽肿性疾病
6 1 2014 11. 哪项可以作为克罗恩病与溃疡性结肠炎的鉴别点:(2.5分) A.便血的程度
B.发热
C.腹痛部位和程度
D.瘘管形成
E.血沉和C反应蛋白升高
D.瘘管形成
6 1 2014 12. 克罗恩病的最常见并发症是(2.5分) A.中毒性休克
B.结肠大出血
C.肠梗阻
D.急性肠穿孔
E.癌变
C.肠梗阻
6 1 2014 13. 关于溃疡性结肠炎的病理改变错误的是:(2.5分) A.病变呈广泛浅小溃疡
B.病变常侵入肌层易出现穿孔及巨结肠
C.炎症反复发作可形成炎性息肉
D.病变反复发作可致结肠变形缩短
E.溃疡性结肠炎可出现癌变
B.病变常侵入肌层易出现穿孔及巨结肠
6 1 2014 14. 下列哪项不是手术治疗溃疡性结肠炎的指征:(2.5分) A.反复发作的轻度溃疡性结肠炎
B.并发大出血
C.并发结肠癌变
D.肠穿孔
E.合并中毒性结肠扩张
A.反复发作的轻度溃疡性结肠炎
6 1 2014 15. 高位小肠梗阻的临床表现特点应为:(2.5分) A.以腹痛和呕吐为著,而腹胀,排便,排气停止不明显
B.以腹痛、呕吐和排便、排气停止为著,但腹胀不明显
C.以呕吐和排便、排气停止为著,腹痛轻微无腹胀
D.存在腹痛、呕吐、腹胀及排气排便停止
E.以腹痛和腹胀为著,无呕吐,无排气排便停止
A.以腹痛和呕吐为著,而腹胀,排便,排气停止不明显
6 1 2014 16. 急性肠梗阻不会出现那个变化:(2.5分) A.肠腔因气体和液体的积聚而膨胀
B.肠壁变薄
C.肠腔压力升高
D.肠壁血运障碍
E.肠壁代偿性肥厚
E.肠壁代偿性肥厚
6 1 2014 17. 我国成人肠梗阻最常见的原因是(2.5分) A.肠扭转
B.肠粘连
C.腹内疝
D.肠套叠
E.肠肿瘤
B.肠粘连
6 1 2014 18. A 54-year-old man has had colicky abdominal pain and protracted vomiting for several days. He has developed progressive moderate abdominal distention, and has not had a bowel movement or pass gas for five days. He has high-pitched, loud bowel sounds that coincide with the colicky pain, and X-ray show distended loops of small bowel and air-fluid levels. Five years ago he had an exploratory laparotomy for a blunt trauma of abdomen. Six hours after being hospitalized and placed on NG suction and IV fluids. He develops fever, leukocytosis, tenderness, and rebound tenderness. What is the most likely diagnosis? (2.5分) A.acute appendicitis
B.Acute pancreatitis
C.Strangulated obstruction of bowel
D.Carcinoma of sigmoid colon
E.Simple obstruction of small intestine
C.Strangulated obstruction of bowel
6 1 2014 19. 粘连性肠梗阻最常见于:(2.5分) A.先天性发育异常
B.胎粪性腹膜炎
C.化脓性腹膜炎
D.腹腔手术后
E.腹腔创伤后
D.腹腔手术后
6 1 2014 20. 患者主诉大便干结伴排便时肛门疼痛伴鲜红血便4周。检查发现典型的肛管后正中肛裂,裂口无潜行、无分泌物;临床诊断肛裂,对于该患者最恰当的处理是:(2.5分) A.温水坐浴、增加膳食纤维,通便药物
B.肛门内括约肌侧方切开术
C.麻醉下行肛门检查和活检
D.Y-V肛门整形
E.结肠镜检查后口服止痛剂和氢化可的松
A.温水坐浴、增加膳食纤维,通便药物
6 1 2014 21. 42岁女性患者,10、12、14年前分别顺产3个小孩,现患有后正中肛裂,无皮赘及肥大肛乳头,经过6周的增加膳食纤维,坐浴以及局部使用0.2%硝酸甘油后症状无显著好转,以下最恰当的治疗方式是:(2.5分) A.口服地尔硫卓
B.局部创面使用肉毒杆菌毒素
C.肛门整形
D.侧方内括约肌切开术
E.侧方外括约肌切开术
D.侧方内括约肌切开术
6 1 2014 22. 38岁男性直肠肿瘤患者在低位前切除术后出现射精功能障碍,最可能是手术时损伤了以下哪类神经:(2.5分) A.盆丛
B.盆内脏神经
C.腹下神经
D.阴部神经
E.直肠下神经
C.腹下神经
6 1 2014 23. 根据肛瘘瘘管的行走部位,可分为4种类型,以下那一个不正确?(2.5分) A.括约肌内型
B.括约肌外型
C.经括约肌
D.括约肌上
E.括约肌间
A.括约肌内型
6 1 2014 24. 以下对肛管部位关于齿状线的描述中,哪一个是错误的?(2.5分) A.齿状线以上的上皮是复层立方上皮(粘膜,属内胚层)
B.齿状线以上的淋巴回流方向为髂内淋巴结、肠系膜下淋巴结
C.齿状线以下的神经为躯体神经(痛觉敏锐)
D.齿状线以上静脉通过阴部内静脉(属下腔静脉系)回流
E.齿状线以下的动脉血液供应为肛门动脉
D.齿状线以上静脉通过阴部内静脉(属下腔静脉系)回流
6 1 2014 25. 反复肛周肿痛伴流脓1年余,体检:右后距离肛门4cm见肉芽样组织外口,直肠指检,扪到条索状,伴有轻压痛,挤压时外口有脓性分泌物流出者,临床应考虑(2.5分) A.内痔
B.外痔
C.肛瘘
D.直肠息肉
E.肛管肿瘤
C.肛瘘
6 1 2014 26. 以下哪项检查优先考虑用于评估直肠癌局部侵犯深度?(2.5分) A.结肠镜
B.气钡双重造影
C.腔内超声检查
D.直肠MRI增强
E.盆腔CT增强
D.直肠MRI增强
6 1 2014 27. 直肠癌最常见的远处血运转移部位是(2.5分) A.锁骨上淋巴结
B.腹膜
C.肝
D.肺
E.脑
C.肝
6 1 2014 28. 直肠癌的主要临床症状和体检不包括(2.5分) A.便血
B.排便习惯改变
C.腹胀、腹痛等不全性肠梗阻
D.直肠指检可发现病变
E.腹部可触及包块
D.直肠指检可发现病变
6 1 2014 29. 下列关于结直肠癌术后随访的叙述,哪一项是正确的:(2.5分) A.术前接受过全结肠镜检查的患者,术后3年进行肠镜复查
B.术后3年内,每3月进行胸腹部CT检查
C.术后5年内,每3月进行胸片和肝脏B超检查
D.手术前未能够进行全结肠镜检查的患者,术后1年进行肠镜复查
E.以上都不对
E.以上都不对
6 1 2014 30. 对于40女性右半结肠癌患者,以下哪项最支持Lynch综合征的诊断:(2.5分) A.侄儿因家族性腺瘤性息肉病行结肠切除术
B.父亲有肢体黑色素瘤病史
C.因子宫内膜癌行子宫切除术的病史
D.祖父在64岁时被诊断为结肠癌
E.有一因子宫颈癌行子宫切除的妹妹
C.因子宫内膜癌行子宫切除术的病史
6 1 2014 31. 请问IBS可能的病因包括: (2.5分) A.精神因素
B.肠道菌群失调
C.感染因素
D.遗传和环境因素
E.以上均是
E.以上均是
6 1 2014 32. 粪便隐血试验阳性者提示出血量多少: (2.5分) A.每日出血量10ml以上
B.每日出血量60ml以上
C.每日出血量30ml以上
D.每日出血量5ml以上
E.每日出血量20ml以上
D.每日出血量5ml以上
6 1 2014 33. 下列哪项不是上消化道出血手术治疗的指征:(2.5分) A.过去有多次大出血,近期反复大出血
B.年龄超过45岁
C.大出血时伴有幽门梗阻
D.大出血同时伴有急性穿孔
E.大出血同时伴有急性弥漫性腹膜炎
B.年龄超过45岁
6 1 2014 34. 引起上消化道出血的最常见的疾病是:(2.5分) A.慢性胃炎
B.消化性溃疡
C.肝硬化食管、胃底静脉曲张破裂
D.胃癌
E.出血性胃炎
B.消化性溃疡
6 1 2014 35. 关于上消化道出血,下列哪种说法不对: (2.5分) A.呕血多见于幽门以上部位出血
B.呕血者有黑便而黑便者不一定有呕血
C.应包括胆道出血
D.胃大部切除后的空肠上段出血也属于上消化道出血
E.上消化道出血均是消化系统疾病所致
E.上消化道出血均是消化系统疾病所致
6 1 2014 36. 处理上消化道大出血休克患者,首先采取哪项措施: (2.5分) A.冰盐水洗胃
B.口服止血药
C.补充血容量
D.电凝或激光治疗
E.手术治疗
C.补充血容量
6 1 2014 37. 除哪项外有助于肝硬化食管胃底静脉曲张破裂出血的诊断: (2.5分) A.门静脉高压
B.肝炎病史
C.节律性疼痛
D.低白蛋白血症
E.脾大
C.节律性疼痛
6 1 2014 38. Which of the following is true regarding Mallory-Weiss syndrome? (2.5分) A.Operative repair usually is necessary
B.Is the most common cause of gastrointestinal bleeding during pregnancy
C.Endoscopy is contraindicated
D.Bleeding stops spontaneously in the majority of cases
E.Accounts for 1/3 of deaths from upper gastrointestinal bleeding
D.Bleeding stops spontaneously in the majority of cases
6 1 2014 39. 肠易激综合征腹痛的规律是:(2.5分) A.无明显规律性;
B.疼痛-排便加重;
C.进食-疼痛-缓解;
D.疼痛进食-缓解;
E.疼痛-排便缓解
D.疼痛进食-缓解;
6 1 2014 40. 以下哪项不是肠易激综合征的症状? (2.5分) A.腹痛
B.排便频率的改变
C.腹胀
D.脂肪泻
E.排便性状的改变
D.脂肪泻
6 1 2015 1. 起生理性调节和激素样作用的胃肠肽不包括 (0.0分) A.促胃液素
B.缩胆囊素
C.促胰液素
D.抑胃肽
E.生长抑素
E.生长抑素
6 1 2015 2. 多种胃肠肽也分布于中枢神经系统中,这种双重分布的肽,被称为 (0.0分) A.脑肠肽
B.糖皮质激素
C.内因子
D.胃泌素
E.胆囊收缩素
A.脑肠肽
6 1 2015 3. 下面不属于代谢综合征的成分是 (0.0分) A.超重/肥胖
B.高血压
C.血脂异常
D.高血糖
E.冠心病
E.冠心病
6 1 2015 4. 炎症性肠病的特征性皮肤表现有(0.0分) A.结节性红斑
B.红斑狼疮
C.天胞疮
D.剥脱性皮炎
E.银屑病
A.结节性红斑
6 1 2015 5. PCOS和脂肪肝的最重要共同发病机制(0.0分) A.脂质代谢紊乱
B.糖代谢紊乱
C.胰岛素抵抗
D.基因多态性
E.蛋白质代谢紊乱
C.胰岛素抵抗
6 1 2015 6. 男性,34岁,低热.右下腹痛3个月,大便不成形,伴乏力,体重下降约7斤。血WBC 7.8×109/L。便常规偶见WBC。ESR 36mm/hr。X线钡餐示钡剂通过回盲部迅速。乙状结肠镜下肠黏膜大致正常。最可能的疾病是(0.0分) A.结肠癌
B.溃疡性肠结核
C.阿米巴痢疾
D.溃疡性结肠炎
E.肠易激综合征
B.溃疡性肠结核
6 1 2015 7. 肠结核患者常在腹部什么部位触到部包块(0.0分) A.脐周
B.右上腹
C.左上腹
D.右下腹
E.左下腹
D.右下腹
6 1 2015 8. 有关结核性腹膜炎临床表现的描述,正确的是(0.0分) A.主要表现为腹痛、腹胀,结核中毒症状少见
B.为局限的慢性化脓性腹膜炎
C.青壮年少见,男女比例约1:2
D.多数可触及腹部包块
E.多数可伴有肝脾结核
C.青壮年少见,男女比例约1:2
6 1 2015 9. 对肠结核最有诊断价值的检查是(0.0分) A.X线钡餐检查发现肠腔狭窄
B.结肠镜检查示回盲部炎症
C.结肠镜下活检找到干酪性上皮样肉芽肿
D.结核菌素试验强阳性
E.粪便中查到结核杆菌
C.结肠镜下活检找到干酪性上皮样肉芽肿
6 1 2015 10. 关于溃疡型肠结核的病理改变,下列哪项是错误的(0.0分) A.病变侵犯肠壁的集合淋巴管和鼓励淋巴滤泡
B.病变发展过程是:渗出病变-干酪病变-溃疡
C.有时病变可深达肌层或浆膜层
D.可累及周围腹膜或邻近肠系膜淋巴结
E.溃疡边缘与基地闭塞性动脉内膜炎少见,故易引起出血。
E.溃疡边缘与基地闭塞性动脉内膜炎少见,故易引起出血。
6 1 2015 11. 下列哪项不是Crohn病的病理改变(0.0分) A.肠粘膜面有多数匐行沟槽样或裂隙状纵行溃疡,可深达基层
B.粘膜隆起呈铺路卵石状
C.干酪性肉芽肿
D.非干酪性肉芽肿
E.可有窦道形成
C.干酪性肉芽肿
6 1 2015 12. 溃疡性结肠炎并发中毒性巨结肠的常见诱因是(0.0分) A.低血镁
B.低血钠
C.低血钾
D.低蛋白血症
E.低血钙
C.低血钾
6 1 2015 13. 溃疡性结肠炎病理改变不包括(0.0分) A.肉芽组织增生
B.炎性息肉
C.非干酪样肉芽肿
D.隐窝脓肿
E.黏膜溃疡形成
C.非干酪样肉芽肿
6 1 2015 14. 下列哪项不是手术治疗溃疡性结肠炎的指征(0.0分) A.反复发作的轻度溃疡性结肠炎
B.并发大出血
C.并发结肠癌变
D.肠穿孔
E.合并中毒性结肠扩张
A.反复发作的轻度溃疡性结肠炎
6 1 2015 15. 哪项可以作为克罗恩病与溃疡性结肠炎的鉴别点(0.0分) A.便血的程度
B.发热
C.腹痛部位和程度
D.瘘管形成
E.血沉和C反应蛋白升高
D.瘘管形成
6 1 2015 16. 高位小肠梗阻的临床表现特点应为(0.0分) A.以腹痛和呕吐为著,而腹胀,排便,排气停止不明显
B.以腹痛、呕吐和排便、排气停止为著,但腹胀不明显
C.以呕吐和排便、排气停止为著,腹痛轻微无腹胀
D.存在腹痛、呕吐、腹胀及排气排便停止
E.以腹痛和腹胀为著,无呕吐,无排气排便停止
A.以腹痛和呕吐为著,而腹胀,排便,排气停止不明显
6 1 2015 17. 绞窄性肠梗阻最容易发生的酸碱失衡类型是(0.0分) A.呼吸性碱中毒
B.代谢性酸中毒
C.代谢性碱中毒
D.呼吸性酸中毒
E.呼吸性酸中毒和代谢性碱中毒
B.代谢性酸中毒
6 1 2015 18. 粘连性肠梗阻最常见于(0.0分) A.先天性发育异常
B.胎粪性腹膜炎
C.化脓性腹膜炎
D.腹腔手术后
E.腹腔创伤后
D.腹腔手术后
6 1 2015 19. A 55-year-old woman goes to the emergency department because of vomiting and severe abdominal cramping of 3 day's duration. Her pain is centered on the umbilicus. She denies being exposed to a viral or bacterial illness. Her medical history includes a previous cholecystectomy and an appendectomy after which she developed an infection. Her abdomen is not tender, but hyperactive, high-pitched peristalsis with rushes coincides with palpable bowel cramping. Abdominal X-ray films taken in the upright positions demonstrate a ladder-like series of distended small bowel loops. Which of the following is the most likely explanation for these findings?(0.0分) A.Adhesions
B.Ascaris infection
C.Cancer
D.Intussusception
E.Volvulus
A.Adhesions
6 1 2015 20. 男性,87岁,粘液稀便2个月;COPD病史30年;体检:腹部平软,无压痛及肿块,大便检查潜血(+);肠镜检查距肛门9cm处的直肠肿块,病理:腺癌;手术前直肠MRI评估无MRF累及,CT检查未见远处转移;应选择哪种手术为佳?(0.0分) A.经腹直肠癌切除,人工肛门,远端封闭
B.拉下式直肠癌切除术
C.经腹直肠癌切除术
D.腹会阴联合直肠癌根治术
E.乙状结肠造瘘术
A.经腹直肠癌切除,人工肛门,远端封闭
6 1 2015 21. 直肠癌最常见的远处血运转移部位是(0.0分) A.锁骨上淋巴结
B.腹膜
C.肝
D.肺
E.脑
C.肝
6 1 2015 22. 直肠癌患者便血的常常表现为(0.0分) A.便血量多而鲜红,便次增多;
B.便血少伴肛门疼痛,大便干结;
C.便污血而疼痛;
D.便血量多而色黑;
E.便血污秽而腥臭,便次增多;
E.便血污秽而腥臭,便次增多;
6 1 2015 23. 直肠癌最基本的诊断方法是(0.0分) A.直肠指检
B.直肠镜
C.乙状结肠镜
D.钡剂灌肠
E.B型超声检查
A.直肠指检
6 1 2015 24. 以下哪个不是肠镜下息肉切除的方法(0.0分) A.Colonoscopy
B.Cold cut
C.Hot cut
D.EMR
E.ESD
A.Colonoscopy
6 1 2015 25. 患者主诉大便干结伴排便时肛门疼痛伴鲜红血便4周。检查发现典型的肛管后正中肛裂,裂口无潜行、无分泌物;临床诊断肛裂,对于该患者最恰当的处理是:(0.0分) A.温水坐浴、增加膳食纤维,通便药物
B.肛门内括约肌侧方切开术
C.麻醉下行肛门检查和活检
D.Y-V肛门整形
E.结肠镜检查后口服止痛剂和氢化可的松
A.温水坐浴、增加膳食纤维,通便药物
6 1 2015 26. 42岁女性患者,10、12、14年前分别顺产3个小孩,现患有后正中肛裂,无皮赘及肥大肛乳头,经过6周的增加膳食纤维,坐浴以及局部使用0.2%硝酸甘油后症状无显著好转,以下最恰当的治疗方式是:(0.0分) A.口服地尔硫卓
B.局部创面使用肉毒杆菌毒素
C.肛门整形
D.侧方内括约肌切开术
E.侧方外括约肌切开术
D.侧方内括约肌切开术
6 1 2015 27. 38岁男性直肠肿瘤患者在低位前切除术后出现射精功能障碍,最可能是手术时损伤了以下哪类神经:(0.0分) A.盆丛
B.盆内脏神经
C.腹下神经
D.阴部神经
E.直肠下神经
C.腹下神经
6 1 2015 28. 根据肛瘘瘘管的行走部位,可分为4种类型,以下那一个不正确?(0.0分) A.括约肌内型
B.括约肌外型
C.经括约肌
D.括约肌上
E.括约肌间
A.括约肌内型
6 1 2015 29. 内痔注射术后3天的患者,突然出现发热,体温38.5度,感觉直肠和下腹部疼痛,实验室检查发现WBC 14000/ul,腹部平片发现腹膜后积气。下一步最恰当的治疗措施是:(0.0分) A.禁食,CT检查
B.直肠镜检查以及直肠灌洗
C.麻醉下直肠检查和局部清创
D.使用广谱抗生素
E.剖腹探查、结肠切除
D.使用广谱抗生素
6 1 2015 30. 反复肛周肿痛伴流脓1年余,体检:右后距离肛门4cm见肉芽样组织外口,直肠指检,扪到条索状,伴有轻压痛,挤压时外口有脓性分泌物流出者,临床应考虑(0.0分) A.内痔
B.外痔
C.肛瘘
D.直肠息肉
E.肛管肿瘤
C.肛瘘
6 1 2015 31. 消化性溃疡并发呕血时,首选下列哪种药物止血:(0.0分) A.甲氰咪胍
B.质子泵抑制剂
C.心得安
D.安络血
E.垂体后叶素
B.质子泵抑制剂
6 1 2015 32. 关于上消化道出血的处理哪项有误(0.0分) A.嘱患者保持安静
B.注意观察尿量变化
C.一律禁食
D.定期复查血象
E.防治失血性休克
C.一律禁食
6 1 2015 33. 引起上消化道出血的最常见的疾病是(0.0分) A.慢性胃炎
B.消化性溃疡
C.肝硬化食管、胃底静脉曲张破裂
D.胃癌
E.出血性胃炎
B.消化性溃疡
6 1 2015 34. 判断上消化道出血部位最好的检查方法是(0.0分) A.内窥镜检查
B.钡餐检查
C.B超检查
D.选择性动脉造影
E.胃液分析
A.内窥镜检查
6 1 2015 35. 除哪项外有助于肝硬化食管胃底静脉曲张破裂出血的诊断: (0.0分) A.门静脉高压
B.肝炎病史
C.节律性疼痛
D.低白蛋白血症
E.脾大
C.节律性疼痛
6 1 2015 36. Which of the following is true regarding Mallory-Weiss syndrome? (0.0分) A.Operative repair usually is necessary
B.Is the most common cause of gastrointestinal bleeding during pregnancy
C.Endoscopy is contraindicated
D.Bleeding stops spontaneously in the majority of cases
E.Accounts for 1/3 of deaths from upper gastrointestinal bleeding
D.Bleeding stops spontaneously in the majority of cases
6 1 2015 37. 肠易激综合征腹痛的规律是: (0.0分) A.无明显规律性;
B.疼痛-排便加重;
C.进食-疼痛-缓解;
D.疼痛进食-缓解;
E.疼痛-排便缓解
D.疼痛进食-缓解;
6 1 2015 38. 以下哪项不是肠易激综合征的症状? (0.0分) A.腹痛
B.排便频率的改变
C.腹胀
D.脂肪泻
E.排便性状的改变
D.脂肪泻
6 1 2015 39. 简述肠易激综合征的治疗原则 : (0.0分) A.建立良好的医患关系
B.一般治疗
C.根据主要症状对症治疗及根据严重程度分级治疗
D.重视心理治疗
E.以上均是
E.以上均是
6 1 2015 40. 请问IBS可能的病因包括:(0.0分) A.精神因素
B.肠道菌群失调
C.感染因素
D.遗传和环境因素
E.以上均是
E.以上均是
6 2 2006 1. The Calot's Triangle is bounded by except: (3.0分) A.Cystic duct.
B.The inferior border of the liver.
C.The common hepatic duct.
D.The left hepatic duct.
E.None of above.
D.The left hepatic duct.
6 2 2006 2. 目前对小肝癌定位诊断的各种方法中最优者是: (3.0分) A.B型超声显象仪检查
B.CT检查
C.放射性核素肝扫描
D.选择性腹腔动脉或肝动脉造影
E.磁共振成象
D.选择性腹腔动脉或肝动脉造影
6 2 2006 3. 女性,57岁,CT示肝左外叶8cm占位,伴门静脉左支栓塞,肝肾功能正常,首选治疗方案:(3.0分) A.经肝动脉化学治疗和栓塞
B.手术切除
C.全身化学治疗和中医治疗
D.肝肿瘤局部无水酒精注射
E.生物治疗
B.手术切除
6 2 2006 4. 肝内胆管出血部位不明可采用: (3.0分) A.肝固有动脉结扎及胆总管引流
B.三腔管压迫
C.胃大部切除
D.肝叶切除
E.肝固有动脉结扎
A.肝固有动脉结扎及胆总管引流
6 2 2006 5. 一名49岁男性,因右季肋部疼痛半月余,加重一天入院,以往健康,检查时发现血HBsAg阳性,AFP500ug/L。支持患原发性肝癌最有意义的体征是:(3.0分) A.肝脏质地硬,有压痛
B.肝右肋下约4.0cm
C.肝区可闻及血管杂音
D.肝区可闻及摩擦音
E.肝区有明显叩击痛
C.肝区可闻及血管杂音
6 2 2006 6. 中晚期肝癌最常见的体征是:(3.0分) A.下肢浮肿
B.腹胀
C.肝肿大
D.肝区痛
E.皮下出血
C.肝肿大
6 2 2006 7. B超显示肝内"牛眼征"常见于下列哪一种疾病(3.0分) A.肝囊肿
B.转移性肝癌
C.肝脓肿
D.原发性肝癌
E.肝包虫病
B.转移性肝癌
6 2 2006 8. Couinaud分段法将肝脏分为8段,其中尾状叶在(3.0分) A.Ⅱ段
B.Ⅰ段
C.Ⅲ段
D.Ⅴ段
E.Ⅷ段
B.Ⅰ段
6 2 2006 9. 早期肝癌是指瘤体:(3.0分) A.单个有包膜
B.3cm以下,不超过2个
C.界限清楚无出血
D.微小而多个
E.以上都是
B.3cm以下,不超过2个
6 2 2006 10. 可造成感染慢性化的肝炎病毒是:(3.0分) A.肝炎病毒A、C、D
B.肝炎病毒A、B、E
C.肝炎病毒A、B、C
D.肝炎病毒B、C、D
E.肝炎病毒C、D、E
D.肝炎病毒B、C、D
6 2 2006 11. 对疑有早期原发性肝癌的患者,应首先用哪一种方法检查较好:(3.0分) A.血清甲胎蛋白动态观察检查
B.同位素肝扫描
C.肝区超声波检查
D.血清γˉ谷氨酰转肽酶
E.选择性肝动脉造影
A.血清甲胎蛋白动态观察检查
6 2 2006 12. 肝硬化引起门脉高压的临床表现是:(3.0分) A.黄疸、腹水、脾肿大
B.腹水、脾肿大、肾功能衰竭
C.腹水、脾肿大、侧枝循环的建立与开放
D.黄疸、腹水、侧枝循环的建立与开放
E.腹水、上消化道出血、侧枝循环的建立与开放
C.腹水、脾肿大、侧枝循环的建立与开放
6 2 2006 13. 下列哪种机制与肝性脑病发生无关? (3.0分) A.氨基酸失衡学说
B.氨中毒学说
C.假神经递质学说
D.GABA/BZ复合体学说
E.水钠潴留
E.水钠潴留
6 2 2006 14. 下列不属于兴奋性神经递质的是(3.0分) A.γ-氨基丁酸
B.去甲肾上腺素
C.门冬氨酸
D.乙酰胆碱
E.多巴胺
A.γ-氨基丁酸
6 2 2006 15. 门脉高压是指门脉压力超过 (3.0分) A.20cm H2O
B.10cm H2O
C.30cm H2O
D.40cm H2O
E.50cm H2O
A.20cm H2O
6 2 2006 16. 在我国和西方国家,造成肝硬化的最主要原因是分别是 (3.0分) A.丙型肝炎和乙型肝炎
B.乙型肝炎和丙型肝炎
C.丙型肝炎和非酒精性脂肪性肝病
D.乙型肝炎和非酒精性脂肪性肝病
E.非酒精性脂肪性肝病和丙型肝炎
D.乙型肝炎和非酒精性脂肪性肝病
6 2 2006 17. 肝硬化的并发症不包括 (4.0分) A.上消化道出血
B.原发性肝癌
C.肝性脑病
D.肝肺综合征
E.尿崩症
E.尿崩症
6 2 2006 18. 男性,45岁,因患肝硬化腹水用速尿后尿量每日超过2000ml,近日出现四肢肌肉软弱无力,伴恶心呕吐,心电图出现传导和节律异常,其原因最可能是 (3.0分) A.高钾血症
B.低钾血症
C.低钠血症
D.高钠血症
E.低镁血症
B.低钾血症
6 2 2006 19. 关于肝硬化的叙述中正确的是(3.0分) A.肝功能不全可表现为脾脏肿大
B.亚急性重症病毒性肝炎多发展为门脉性肝硬化
C.病变特点是肝细胞坏死、纤维组织增生和假小叶形成
D.门脉高压可表现为出血倾向
E.肝硬化不发生癌变
C.病变特点是肝细胞坏死、纤维组织增生和假小叶形成
6 2 2006 20. 下列属于肝硬化门脉高压表现的是(3.0分) A.肝肿大
B.脾肿大
C.肝掌
D.蜘蛛痣
E.男性乳房发育
B.脾肿大
6 2 2006 21. 肝硬化患者出现消化道出血时,下列可诱发肝性脑病的是(3.0分) A.输生理盐水
B.水电解质紊乱致低钾血症
C.内镜下止血
D.静脉注射止血药
E.口服去甲肾上腺素
B.水电解质紊乱致低钾血症
6 2 2006 22. 查体于肿大的肝脏表面听到血管杂音时,应考虑 (3.0分) A.门脉血栓形成
B.布加综合征
C.肝硬化失代偿期
D.原发性肝癌
E.慢性肝炎
D.原发性肝癌
6 2 2006 23. 男性,38岁,患肝硬化3年,一周来畏寒发热,体温38℃左右,全腹痛,腹部明显膨胀,尿量500ml/d。住院后经检查有以下体征,对目前病情判断最有意义的是 (3.0分) A.腹壁静脉曲张呈海蛇头样
B.蜘蛛痣及肝掌
C.脾肿大
D.全腹压痛及反跳痛
E.腹部移动性浊音阳性
D.全腹压痛及反跳痛
6 2 2006 24. 下列关于门脉性肝硬化特点的叙述,不正确的是(3.0分) A.可由酒精性肝病发展而来
B.病变早期结节间的纤维间隔较细
C.病变可继续发展使结节形态不规则
D.结节大小较均匀
E.结节直径多大于3mm
E.结节直径多大于3mm
6 2 2006 25. 与肝细胞性肝癌相比,下列哪项不是胆管细胞性肝癌的特点?(3.0分) A.大体观呈灰白色
B.产生胆汁
C.老年多见
D.AFP呈阴性表达
E.多不伴有肝硬化
B.产生胆汁
6 2 2006 26. Portal hypertension in cirrhosis is caused by the following, except for(3.0分) A.elevated level of blood ammonia
B.renal retension of sodium and water due to secondary hyperaldostenosis
C.increased resistance to portal flow at the level of the sinsoids
D.compression of central vein and branches of hepatic vein by expansile parenchyma nodules
E.anastomoses between the arterial and portal veins
A.elevated level of blood ammonia
6 2 2006 27. Fatty change is most likely appeared in hepatitis of:(3.0分) A.HBV
B.HAV
C.HCV
D.HDV
E.HEV
C.HCV
6 2 2006 28. Which of the following statement about hepatitis A virus is incorrect ?(3.0分) A.dose not cause cirrhosis
B.no carrier states
C.cause chronic hepatitis
D.fecal-oral transmission
E.no connection with hepatocellular carcinoma
C.cause chronic hepatitis
6 2 2006 29. 目前认为乙型肝炎病毒引起肝细胞损害的主要机制是:(3.0分) A.病毒损坏肝细胞的线粒体
B.病毒直接破坏肝细胞膜
C.病毒代谢产物的损害作用
D.病毒激活机体细胞免疫反应损害病毒感染的肝细胞
E.通过抗原抗体反应激活补体
D.病毒激活机体细胞免疫反应损害病毒感染的肝细胞
6 2 2006 30. 慢性肝炎或HBsAg携带者电镜下见HBsAg主要存在于:(3.0分) A.核糖体
B.线粒体
C.滑面内质网内
D.高尔基体
E.核内
C.滑面内质网内
6 2 2006 31. 急性重型肝炎(急性肝萎缩)与亚急性重型肝炎(亚急性肝萎缩)肉眼观形态主要区别是:(3.0分) A.有无再生结节形成
B.肝实质黄染的程度
C.肝包膜皱缩的程度
D.肝体积缩小的程度
E.出血程度
A.有无再生结节形成
6 2 2006 32. 不属于肝硬化门脉高压的临床表现的是:(3.0分) A.胃肠道淤血
B.淤血性脾肿大
C.蜘蛛痣
D.腹水
E.食管下段静脉曲张
C.蜘蛛痣
6 2 2006 33. 肝硬化并发肝性脑病的主要原因是:(3.0分) A.凝血机制障碍所至出血
B.体内雌激素水平升高
C.并发感染引起
D.代谢性酸中毒
E.血氨升高或中枢神经系统氨基酸代谢紊乱
E.血氨升高或中枢神经系统氨基酸代谢紊乱
6 2 2007 1. 下列属于肝硬化门脉高压表现的是(2.5分) A.肝掌
B.肝肿大
C.脾肿大
D.蜘蛛痣
E.性乳房发育
C.脾肿大
6 2 2007 2. 可造成感染慢性化的肝炎病毒是:(2.5分) A.肝炎病毒A、B、E
B.肝炎病毒A、B、C
C.肝炎病毒A、C、D
D.肝炎病毒B、C、D
E.肝炎病毒C、D、E
D.肝炎病毒B、C、D
6 2 2007 3. 早期肝癌是指瘤体 (2.5分) A.界限清楚无出血
B.3cm以下,不超过2个
C.微小而多个
D.单个有包膜
E.以上都是
B.3cm以下,不超过2个
6 2 2007 4. 不符合病毒性肝炎基本病变的是 (2.5分) A.肝细胞变性坏死
B.Kupffer细胞异型增生
C.纤维组织增生
D.慢性炎细胞浸润
E.小胆管增生
B.Kupffer细胞异型增生
6 2 2007 5. 肝细胞大片崩解坏死并有肝细胞结节状再生者见于 (2.5分) A.急性普通型肝炎
B.慢性活动性肝炎
C.亚急性重型肝炎
D.慢性持续性肝炎
E.急性重型肝炎
C.亚急性重型肝炎
6 2 2007 6. 上消化道出血的首选诊断方法是:(2.5分) A.鼻胃管或三腔管检查
B.X线钡餐检查
C.选择性腹腔动脉或肠系膜上动脉造影
D.早期内镜检查
E.核素检查
D.早期内镜检查
6 2 2007 7. 男性,45岁,因患肝硬化腹水用速尿后尿量每日超过2000ml,近日出现四肢肌肉软弱无力,伴恶心呕吐,心电图出现传导和节律异常,其原因最可能是(2.5分) A.高钠血症
B.高钾血症
C.低钠血症
D.低钾血症
E.低镁血症
D.低钾血症
6 2 2007 8. 肝硬化腹水治疗,一般不主张采用(2.5分) A.卧床休息
B.低盐饮食
C.高蛋白饮食
D.强烈利尿
E.腹水浓缩回输
D.强烈利尿
6 2 2007 9. 下列哪项对诊断门静脉高压最有特征性? (2.5分) A.蜘蛛痣
B.腹水
C.侧支循环的建立及开放
D.脾大
E.脾功能亢进表现
C.侧支循环的建立及开放
6 2 2007 10. 哪一种与肝癌无关 (2.5分) A.黄曲霉菌
B.乙型肝炎
C.亚硝胺
D.脂肪变性
E.肝硬化
D.脂肪变性
6 2 2007 11. 目前对小肝癌定位诊断的各种方法中最优者是: (2.5分) A.CT检查
B.B型超声显象仪检查
C.选择性腹腔动脉或肝动脉造影
D.放射性核素肝扫描
E.磁共振成象
C.选择性腹腔动脉或肝动脉造影
6 2 2007 12. 对疑有早期原发性肝癌的患者,应首先用哪一种方法检查较好:(2.5分) A.同位素肝扫描
B.血清甲胎蛋白动态观察检查
C.血清γˉ谷氨酰转肽酶
D.肝区超声波检查
E.选择性肝动脉造影
B.血清甲胎蛋白动态观察检查
6 2 2007 13. 女性,57岁,CT示肝左外叶8cm占位,伴门静脉左支栓塞,肝肾功能正常,首选治疗方案:(2.5分) A.经肝动脉化学治疗和栓塞
B.手术切除
C.全身化学治疗和中医治疗
D.肝肿瘤局部无水酒精注射
E.生物治疗
B.手术切除
6 2 2007 14. 原发性肝癌是指 (2.5分) A.肝细胞和肝外胆管上皮发生的癌
B.肝细胞发生的癌
C.胆管上皮发生的癌
D.肝细胞和肝内胆管上皮发生的癌
E.来自Kupffer细胞的恶性肿瘤
D.肝细胞和肝内胆管上皮发生的癌
6 2 2007 15. 肝硬化引起门脉高压的临床表现是:(2.5分) A.黄疸、腹水、侧枝循环的建立与开放
B.腹水、脾肿大、肾功能衰竭
C.黄疸、腹水、脾肿大
D.腹水、脾肿大、侧枝循环的建立与开放
E.腹水、上消化道出血、侧枝循环的建立与开放
D.腹水、脾肿大、侧枝循环的建立与开放
6 2 2007 16. 我国与原发性肝癌发病最密切的因素是: (2.5分) A.黄曲霉素
B.亚硝胺
C.嗜酒
D.病毒性肝炎
E.吸烟
D.病毒性肝炎
6 2 2007 17. 查体于肿大的肝脏表面听到血管杂音时,应考虑(2.5分) A.原发性肝癌
B.门脉血栓形成
C.肝硬化失代偿期
D.布加综合征
E.慢性肝炎
A.原发性肝癌
6 2 2007 18. 下列哪种机制与肝性脑病发生无关? (2.5分) A.氨基酸失衡学说
B.氨中毒学说
C.假神经递质学说
D.GABA/BZ复合体学说
E.水钠潴留
E.水钠潴留
6 2 2007 19. 碎片状坏死主要见于 (2.5分) A.急性重型肝炎
B.慢性肝炎
C.亚急性重型肝炎
D.急性肝炎
E.肝硬化
B.慢性肝炎
6 2 2007 20. 常伴皮肤瘙痒的肝硬化是(2.5分) A.小结节性肝硬化
B.淤血性肝硬化
C.大结节性肝硬化
D.原发性胆汁性肝硬化
E.日本血吸虫性肝硬化
D.原发性胆汁性肝硬化
6 2 2007 21. 门脉高压是指门脉压力超过(2.5分) A.40cm H2O
B.20cm H2O
C.30cm H2O
D.10cm H2O
E.50cm H2O
B.20cm H2O
6 2 2007 22. 下列哪个部位不存在门静脉系与腔静脉系之间的交通支:(2.5分) A.胃底、食管下端
B.食管中段、胃底
C.直肠下部,肛管
D.前腹壁
E.腹膜后
B.食管中段、胃底
6 2 2007 23. 25.B超显示肝内"牛眼征"常见于下列哪一种疾病 (2.5分) A.转移性肝癌
B.原发性肝癌
C.肝脓肿
D.肝囊肿
E.肝包虫病
A.转移性肝癌
6 2 2007 24. 门脉高压症发生消化道出血的主要原因是:(2.5分) A.腹壁静脉曲张
B.脾肿大破裂
C.痔静脉曲张
D.食管下段和胃底静脉曲张
D.食管下段和胃底静脉曲张
6 2 2007 25. 自发性细菌性腹膜炎常见致病菌应是(2.5分) A.溶血链球菌和肺炎双球菌
B.大肠杆菌和副大肠杆菌
C.厌氧菌和链球菌
D.大肠杆菌和厌氧菌
E.绿脓杆菌和葡萄球菌
A.溶血链球菌和肺炎双球菌
6 2 2007 26. 原发性肝癌早期转移途径为:(2.5分) A.肺内转移
B.肝内转移
C.直接浸润转移
D.淋巴转移
E.骨转移
B.肝内转移
6 2 2007 27. 在我国引起肝硬变最主要原因是: (2.5分) A.病毒性肝炎
B.慢性淤胆
C.慢性酒精中毒
D.寄生虫感染
E.营养不良
A.病毒性肝炎
6 2 2007 28. 下列不属于兴奋性神经递质的是 (2.5分) A.γ-氨基丁酸
B.乙酰胆碱
C.去甲肾上腺素
D.门冬氨酸
E.多巴胺
A.γ-氨基丁酸
6 2 2007 29. 下列属于芳香族氨基酸的是(2.5分) A.亮氨酸
B.异亮氨酸
C.色氨酸
D.缬氨酸
E.苏氨酸
C.色氨酸
6 2 2007 30. 肝硬化腹水生成的机制中,不正确的说法是 (2.5分) A.肝脏合成清蛋白减少
B.门静脉淤血、血管壁通透性增强
C.结缔组织增生、肝静脉受压
D.血内抗利尿激素、醛固酮增高
E.肝窦内压力升高
C.结缔组织增生、肝静脉受压
6 2 2007 31. 肝硬化的并发症不包括 (2.5分) A.肝性脑病
B.上消化道出血
C.原发性肝癌
D.肝肺综合征
E.尿崩症
E.尿崩症
6 2 2007 32. 关于病毒性肝炎,下列哪项是错误的 (2.5分) A.急性普通型肝炎以广泛的肝细胞变性为主
B.各型肝炎病变的性质完全相同
C.以肝实质细胞变性坏死为主要病变的传染病
D.桥接坏死常见于中、重度慢性肝炎
E.大片的溶解性坏死是重型肝炎的病变特征
B.各型肝炎病变的性质完全相同
6 2 2007 33. 肝内胆管出血部位不明可采用: (2.5分) A.三腔管压迫
B.肝固有动脉结扎及胆总管引流
C.肝叶切除
D.胃大部切除
E.肝固有动脉结扎
B.肝固有动脉结扎及胆总管引流
6 2 2007 34. 男性,38岁,患肝硬化3年,一周来畏寒发热,体温38℃左右,全腹痛,腹部明显膨胀,尿量500ml/d。住院后经检查有以下体征,对目前病情判断最有意义的是 (2.5分) A.腹壁静脉曲张呈海蛇头样
B.蜘蛛痣及肝掌
C.脾肿大
D.全腹压痛及反跳痛
E.腹部移动性浊音阳性
D.全腹压痛及反跳痛
6 2 2007 35. 目前肝癌早期宜采用哪一种治疗为主: (2.5分) A.手术切除
B.化学疗法
C.放射疗法
D.中医中药
E.中医中药
A.手术切除
6 2 2007 36. 早期肝癌是指瘤体:(2.5分) A.3cm以下,不超过2个
B.微小而多个
C.界限清楚无出血
D.单个有包膜
E.以上都是
A.3cm以下,不超过2个
6 2 2007 37. 男,55岁,进行性黄疸,肝功能欠佳,PT明显延长,宜选用哪种检查:(2.5分) A.碘番酸口服胆囊造影
B.内窥镜逆行胰胆管造影(ERCP)
C.经皮肝穿刺胆管造影(PTC)
D.静脉胆囊造影
E.胆囊穿刺置管造影
B.内窥镜逆行胰胆管造影(ERCP)
6 2 2007 38. 肝硬化患者出现消化道出血时,下列可诱发肝性脑病的是(2.5分) A.内镜下止血
B.输生理盐水
C.水电解质紊乱致低钾血症
D.静脉注射止血药
E.口服去甲肾上腺素
C.水电解质紊乱致低钾血症
6 2 2007 39. 关于肝细胞气球样变细胞中下列哪一项改变是错误的 (2.5分) A.线立体肿胀、嵴消失
B.细胞内糖原增多
C.细胞内水分增多
D.内质网扩张、囊泡变性
E.粗面内质网核蛋白体颗粒消失
B.细胞内糖原增多
6 2 2007 40. 有关肝性脑病错误的方法是(2.5分) A.生理盐水灌肠
B.弱酸溶液灌肠
C.乳果糖灌肠
D.肥皂水灌肠
E.硫酸镁导泻
D.肥皂水灌肠
6 2 2008 1. 下列哪种机制与肝性脑病发生无关? (2.0分) A.氨中毒学说
B.假神经递质学说
C.GABA/BZ复合体学说
D.氨基酸失衡学说
E.水钠潴留
E.水钠潴留
6 2 2008 2. 以下不是细菌性肝脓肿的常见临床表现是: (2.0分) A.寒战、高烧
B.肝区剧烈疼痛
C.肋下可及肿大肝脏
D.乏力、食欲不振
E.白细胞计数增高、明显左移
B.肝区剧烈疼痛
6 2 2008 3. 对阿米巴肝脓肿的有关论述,不正确的是: (2.0分) A.继发于阿米巴痢疾后
B.病情急剧,中毒症状严重
C.部分病人粪中查有阿米巴滋养体
D.多见棕褐色脓液,无臭味
E.白细胞计数增高不明显
B.病情急剧,中毒症状严重
6 2 2008 4. 对细菌性肝脓肿的治疗,不正确的是: (2.0分) A.应使用较大剂量抗生素
B.单个较大脓肿可经皮穿刺置管引流
C.脓肿穿破胸腔应同时引流胸腔
D.慢性局限性厚壁脓肿可行肝叶切除
E.多发性脓肿宜经腹腔切开引流
E.多发性脓肿宜经腹腔切开引流
6 2 2008 5. 细菌性肝脓肿治疗中一般不需要手术切开引流的脓肿是: (2.0分) A.较大表浅易穿破的脓肿
B.已穿破腹腔或胸腔的脓肿
C.位于肝右后叶壁厚的脓肿
D.位于肝左叶、穿刺易污染腹腔的脓肿
E.慢性迁延性脓肿
C.位于肝右后叶壁厚的脓肿
6 2 2008 6. 细菌性肝脓肿中不是常见的致病菌是: (2.0分) A.大肠杆菌
B.金黄色葡萄球菌
C.厌氧链球菌
D.表皮葡萄球菌
E.类杆菌属
D.表皮葡萄球菌
6 2 2008 7. 除下列哪种肝脏疾病外是常见需要和肝脓肿相鉴别的: (2.0分) A.肝囊肿
B.胆管细胞癌
C.转移性肝癌
D.肝腺瘤
E.肝包虫病
D.肝腺瘤
6 2 2008 8. 下列不属于兴奋性神经递质的是(2.0分) A.γ-氨基丁酸
B.去甲肾上腺素
C.乙酰胆碱
D.门冬氨酸
E.多巴胺
A.γ-氨基丁酸
6 2 2008 9. 下列属于芳香族氨基酸的是(2.0分) A.缬氨酸
B.亮氨酸
C.异亮氨酸
D.色氨酸
E.苏氨酸
D.色氨酸
6 2 2008 10. 门脉高压是指门脉压力超过(2.0分) A.10cm H2O
B.20cm H2O
C.30cm H2O
D.40cm H2O
E.50cm H2O
B.20cm H2O
6 2 2008 11. 在我国和西方国家,造成肝硬化的最主要原因是分别是(2.0分) A.乙型肝炎和非酒精性脂肪性肝病
B.乙型肝炎和丙型肝炎
C.丙型肝炎和非酒精性脂肪性肝病
D.丙型肝炎和乙型肝炎
E.非酒精性脂肪性肝病和丙型肝炎
A.乙型肝炎和非酒精性脂肪性肝病
6 2 2008 12. 肝硬化的并发症不包括(2.0分) A.肝性脑病
B.上消化道初学
C.肝肺综合征
D.原发性肝癌
E.尿崩症
E.尿崩症
6 2 2008 13. 自发性细菌性腹膜炎常见致病菌应是(2.0分) A.大肠杆菌和副大肠杆菌
B.溶血链球菌和肺炎双球菌
C.厌氧菌和链球菌
D.大肠杆菌和厌氧菌
E.绿脓杆菌和葡萄球菌
B.溶血链球菌和肺炎双球菌
6 2 2008 14. 男性,45岁,因患肝硬化腹水用速尿后尿量每日超过2000ml,近日出现四肢肌肉软弱无力,伴恶心呕吐,心电图出现传导和节律异常,其原因最可能是(2.0分) A.低钾血症
B.高钾血症
C.低钠血症
D.高钠血症
E.低镁血症
A.低钾血症
6 2 2008 15. 男性,38岁,患肝硬化3年,一周来畏寒发热,体温38℃左右,全腹痛,腹部明显膨胀,尿量500ml/d。住院后经检查有以下体征,对目前病情判断最有意义的是(2.0分) A.蜘蛛痣及肝掌
B.腹壁静脉曲张呈海蛇头样
C.脾肿大
D.全腹压痛及反跳痛
E.腹部移动性浊音阳性
D.全腹压痛及反跳痛
6 2 2008 16. 关于肝硬化的叙述中正确的是(2.0分) A.亚急性重症病毒性肝炎多发展为门脉性肝硬化
B.病变特点是肝细胞坏死、纤维组织增生和假小叶形成
C.门脉高压可表现为出血倾向
D.肝功能不全可表现为脾脏肿大
E.肝硬化不发生癌变
B.病变特点是肝细胞坏死、纤维组织增生和假小叶形成
6 2 2008 17. 治疗肝硬化并上消化道出血(2.0分) A.休息,低盐饮食,限制入量,补充蛋白,给利尿剂
B.休息,高热量高蛋白饮食,护肝治疗
C.休息,高热量限制蛋白饮食,输入支链氨基酸
D.休息,禁食,积极补足血容量及采用各种止血措施
E.休息,控制输液量,纠正电解质紊乱,限制蛋白质
D.休息,禁食,积极补足血容量及采用各种止血措施
6 2 2008 18. 下列属于肝硬化门脉高压表现的是(2.0分) A.肝掌
B.蜘蛛痣
C.脾肿大
D.肝肿大
E.男性乳房发育
C.脾肿大
6 2 2008 19. 肝硬化腹水治疗,一般不主张采用(2.0分) A.高蛋白饮食
B.低盐饮食
C.卧床休息
D.强烈利尿
E.腹水浓缩回输
D.强烈利尿
6 2 2008 20. 常伴皮肤瘙痒的肝硬化是(2.0分) A.小结节性肝硬化
B.大结节性肝硬化
C.淤血性肝硬化
D.原发性胆汁性肝硬化
E.日本血吸虫性肝硬化
D.原发性胆汁性肝硬化
6 2 2008 21. 肝硬化患者出现消化道出血时,下列可诱发肝性脑病的是(2.0分) A.输生理盐水
B.水电解质紊乱致低钾血症
C.静脉注射止血药
D.内镜下止血
E.口服去甲肾上腺素
B.水电解质紊乱致低钾血症
6 2 2008 22. 查体于肿大的肝脏表面听到血管杂音时,应考虑(2.0分) A.布加综合征
B.门脉血栓形成
C.肝硬化失代偿期
D.原发性肝癌
E.慢性肝炎
D.原发性肝癌
6 2 2008 23. 关于肝细胞气球样变细胞中下列哪一项改变是错误的(2.0分) A.细胞内糖原增多
B.细胞内水分增多
C.内质网扩张、囊泡变性
D.线立体肿胀、嵴消失
E.粗面内质网核蛋白体颗粒消失
A.细胞内糖原增多
6 2 2008 24. 造成感染慢性化的肝炎病毒是(2.0分) A.肝炎病毒A.B.C
B.肝炎病毒A.B.E
C.肝炎病毒A.C.D
D.肝炎病毒B.C.D
E.肝炎病毒C.D.E
D.肝炎病毒B.C.D
6 2 2008 25. 关于病毒性肝炎,下列哪项是错误的(2.0分) A.以肝实质细胞变性坏死为主要病变的传染病
B.各型肝炎病变的性质完全相同
C.急性普通型肝炎以广泛的肝细胞变性为主
D.桥接坏死常见于中、重度慢性肝炎
E.大片的溶解性坏死是重型肝炎的病变特征
B.各型肝炎病变的性质完全相同
6 2 2008 26. 引起门脉高压的原因一般不包括(2.0分) A.假小叶压迫小叶下静脉
B.纤维组织压迫中央静脉
C.肝窦狭窄、闭合
D.肝动脉和门静脉之间形成吻合支
E.肝动脉与肝静脉之间形成吻合支
E.肝动脉与肝静脉之间形成吻合支
6 2 2008 27. 早期肝癌是指瘤体(2.0分) A.单个有包膜
B.3cm以下,不超过2个
C.微小而多个
D.界限清楚无出血
E.以上都是
B.3cm以下,不超过2个
6 2 2008 28. 肝细胞癌患者最有诊断意义的血清学指标是(2.0分) A.甲胎蛋白
B.癌胚抗原
C.碱性磷酸酶
D.酸性磷酸酶
E.γ-谷氨酰转肽酶
A.甲胎蛋白
6 2 2008 29. 原发性肝癌是指(2.0分) A.胆管上皮发生的癌
B.肝细胞和肝内胆管上皮发生的癌
C.肝细胞发生的癌
D.肝细胞和肝外胆管上皮发生的癌
E.来自Kupffer细胞的恶性肿瘤
B.肝细胞和肝内胆管上皮发生的癌
6 2 2008 30. 急性重型肝炎的病变性质是(2.0分) A.以肝细胞变性为主的炎症
B.以肝细胞坏死为主的炎症
C.以肝细胞增生为主的炎症
D.以汇管区渗出为主的炎症
E.以汇管区增生为主的炎症
B.以肝细胞坏死为主的炎症
6 2 2008 31. 肝细胞大片崩解坏死并有肝细胞结节状再生者见于(2.0分) A.慢性持续性肝炎
B.慢性活动性肝炎
C.亚急性重型肝炎
D.急性普通型肝炎
E.急性重型肝炎
C.亚急性重型肝炎
6 2 2008 32. 肝硬化腹水生成的机制中,不正确的说法是(2.0分) A.肝脏合成清蛋白减少
B.血内抗利尿激素、醛固酮增高
C.结缔组织增生、肝静脉受压
D.门静脉淤血、血管壁通透性增强
E.肝窦内压力升高
C.结缔组织增生、肝静脉受压
6 2 2008 33. 肝硬化形成过程的基本病理变化不包括(2.0分) A.肝细胞溶解性坏死
B.肝细胞结节状再生
C.肝细胞嗜酸性坏死
D.肝细胞桥接坏死
E.纤维组织增生
C.肝细胞嗜酸性坏死
6 2 2008 34. 碎片状坏死主要见于(2.0分) A.急性肝炎
B.慢性肝炎
C.亚急性重型肝炎
D.急性重型肝炎
E.肝硬化
B.慢性肝炎
6 2 2008 35. 桥接坏死常见于(2.0分) A.急性重型肝炎
B.慢性轻度肝炎
C.慢性重度肝炎
D.急性普通型肝炎
E.亚急性重型肝炎
C.慢性重度肝炎
6 2 2008 36. 下列哪项既与门脉高压有关,又与肝功能障碍有关(2.0分) A.侧支循环建立
B.胃肠道淤血水肿
C.脾大
D.黄疸
E.腹水
E.腹水
6 2 2008 37. 下列哪项对诊断门静脉高压最有特征性?(2.0分) A.脾大
B.腹水
C.侧支循环的建立及开放
D.蜘蛛痣
E.脾功能亢进表现
C.侧支循环的建立及开放
6 2 2008 38. 哪一种与肝癌无关(2.0分) A.黄曲霉菌
B.亚硝胺
C.乙型肝炎
D.脂肪变性
E.肝硬化
D.脂肪变性
6 2 2008 39. The Calot's Triangle is bounded by except: (2.0分) A.Cystic duct.
B.The left hepatic duct.
C.The common hepatic duct.
D.The inferior border of the liver.
E.None of above.
B.The left hepatic duct.
6 2 2008 40. 目前对小肝癌定位诊断的各种方法中最优者是:(2.0分) A.选择性腹腔动脉或肝动脉造影
B.B型超声显象仪检查
C.CT检查
D.放射性核素肝扫描
E.磁共振成象
A.选择性腹腔动脉或肝动脉造影
6 2 2008 41. 女性,57岁,CT示肝左外叶8cm占位,伴门静脉左支栓塞,肝肾功能正常,首选治疗方案:(2.0分) A.手术切除
B.经肝动脉化学治疗和栓塞
C.全身化学治疗和中医治疗
D.肝肿瘤局部无水酒精注射
E.生物治疗
A.手术切除
6 2 2008 42. 肝内胆管出血部位不明可采用:(2.0分) A.三腔管压迫
B.肝固有动脉结扎及胆总管引流
C.胃大部切除
D.肝叶切除
E.肝固有动脉结扎
B.肝固有动脉结扎及胆总管引流
6 2 2008 43. 一名49岁男性,因右季肋部疼痛半月余,加重一天入院,以往健康,检查时发现血HBsAg阳性,AFP500ug/L。支持患原发性肝癌最有意义的体征是:(2.0分) A.肝右肋下约4.0cm
B.肝脏质地硬,有压痛
C.肝区可闻及血管杂音
D.肝区可闻及摩擦音
E.肝区有明显叩击痛
C.肝区可闻及血管杂音
6 2 2008 44. 中晚期肝癌最常见的体征是:(2.0分) A.肝区痛
B.腹胀
C.肝肿大
D.下肢浮肿
E.皮下出血
C.肝肿大
6 2 2008 45. 原发性肝癌的主要并发症不包括:(2.0分) A.肝性脑病
B.癌肿破裂出血
C.上消化道出血
D.低血糖症
E.继发感染
D.低血糖症
6 2 2008 46. 男,55岁,进行性黄疸,肝功能欠佳,PT明显延长,宜选用哪种检查:(2.0分) A.碘番酸口服胆囊造影
B.静脉胆囊造影
C.经皮肝穿刺胆管造影(PTC)
D.内窥镜逆行胰胆管造影(ERCP)
E.胆囊穿刺置管造影
D.内窥镜逆行胰胆管造影(ERCP)
6 2 2008 47. 上消化道出血的首选诊断方法是:(2.0分) A.鼻胃管或三腔管检查
B.早期内镜检查
C.选择性腹腔动脉或肠系膜上动脉造影
D.X线钡餐检查
E.核素检查
B.早期内镜检查
6 2 2008 48. 下列哪个部位不存在门静脉系与腔静脉系之间的交通支:(2.0分) A.胃底、食管下端
B.直肠下部,肛管
C.食管中段、胃底
D.前腹壁
E.腹膜后
C.食管中段、胃底
6 2 2008 49. 原发性肝癌早期转移途径为:(2.0分) A.肺内转移
B.淋巴转移
C.直接浸润转移
D.肝内转移
E.骨转移
D.肝内转移
6 2 2008 50. 目前肝癌早期宜采用哪一种治疗为主:(2.0分) A.放射疗法
B.化学疗法
C.手术切除
D.中医中药
E.中医中药
C.手术切除
6 2 2009 1. 查体于肿大的肝脏表面听到血管杂音时,应考虑 (2.5分) A.布加综合征
B.门脉血栓形成
C.肝硬化失代偿期
D.原发性肝癌
E.慢性肝炎
D.原发性肝癌
6 2 2009 2. 自发性细菌性腹膜炎常见致病菌应是(2.5分) A.大肠杆菌和副大肠杆菌
B.溶血链球菌和肺炎双球菌
C.厌氧菌和链球菌
D. 大肠杆菌和厌氧菌
E. 绿脓杆菌和葡萄球菌
B.溶血链球菌和肺炎双球菌
6 2 2009 3. 男性,45岁,因患肝硬化腹水用速尿后尿量每日超过2000ml,近日出现四肢肌肉软弱无力,伴恶心呕吐,心电图出现传导和节律异常,其原因最可能是(2.5分) A. 低钾血症
B. 高钾血症
C. 低钠血症
D. 高钠血症
E. 低镁血症
A. 低钾血症
6 2 2009 4. 男性,38岁,患肝硬化3年,一周来畏寒发热,体温38℃左右,全腹痛,腹部明显膨胀,尿量500ml/d。住院后经检查有以下体征,对目前病情判断最有意义的是(2.5分) A. 蜘蛛痣及肝掌
B. 腹壁静脉曲张呈海蛇头样
C. 脾肿大
D. 全腹压痛及反跳痛
E. 腹部移动性浊音阳性
D. 全腹压痛及反跳痛
6 2 2009 5. 治疗肝硬化并上消化道出血(2.5分) A.休息,低盐饮食,限制入量,补充蛋白,给利尿剂
B.休息,高热量高蛋白饮食,护肝治疗
C.休息,高热量限制蛋白饮食,输入支链氨基酸
D.休息,禁食,积极补足血容量及采用各种止血措施
E.休息,控制输液量,纠正电解质紊乱,限制蛋白质
D.休息,禁食,积极补足血容量及采用各种止血措施
6 2 2009 6. 常伴皮肤瘙痒的肝硬化是(2.5分) A.小结节性肝硬化
B.大结节性肝硬化
C.淤血性肝硬化
D.原发性胆汁性肝硬化
E.日本血吸虫性肝硬化
D.原发性胆汁性肝硬化
6 2 2009 7. 下列不属于兴奋性神经递质的是(2.5分) A.γ-氨基丁酸
B.去甲肾上腺素
C.乙酰胆碱
D.门冬氨酸
E.多巴胺
A.γ-氨基丁酸
6 2 2009 8. 下列属于芳香族氨基酸的是 (2.5分) A.缬氨酸
B.亮氨酸
C.异亮氨酸
D.色氨酸
E.苏氨酸
D.色氨酸
6 2 2009 9. 门脉高压是指门脉压力超过 (2.5分) A.10cm H2O
B.20cm H2O
C.30cm H2O
D.40cm H2O
E.50cm H2O
B.20cm H2O
6 2 2009 10. 对细菌性肝脓肿的治疗,不正确的是: (2.5分) A.应使用较大剂量抗生素
B.单个较大脓肿可经皮穿刺置管引流
C.脓肿穿破胸腔应同时引流胸腔
D.慢性局限性厚壁脓肿可行肝叶切除
E.多发性脓肿宜经腹腔切开引流
E.多发性脓肿宜经腹腔切开引流
6 2 2009 11. 细菌性肝脓肿治疗中一般不需要手术切开引流的脓肿是: (2.5分) A.较大表浅易穿破的脓肿
B.已穿破腹腔或胸腔的脓肿
C.位于肝右后叶壁厚的脓肿
D.位于肝左叶、穿刺易污染腹腔的脓肿
E.慢性迁延性脓肿
C.位于肝右后叶壁厚的脓肿
6 2 2009 12. 除下列哪种肝脏疾病外是常见需要和肝脓肿相鉴别的: (2.5分) A.肝囊肿
B.胆管细胞癌
C.转移性肝癌
D.肝腺瘤
E.肝包虫
D.肝腺瘤
6 2 2009 13. 细菌性肝脓肿的主要感染途径是: (2.5分) A.胆道
B.肝动脉
C.门静脉
D.淋巴管
E.肝破损裂口
A.胆道
6 2 2009 14. 以下不是细菌性肝脓肿的常见临床表现是: (2.5分) A.寒战、高烧
B.肝区剧烈疼痛
C.肋下可及肿大肝脏
D.乏力、食欲不振
E.白细胞计数增高、明显左移
B.肝区剧烈疼痛
6 2 2009 15. 对阿米巴肝脓肿的有关论述,不正确的是: (2.5分) A.继发于阿米巴痢疾后
B.病情急剧,中毒症状严重
C.部分病人粪中查有阿米巴滋养体
D.多见棕褐色脓液,无臭味
E.白细胞计数增高不明显
B.病情急剧,中毒症状严重
6 2 2009 16. The Calot's Triangle is bounded by except:(2.5分) A.Cystic duct.
B.The left hepatic duct.
C.The common hepatic duct.
D.The inferior border of the liver.
E.None of above.
B.The left hepatic duct.
6 2 2009 17. 女性,57岁,CT示肝左外叶8cm占位,伴门静脉左支栓塞,肝肾功能正常,首选治疗方案:(2.5分) A.手术切除
B.经肝动脉化学治疗和栓塞
C.全身化学治疗和中医治疗
D.肝肿瘤局部无水酒精注射
E.生物治疗
A.手术切除
6 2 2009 18. 肝内胆管出血部位不明可采用:(2.5分) A.三腔管压迫
B.肝固有动脉结扎及胆总管引流
C.胃大部切除
D.肝叶切除
E.肝固有动脉结扎
B.肝固有动脉结扎及胆总管引流
6 2 2009 19. 一名49岁男性,因右季肋部疼痛半月余,加重一天入院,以往健康,检查时发现血HBsAg阳性,AFP500ug/L。支持患原发性肝癌最有意义的体征是:(2.5分) A.肝右肋下约4.0cm
B.肝脏质地硬,有压痛
C.肝区可闻及血管杂音
D. 肝区可闻及摩擦音
E. 肝区有明显叩击痛
C.肝区可闻及血管杂音
6 2 2009 20. 中晚期肝癌最常见的体征是:(2.5分) A.肝区痛
B.腹胀
C.肝肿大
D.下肢浮肿
E.皮下出血
C.肝肿大
6 2 2009 21. 原发性肝癌的主要并发症不包括:(2.5分) A.肝性脑病
B.癌肿破裂出血
C.上消化道出血
D.低血糖症
E.继发感染
D.低血糖症
6 2 2009 22. 男,55岁,进行性黄疸,肝功能欠佳,PT明显延长,宜选用哪种检查:(2.5分) A.碘番酸口服胆囊造影
B.静脉胆囊造影
C.经皮肝穿刺胆管造影(PTC)
D.内窥镜逆行胰胆管造影(ERCP)
E.胆囊穿刺置管造影
D.内窥镜逆行胰胆管造影(ERCP)
6 2 2009 23. 肝硬化患者出现血性腹水,但无腹痛及发烧。首先考虑可能并发:(2.5分) A.原发性腹膜炎
B.原发性肝癌
C.结核性腹膜炎
D.肝肾综合征
E.门静脉血栓形成
B.原发性肝癌
6 2 2009 24. 肝癌血行肝外转移最多见于:(2.5分) A.肺
B.胰
C.脑
D.肾
E.脑
A.肺
6 2 2009 25. 肝硬化引起门脉高压的临床表现是:(2.5分) A.黄疸、腹水、脾肿大
B.腹水、脾肿大、肾功能衰竭
C.黄疸、腹水、侧枝循环的建立与开放
D.腹水、脾肿大、侧枝循环的建立与开放
E.腹水、上消化道出血、侧枝循环的建立与开放
D.腹水、脾肿大、侧枝循环的建立与开放
6 2 2009 26. 下面哪一项不符合干扰素治疗乙肝炎的指征:(2.5分) A.血清胆红素在正常上限的2倍
B.HBV DNA 阳性
C.HBeAg 阳性
D.ALT在正常上限的2-10倍之间
E.病理检查有活动性炎症
A.血清胆红素在正常上限的2倍
6 2 2009 27. Couinaud分段法将肝脏分为8段,其中尾状叶在:(2.5分) A.Ⅰ段
B.Ⅱ段
C.Ⅲ段
D.Ⅴ段
E.Ⅷ段
A.Ⅰ段
6 2 2009 28. 早期肝癌是指瘤体:(2.5分) A.单个有包膜
B.3cm以下,不超过2个
C.微小而多个
D.界限清楚无出血
E.以上都是
B.3cm以下,不超过2个
6 2 2009 29. 肝细胞弥漫性水样变性主要见于 (2.5分) A.急性重型肝炎
B.亚急性重型肝炎
C.慢性持续性肝炎
D.急性普通性肝炎
E.胆汁淤滞性肝炎
D.急性普通性肝炎
6 2 2009 30. 肝细胞呈碎片状坏死或形成桥接坏死见于 (2.5分) A.急性重型肝炎
B.亚急性重型肝炎
C.急性普通型肝炎
D.慢性活动性肝炎
E.慢性持续性肝炎
D.慢性活动性肝炎
6 2 2009 31. 甲型肝炎病毒(HAV)的主要传播途径是 (2.5分) A.经血传播
B.经消化道传播
C.经呼吸道传播
D.性传播
E.母婴传播
B.经消化道传播
6 2 2009 32. 应用三腔管压迫止血,哪项不正确(2.5分) A.先向胃气囊充气150-200ml
B.管端悬以重量约1.0kg的物品,作牵引压迫
C.若仍有出血,再向食管气囊注气100-150ml
D.放置三腔管的时间不宜持续超过3-5天
E.每隔12小时,应将气囊放空10-20分钟
B.管端悬以重量约1.0kg的物品,作牵引压迫
6 2 2009 33. 目前肝癌早期宜采用哪一种治疗为主:(2.5分) A.放射疗法
B.化学疗法
C.手术切除
D.中医中药
E.中医中药
C.手术切除
6 2 2009 34. 对疑有早期原发性肝癌的患者,应首先用哪一种方法检查较好:(2.5分) A.同位素肝扫描
B.血清甲胎蛋白动态观察检查
C.肝区超声波检查
D.血清γˉ谷氨酰转肽酶
E.选择性肝动脉造影
B.血清甲胎蛋白动态观察检查
6 2 2009 35. 下列哪种机制与肝性脑病发生无关? (2.5分) A.氨中毒学说
B.假神经递质学说
C.GABA/BZ复合体学说
D.氨基酸失衡学说
E.水钠潴留
E.水钠潴留
6 2 2009 36. 有关肝性脑病错误的方法是(2.5分) A.生理盐水灌肠
B.弱酸溶液灌肠
C.肥皂水灌肠
D.乳果糖灌肠
E.硫酸镁导泻
C.肥皂水灌肠
6 2 2009 37. 肝硬化腹水治疗,一般不主张采用(2.5分) A.高蛋白饮食
B.低盐饮食
C.卧床休息
D.强烈利尿
E.腹水浓缩回输
D.强烈利尿
6 2 2009 38. 肝细胞癌患者最有诊断意义的血清学指标是 (2.5分) A.甲胎蛋白
B.癌胚抗原
C.碱性磷酸酶
D.酸性磷酸酶
E.γ-谷氨酰转肽酶
A.甲胎蛋白
6 2 2009 39. 原发性肝癌是指 (2.5分) A.胆管上皮发生的癌
B.肝细胞和肝内胆管上皮发生的癌
C.肝细胞发生的癌
D.肝细胞和肝外胆管上皮发生的癌
E.来自Kupffer细胞的恶性肿瘤
B.肝细胞和肝内胆管上皮发生的癌
6 2 2009 40. 肝硬化形成过程的基本病理变化不包括 (2.5分) A.肝细胞溶解性坏死
B.肝细胞结节状再生
C.肝细胞嗜酸性坏死
D.肝细胞桥接坏死
E.纤维组织增生
C.肝细胞嗜酸性坏死
6 2 2010 1. Three weeks after a meal at the Trucker's Cafe, a 28-year-old man develops malaise, fatigue,and loss of appetite. He notes passing dark urine. On physical examination, he has mild scleral icterus and right upper quadrant tenderness. Laboratory studies show serum AST of 62U/L and ALT of 58U/L. The total bilirubin concentration is 3.9 mg/dL, and the direct bilirubin concentration is 3.9mg/dL. His symptoms abate over the next 3 weeks. On returning to the cafe, he finds that it has been closed by the city's health department. Which of the following serologic test results is most likely to be positive in this patient? (2.5分) A.Anti-HBs
B.IgM anti-HDV
C.Anti-HCV
D.IgM anti-HAV
E.Anti-HBc
D.IgM anti-HAV
6 2 2010 2. A 53-year-old man comes to the emergency department because of marked hematemesis that has continued for the past 3 hours. On physical examination, he has a temperature of 35.9℃, pulse of 90/45 mm Hg. He has a distended abdomen with a fluid wave, and the spleen tip is palpable. What liver disease is most likely to be present in this patient? (2.5分) A.Cirrhosis
B.Cholangiocarcinoma
C.Massive hepatic necrosis
D.Fatty change
E.HAV infection
A.Cirrhosis
6 2 2010 3. A 27-year-old man with a history of intravenous drug use has been known to be infected with hepatitis B virus for the past 6 years and not ill. He is seen in the emergency department because he has had nausea, vomiting, and passage of dark-colored urine for the past week. Physical examination shows scleral icterus and mild jaundice. There are recent and healed track marks in the right antecubital fossa. Neurologic examination shows a confused, somnolent man oriented only to person. He exhibits asterixis. Laboratory studies show total protein, 5g/dL; albumin, 2.7g/dL; AST, 2342U/L; ALT, 2150U/L; alkaline phosphatase, 233U/L; total bilirubin, 8.3mg/dL; and direct bilirubin, 4.5mg/dL. What is the most likely diagnosis? (2.5分) A.Chronic HAV infection
B.Chronic HBV infection
C.Chronic HCV infection
D.Coinfection with HBV and HDV
E.Superinfection of chronic HBV with HDV
E.Superinfection of chronic HBV with HDV
6 2 2010 4. 肝细胞呈碎片状坏死或形成桥接坏死见于(2.5分) A.急性重型肝炎
B.亚急性重型肝炎
C.急性普通型肝炎
D.慢性活动性肝炎
E.慢性持续性肝炎
D.慢性活动性肝炎
6 2 2010 5. 下列哪项是确诊门静脉性肝硬化最可靠的依据? (2.5分) A.血浆白蛋白、球蛋白比例倒置
B.肝功能明显不正常
C.肝穿刺病理有假小叶形成
D.肝、脾大
E.肝脏明显增大
C.肝穿刺病理有假小叶形成
6 2 2010 6. 下列关于肝脏的功能,不正确的是(2.5分) A.分泌功能
B.代谢功能
C.解毒作用
D.合成功能
E.氧合功能
E.氧合功能
6 2 2010 7. 女性,57岁,CT示肝左外叶8cm占位,无血管侵犯,肝肾功能正常,首选治疗方案(2.5分) A.手术切除
B.经肝动脉化学治疗和栓塞
C.全身化学治疗和中医治疗
D.肝肿瘤局部无水酒精注射
E.生物治疗
A.手术切除
6 2 2010 8. 一名49岁男性,因右季肋部疼痛半月余,加重一天入院,以往健康,检查时发现血HBsAg阳性,AFP500ug/L。下面哪一种辅助检查不利于明确诊断(2.5分) A.B超
B.X片
C.CT
D.磁共振
E.PET-CT
B.X片
6 2 2010 9. 原发性肝癌的主要并发症不包括(2.5分) A.肝性脑病
B.癌肿破裂出血
C.上消化道出血
D.低血糖症
E.继发感染
D.低血糖症
6 2 2010 10. 上消化道出血的首选诊断方法是(2.5分) A.鼻胃管或三腔管检查
B.早期内镜检查
C.选择性腹腔动脉或肠系膜上动脉造影
D.X线钡餐检查
E.核素检查
B.早期内镜检查
6 2 2010 11. 肝硬化患者出现血性腹水,但无腹痛及发烧。首先考虑可能并发(2.5分) A.原发性腹膜炎
B.原发性肝癌
C.结核性腹膜炎
D.肝肾综合征
E.门静脉血栓形成
B.原发性肝癌
6 2 2010 12. 目前肝癌早期宜采用哪一种治疗为主(2.5分) A.放射疗法
B.化学疗法
C.手术切除
D.中医中药
E.中医中药
C.手术切除
6 2 2010 13. 肝癌血行肝外转移最多见于(2.5分) A.肺
B.胰
C.脑
D.肾
E.脑
A.肺
6 2 2010 14. 对疑有早期原发性肝癌的患者,应首先用哪一种方法检查较好(2.5分) A.同位素肝扫描
B.血清甲胎蛋白动态观察检查
C.肝区超声波检查
D.血清γˉ谷氨酰转肽酶
E.选择性肝动脉造影
B.血清甲胎蛋白动态观察检查
6 2 2010 15. 肝移植的适应症不包括(2.5分) A.急性肝衰竭
B.重型肝炎
C.先天性胆道闭锁
D.肝硬化失代偿合并小肝癌
E.肝脏巨大血管瘤
E.肝脏巨大血管瘤
6 2 2010 16. 门脉高压症发生消化道出血的主要原因是(2.5分) A.痔静脉曲张
B.脾肿大破裂
C.腹壁静脉曲张
D.食管下段和胃底静脉曲张
D.食管下段和胃底静脉曲张
6 2 2010 17. Couinaud分段法将肝脏分为8段,其中尾状叶在(2.5分) A.Ⅰ段
B.Ⅱ段
C.Ⅲ段
D.Ⅴ段
E.Ⅷ段
A.Ⅰ段
6 2 2010 18. Couinaud肝脏分段法是根据以下哪项解剖结构为基础(2.5分) A.肝动脉、门静脉
B.门静脉、胆道
C.肝静脉、胆道
D.肝静脉、门静脉
E.Glisson纤维鞘
D.肝静脉、门静脉
6 2 2010 19. 肝癌最主要的转移途径是(2.5分) A.直接侵犯
B.淋巴转移
C.血行转移
D.腹腔内种植
E.肝内转移
E.肝内转移
6 2 2010 20. 下列哪项不是肝硬变腹水形成机制(2.5分) A.低蛋白血症
B.门静脉压力增高
C.肝内淋巴的产生增多
D.肝静脉阻力增高
E.醛固酮分泌减少
D.肝静脉阻力增高
6 2 2010 21. 下列哪种机制与肝性脑病发生无关(2.5分) A.氨中毒学说
B.假神经递质学说
C.GABA/BZ复合体学说
D.氨基酸失衡学说
E.水钠潴留
E.水钠潴留
6 2 2010 22. 下列属于芳香族氨基酸的是(2.5分) A.缬氨酸
B.亮氨酸
C.异亮氨酸
D.色氨酸
E.苏氨酸
D.色氨酸
6 2 2010 23. 肝硬化腹水形成的原因不包括(2.5分) A.门静脉压力增高
B.血浆胶体渗透压降低
C.肝脏淋巴液失平衡
D.甲状腺功能减退
E.低蛋白血症
D.甲状腺功能减退
6 2 2010 24. The complication of liver cirrhosis is not including(2.5分) A.Hepatic coma
B.Upper GI bleeding
C.Liver-lung syndrome
D.Hepatocellular carcinoma
E.Diabetes insipidus
E.Diabetes insipidus
6 2 2010 25. 男性,45岁,因患肝硬化腹水用速尿后尿量每日超过2000ml,近日出现四肢肌肉软弱无力,伴恶心呕吐,心电图出现传导和节律异常,其原因最可能是(2.5分) A.低钾血症
B.高钾血症
C.低钠血症
D.高钠血症
E.低镁血症
A.低钾血症
6 2 2010 26. 关于肝硬化的叙述中正确的是(2.5分) A.亚急性重症病毒性肝炎多发展为门脉性肝硬化
B.病变特点是肝细胞坏死、纤维组织增生和假小叶形成
C.门脉高压可表现为出血倾向
D.肝功能不全可表现为脾脏肿大
E.肝硬化不发生癌变
B.病变特点是肝细胞坏死、纤维组织增生和假小叶形成
6 2 2010 27. 下列属于肝硬化门脉高压表现的是(2.5分) A.肝掌
B.蜘蛛痣
C.脾肿大
D.肝肿大
E.男性乳房发育
C.脾肿大
6 2 2010 28. 查体于肿大的肝脏表面听到血管杂音时,应考虑(2.5分) A.布加综合征
B.门脉血栓形成
C.肝硬化失代偿期
D.原发性肝癌
E.慢性肝炎
D.原发性肝癌
6 2 2010 29. 有关肝性脑病错误的方法是(2.5分) A.生理盐水灌肠
B.弱酸溶液灌肠
C.肥皂水灌肠
D.乳果糖灌肠
E.硫酸镁导泻
C.肥皂水灌肠
6 2 2010 30. 常伴皮肤瘙痒的肝硬化是(2.5分) A.小结节性肝硬化
B.大结节性肝硬化
C.淤血性肝硬化
D.原发性胆汁性肝硬化
E.日本血吸虫性肝硬化
D.原发性胆汁性肝硬化
6 2 2010 31. A 57-year-old woman is found to have a liver mass by CT scan. Which of the followings confirms that she is more likely to have a pyogenic liver abscess than malignant tumor? (2.5分) A.She was diagnosed as liver cirrhosis previously.
B.Jaundice is absent.
C.Slightly yellow and turbid liquid is obtained by BUS guided aspiration.
D.She has a history of biliary tract disease.
E.There is a rapid response to metronidazole.
C.Slightly yellow and turbid liquid is obtained by BUS guided aspiration.
6 2 2010 32. A 64-year-old man was consulted because of a fever of 39°C, general malaise, nausea, and right upper quadrant (RUQ) discomfort. On physical examination, jaundice and RUQ tenderness are noted, and enlarged liver can be palpated. Chest x-ray shows elevated right diaphragm. Which of the followings indicates amebic liver abscess? (2.5分) A.Laboratory tests show leukocytosis.
B.Brown pus was aspirated.
C.Previous history of calculous cholecystitis.
D.He was diagnosed as diabetes mellitus 5 years ago.
E.Jaundice is presented.
B.Brown pus was aspirated.
6 2 2010 33. A 28-year-old man presents with right upper quadrant pain. His WBC and ALP are raised with a normal serum bilirubin. Serology for antibodies to Entamoeba histolytica is positive. Abdominal ultrasonography showed a solitary abscess measuring 6 cm in diameter in the right lobe of the liver. He had travelled to India 4 months ago. Which therapy is the first choice? (2.5分) A.Chloroquine
B.Metronidazole
C.Percutaneous drainage
D.Resection
E.Deroofing
B.Metronidazole
6 2 2010 34. 阿米巴原虫是沿何途径进入肝内,形成阿米巴肝脓肿的?(2.5分) A.肝静脉
B.肝动脉
C.胆道
D.淋巴道
E.门静脉属支
E.门静脉属支
6 2 2010 35. 阿米巴肝脓肿穿刺引流时往往采用封闭式引流,其主要原因是(2.5分) A.脓肿往往很大,脓液量大
B.脓肿常合并细菌感染
C.为避免脓肿破坏周边血管引起的出血堵塞引流管
D.脓肿多位于右肝近膈顶部,穿刺易引起气胸
E.为预防继发细菌感染
E.为预防继发细菌感染
6 2 2010 36. 应用三腔管压迫止血,哪项不正确(2.5分) A.先向胃气囊充气150-200ml
B.管端悬以重量约1.0kg的物品,作牵引压迫
C.若仍有出血,再向食管气囊注气100-150ml
D.放置三腔管的时间不宜持续超过3-5天
E.每隔12小时,应将气囊放空10-20分钟
B.管端悬以重量约1.0kg的物品,作牵引压迫
6 2 2010 37. 我国原发性肝癌的发病主要与下列哪个因素有关(2.5分) A.慢性乙型病毒性肝炎
B.丙型病毒性肝炎
C.营养不良
D.遗传因素
E.酒精摄入
A.慢性乙型病毒性肝炎
6 2 2010 38. 早期肝癌是指(2.5分) A.直径在2cm以内,少于2个结节
B.直径在3cm以内,少于2个结节
C.直径在2cm以内,少于3个结节
D.癌未累及被膜
E.癌局限于小叶内
B.直径在3cm以内,少于2个结节
6 2 2010 39. Which marker has the highest sensitivity and specificity for hepatocellular carcinoma? (2.5分) A.AFP
B.CEA
C.CA125
D.CA199
E.PSA
A.AFP
6 2 2010 40. 目前细菌性肝脓肿的感染途径最常见的是何种途径?(2.5分) A.肝动脉
B.脐静脉
C.门静脉
D.胆道
E.损伤处直接进入
D.胆道
6 2 2011 1. 应用三腔管压迫止血,哪项不正确(2.5分) A.先向胃气囊充气150-200ml
B.管端悬以重量约1.0kg的物品,作牵引压迫
C.若仍有出血,再向食管气囊注气100-150ml
D.放置三腔管的时间不宜持续超过3-5天
E.每隔12小时,应将气囊放空10-20分钟
B.管端悬以重量约1.0kg的物品,作牵引压迫
6 2 2011 2. 下列哪种说法不符合细菌性肝脓肿的常见表现: (2.5分) A.有慢性腹泻病史
B.病灶多为多发,好发于肝右叶
C.白细胞计数可增加
D.脓液呈黄色,有臭味
E.抗生素治疗多有效
A.有慢性腹泻病史
6 2 2011 3. 肝硬化腹水治疗,一般不主张采用(2.5分) A.高蛋白饮食
B.低盐饮食
C.卧床休息
D.强烈利尿
E.腹水浓缩回输
D.强烈利尿
6 2 2011 4. 第三肝门处进入下腔静脉的血管是(2.5分) A.门静脉
B.肝短静脉
C.肝静脉
D.肝动脉
E.胃网膜静脉
B.肝短静脉
6 2 2011 5. A 65-year-old man is found to have a liver abscess by CT scan. Which of the followings indicates that he is more likely to have a pyogenic than amebic liver abscess? (2.5分) A.He travelled to South American recently.
B.Jaundice is absent.
C.He has associated diarrhea.
D.He has a history of biliary tract disease.
E.There is a rapid response to metronidazole.
D.He has a history of biliary tract disease.
6 2 2011 6. A 34-year-old man presented to the Emergency Department with a 3-day history of abdominal pain, fever, malaise and jaundice. On examination he was pyrexial and jaundice with no other abdominal signs. An ultrasound scan of the liver showed a 5-cm hypoechoic lesion with a fluid level. Which is the most likely diagnosis? (2.5分) A.Haemangioma
B.Liver abscess
C.Hepatic metastasis
D.Hydatid cyst
E.Hepatocellular carcinoma (HCC)
B.Liver abscess
6 2 2011 7. 关于细菌性肝脓肿,下列哪项叙述正确(2.5分) A.大部分为胆源性肝脓肿
B.致病菌多为革兰氏阴性菌
C.脓液多为棕褐色,涂片可能无细菌
D.多见于肝右叶,单发
E.手术引流是唯一有效方法
A.大部分为胆源性肝脓肿
6 2 2011 8. 下列哪项不属于细菌性肝脓肿的经验性抗生素治疗必需覆盖的病原微生物(2.5分) A.大肠杆菌
B.肺炎克雷伯菌
C.金黄色葡萄球菌
D.厌氧性链球菌
E.分支杆菌
E.分支杆菌
6 2 2011 9. 一位68岁的男性患者因主诉右上腹痛伴高热三天来医院就诊,问诊得知他患有2型糖尿病多年,血常规检查发现该患者的白细胞计数升高至14×109/L,肝功能检查发现转氨酶轻度升高。腹部B超检查发现患者右肝内有一约6cm大小的病灶。该患者被诊断为细菌性肝脓肿,他的下一步的治疗方案是(2.5分) A.口服抗生素治疗
B.超声引导下的肝脓肿穿刺引流
C.广谱抗生素+超声引导下的肝脓肿穿刺引流
D.开腹肝脓肿切开引流术
E.脓肿所在的肝叶切除手术
C.广谱抗生素+超声引导下的肝脓肿穿刺引流
6 2 2011 10. 关于细菌性肝脓肿,下列哪项叙述正确? (2.5分) A.大部分为胆源性肝脓肿
B.致病菌多为革兰氏阳性菌
C.脓液多为棕褐色,涂片可能无细菌
D.多见于肝右叶,单发
E.手术引流是唯一有效方法
A.大部分为胆源性肝脓肿
6 2 2011 11. 下列哪种机制与肝性脑病发生无关? (2.5分) A.氨中毒学说
B.假神经递质学说
C.GABA/BZ复合体学说
D.氨基酸失衡学说
E.水钠潴留
E.水钠潴留
6 2 2011 12. 下列属于芳香族氨基酸的是(2.5分) A.缬氨酸
B.亮氨酸
C.异亮氨酸
D.色氨酸
E.苏氨酸
D.色氨酸
6 2 2011 13. 肝硬化腹水形成的原因不包括(2.5分) A.门静脉压力增高
B.血浆胶体渗透压降低
C.肝脏淋巴液失平衡
D.甲状腺功能减退
E.低蛋白血症
D.甲状腺功能减退
6 2 2011 14. The complication of liver cirrhosis is not including(2.5分) A.Hepatic coma
B.Upper GI bleeding
C.Liver-lung syndrome
D.Hepatocellular carcinoma
E.Diabetes insipidus
E.Diabetes insipidus
6 2 2011 15. 男性,45岁,因患肝硬化腹水用速尿后尿量每日超过2000ml,近日出现四肢肌肉软弱无力,伴恶心呕吐,心电图出现传导和节律异常,其原因最可能是(2.5分) A.低钾血症
B.高钾血症
C.低钠血症
D.高钠血症
E.低镁血症
A.低钾血症
6 2 2011 16. 关于肝硬化的叙述中正确的是(2.5分) A.亚急性重症病毒性肝炎多发展为门脉性肝硬化
B.病变特点是肝细胞坏死、纤维组织增生和假小叶形成
C.门脉高压可表现为出血倾向
D.肝功能不全可表现为脾脏肿大
E.肝硬化不发生癌变
B.病变特点是肝细胞坏死、纤维组织增生和假小叶形成
6 2 2011 17. 下列属于肝硬化门脉高压表现的是(2.5分) A.肝掌
B.蜘蛛痣
C.脾肿大
D.肝肿大
E.男性乳房发育
C.脾肿大
6 2 2011 18. 常伴皮肤瘙痒的肝硬化是(2.5分) A.小结节性肝硬化
B.大结节性肝硬化
C.淤血性肝硬化
D.原发性胆汁性肝硬化
E.日本血吸虫性肝硬化
D.原发性胆汁性肝硬化
6 2 2011 19. 查体于肿大的肝脏表面听到血管杂音时,应考虑(2.5分) A.布加综合征
B.门脉血栓形成
C.肝硬化失代偿期
D.原发性肝癌
E.慢性肝炎
D.原发性肝癌
6 2 2011 20. 肝硬化患者两个月前发现腹水,昨天突然出现剧烈腹痛、伴呕吐、发热、腹水迅速增加,并排出少量暗红色血便,最可能出现哪种情况(2.5分) A.食管静脉曲张破裂出血
B.急性胰腺炎
C.合并肝癌破裂
D.门静脉血栓形成
E.合并脾破裂出血
D.门静脉血栓形成
6 2 2011 21. 关于肝细胞气球样变细胞中下列哪一项改变是错误的 (2.5分) A.细胞内糖原增多
B.细胞内水分增多
C.内质网扩张、囊泡变性
D.线立体肿胀、嵴消失
E.粗面内质网核蛋白体颗粒消失
A.细胞内糖原增多
6 2 2011 22. 关于病毒性肝炎,下列哪项是错误的(2.5分) A.以肝实质细胞变性坏死为主要病变的传染病
B.各型肝炎病变的性质完全相同
C.急性普通型肝炎以广泛的肝细胞变性为主
D.桥接坏死常见于中、重度慢性肝炎
E.大片的溶解性坏死是重型肝炎的病变特征
B.各型肝炎病变的性质完全相同
6 2 2011 23. 早期肝癌是指瘤体(2.5分) A.单个有包膜
B.3cm以下,不超过2个
C.微小而多个
D.界限清楚无出血
E.以上都是
B.3cm以下,不超过2个
6 2 2011 24. 原发性肝癌是指(2.5分) A.胆管上皮发生的癌
B.肝细胞和肝内胆管上皮发生的癌
C.肝细胞发生的癌
D.肝细胞和肝外胆管上皮发生的癌
E.来自Kupffer细胞的恶性肿瘤
B.肝细胞和肝内胆管上皮发生的癌
6 2 2011 25. 肝细胞大片崩解坏死并有肝细胞结节状再生者见于 (2.5分) A.慢性持续性肝炎
B.慢性活动性肝炎
C.亚急性重型肝炎
D.急性普通型肝炎
E.急性重型肝炎
C.亚急性重型肝炎
6 2 2011 26. 肝硬化形成过程的基本病理变化不包括 (2.5分) A.肝细胞溶解性坏死
B.肝细胞结节状再生
C.肝细胞嗜酸性坏死
D.肝细胞桥接坏死
E.纤维组织增生
C.肝细胞嗜酸性坏死
6 2 2011 27. 碎片状坏死主要见于(2.5分) A.急性肝炎
B.慢性肝炎
C.亚急性重型肝炎
D.急性重型肝炎
E.肝硬化
B.慢性肝炎
6 2 2011 28. 下列哪项既与门脉高压有关,又与肝功能障碍有关(2.5分) A.侧支循环建立
B.胃肠道淤血水肿
C.脾大
D.黄疸
E.腹水
E.腹水
6 2 2011 29. 哪一种与肝癌无关 (2.5分) A.黄曲霉菌
B.亚硝胺
C.乙型肝炎
D.脂肪变性
E.肝硬化
D.脂肪变性
6 2 2011 30. 门脉性肝硬化的形态学特点不包括(2.5分) A.结节较小而大小较均匀
B.结节较大而大小较均匀
C.纤维间隔较窄
D.肝细胞结节状再生
E.以上都是
B.结节较大而大小较均匀
6 2 2011 31. 目前对小肝癌定位诊断的各种方法中最优者是(2.5分) A.选择性腹腔动脉或肝动脉造影
B.B型超声显象仪检查
C.CT检查
D.放射性核素肝扫描
E.磁共振成象
A.选择性腹腔动脉或肝动脉造影
6 2 2011 32. 肝内胆管大出血且部位不明可采用(2.5分) A.三腔管压迫
B.胆囊切除
C.胃大部切除
D.肝叶切除
E.肝固有动脉结扎
E.肝固有动脉结扎
6 2 2011 33. 中晚期肝癌最常见的体征是(2.5分) A.肝区痛
B.腹胀
C.肝肿大
D.下肢浮肿
E.皮下出血
C.肝肿大
6 2 2011 34. 男,55岁,进行性黄疸,肝功能欠佳,PT明显延长,宜选用哪种检查(2.5分) A.碘番酸口服胆囊造影
B.静脉胆囊造影
C.经皮肝穿刺胆管造影(PTC)
D.内窥镜逆行胰胆管造影(ERCP)
E.胆囊穿刺置管造影
D.内窥镜逆行胰胆管造影(ERCP)
6 2 2011 35. 下列哪个部位不存在门静脉系与腔静脉系之间的交通支(2.5分) A.胃底、食管下端
B.直肠下部,肛管
C.食管中段、胃底
D.前腹壁
E.腹膜后
C.食管中段、胃底
6 2 2011 36. 不属于肝脏良性肿瘤为(2.5分) A.肝囊肿
B.肝血管瘤
C.局灶结节性增生
D.肝腺瘤
E.肝母细胞瘤
E.肝母细胞瘤
6 2 2011 37. 我国与原发性肝癌发病无明显相关性的因素是(2.5分) A.病毒性肝炎
B.亚硝胺
C.嗜酒
D.黄曲霉素
E.吸烟
E.吸烟
6 2 2011 38. 引起细菌性肝脓肿最常见的原因是 (2.5分) A.坏脓性阑尾炎
B.开放性的肝损伤
C.右膈下脓肿
D.全身化脓性感染
E.胆道的化脓性感染
E.胆道的化脓性感染
6 2 2011 39. 肝硬化引起门脉高压的临床表现是(2.5分) A.黄疸、腹水、脾肿大
B.腹水、脾肿大、肾功能衰竭
C.黄疸、腹水、侧枝循环的建立与开放
D.腹水、脾肿大、侧枝循环的建立与开放
E.腹水、上消化道出血、侧枝循环的建立与开放
D.腹水、脾肿大、侧枝循环的建立与开放
6 2 2011 40. Glisson鞘内包含的结构主要有(2.5分) A.肝动脉、门静脉、胆管
B.肝动脉、肝静脉、胆管
C.肝动脉、肝静脉、门静脉
D.肝静脉、门静脉、胆管
E.肝动脉、肝静脉、肝圆韧带
A.肝动脉、门静脉、胆管
6 2 2012 1. 关于肝细胞气球样变细胞中下列哪一项改变是错误的 (2.5分) A.细胞内糖原增多
B.细胞内水分增多
C.内质网扩张、囊泡变性
D.线立体肿胀、嵴消失
E.粗面内质网核蛋白体颗粒消失
A.细胞内糖原增多
6 2 2012 2. 引起门脉高压的原因一般不包括 (2.5分) A.假小叶压迫小叶下静脉
B.纤维组织压迫中央静脉
C.肝窦狭窄、闭合
D.肝动脉和门静脉之间形成吻合支
E.肝动脉与肝静脉之间形成吻合支
E.肝动脉与肝静脉之间形成吻合支
6 2 2012 3. 肝细胞癌患者最有诊断意义的血清学指标是 (2.5分) A.甲胎蛋白
B.癌胚抗原
C.碱性磷酸酶
D.酸性磷酸酶
E.γ-谷氨酰转肽酶
A.甲胎蛋白
6 2 2012 4. 急性重型肝炎的病变性质是 (2.5分) A.以肝细胞变性为主的炎症
B.以肝细胞坏死为主的炎症
C.以肝细胞增生为主的炎症
D.以汇管区渗出为主的炎症
E.以汇管区增生为主的炎症
B.以肝细胞坏死为主的炎症
6 2 2012 5. 肝硬化腹水生成的机制中,不正确的说法是 (2.5分) A.肝脏合成清蛋白减少
B.血内抗利尿激素、醛固酮增高
C.结缔组织增生、肝静脉受压
D.门静脉淤血、血管壁通透性增强
E.肝窦内压力升高
C.结缔组织增生、肝静脉受压
6 2 2012 6. 不符合病毒性肝炎基本病变的是 (2.5分) A.肝细胞变性坏死
B.慢性炎细胞浸润
C.纤维组织增生
D.Kupffer细胞异型增生
E.小胆管增生
D.Kupffer细胞异型增生
6 2 2012 7. 桥接坏死常见于 (2.5分) A.急性重型肝炎
B.慢性轻度肝炎
C.慢性重度肝炎
D.急性普通型肝炎
E.亚急性重型肝炎
C.慢性重度肝炎
6 2 2012 8. 下列哪项对诊断门静脉高压最有特征性? (2.5分) A.脾大
B.腹水
C.侧支循环的建立及开放
D.蜘蛛痣
E.脾功能亢进表现
C.侧支循环的建立及开放
6 2 2012 9. 下列关于肝脏的功能,不正确的是: (2.5分) A.分泌功能
B.代谢功能
C.解毒作用
D.合成功能
E.氧合功能
E.氧合功能
6 2 2012 10. 女性,57岁,CT示肝左外叶8cm占位,无血管侵犯,肝肾功能正常,首选治疗方案:(2.5分) A.手术切除
B.经肝动脉化学治疗和栓塞
C.全身化学治疗和中医治疗
D.肝肿瘤局部无水酒精注射
E.生物治疗
A.手术切除
6 2 2012 11. 一名49岁男性,因右季肋部疼痛半月余,加重一天入院,以往健康,检查时发现血HBsAg阳性,AFP500ug/L。下面哪一种辅助检查不利于明确诊断:(2.5分) A.B超
B.X片
C.CT
D.磁共振
E.PET-CT
B.X片
6 2 2012 12. 原发性肝癌的主要并发症不包括:(2.5分) A.肝性脑病
B.癌肿破裂出血
C.上消化道出血
D.低血糖症
E.继发感染
D.低血糖症
6 2 2012 13. 上消化道出血的首选诊断方法是:(2.5分) A.鼻胃管或三腔管检查
B.早期内镜检查
C.选择性腹腔动脉或肠系膜上动脉造影
D.X线钡餐检查
E.核素检查
B.早期内镜检查
6 2 2012 14. 肝硬化患者出现血性腹水,但无腹痛及发烧。首先考虑可能并发:(2.5分) A.原发性腹膜炎
B.原发性肝癌
C.结核性腹膜炎
D.肝肾综合征
E.门静脉血栓形成
B.原发性肝癌
6 2 2012 15. 目前肝癌早期宜采用哪一种治疗为主: (2.5分) A.放射疗法
B.化学疗法
C.手术切除
D.中医中药
E.中医中药
C.手术切除
6 2 2012 16. 肝癌血行肝外转移最多见于:(2.5分) A.肺
B.胰
C.脑
D.肾
E.脑
A.肺
6 2 2012 17. 对疑有早期原发性肝癌的患者,应首先用哪一种方法检查较好:(2.5分) A.同位素肝扫描
B.血清甲胎蛋白动态观察检查
C.肝区超声波检查
D.血清γˉ谷氨酰转肽酶
E.选择性肝动脉造影
B.血清甲胎蛋白动态观察检查
6 2 2012 18. 肝移植的适应症不包括:(2.5分) A.急性肝衰竭
B.重型肝炎
C.先天性胆道闭锁
D.肝硬化失代偿合并小肝癌
E.肝脏巨大血管瘤
E.肝脏巨大血管瘤
6 2 2012 19. 门脉高压症发生消化道出血的主要原因是:(2.5分) A.痔静脉曲张
B.脾肿大破裂
C.腹壁静脉曲张
D.食管下段和胃底静脉曲张
D.食管下段和胃底静脉曲张
6 2 2012 20. Couinaud分段法将肝脏分为8段,其中尾状叶在:(2.5分) A.Ⅰ段
B.Ⅱ段
C.Ⅲ段
D.Ⅴ段
E.Ⅷ段
A.Ⅰ段
6 2 2012 21. Couinaud肝脏分段法是根据以下哪项解剖结构为基础: (2.5分) A.肝动脉、门静脉
B.门静脉、胆道
C.肝静脉、胆道
D.肝静脉、门静脉
E.Glisson纤维鞘
D.肝静脉、门静脉
6 2 2012 22. 肝癌最主要的转移途径是:(2.5分) A.直接侵犯
B.淋巴转移
C.血行转移
D.腹腔内种植
E.肝内转移
E.肝内转移
6 2 2012 23. 下列哪项不是肝硬变腹水形成机制: (2.5分) A.低蛋白血症
B.门静脉压力增高
C.肝内淋巴的产生增多
D.肝静脉阻力增高
E.醛固酮分泌减少
D.肝静脉阻力增高
6 2 2012 24. 下列不属于兴奋性神经递质的是(2.5分) A.γ-氨基丁酸
B.去甲肾上腺素
C.乙酰胆碱
D.门冬氨酸
E.多巴胺
A.γ-氨基丁酸
6 2 2012 25. Portal hypertension is defined as the portal pressure higher than (2.5分) A.10cm H2O
B.20cm H2O
C.30cm H2O
D.40cm H2O
E.50cm H2O
B.20cm H2O
6 2 2012 26. The major reasons for liver cirrhosis in China and Western countries are(2.5分) A.HBV and nonalcoholic fatty liver disease
B.HBV and HCV
C.HCV and nonalcoholic fatty liver disease
D.HCV and HBV
E.nonalcoholic fatty liver disease and HCV
A.HBV and nonalcoholic fatty liver disease
6 2 2012 27. 自发性细菌性腹膜炎常见致病菌应是(2.5分) A.大肠杆菌和副大肠杆菌
B.溶血链球菌和肺炎双球菌
C.厌氧菌和链球菌
D.大肠杆菌和厌氧菌
E.绿脓杆菌和葡萄球菌
B.溶血链球菌和肺炎双球菌
6 2 2012 28. 男性,38岁,患肝硬化3年,一周来畏寒发热,体温38℃左右,全腹痛,腹部明显膨胀,尿量500ml/d。住院后经检查有以下体征,对目前病情判断最有意义的是(2.5分) A.蜘蛛痣及肝掌
B.腹壁静脉曲张呈海蛇头样
C.脾肿大
D.全腹压痛及反跳痛
E.腹部移动性浊音阳性
D.全腹压痛及反跳痛
6 2 2012 29. 治疗肝硬化并上消化道出血(2.5分) A.休息,低盐饮食,限制入量,补充蛋白,给利尿剂
B.休息,高热量高蛋白饮食,护肝治疗
C.休息,高热量限制蛋白饮食,输入支链氨基酸
D.休息,禁食,积极补足血容量及采用各种止血措施
E.休息,控制输液量,纠正电解质紊乱,限制蛋白质
D.休息,禁食,积极补足血容量及采用各种止血措施
6 2 2012 30. 肝硬化腹水治疗,一般不主张采用(2.5分) A.高蛋白饮食
B.低盐饮食
C.卧床休息
D.强烈利尿
E.腹水浓缩回输
D.强烈利尿
6 2 2012 31. 肝硬化患者出现消化道出血时,下列可诱发肝性脑病的是(2.5分) A.输生理盐水
B.水电解质紊乱致低钾血症
C.静脉注射止血药
D.内镜下止血
E.口服去甲肾上腺素
B.水电解质紊乱致低钾血症
6 2 2012 32. 有关肝性脑病治疗的错误方法是(2.5分) A.生理盐水灌肠
B.弱酸溶液灌肠
C.肥皂水灌肠
D.乳果糖灌肠
E.硫酸镁导泻
C.肥皂水灌肠
6 2 2012 33. Which marker has the highest sensitivity and specificity for hepatocellular carcinoma? ()(2.5分) A.AFP
B.CEA
C.CA125
D.CA199
E.PSA
A.AFP
6 2 2012 34. 应用三腔管压迫止血,哪项不正确(2.5分) A.先向胃气囊充气150-200ml
B.管端悬以重量约1.0kg的物品,作牵引压迫
C.若仍有出血,再向食管气囊注气100-150ml
D.放置三腔管的时间不宜持续超过3-5天
E.每隔12小时,应将气囊放空10-20分钟
B.管端悬以重量约1.0kg的物品,作牵引压迫
6 2 2012 35. 门脉高压症的主要临床表现:(2.5分) A.腹胀食欲减退
B.食管胃底静脉伴出血
C.血白细胞和血小板减少
D.肝肿大
E.肝功能减退
B.食管胃底静脉伴出血
6 2 2012 36. A 78-year-old woman undergoes successful endoscopic stenting of the common bile duct for obstructive jaundice secondary to an cholangiocarcinoma. Two weeks later, she consults her physician because of a fever of 39°C, general malaise, nausea, and right upper quadrant (RUQ) discomfort. On physical examination, jaundice and RUQ tenderness are noted. Laboratory test results show leukocytosis, anemia, and an elevated serum bilirubin level. Chest x-ray shows no acute infiltrate, but the right diaphragm is elevated. What is the most likely diagnosis?(2.5分) A.Cholangitis
B.Liver abscess
C.Acute calculous cholecystitis
D.Liver metastasis
E.Pneumonia
B.Liver abscess
6 2 2012 37. A 35-year-old man presents with right upper quadrant pain. He had travelled to India 6 months ago. His WBC and ALP are raised with a normal serum bilirubin. Serology for antibodies to Entamoeba histolytica is positive. Abdominal ultrasonography showed a solitary abscess measuring 8 cm in diameter in the right lobe of the liver. Which is preferred therapy?(2.5分) A.Metronidazole alone
B.Percutaneous drainage
C.Resection
D.Deroofing
E.Observation alone
A.Metronidazole alone
6 2 2012 38. 下列哪种说法不符合阿米巴性肝脓肿的常见表现:(2.5分) A.有慢性腹泻病史
B.病灶较大,多为单发,好发于肝左叶
C.白细胞计数可增加
D.脓液呈巧克力色,细菌培养常为阴性
E.甲硝唑单药治疗有效
B.病灶较大,多为单发,好发于肝左叶
6 2 2012 39. 下列哪种疾病与肝脓肿发生的相关性最小:(2.5分) A.伴有反复胆管炎的肝内胆管结石
B.急性化脓性阑尾炎
C.亚急性细菌性心内膜炎
D.糖尿病
E.肝囊肿
E.肝囊肿
6 2 2012 40. 下列哪一项不是典型肝脓肿在CT图像上的表现?(2.5分) A.平扫示低密度占位,中心区CT值略高于水
B.多为圆或椭圆形,部分腔内有隔
C.多数病灶边缘不清楚
D.脓肿周围出现不同密度环征
E.增强扫描脓肿壁无强化
E.增强扫描脓肿壁无强化
6 2 2013 1. 关于肝细胞气球样变细胞中下列哪一项改变是错误的 (2.5分) A.细胞内糖原增多
B.细胞内水分增多
C.内质网扩张、囊泡变性
D.线立体肿胀、嵴消失
E.粗面内质网核蛋白体颗粒消失
A.细胞内糖原增多
6 2 2013 2. 引起门脉高压的原因一般不包括 (2.5分) A.假小叶压迫小叶下静脉
B.纤维组织压迫中央静脉
C.肝窦狭窄、闭合
D.肝动脉和门静脉之间形成吻合支
E.肝动脉与肝静脉之间形成吻合支
E.肝动脉与肝静脉之间形成吻合支
6 2 2013 3. 肝细胞癌患者最有诊断意义的血清学指标是 (2.5分) A.甲胎蛋白
B.癌胚抗原
C.碱性磷酸酶
D.酸性磷酸酶
E.γ-谷氨酰转肽酶
A.甲胎蛋白
6 2 2013 4. 急性重型肝炎的病变性质是 (2.5分) A.以肝细胞变性为主的炎症
B.以肝细胞坏死为主的炎症
C.以肝细胞增生为主的炎症
D.以汇管区渗出为主的炎症
E.以汇管区增生为主的炎症
B.以肝细胞坏死为主的炎症
6 2 2013 5. 肝硬化腹水生成的机制中,不正确的说法是 (2.5分) A.肝脏合成清蛋白减少
B.血内抗利尿激素、醛固酮增高
C.结缔组织增生、肝静脉受压
D.门静脉淤血、血管壁通透性增强
E.肝窦内压力升高
C.结缔组织增生、肝静脉受压
6 2 2013 6. 不符合病毒性肝炎基本病变的是 (2.5分) A.肝细胞变性坏死
B.慢性炎细胞浸润
C.纤维组织增生
D.Kupffer细胞异型增生
E.小胆管增生
D.Kupffer细胞异型增生
6 2 2013 7. 桥接坏死常见于 (2.5分) A.急性重型肝炎
B.慢性轻度肝炎
C.慢性重度肝炎
D.急性普通型肝炎
E.亚急性重型肝炎
C.慢性重度肝炎
6 2 2013 8. 下列哪项对诊断门静脉高压最有特征性? (2.5分) A.脾大
B.腹水
C.侧支循环的建立及开放
D.蜘蛛痣
E.脾功能亢进表现
C.侧支循环的建立及开放
6 2 2013 9. 下列关于肝脏的功能,不正确的是: (2.5分) A.分泌功能
B.代谢功能
C.解毒作用
D.合成功能
E.氧合功能
E.氧合功能
6 2 2013 10. 女性,57岁,CT示肝左外叶8cm占位,无血管侵犯,肝肾功能正常,首选治疗方案:(2.5分) A.手术切除
B.经肝动脉化学治疗和栓塞
C.全身化学治疗和中医治疗
D.肝肿瘤局部无水酒精注射
E.生物治疗
A.手术切除
6 2 2013 11. 一名49岁男性,因右季肋部疼痛半月余,加重一天入院,以往健康,检查时发现血HBsAg阳性,AFP500ug/L。下面哪一种辅助检查不利于明确诊断:(2.5分) A.B超
B.X片
C.CT
D.磁共振
E.PET-CT
B.X片
6 2 2013 12. 原发性肝癌的主要并发症不包括:(2.5分) A.肝性脑病
B.癌肿破裂出血
C.上消化道出血
D.低血糖症
E.继发感染
D.低血糖症
6 2 2013 13. 上消化道出血的首选诊断方法是:(2.5分) A.鼻胃管或三腔管检查
B.早期内镜检查
C.选择性腹腔动脉或肠系膜上动脉造影
D.X线钡餐检查
E.核素检查
B.早期内镜检查
6 2 2013 14. 肝硬化患者出现血性腹水,但无腹痛及发烧。首先考虑可能并发:(2.5分) A.原发性腹膜炎
B.原发性肝癌
C.结核性腹膜炎
D.肝肾综合征
E.门静脉血栓形成
B.原发性肝癌
6 2 2013 15. 目前肝癌早期宜采用哪一种治疗为主: (2.5分) A.放射疗法
B.化学疗法
C.手术切除
D.中医中药
E.中医中药
C.手术切除
6 2 2013 16. 肝癌血行肝外转移最多见于:(2.5分) A.肺
B.胰
C.脑
D.肾
E.脑
A.肺
6 2 2013 17. 对疑有早期原发性肝癌的患者,应首先用哪一种方法检查较好:(2.5分) A.同位素肝扫描
B.血清甲胎蛋白动态观察检查
C.肝区超声波检查
D.血清γˉ谷氨酰转肽酶
E.选择性肝动脉造影
B.血清甲胎蛋白动态观察检查
6 2 2013 18. 肝移植的适应症不包括:(2.5分) A.急性肝衰竭
B.重型肝炎
C.先天性胆道闭锁
D.肝硬化失代偿合并小肝癌
E.肝脏巨大血管瘤
E.肝脏巨大血管瘤
6 2 2013 19. 门脉高压症发生消化道出血的主要原因是:(2.5分) A.痔静脉曲张
B.脾肿大破裂
C.腹壁静脉曲张
D.食管下段和胃底静脉曲张
D.食管下段和胃底静脉曲张
6 2 2013 20. Couinaud分段法将肝脏分为8段,其中尾状叶在:(2.5分) A.Ⅰ段
B.Ⅱ段
C.Ⅲ段
D.Ⅴ段
E.Ⅷ段
A.Ⅰ段
6 2 2013 21. Couinaud肝脏分段法是根据以下哪项解剖结构为基础: (2.5分) A.肝动脉、门静脉
B.门静脉、胆道
C.肝静脉、胆道
D.肝静脉、门静脉
E.Glisson纤维鞘
D.肝静脉、门静脉
6 2 2013 22. 肝癌最主要的转移途径是:(2.5分) A.直接侵犯
B.淋巴转移
C.血行转移
D.腹腔内种植
E.肝内转移
E.肝内转移
6 2 2013 23. 下列哪项不是肝硬变腹水形成机制: (2.5分) A.低蛋白血症
B.门静脉压力增高
C.肝内淋巴的产生增多
D.肝静脉阻力增高
E.醛固酮分泌减少
D.肝静脉阻力增高
6 2 2013 24. The major reasons for liver cirrhosis in China and Western countries are (2.5分) A.HBV and nonalcoholic fatty liver disease
B.HBV and HCV
C.HCV and nonalcoholic fatty liver disease
D.HCV and HBV
E.nonalcoholic fatty liver disease and HCV
A.HBV and nonalcoholic fatty liver disease
6 2 2013 25. Which marker has the highest sensitivity and specificity for hepatocellular carcinoma?(2.5分) A.AFP
B.CEA
C.CA125
D.CA199
E.PSA
A.AFP
6 2 2013 26. 下列不属于兴奋性神经递质的是(2.5分) A.γ-氨基丁酸
B.去甲肾上腺素
C.乙酰胆碱
D.门冬氨酸
E.多巴胺
A.γ-氨基丁酸
6 2 2013 27. 肝硬化腹水形成的原因不包括(2.5分) A.门静脉压力增高
B.血浆胶体渗透压降低
C.肝脏淋巴液失平衡
D.甲状腺功能减退
E.低蛋白血症
D.甲状腺功能减退
6 2 2013 28. 自发性细菌性腹膜炎常见致病菌应是 (2.5分) A.大肠杆菌和副大肠杆菌
B.溶血链球菌和肺炎双球菌
C.厌氧菌和链球菌
D.大肠杆菌和厌氧菌
E.绿脓杆菌和葡萄球菌
B.溶血链球菌和肺炎双球菌
6 2 2013 29. 治疗肝硬化并上消化道出血(2.5分) A.休息,低盐饮食,限制入量,补充蛋白,给利尿剂
B.休息,高热量高蛋白饮食,护肝治疗
C.休息,高热量限制蛋白饮食,输入支链氨基酸
D.休息,禁食,积极补足血容量及采用各种止血措施
E.休息,控制输液量,纠正电解质紊乱,限制蛋白质
D.休息,禁食,积极补足血容量及采用各种止血措施
6 2 2013 30. 肝硬化患者两个月前发现腹水,昨天突然出现剧烈腹痛、伴呕吐、发热、腹水迅速增加,并排出少量暗红色血便,最可能出现哪种情况(2.5分) A.食管静脉曲张破裂出血
B.急性胰腺炎
C.合并肝癌破裂
D.门静脉血栓形成
E.合并脾破裂出血
D.门静脉血栓形成
6 2 2013 31. 肝肾综合征主要与下列哪项因素有关(2.5分) A.肾小管坏死
B.肾小球坏死
C.肾皮质血流量减少
D.肾间质炎性病变
E.胆红素对肾脏的毒性
C.肾皮质血流量减少
6 2 2013 32. 肝硬化患者有出血倾向,与下列哪项因素关系最小(2.5分) A.肝合成凝血因子减少
B.低白蛋白血症
C.毛细血管脆性增加
D.脾功能亢进
E.胃肠道黏膜炎症糜烂
B.低白蛋白血症
6 2 2013 33. 男性,38岁,患肝硬化3年,一周来畏寒发热,体温38℃左右,全腹痛,腹部明显膨胀,尿量500ml/d。住院后经检查有以下体征,对目前病情判断最有意义的是(2.5分) A.蜘蛛痣及肝掌
B.腹壁静脉曲张呈海蛇头样
C.脾肿大
D.全腹压痛及反跳痛
E.腹部移动性浊音阳性
D.全腹压痛及反跳痛
6 2 2013 34. 应用三腔管压迫止血,哪项不正确(2.5分) A.先向胃气囊充气150-200ml
B.管端悬以重量约1.0kg的物品,作牵引压迫
C.若仍有出血,再向食管气囊注气100-150ml
D.放置三腔管的时间不宜持续超过3-5天
E.每隔12小时,应将气囊放空10-20分钟
B.管端悬以重量约1.0kg的物品,作牵引压迫
6 2 2013 35. 门脉高压症的主要临床表现:(2.5分) A.腹胀食欲减退
B.食管胃底静脉伴出血
C.血白细胞和血小板减少
D.肝肿大
E.肝功能减退
B.食管胃底静脉伴出血
6 2 2013 36. A 78-year-old woman undergoes successful endoscopic stenting of the common bile duct for obstructive jaundice secondary to an cholangiocarcinoma. Two weeks later, she consults her physician because of a fever of 39°C, general malaise, nausea, and right upper quadrant (RUQ) discomfort. On physical examination, jaundice and RUQ tenderness are noted. Laboratory test results show leukocytosis, anemia, and an elevated serum bilirubin level. Chest x-ray shows no acute infiltrate, but the right diaphragm is elevated. What is the most likely diagnosis?(2.5分) A.Cholangitis
B.Liver abscess
C.Acute calculous cholecystitis
D.Liver metastasis
E.Pneumonia
B.Liver abscess
6 2 2013 37. A 35-year-old man presents with right upper quadrant pain. He had travelled to India 6 months ago. His WBC and ALP are raised with a normal serum bilirubin. Serology for antibodies to Entamoeba histolytica is positive. Abdominal ultrasonography showed a solitary abscess measuring 8 cm in diameter in the right lobe of the liver. Which is preferred therapy?(2.5分) A.Metronidazole alone
B.Percutaneous drainage
C.Resection
D.Deroofing
E.Observation alone
A.Metronidazole alone
6 2 2013 38. 下列哪种说法不符合阿米巴性肝脓肿的常见表现:(2.5分) A.有慢性腹泻病史
B.病灶较大,多为单发,好发于肝左叶
C.白细胞计数可增加
D.脓液呈巧克力色,细菌培养常为阴性
E.甲硝唑单药治疗有效
B.病灶较大,多为单发,好发于肝左叶
6 2 2013 39. 下列哪种疾病与肝脓肿发生的相关性最小:(2.5分) A.伴有反复胆管炎的肝内胆管结石
B.急性化脓性阑尾炎
C.亚急性细菌性心内膜炎
D.糖尿病
E.肝囊肿
E.肝囊肿
6 2 2013 40. 下列哪一项不是典型肝脓肿在CT图像上的表现?(2.5分) A.平扫示低密度占位,中心区CT值略高于水
B.多为圆或椭圆形,部分腔内有隔
C.多数病灶边缘不清楚
D.脓肿周围出现不同密度环征
E.增强扫描脓肿壁无强化
E.增强扫描脓肿壁无强化
6 2 2014 1. 关于肝细胞气球样变细胞中下列哪一项改变是错误的 (2.5分) A.细胞内糖原增多
B.细胞内水分增多
C.内质网扩张、囊泡变性
D.线立体肿胀、嵴消失
E.粗面内质网核蛋白体颗粒消失
A.细胞内糖原增多
6 2 2014 2. 引起门脉高压的原因一般不包括 (2.5分) A.假小叶压迫小叶下静脉
B.纤维组织压迫中央静脉
C.肝窦狭窄、闭合
D.肝动脉和门静脉之间形成吻合支
E.肝动脉与肝静脉之间形成吻合支
E.肝动脉与肝静脉之间形成吻合支
6 2 2014 3. 肝细胞癌患者最有诊断意义的血清学指标是 (2.5分) A.甲胎蛋白
B.癌胚抗原
C.碱性磷酸酶
D.酸性磷酸酶
E.γ-谷氨酰转肽酶
A.甲胎蛋白
6 2 2014 4. 急性重型肝炎的病变性质是 (2.5分) A.以肝细胞变性为主的炎症
B.以肝细胞坏死为主的炎症
C.以肝细胞增生为主的炎症
D.以汇管区渗出为主的炎症
E.以汇管区增生为主的炎症
B.以肝细胞坏死为主的炎症
6 2 2014 5. 肝硬化腹水生成的机制中,不正确的说法是 (2.5分) A.肝脏合成清蛋白减少
B.血内抗利尿激素、醛固酮增高
C.结缔组织增生、肝静脉受压
D.门静脉淤血、血管壁通透性增强
E.肝窦内压力升高
C.结缔组织增生、肝静脉受压
6 2 2014 6. 不符合病毒性肝炎基本病变的是 (2.5分) A.肝细胞变性坏死
B.慢性炎细胞浸润
C.纤维组织增生
D.Kupffer细胞异型增生
E.小胆管增生
D.Kupffer细胞异型增生
6 2 2014 7. 桥接坏死常见于 (2.5分) A.急性重型肝炎
B.慢性轻度肝炎
C.慢性重度肝炎
D.急性普通型肝炎
E.亚急性重型肝炎
C.慢性重度肝炎
6 2 2014 8. 下列哪项对诊断门静脉高压最有特征性? (2.5分) A.脾大
B.腹水
C.侧支循环的建立及开放
D.蜘蛛痣
E.脾功能亢进表现
C.侧支循环的建立及开放
6 2 2014 9. 下列关于肝脏的功能,不正确的是: (2.5分) A.分泌功能
B.代谢功能
C.解毒作用
D.合成功能
E.氧合功能
E.氧合功能
6 2 2014 10. 女性,57岁,CT示肝左外叶8cm占位,无血管侵犯,肝肾功能正常,首选治疗方案:(2.5分) A.手术切除
B.经肝动脉化学治疗和栓塞
C.全身化学治疗和中医治疗
D.肝肿瘤局部无水酒精注射
E.生物治疗
A.手术切除
6 2 2014 11. 一名49岁男性,因右季肋部疼痛半月余,加重一天入院,以往健康,检查时发现血HBsAg阳性,AFP500ug/L。下面哪一种辅助检查不利于明确诊断:(2.5分) A.B超
B.X片
C.CT
D.磁共振
E.PET-CT
B.X片
6 2 2014 12. 原发性肝癌的主要并发症不包括:(2.5分) A.肝性脑病
B.癌肿破裂出血
C.上消化道出血
D.低血糖症
E.继发感染
D.低血糖症
6 2 2014 13. 上消化道出血的首选诊断方法是:(2.5分) A.鼻胃管或三腔管检查
B.早期内镜检查
C.选择性腹腔动脉或肠系膜上动脉造影
D.X线钡餐检查
E.核素检查
B.早期内镜检查
6 2 2014 14. 肝硬化患者出现血性腹水,但无腹痛及发烧。首先考虑可能并发:(2.5分) A.原发性腹膜炎
B.原发性肝癌
C.结核性腹膜炎
D.肝肾综合征
E.门静脉血栓形成
B.原发性肝癌
6 2 2014 15. 目前肝癌早期宜采用哪一种治疗为主: (2.5分) A.放射疗法
B.化学疗法
C.手术切除
D.中医中药
E.中医中药
C.手术切除
6 2 2014 16. 肝癌血行肝外转移最多见于:(2.5分) A.肺
B.胰
C.脑
D.肾
E.脑
A.肺
6 2 2014 17. 对疑有早期原发性肝癌的患者,应首先用哪一种方法检查较好:(2.5分) A.同位素肝扫描
B.血清甲胎蛋白动态观察检查
C.肝区超声波检查
D.血清γˉ谷氨酰转肽酶
E.选择性肝动脉造影
B.血清甲胎蛋白动态观察检查
6 2 2014 18. 肝移植的适应症不包括:(2.5分) A.急性肝衰竭
B.重型肝炎
C.先天性胆道闭锁
D.肝硬化失代偿合并小肝癌
E.肝脏巨大血管瘤
E.肝脏巨大血管瘤
6 2 2014 19. 门脉高压症发生消化道出血的主要原因是:(2.5分) A.痔静脉曲张
B.脾肿大破裂
C.腹壁静脉曲张
D.食管下段和胃底静脉曲张
D.食管下段和胃底静脉曲张
6 2 2014 20. Couinaud分段法将肝脏分为8段,其中尾状叶在:(2.5分) A.Ⅰ段
B.Ⅱ段
C.Ⅲ段
D.Ⅴ段
E.Ⅷ段
A.Ⅰ段
6 2 2014 21. Couinaud肝脏分段法是根据以下哪项解剖结构为基础: (2.5分) A.肝动脉、门静脉
B.门静脉、胆道
C.肝静脉、胆道
D.肝静脉、门静脉
E.Glisson纤维鞘
D.肝静脉、门静脉
6 2 2014 22. 肝癌最主要的转移途径是:(2.5分) A.直接侵犯
B.淋巴转移
C.血行转移
D.腹腔内种植
E.肝内转移
E.肝内转移
6 2 2014 23. 下列哪项不是肝硬变腹水形成机制: (2.5分) A.低蛋白血症
B.门静脉压力增高
C.肝内淋巴的产生增多
D.肝静脉阻力增高
E.醛固酮分泌减少
D.肝静脉阻力增高
6 2 2014 24. The major reasons for liver cirrhosis in China and Western countries are(2.5分) A.HBV and nonalcoholic fatty liver disease
B.HBV and HCV
C.HCV and nonalcoholic fatty liver disease
D.HCV and HBV
E.nonalcoholic fatty liver disease and HCV
A.HBV and nonalcoholic fatty liver disease
6 2 2014 25. Which marker has the highest sensitivity and specificity for hepatocellular carcinoma? (2.5分) A.AFP
B.CEA
C.CA125
D.CA199
E.PSA
A.AFP
6 2 2014 26. 下列不属于兴奋性神经递质的是(2.5分) A.γ-氨基丁酸
B.去甲肾上腺素
C.乙酰胆碱
D.门冬氨酸
E.多巴胺
A.γ-氨基丁酸
6 2 2014 27. 肝硬化腹水形成的原因不包括(2.5分) A.门静脉压力增高
B.血浆胶体渗透压降低
C.肝脏淋巴液失平衡
D.甲状腺功能减退
E.低蛋白血症
D.甲状腺功能减退
6 2 2014 28. 自发性细菌性腹膜炎常见致病菌应是 (2.5分) A.大肠杆菌和副大肠杆菌
B.溶血链球菌和肺炎双球菌
C.厌氧菌和链球菌
D.大肠杆菌和厌氧菌
E.绿脓杆菌和葡萄球菌
B.溶血链球菌和肺炎双球菌
6 2 2014 29. 治疗肝硬化并上消化道出血(2.5分) A.休息,低盐饮食,限制入量,补充蛋白,给利尿剂
B.休息,高热量高蛋白饮食,护肝治疗
C.休息,高热量限制蛋白饮食,输入支链氨基酸
D.休息,禁食,积极补足血容量及采用各种止血措施
E.休息,控制输液量,纠正电解质紊乱,限制蛋白质
D.休息,禁食,积极补足血容量及采用各种止血措施
6 2 2014 30. 肝硬化患者两个月前发现腹水,昨天突然出现剧烈腹痛、伴呕吐、发热、腹水迅速增加,并排出少量暗红色血便,最可能出现哪种情况 (2.5分) A.食管静脉曲张破裂出血
B.急性胰腺炎
C.合并肝癌破裂
D.门静脉血栓形成
E.合并脾破裂出血
D.门静脉血栓形成
6 2 2014 31. 肝肾综合征主要与下列哪项因素有关(2.5分) A.肾小管坏死
B.肾小球坏死
C.肾皮质血流量减少
D.肾间质炎性病变
E.胆红素对肾脏的毒性
C.肾皮质血流量减少
6 2 2014 32. 肝硬化患者有出血倾向,与下列哪项因素关系最小(2.5分) A.肝合成凝血因子减少
B.低白蛋白血症
C.毛细血管脆性增加
D.脾功能亢进
E.胃肠道黏膜炎症糜烂
B.低白蛋白血症
6 2 2014 33. 男性,38岁,患肝硬化3年,一周来畏寒发热,体温38℃左右,全腹痛,腹部明显膨胀,尿量500ml/d。住院后经检查有以下体征,对目前病情判断最有意义的是 (2.5分) A.蜘蛛痣及肝掌
B.腹壁静脉曲张呈海蛇头样
C.脾肿大
D.全腹压痛及反跳痛
E.腹部移动性浊音阳性
D.全腹压痛及反跳痛
6 2 2014 34. 应用三腔管压迫止血,哪项不正确(2.5分) A.先向胃气囊充气150-200ml
B.管端悬以重量约1.0kg的物品,作牵引压迫
C.若仍有出血,再向食管气囊注气100-150ml
D.放置三腔管的时间不宜持续超过3-5天
E.每隔12小时,应将气囊放空10-20分钟
B.管端悬以重量约1.0kg的物品,作牵引压迫
6 2 2014 35. 门脉高压症的主要临床表现:(2.5分) A.腹胀食欲减退
B.食管胃底静脉伴出血
C.血白细胞和血小板减少
D.肝肿大
E.肝功能减退
B.食管胃底静脉伴出血
6 2 2014 36. A 78-year-old woman undergoes successful endoscopic stenting of the common bile duct for obstructive jaundice secondary to an cholangiocarcinoma. Two weeks later, she consults her physician because of a fever of 39°C, general malaise, nausea, and right upper quadrant (RUQ) discomfort. On physical examination, jaundice and RUQ tenderness are noted. Laboratory test results show leukocytosis, anemia, and an elevated serum bilirubin level. Chest x-ray shows no acute infiltrate, but the right diaphragm is elevated. What is the most likely diagnosis?(2.5分) A.Cholangitis
B.Liver abscess
C.Acute calculous cholecystitis
D.Liver metastasis
E.Pneumonia
B.Liver abscess
6 2 2014 37. A 35-year-old man presents with right upper quadrant pain. He had travelled to India 6 months ago. His WBC and ALP are raised with a normal serum bilirubin. Serology for antibodies to Entamoeba histolytica is positive. Abdominal ultrasonography showed a solitary abscess measuring 8 cm in diameter in the right lobe of the liver. Which is preferred therapy?(2.5分) A.Metronidazole alone
B.Percutaneous drainage
C.Resection
D.Deroofing
E.Observation alone
A.Metronidazole alone
6 2 2014 38. 下列哪种说法不符合阿米巴性肝脓肿的常见表现:(2.5分) A.有慢性腹泻病史
B.病灶较大,多为单发,好发于肝左叶
C.白细胞计数可增加
D.脓液呈巧克力色,细菌培养常为阴性
E.甲硝唑单药治疗有效
B.病灶较大,多为单发,好发于肝左叶
6 2 2014 39. 下列哪种疾病与肝脓肿发生的相关性最小:(2.5分) A.伴有反复胆管炎的肝内胆管结石
B.急性化脓性阑尾炎
C.亚急性细菌性心内膜炎
D.糖尿病
E.肝囊肿
E.肝囊肿
6 2 2014 40. 下列哪一项不是典型肝脓肿在CT图像上的表现?(2.5分) A.平扫示低密度占位,中心区CT值略高于水
B.多为圆或椭圆形,部分腔内有隔
C.多数病灶边缘不清楚
D.脓肿周围出现不同密度环征
E.增强扫描脓肿壁无强化
E.增强扫描脓肿壁无强化
6 2 2015 1. 关于肝细胞气球样变细胞中下列哪一项改变是错误的 (2.5分) A.细胞内糖原增多
B.细胞内水分增多
C.内质网扩张、囊泡变性
D.线立体肿胀、嵴消失
E.粗面内质网核蛋白体颗粒消失
A.细胞内糖原增多
6 2 2015 2. 引起门脉高压的原因一般不包括 (2.5分) A.假小叶压迫小叶下静脉
B.纤维组织压迫中央静脉
C.肝窦狭窄、闭合
D.肝动脉和门静脉之间形成吻合支
E.肝动脉与肝静脉之间形成吻合支
E.肝动脉与肝静脉之间形成吻合支
6 2 2015 3. 肝细胞癌患者最有诊断意义的血清学指标是 (2.5分) A.甲胎蛋白
B.癌胚抗原
C.碱性磷酸酶
D.酸性磷酸酶
E.γ-谷氨酰转肽酶
A.甲胎蛋白
6 2 2015 4. 急性重型肝炎的病变性质是 (2.5分) A.以肝细胞变性为主的炎症
B.以肝细胞坏死为主的炎症
C.以肝细胞增生为主的炎症
D.以汇管区渗出为主的炎症
E.以汇管区增生为主的炎症
B.以肝细胞坏死为主的炎症
6 2 2015 5. 肝硬化腹水生成的机制中,不正确的说法是 (2.5分) A.肝脏合成清蛋白减少
B.血内抗利尿激素、醛固酮增高
C.结缔组织增生、肝静脉受压
D.门静脉淤血、血管壁通透性增强
E.肝窦内压力升高
C.结缔组织增生、肝静脉受压
6 2 2015 6. 不符合病毒性肝炎基本病变的是 (2.5分) A.肝细胞变性坏死
B.慢性炎细胞浸润
C.纤维组织增生
D.Kupffer细胞异型增生
E.小胆管增生
D.Kupffer细胞异型增生
6 2 2015 7. 桥接坏死常见于 (2.5分) A.急性重型肝炎
B.慢性轻度肝炎
C.慢性重度肝炎
D.急性普通型肝炎
E.亚急性重型肝炎
C.慢性重度肝炎
6 2 2015 8. 下列哪项对诊断门静脉高压最有特征性? (2.5分) A.脾大
B.腹水
C.侧支循环的建立及开放
D.蜘蛛痣
E.脾功能亢进表现
C.侧支循环的建立及开放
6 2 2015 9. 下列关于肝脏的功能,不正确的是: (2.5分) A.分泌功能
B.代谢功能
C.解毒作用
D.合成功能
E.氧合功能
E.氧合功能
6 2 2015 10. 女性,57岁,CT示肝左外叶8cm占位,无血管侵犯,肝肾功能正常,首选治疗方案:(2.5分) A.手术切除
B.经肝动脉化学治疗和栓塞
C.全身化学治疗和中医治疗
D.肝肿瘤局部无水酒精注射
E.生物治疗
A.手术切除
6 2 2015 11. 一名49岁男性,因右季肋部疼痛半月余,加重一天入院,以往健康,检查时发现血HBsAg阳性,AFP500ug/L。下面哪一种辅助检查不利于明确诊断:(2.5分) A.B超
B.X片
C.CT
D.磁共振
E.PET-CT
B.X片
6 2 2015 12. 原发性肝癌的主要并发症不包括:(2.5分) A.肝性脑病
B.癌肿破裂出血
C.上消化道出血
D.低血糖症
E.继发感染
D.低血糖症
6 2 2015 13. 上消化道出血的首选诊断方法是:(2.5分) A.鼻胃管或三腔管检查
B.早期内镜检查
C.选择性腹腔动脉或肠系膜上动脉造影
D.X线钡餐检查
E.核素检查
B.早期内镜检查
6 2 2015 14. 肝硬化患者出现血性腹水,但无腹痛及发烧。首先考虑可能并发:(2.5分) A.原发性腹膜炎
B.原发性肝癌
C.结核性腹膜炎
D.肝肾综合征
E.门静脉血栓形成
B.原发性肝癌
6 2 2015 15. 目前肝癌早期宜采用哪一种治疗为主: (2.5分) A.放射疗法
B.化学疗法
C.手术切除
D.中医中药
E.中医中药
C.手术切除
6 2 2015 16. 肝癌血行肝外转移最多见于:(2.5分) A.肺
B.胰
C.脑
D.肾
E.脑
A.肺
6 2 2015 17. 对疑有早期原发性肝癌的患者,应首先用哪一种方法检查较好:(2.5分) A.同位素肝扫描
B.血清甲胎蛋白动态观察检查
C.肝区超声波检查
D.血清γˉ谷氨酰转肽酶
E.选择性肝动脉造影
B.血清甲胎蛋白动态观察检查
6 2 2015 18. 肝移植的适应症不包括:(2.5分) A.急性肝衰竭
B.重型肝炎
C.先天性胆道闭锁
D.肝硬化失代偿合并小肝癌
E.肝脏巨大血管瘤
E.肝脏巨大血管瘤
6 2 2015 19. 门脉高压症发生消化道出血的主要原因是:(2.5分) A.痔静脉曲张
B.脾肿大破裂
C.腹壁静脉曲张
D.食管下段和胃底静脉曲张
D.食管下段和胃底静脉曲张
6 2 2015 20. Couinaud分段法将肝脏分为8段,其中尾状叶在:(2.5分) A.Ⅰ段
B.Ⅱ段
C.Ⅲ段
D.Ⅴ段
E.Ⅷ段
A.Ⅰ段
6 2 2015 21. Couinaud肝脏分段法是根据以下哪项解剖结构为基础: (2.5分) A.肝动脉、门静脉
B.门静脉、胆道
C.肝静脉、胆道
D.肝静脉、门静脉
E.Glisson纤维鞘
D.肝静脉、门静脉
6 2 2015 22. 肝癌最主要的转移途径是:(2.5分) A.直接侵犯
B.淋巴转移
C.血行转移
D.腹腔内种植
E.肝内转移
E.肝内转移
6 2 2015 23. 下列哪项不是肝硬变腹水形成机制: (2.5分) A.低蛋白血症
B.门静脉压力增高
C.肝内淋巴的产生增多
D.肝静脉阻力增高
E.醛固酮分泌减少
D.肝静脉阻力增高
6 2 2015 24. The major reasons for liver cirrhosis in China and Western countries are (2.5分) A.HBV and nonalcoholic fatty liver disease
B.HBV and HCV
C.HCV and nonalcoholic fatty liver disease
D.HCV and HBV
E.nonalcoholic fatty liver disease and HCV
A.HBV and nonalcoholic fatty liver disease
6 2 2015 25. Which marker has the highest sensitivity and specificity for hepatocellular carcinoma? (2.5分) A.AFP
B.CEA
C.CA125
D.CA199
E.PSA
A.AFP
6 2 2015 26. 下列不属于兴奋性神经递质的是(2.5分) A.γ-氨基丁酸
B.去甲肾上腺素
C.乙酰胆碱
D.门冬氨酸
E.多巴胺
A.γ-氨基丁酸
6 2 2015 27. 肝硬化腹水形成的原因不包括(2.5分) A.门静脉压力增高
B.血浆胶体渗透压降低
C.肝脏淋巴液失平衡
D.甲状腺功能减退
E.低蛋白血症
D.甲状腺功能减退
6 2 2015 28. 自发性细菌性腹膜炎常见致病菌应是 (2.5分) A.大肠杆菌和副大肠杆菌
B.溶血链球菌和肺炎双球菌
C.厌氧菌和链球菌
D.大肠杆菌和厌氧菌
E.绿脓杆菌和葡萄球菌
B.溶血链球菌和肺炎双球菌
6 2 2015 29. 治疗肝硬化并上消化道出血(2.5分) A.休息,低盐饮食,限制入量,补充蛋白,给利尿剂
B.休息,高热量高蛋白饮食,护肝治疗
C.休息,高热量限制蛋白饮食,输入支链氨基酸
D.休息,禁食,积极补足血容量及采用各种止血措施
E.休息,控制输液量,纠正电解质紊乱,限制蛋白质
D.休息,禁食,积极补足血容量及采用各种止血措施
6 2 2015 30. 肝硬化患者两个月前发现腹水,昨天突然出现剧烈腹痛、伴呕吐、发热、腹水迅速增加,并排出少量暗红色血便,最可能出现哪种情况 (2.5分) A.食管静脉曲张破裂出血
B.急性胰腺炎
C.合并肝癌破裂
D.门静脉血栓形成
E.合并脾破裂出血
D.门静脉血栓形成
6 2 2015 31. 查体于肿大的肝脏表面听到血管杂音时,应考虑(2.5分) A.布加综合征
B.门脉血栓形成
C.肝硬化失代偿期
D.原发性肝癌
E.慢性肝炎
D.原发性肝癌
6 2 2015 32. 肝硬化腹水治疗,一般不主张采用(2.5分) A.高蛋白饮食
B.低盐饮食
C.卧床休息
D.强烈利尿
E.腹水浓缩回输
D.强烈利尿
6 2 2015 33. 男性,38岁,患肝硬化3年,一周来畏寒发热,体温38℃左右,全腹痛,腹部明显膨胀,尿量500ml/d。住院后经检查有以下体征,对目前病情判断最有意义的是(2.5分) A.蜘蛛痣及肝掌
B.腹壁静脉曲张呈海蛇头样
C.脾肿大
D.全腹压痛及反跳痛
E.腹部移动性浊音阳性
D.全腹压痛及反跳痛
6 2 2015 34. 应用三腔管压迫止血,哪项不正确(2.5分) A.先向胃气囊充气150-200ml
B.管端悬以重量约1.0kg的物品,作牵引压迫
C.若仍有出血,再向食管气囊注气100-150ml
D.放置三腔管的时间不宜持续超过3-5天
E.每隔12小时,应将气囊放空10-20分钟
B.管端悬以重量约1.0kg的物品,作牵引压迫
6 2 2015 35. 门脉高压症的主要临床表现:(2.5分) A.腹胀食欲减退
B.食管胃底静脉伴出血
C.血白细胞和血小板减少
D.肝肿大
E.肝功能减退
B.食管胃底静脉伴出血
6 2 2015 36. A 78-year-old woman undergoes successful endoscopic stenting of the common bile duct for obstructive jaundice secondary to an cholangiocarcinoma. Two weeks later, she consults her physician because of a fever of 39°C, general malaise, nausea, and right upper quadrant (RUQ) discomfort. On physical examination, jaundice and RUQ tenderness are noted. Laboratory test results show leukocytosis, anemia, and an elevated serum bilirubin level. Chest x-ray shows no acute infiltrate, but the right diaphragm is elevated. What is the most likely diagnosis?(2.5分) A.Cholangitis
B.Liver abscess
C.Acute calculous cholecystitis
D.Liver metastasis
E.Pneumonia
B.Liver abscess
6 2 2015 37. A 35-year-old man presents with right upper quadrant pain. He had travelled to India 6 months ago. His WBC and ALP are raised with a normal serum bilirubin. Serology for antibodies to Entamoeba histolytica is positive. Abdominal ultrasonography showed a solitary abscess measuring 8 cm in diameter in the right lobe of the liver. Which is preferred therapy?(2.5分) A.Metronidazole alone
B.Percutaneous drainage
C.Resection
D.Deroofing
E.Observation alone
A.Metronidazole alone
6 2 2015 38. 下列哪种说法不符合阿米巴性肝脓肿的常见表现:(2.5分) A.有慢性腹泻病史
B.病灶较大,多为单发,好发于肝左叶
C.白细胞计数可增加
D.脓液呈巧克力色,细菌培养常为阴性
E.甲硝唑单药治疗有效
B.病灶较大,多为单发,好发于肝左叶
6 2 2015 39. 下列哪种疾病与肝脓肿发生的相关性最小:(2.5分) A.伴有反复胆管炎的肝内胆管结石
B.急性化脓性阑尾炎
C.亚急性细菌性心内膜炎
D.糖尿病
E.肝囊肿
E.肝囊肿
6 2 2015 40. 一位68岁的男性患者因主诉右上腹痛伴高热三天来医院就诊,问诊得知他患有2型糖尿病多年,血常规检查发现该患者的白细胞计数升高至14×109/L,肝功能检查发现转氨酶轻度升高。腹部B超检查发现患者右肝内有一约6cm大小的病灶。该患者被诊断为细菌性肝脓肿,他的下一步的治疗方案是:(2.5分) A.口服抗生素治疗
B.超声引导下的肝脓肿穿刺引流
C.广谱抗生素+超声引导下的肝脓肿穿刺引流
D.开腹肝脓肿切开引流术
E.脓肿所在的肝叶切除手术
C.广谱抗生素+超声引导下的肝脓肿穿刺引流
6 3 2006 1. 男性,50岁,3年前曾行胆总管切开取石术,现持续黄疸一个月,肝大肋下3cm,血清胆红素为70μmol.L,BUS显示胆总管直径2cm,下述何种检查方法是最优: (3.0分) A.口服法胆囊造影术
B.CT
C.低张性十二指肠造影
D.静脉法胆道造影术
E.PTC
E.PTC
6 3 2006 2. 男性,15岁,突发心窝部阵发性钻顶样疼痛6小时,疼痛时大汗淋漓,辗转不安,痛止时又平息如常,体检:剑突偏右方有深在压痛,无腹肌紧张及反跳痛,为明确诊断应采取简单安全的方法是(3.0分) A.十二指肠引流液检查
B.右上腹X线平片
C.测定血清淀粉酶
D.BUS
E.ERCP
D.BUS
6 3 2006 3. 女性,42岁,3年来经常夜间上腹部不适,2日前进油腻食,突然右上腹部阵发性绞痛伴恶心,入院时体温38℃,巩膜轻度黄染,右上腹肌紧张,压痛明显,肠鸣音减弱,WBC16×10的9次方/L,血清淀粉酶为128温氏单位,应首先考虑诊断为何种疾病(3.0分) A.急性胰腺炎
B.高位急性阑尾炎
C.溃疡病穿孔
D.急性化脓性胆囊炎
E.胆道蛔虫症
D.急性化脓性胆囊炎
6 3 2006 4. 女性,47岁,胆囊结石病史4年,曾先后发作性胆绞痛4次,BUS显示胆囊内充满型结石。首选的治疗方法是下列哪一种(3.0分) A.口服熊去氧胆酸片
B.口服排石饮液
C.口服利胆素片
D.胆囊切除术
E.经皮胆镜取石术
D.胆囊切除术
6 3 2006 5. 男性,43岁,突然右上腹绞痛,并出现寒战高热,于16小时后出现明显黄疸。检查:意识尚清楚,巩膜及周身黄染,BP130/70mmHg,体温38.5℃,右上腹轻度压痛,此病最可能为(3.0分) A.Grey-Turner征
B.Murphy征
C.Charcot征
D.Courvoisier征
E.Reynauld征
C.Charcot征
6 3 2006 6. 男性,52岁,右上腹痛已半年,近半个月出现黄疸,体重下降,食欲减退。既往患胆囊结石已5年。检查:巩膜及全身皮肤黄染,右上腹扪及的6cm×6cm肿块,无移动性,BUS显示胆囊部位有一实质性占位病变,并可见强光团及声影。此病人可能的诊断是(3.0分) A.高位胆管癌
B.胆囊息肉
C.胆囊癌并胆囊结石
D.充满型胆囊结石
C.胆囊癌并胆囊结石
6 3 2006 7. 急性梗阻性化脓性胆管炎的治疗一般不宜采取(3.0分) A.纠正水电解质和酸碱平衡失调
B.胆囊造瘘术
C.胆总管探查取石去除病灶
D.使用有效足量的抗生素
E.及时合理使用多巴胺
B.胆囊造瘘术
6 3 2006 8. A 54-year-old male alcoholic presents with the sudden onset of severe, constant epigastric pain that radiates to his midback. Further evaluation finds fever, steatorrhea, and discoloration around his flank and umbilicus. Laboratory tests find elevated serum levels of amylase and lipase. What is the most likely cause of these findings?(3.0分) A.Acute cholangitis
B.Acute cholecystitis
C.Acute diverticulitis
D.Acute appendicitis
E.Acute pancreatitis
E.Acute pancreatitis
6 3 2006 9. A 45-year-old obese woman with cholelithiasis presents to the emergency room complaining of nausea and vomiting for 2 days, along with severe continuous midabdominal pain. She has a low-grade fever and the ER physician finds that she has a slightly elevated WBC count (12,000) and an elevated serum amylase. The most likely diagnosis is(3.0分) A.Hepatitis
B.Early phase of acute appendicitis
C.Ruptured abdominal aortic aneurysm
D.Peptic ulcer disease
E.Acute pancreatitis
E.Acute pancreatitis
6 3 2006 10. A 35-year-old woman presents with pancreatitis. Subsequent endoscopic retrograde cholangiopancreatography (ERCP) reveals the congenital cystic anomaly of her biliary system illustrated in the film below. Which of the following statements regarding this problem is true? (3.0分) A.Treatment consists of internal drainage via choledochoduodenostomy
B.Most patients present with the classic triad of epigastric pain, an abdominal mass, and jaundice
C.Malignant changes may occur within this structure
D.Cystic dilation of the intrahepatic biliary tree may coexist and is managed in a similar fashion
E.Surgery should be reserved for symptomatic patients
C.Malignant changes may occur within this structure
6 3 2006 11. 肝外胆道的解剖特点中,下列哪项是错误的(3.0分) A.Oddi括约肌由胆胰管壶腹部括约肌构成
B.胆囊动脉常有变异
C.胆总管末端多与主胰管汇合
D.胆囊管常有变异
E.胆囊分为颈体底三部
A.Oddi括约肌由胆胰管壶腹部括约肌构成
6 3 2006 12. 肝外胆管癌发生率最高的部位是(3.0分) A.中段胆管
B.下段胆管
C.左肝管
D.上段胆管
E.胆囊管
D.上段胆管
6 3 2006 13. Courvoisier征阳性的疾病最可能是(3.0分) A.胆管上段癌
B.胆管下端癌
C.胆囊管癌
D.右肝管Ⅱ级支癌
E.胆囊体癌
B.胆管下端癌
6 3 2006 14. Nevin分期的III 期表述为:(3.0分) A.癌组织侵犯胆囊壁全层并有淋巴结转移
B.癌组织侵犯胆囊黏膜和肌层
C.癌组织仅限于胆囊黏膜
D.癌组织侵及胆囊壁全层, 即黏膜、肌层和浆膜层
E.癌组织直接侵犯肝脏或有肝转移, 或者有任何器官的转移
D.癌组织侵及胆囊壁全层, 即黏膜、肌层和浆膜层
6 3 2006 15. 病灶超过胆管分叉,主要累及右肝管并达到二级胆管开口处,左肝管情况尚好,属于Klatskin瘤Bismuth分型的哪一型?(3.0分) A.Bismuth IIIb型
B.BismuthⅡ型
C.Bismuth IIIa型
D.BismuthⅠ型
E.Bismuth Ⅳ
C.Bismuth IIIa型
6 3 2006 16. 男性,52岁,右上腹痛已半年,近半个月出现黄疸,体重下降,食欲减退。既往患胆囊结石已5年。检查:巩膜及全身皮肤黄染,右上腹扪及的6cm×6cm肿块,无移动性,BUS显示胆囊部位有一实质性占位病变,并可见强光团及声影。此病人可能的诊断是(3.0分) A.充满型胆囊结石
B.高位胆管癌
C.胆囊癌并胆囊结石
D.胆囊息肉
E.幽门管癌并肝十二指肠韧带淋巴结转移
C.胆囊癌并胆囊结石
6 3 2006 17. 男性,64岁,进行性无痛性黄疸1月,上腹胀痛,体重减轻。检查:巩膜及周身皮肤黄染,P90次/分,BP140/70mmHg,血清总胆红素为70μmol/L,Courvoisier征阳性。此病人首先考虑诊断为 (3.0分) A.中、下段胆管癌
B.肝门部胆管癌
C.胆囊癌
D.胆囊结石
E.胆总管结石
A.中、下段胆管癌
6 3 2006 18. 胰腺癌切除率低的主要原因为(3.0分) A.年老,体弱
B.癌的恶性程度高
C.并发胆道疾病
D.癌直接浸润和转移
E.手术复杂
D.癌直接浸润和转移
6 3 2006 19. 胰腺癌最常见的组织类型为(4.0分) A.导管细胞癌
B.腺泡细胞癌
C.多形性腺癌
D.纤维细胞腺癌
E.粘液癌
A.导管细胞癌
6 3 2006 20. 胰腺癌最好发的部位是(3.0分) A.胰腺尾部
B.胰腺头部
C.全胰腺
D.胰腺体部
E.异位胰腺
B.胰腺头部
6 3 2006 21. 壶腹部癌的临床特点是较早出现(3.0分) A.黄疸、寒战、发热
B.消化道症状
C.上腹痛及脊背痛
D.转移症状
E.贫血消瘦
A.黄疸、寒战、发热
6 3 2006 22. 男性,35岁黄疸已1月,右上腹轻微胀痛,食欲不振,经内科治疗无效。查体:肝大,胆囊增大,血胆红素17μmol/L,AST70单位,AKP45单位,AFP>5ng/ml,可能诊断是(3.0分) A.肝硬化晚期
B.胆总管结石梗阻
C.肝癌
D.黄疸性肝炎
E.壶腹周围癌
E.壶腹周围癌
6 3 2006 23. 以下哪项不是胰腺癌高危因素: (3.0分) A.突发糖尿病者
B.胰腺癌家族史
C.慢性胰腺炎
D.吸烟、大量饮酒
E.胆道结石
E.胆道结石
6 3 2006 24. 关于急腹症的手术指征,下列哪项错误(3.0分) A.急性阑尾炎
B.急性水肿性胰腺炎
C.急性肠扭转
D.急性胃穿孔
E.急性完全性机械性肠梗阻
B.急性水肿性胰腺炎
6 3 2006 25. 关于牵涉痛,下列哪种说法是错误(3.0分) A.急性阑尾炎牵涉痛表现在脐周/上腹;
B.胆绞痛向右肩及右肩胛区放射痛;
C.急性胰腺炎伴左肩痛;
D.输尿管结石牵涉痛为同侧会阴部及大腿内侧;
E.胃溃疡穿孔右下腹放射痛。
E.胃溃疡穿孔右下腹放射痛。
6 3 2006 26. 下列哪种疾病需要紧急手术(3.0分) A.急性单纯性机械性肠梗阻
B.消化道穿孔并弥漫性腹膜炎
C.急性胆囊炎、胆管炎
D.消化性溃疡空腹穿孔或小穿孔已闭合,腹膜炎局限
E.术后吻合口、缝合口漏,腹膜炎局限且引流通畅
B.消化道穿孔并弥漫性腹膜炎
6 3 2006 27. 患者男,65岁,突发腹部绞痛伴恶心呕吐5小时入院。既往有风湿性心脏病伴房颤病史。入院查体:T:38.3℃,P:110次/分,R:25次/分,BP:135/75mmHg。急性面容,腹部稍膨隆,全腹压痛、反跳痛和肌紧张不明显,肠鸣音未闻及。为明确诊断诊断,需进行下列哪项检查(3.0分) A.胸片
B.腹平片
C.心电图
D.腹部CTA
E.腹部B超
D.腹部CTA
6 3 2006 28. 关于躯体性腹痛错误的是:(3.0分) A.可因咳嗽或改变体位而加重,腹式呼吸受限
B.具有脊髓节段性分布,定位较准确
C.程度剧烈而持续
D.常常提示炎症累及腹膜脏层
E.可伴有局部压痛、反跳痛和肌紧张
D.常常提示炎症累及腹膜脏层
6 3 2006 29. 患者男,11岁,右下腹持续性痛阵发性加剧5天。伴发热,呕吐1次。入院查体:T:38.5℃,P:111次/分,R:23次/分,BP:103/62mmHg。急性面容,皮肤巩膜无黄染。腹部平坦,右下腹扪及一约5cm×5cm大小肿块,压痛,局部肌紧张。肠鸣音正常。血常规中WBC:16×109/L,N:85%。该患者最可能的诊断是:(3.0分) A.急性单纯性阑尾炎
B.结肠癌
C.肠套叠
D.阑尾周围脓肿
E.粪石性肠梗阻
D.阑尾周围脓肿
6 3 2006 30. 以下哪项不一定是婴幼儿急性腹泻病(3.0分) A.便秘幼儿近3天日解2次稀水便
B.2月龄母乳喂养儿大便5次/日
C.幼儿大便呈粘液脓血样,病程7日
D.3岁幼儿大便呈水样,>3次/日,病程3日
E.人工喂养儿大便突呈稀水样
B.2月龄母乳喂养儿大便5次/日
6 3 2006 31. 下列哪项与急性腹泻的严重性无关(3.0分) A.年龄<6月
B.脱水
C.发热
D.呕吐
E.病程
E.病程
6 3 2006 32. 哪项说法是错误的(3.0分) A.医院获得性腹泻往往比社区获得的腹泻更严重
B.在2周内接触腹泻病人是病毒性腹泻主要风险因素
C.出国、接触腹泻病人、住院、失业、父母文化水平低是引起腹泻的风险增加的因素
D.年龄小、免疫缺陷、营养不良、医学知识教育差的患儿更加易致轮状病毒感染
E.母乳喂养的婴儿对轮状病毒感染有较大的保护性
A.医院获得性腹泻往往比社区获得的腹泻更严重
6 3 2006 33. 关于腹泻病人的水电解质紊乱,以下说法哪项是不合理(3.0分) A.急性腹泻的脱水大多为等渗性脱水
B.所有腹泻病人均应该行血气及电解质的分析
C.pH、BE、电解质并不能预测脱水的严重性
D.中度以上脱水、病史体征与腹泻程度不一致、需静脉补液者需行电解质的测定
E.怀疑高钠血症、12小时以上无尿者需要血气电解质检测
B.所有腹泻病人均应该行血气及电解质的分析
6 3 2007 1. 急性梗阻性化脓性胆管炎最常见的原因是(2.5分) A.胆总管癌
B.胆总管末端狭窄
C.胆道出血继发感染
D.胆总管结石
E.先天性胆总管扩张症
D.胆总管结石
6 3 2007 2. 男性,15岁,突发心窝部阵发性钻顶样疼痛6小时,疼痛时大汗淋漓,辗转不安,痛止时又平息如常,体检:剑突偏右方有深在压痛,无腹肌紧张及反跳痛,为明确诊断应采取简单安全的方法是(2.5分) A.BUS
B.十二指肠引流液检查
C.测定血清淀粉酶
D.右上腹X线平片
E.ERCP
A.BUS
6 3 2007 3. 下列那个特点不是切口疝的特点:(2.5分) A.可用假体加强腹壁缺损区
B.可能有食欲减退、恶心等症状
C.发病率在各种腹外疝中排在第三位
D.一般发生在手术瘢痕区
E.易发生嵌顿
E.易发生嵌顿
6 3 2007 4. 女性43岁,突发右上腹剧痛并阵发性加剧3天,寒战高热,恶心呕吐。体检:全身黄染,体温39℃,脉搏120次/分,血压80/60mmHg,谵妄,神志不清,剑突偏左腹肌紧张,肝肋下2cm,WBC15×109/L,中性80%血清总胆红素50μmol/L,本例治疗首选(2.5分) A.胆囊造口术
B.胆囊切除,腹腔引流术
C.抗休克,抗感染治疗
D.胆总管减压引流
E.溃疡病穿孔缝合
D.胆总管减压引流
6 3 2007 5. 治疗股疝一般选用的手术是:(2.5分) A.Mcvay法
B.Halsted法
C.Ferguson法
D.Bassini法
E.Shouldice法
A.Mcvay法
6 3 2007 6. Laparoscopic cholecystectomy is indicated for symptomatic gallstones in which of the following conditions? (2.5分) A.Morbid obesity
B.Prior upper abdominal surgery
C.Suspected carcinoma of the gallbladder
D.Cirrhosis
E.Coagulopathy
A.Morbid obesity
6 3 2007 7. 男性,52岁,右上腹痛已半年,近半个月出现黄疸,体重下降,食欲减退。既往患胆囊结石已5年。检查:巩膜及全身皮肤黄染,右上腹扪及的6cm×6cm肿块,无移动性,BUS显示胆囊部位有一实质性占位病变,并可见强光团及声影。此病人可能的诊断是(2.5分) A.胆囊癌并胆囊结石
B.高位胆管癌
C.胆囊息肉
D.充满型胆囊结石
A.胆囊癌并胆囊结石
6 3 2007 8. A 45-year-old obese woman with cholelithiasis presents to the emergency room complaining of nausea and vomiting for 2 days, along with severe continuous midabdominal pain. She has a low-grade fever and the ER physician finds that she has a slightly elevat(2.5分) A.Hepatitis
B.Ruptured abdominal aortic aneurysm
C.Peptic ulcer disease
D.Early phase of acute appendicitis
E.Acute pancreatitis
E.Acute pancreatitis
6 3 2007 9. 男性,75岁,腹痛腹胀、呕吐、肛门停止排气排便2天。全腹压痛明显,腹部平片示肠管可见多处液平面。首先考虑:(2.5分) A.急性胰腺炎
B.胃溃疡穿孔
C.急性肠梗阻
D.急性阑尾炎
E.急性胆囊炎
C.急性肠梗阻
6 3 2007 10. 女性,42岁,3年来经常夜间上腹部不适,2日前进油腻食,突然右上腹部阵发性绞痛伴恶心,入院时体温38℃,巩膜轻度黄染,右上腹肌紧张,压痛明显,肠鸣音减弱,WBC16×109/L,血清淀粉酶为128温氏单位,应首先考虑诊断为何种疾病(2.5分) A.高位急性阑尾炎
B.溃疡病穿孔
C.急性胰腺炎
D.急性化脓性胆囊炎
E.胆道蛔虫症
D.急性化脓性胆囊炎
6 3 2007 11. 诊断胆囊结石的简单而可靠的方法是(2.5分) A.ERCP
B.BUS
C.PTC
D.口服法胆囊造影
E.十二指肠引流术
B.BUS
6 3 2007 12. 以下哪项是胰腺癌手术切除的禁忌症: (2.5分) A.肠系膜上静脉肿瘤包绕侵犯
B.后腹膜淋巴结肿大
C.肠系膜上动脉肿瘤包绕侵犯
D.肿瘤大于4cm
E.肿瘤侵犯十二指肠、胆总管
C.肠系膜上动脉肿瘤包绕侵犯
6 3 2007 13. 腹外疝患者手术时发现疝囊位于腹壁下动脉内侧,应诊断为: (2.5分) A.股疝
B.腹股沟直疝
C.腹股沟斜疝
D.脐疝
E.白线疝
B.腹股沟直疝
6 3 2007 14. 以下哪项不一定是婴幼儿急性腹泻病(2.5分) A.幼儿大便呈粘液脓血样,病程7日
B.2月龄母乳喂养儿大便5次/日
C.3岁幼儿大便呈水样,>3次/日,病程3日
D.便秘幼儿近3天日解2次稀水便
E.人工喂养儿大便突呈稀水样
B.2月龄母乳喂养儿大便5次/日
6 3 2007 15. 在临床上,休克不易纠正的急腹症是(2.5分) A.溃疡病急性穿孔
B.阑尾炎穿孔
C.肠伤寒穿孔
D.急性水肿性胰腺炎
E.急性梗阻性化脓性胆管炎
E.急性梗阻性化脓性胆管炎
6 3 2007 16. 胰腺癌最常见的组织类型为(2.5分) A.纤维细胞腺癌
B.多形性腺癌
C.腺泡细胞癌
D.导管细胞癌
E.粘液癌
D.导管细胞癌
6 3 2007 17. 胰腺癌最好发的部位是(2.5分) A.胰腺尾部
B.胰腺头部
C.全胰腺
D.胰腺体部
E.异位胰腺
B.胰腺头部
6 3 2007 18. 35岁黄疸已1月,右上腹轻微胀痛,食欲不振,经内科治疗无效。查体:肝大,胆囊增大,血胆红素17μmol/L,AST70单位,AKP45单位,AFP>5ng/ml,可能诊断是(2.5分) A.肝癌
B.胆总管结石梗阻
C.黄疸性肝炎
D.肝硬化晚期
E.壶腹周围癌
E.壶腹周围癌
6 3 2007 19. 以下哪项不是胰腺癌高危因素: (2.5分) A.胰腺癌家族史
B.突发糖尿病者
C.吸烟、大量饮酒
D.慢性胰腺炎
E.胆道结石
E.胆道结石
6 3 2007 20. 胆囊动脉多源于(2.5分) A.肝左动脉
B.肝固有动脉
C.肝右动脉
D.胃十二指肠动脉
E.胃右动脉
C.肝右动脉
6 3 2007 21. 黄疸开始有波动,以后加深并伴大便潜血阳性,常见于(2.5分) A.胰头癌
B.慢性胆囊炎
C.急性化脓性胆管炎
D.急性胰腺炎
E.壶腹部癌
E.壶腹部癌
6 3 2007 22. 腹股沟加强前壁的手术方法有:(2.5分) A.Bassini法
B.Ferguson法
C.McVay法
D.高位结扎法
E.Halsted法
B.Ferguson法
6 3 2007 23. 男性,50岁,3年前曾行胆总管切开取石术,现持续黄疸一个月,肝大肋下3cm,血清胆红素为70μmol/L,BUS显示胆总管直径2cm,下述何种检查方法是最优:(2.5分) A.口服法胆囊造影术
B.CT
C.低张性十二指肠造影
D.静脉法胆道造影术
E.PTC
E.PTC
6 3 2007 24. Charcot征是指(2.5分) A.上腹痛、发热、休克
B.上腹痛、黑便、呕血
C.上腹痛、畏寒发热、黄疸
D.上腹痛、畏寒发热、休克
E.上腹痛、畏寒发热、黄疸、休克、神志不清
C.上腹痛、畏寒发热、黄疸
6 3 2007 25. 女性,55岁,上腹部突然持续疼痛,阵发性加剧,拒按难忍,伴有腰背胀痛、恶心、呕吐48小时入院。体检:稍腹胀,上腹部压痛,反跳痛,肌紧张,肠鸣音减弱,WBC15×109/L,N0.90,血淀粉酶32u(温氏单位),尿淀粉酶大于1200u(温氏单位)。首先考虑(2.5分) A.胆石症、胆道感染
B.十二指肠溃疡穿孔
C.绞窄性肠梗阻
D.急性胰腺炎
E.急性弥漫性腹膜炎
D.急性胰腺炎
6 3 2007 26. 1岁男孩,突发哭闹不休,面色苍白,片刻后如常,症状多次出现,近1天来解果酱样大便,首先应考虑(2.5分) A.肠炎
B.肠扭转
C.肠套叠
D.肠蛔虫
E.肠系膜淋巴结炎
C.肠套叠
6 3 2007 27. 胰岛素瘤来源于胰腺的: (2.5分) A.B细胞
B.A细胞
C.D细胞
D.G细胞
E.D1细胞
A.B细胞
6 3 2007 28. 急性胆囊炎最严重的并发症是(2.5分) A.急性胰腺炎
B.胆囊积脓
C.胆囊坏疸穿死致胆汁性腹膜炎
D.细菌性肝脓肿
E.胆囊十二指肠内瘘
C.胆囊坏疸穿死致胆汁性腹膜炎
6 3 2007 29. 下列哪种合并症不影响疝修补术的疗效:(2.5分) A.前列腺增生
B.慢性咳嗽
C.便闭
D.大量腹水
E.急性膀胱炎
E.急性膀胱炎
6 3 2007 30. 男性,28岁,突感上腹部剧痛,于发病45分钟后来诊。体检:血压110/80mmHg,心率100次/分,板状腹,肠鸣音消失,血常规:血红蛋白120g/L ,WBC7×109/L,N0.72。尿淀粉酶128u/L。最有意义的进一步检查是(2.5分) A.腹部CT检查
B.腹立位平片
C.腹腔滴注灌洗
D.腹部B超检查
E.生化检查
B.腹立位平片
6 3 2007 31. 男性,43岁,突然右上腹绞痛,并出现寒战高热,于16小时后出现明显黄疸。检查:意识尚清楚,巩膜及周身黄染,BP130/70mmHg,体温38.5℃,右上腹轻度压痛,此病最可能为(2.5分) A.Murphy征
B.Courvoisier征
C.Charcot征
D.Grey-Turner征
E.Reynold征
C.Charcot征
6 3 2007 32. 胆囊结石临床表现各异,主要取决于(2.5分) A.体位的改变或静卧状态
B.结石的大小及部位、嵌顿与否
C.结石的大小、部位、梗阻与否、有无感染
D.进油腻食物后
E.结石嵌顿于胆囊颈部
C.结石的大小、部位、梗阻与否、有无感染
6 3 2007 33. Murphy征阳性则提示(2.5分) A.细菌性肝脓肿
B.左肝管结石
C.肝总管结石
D.急性胆管炎
E.急性胆囊炎
E.急性胆囊炎
6 3 2007 34. 嵌顿疝和绞窄性疝的区别是:(2.5分) A.疝内容有无动脉血运障碍
B.疝内容不能还纳的时间
C.疝囊有无压痛
D.是否有休克
E.是否有肠梗阻表现
A.疝内容有无动脉血运障碍
6 3 2007 35. 传统修补手术治疗股疝常采用的术式:(2.5分) A.McVay法
B.Ferguson法
C.Bassini法
D.Halsted法
E.Shouldice法
A.McVay法
6 3 2007 36. 女性,43岁,因右上腹阵发性绞痛,伴恶心呕吐4小时来院。检查:体温37℃,右上腹深在轻度压痛,无腹肌紧张,Murphy征阴性,为确诊进一步检查应首选(2.5分) A.测血清淀粉酶
B.腹部X线平片
C.BUS
D.白细胞计数和分类
E.PTC
C.BUS
6 3 2007 37. 我国,引起婴幼儿腹泻的最常见的病原体是(2.5分) A.空肠弯曲菌
B.沙门氏菌
C.志贺氏菌
D.轮状病毒
E.诺如病毒
D.轮状病毒
6 3 2007 38. 对脱水的评估,最为有效的体征是(2.5分) A.皮肤弹性、血压、眼窝凹陷
B.精神状况、心率、前囟凹陷
C.尿量减少、血压、心率
D.眼窝凹陷、少尿、末梢循环
E.皮肤弹性、毛细血管充盈时间、呼吸方式
E.皮肤弹性、毛细血管充盈时间、呼吸方式
6 3 2007 39. 高钾血症时,血钾浓度为(2.5分) A.>4.0mmol/L
B.>6.0mmol/L
C.>5.5mmol/L
D.>5.0mmol/L
E.<3.5 mmol/L
C.>5.5mmol/L
6 3 2007 40. 关于口服补液,正确的是(2.5分) A.中度脱水要求10-20ml/kg/h补充,基本每15分钟5ml/kg
B.患儿无法进行口服补液时,可采用鼻胃管途径补液
C.口服补液应作为儿童急性胃肠炎的一线治疗手段
D.1小时内无ORS补充或脱水状况加重则进行鼻胃管补液
E.以上都正确
E.以上都正确
6 3 2008 1. 黄疸病人合并肿大而无触痛的胆囊时,最有可能是(2.0分) A.急性胆囊炎
B.慢性胆囊炎,胆囊积水
C.胆囊颈部结石嵌顿
D.中下段胆管癌
E.胆总管下段结石
D.中下段胆管癌
6 3 2008 2. 肝外胆管癌发生率最高的部位是: (2.0分) A.左肝管
B.下段胆管
C.中段胆管
D.上段胆管
E.胆囊管
D.上段胆管
6 3 2008 3. 急性梗阻性化脓性胆管炎最常见的原因是:(2.0分) A.胆总管结石
B.胆总管末端狭窄
C.胆道出血继发感染
D.胆总管癌
E.先天性胆总管扩张
A.胆总管结石
6 3 2008 4. 男,65岁。皮肤巩膜黄染进行性加重1个月,自述尿色深黄,粪便呈灰白色,查体:胆囊无肿大,murphy征阴性,腹部未触及肿块,首先考虑: (2.0分) A.胰头癌
B.胆总管下端癌
C.乏特壶腹癌
D.肝门部胆管癌
E.十二指肠腺癌
D.肝门部胆管癌
6 3 2008 5. 男,57岁。皮肤黄染,尿色变深伴皮肤瘙痒3周。查体:皮肤巩膜变黄,右上腹可触及无痛性圆形肿块,随呼吸上下活动。该肿块可能为: (2.0分) A.胆总管囊肿
B.肝脏囊肿
C.肿大胆囊
D.胰头癌
E.胆管癌
C.肿大胆囊
6 3 2008 6. Courvoisier征阳性的疾病最可能是:(2.0分) A.胆管上段癌
B.胆管下端癌
C.右肝管II级支癌
D.胆囊管癌
E.胆囊体癌
B.胆管下端癌
6 3 2008 7. 男性,52岁,右上腹痛已半年,近半个月出现黄疸,体重下降,食欲减退。既往患胆囊结石已5年。体查:巩膜及全身皮肤黄染,右上腹扪及6cm×6cm肿块,无移动性,BUS显示胆囊部位有一实质性占位病变,并可见强光团及声影。可能诊断: (2.0分) A.胆囊息肉
B.高位胆管癌
C.胆囊癌并胆囊结石
D.充满型胆囊结石
E.幽门管癌并肝十二指肠韧带淋巴结转移
C.胆囊癌并胆囊结石
6 3 2008 8. 肝外胆道解剖特点,下列哪项是错误的: (2.0分) A.胆囊管常有变异
B.胆囊动脉常有变异
C.胆总管末端多与主胰管汇合
D.ODDi括约肌由胆胰管壶腹部括约肌构成
E.胆囊分为颈体底三部分
D.ODDi括约肌由胆胰管壶腹部括约肌构成
6 3 2008 9. 急腹症最重要的体格检查方法是(2.0分) A.望诊
B.触诊
C.叩诊
D.听诊
E.直肠指检
B.触诊
6 3 2008 10. 诊断急性胆囊炎首选的检查是:(2.0分) A.腹部平片
B.腹部CT
C.腹部B超
D.动脉造影
E.心电图
C.腹部B超
6 3 2008 11. 常伴有感染性休克的急腹症是:(2.0分) A.急性阑尾炎
B.急性胆囊炎
C.急性化脓性胆管炎
D.急性水肿型胰腺炎
E.急性粘连性肠梗阻
C.急性化脓性胆管炎
6 3 2008 12. 下列急腹症中血尿淀粉酶明显升高为其典型特点的是:(2.0分) A.急性阑尾炎
B.急性胆囊炎
C.急性胆管炎
D.急性胰腺炎
E.急性肠梗阻
D.急性胰腺炎
6 3 2008 13. 下列关于腹痛的描述错误的是:(2.0分) A.机械性肠梗阻可表现为阵发性疼痛
B.急性阑尾炎早期可表现为上腹痛
C.急性胆囊炎可表现为剑突下痛
D.输尿管下段结石可出现会阴部放射痛
E.急性胰腺炎上腹痛可伴右肩痛
E.急性胰腺炎上腹痛可伴右肩痛
6 3 2008 14. 常常出现严重的腹痛与轻微的体征不相称急腹症的是:(2.0分) A.急性阑尾炎和急性胆囊炎
B.胃穿孔和肠系膜上动脉栓塞
C.急性阑尾炎和胃穿孔
D.胆道蛔虫和肠系膜上动脉栓塞
E.急性胰腺炎和机械性肠梗阻
D.胆道蛔虫和肠系膜上动脉栓塞
6 3 2008 15. 男,45岁。腹部剧烈疼痛2小时入院。既往有剑突下疼痛病史5年,夜间疼痛加重。查体:心率107次/分,血压97/59mmHg。腹平,全腹压痛反跳痛,板状腹,肠鸣音消失。该患者最有意义的检查是:(2.0分) A.血清淀粉酶
B.尿淀粉酶
C.腹部B超
D.立位腹平片
E.血常规
D.立位腹平片
6 3 2008 16. 男性,50岁,3年前曾行胆总管切开取石术,现持续黄疸一个月,肝大肋下3cm,血清胆红素为70μmol/L,BUS显示胆总管直径2cm,下述何种检查方法是最优:(2.0分) A.口服法胆囊造影术
B.静脉法胆道造影术
C.低张性十二指肠造影
D.CT
E.PTC
E.PTC
6 3 2008 17. 男性,15岁,突发心窝部阵发性钻顶样疼痛6小时,疼痛时大汗淋漓,辗转不安,痛止时又平息如常,体检:剑突偏右方有深在压痛,无腹肌紧张及反跳痛,为明确诊断应采取简单安全的方法是 (2.0分) A.BUS
B.右上腹X线平片
C.测定血清淀粉酶
D.十二指肠引流液检查
E.ERCP
A.BUS
6 3 2008 18. 女性,42岁,3年来经常夜间上腹部不适,2日前进油腻食,突然右上腹部阵发性绞痛伴恶心,入院时体温38℃,巩膜轻度黄染,右上腹肌紧张,压痛明显,肠鸣音减弱,WBC16×109/L,血清淀粉酶为128温氏单位,应首先考虑诊断为何种疾病(2.0分) A.高位急性阑尾炎
B.急性胰腺炎
C.溃疡病穿孔
D.急性化脓性胆囊炎
E.胆道蛔虫症
D.急性化脓性胆囊炎
6 3 2008 19. 女性,45岁,突发右上腹及心窝部刀割样绞痛伴阵发性加剧一天,发病后12小时寒战、高热,巩膜黄染,剑突偏右侧深压痛,右上腹轻度肌紧张,体温38℃,WBC 14×109/L血清总胆红素30μmol/L,尿胆原(-),尿胆素(++),应诊断为(2.0分) A.溃疡病穿孔
B.急性胰腺炎
C.急性胆囊炎
D.胆总管结石
E.高位阑尾炎
D.胆总管结石
6 3 2008 20. 女性,43岁,因右上腹阵发性绞痛,伴恶心呕吐4小时来院。检查:体温37℃,右上腹深在轻度压痛,无腹肌紧张,Murphy征阴性,为确诊进一步检查应首选(2.0分) A.白细胞计数和分类
B.腹部X线平片
C.BUS
D.测血清淀粉酶
E.PTC
C.BUS
6 3 2008 21. 女性,47岁,胆囊结石病史4年,曾先后发作性胆绞痛4次,BUS显示胆囊内充满型结石。首选的治疗方法是下列哪一种(2.0分) A.口服排石饮液
B.口服熊去氧胆酸片
C.口服利胆素片
D.胆囊切除术
E.经皮胆镜取石术
D.胆囊切除术
6 3 2008 22. 胆囊动脉多源于(2.0分) A.肝左动脉
B.肝固有动脉
C.胃十二指肠动脉
D.肝右动脉
E.胃右动脉
D.肝右动脉
6 3 2008 23. 梗阻性黄疸时,BUS显示肝内胆管扩张,胆总管直径2cm,进一步明确梗阻部位的检查是(2.0分) A.放射性核素胰腺扫描
B.低张十二指肠造影
C.PTC
D.ERCP
E.MRI
C.PTC
6 3 2008 24. 梗阻性黄疸时,BUS显示胆总管和肝内胆管均不扩张,为明确诊断应选择哪项检查(2.0分) A.放射性核素胰腺扫描
B.PTC
C.低张十二指肠造影
D.ERCP
E.CT
D.ERCP
6 3 2008 25. 胆囊结石临床表现各异,主要取决于(2.0分) A.结石的大小、部位、梗阻与否、有无感染
B.结石的大小及部位、嵌顿与否
C.体位的改变或静卧状态
D.进油腻食物后
E.结石嵌顿于胆囊颈部
A.结石的大小、部位、梗阻与否、有无感染
6 3 2008 26. Charcot三联征间歇发作最大的可能是(2.0分) A.壶腹部癌
B.肝细胞癌
C.胆总管结石
D.黄疸型肝炎
E.细菌性肝脓肿
C.胆总管结石
6 3 2008 27. 胆管结石和急性胆管炎急性发作的典型症状是(2.0分) A.腹痛、呕吐、寒热
B.腹痛、呕吐、黄疸
C.腹痛、黄疸、腹泻
D.腹痛、腹胀、昏迷
E.腹痛、黄疸、寒热
E.腹痛、黄疸、寒热
6 3 2008 28. 胆道手术时,下述哪项不是胆总管探查的指征(2.0分) A.胆总管,肝总管、肝胆管结石
B.黄疸或黄疸病史
C.胰头肿大、变硬
D.胆总管增厚变粗
E.胆囊积液
E.胆囊积液
6 3 2008 29. 急性胆囊炎最严重的并发症是(2.0分) A.细菌性肝脓肿
B.胆囊积脓
C.胆囊坏疸穿死致胆汁性腹膜炎
D.急性胰腺炎
E.胆囊十二指肠内瘘
C.胆囊坏疸穿死致胆汁性腹膜炎
6 3 2008 30. 急性梗阻性化脓性胆管炎最常见的原因是(2.0分) A.胆总管结石
B.胆总管末端狭窄
C.胆道出血继发感染
D.胆总管癌
E.先天性胆总管扩张症
A.胆总管结石
6 3 2008 31. A 54-year-old male alcoholic presents with the sudden onset of severe, constant epigastric pain that radiates to his midback. Further evaluation finds fever, steatorrhea, and discoloration around his flank and umbilicus. Laboratory tests find elevated serum levels of amylase and lipase. What is the most likely cause of these findings? (2.0分) A.Acute appendicitis
B.Acute cholangitis
C.Acute cholecystitis
D.Acute diverticulitis
E.Acute pancreatitis
E.Acute pancreatitis
6 3 2008 32. A 45-year-old obese woman with cholelithiasis presents to the emergency room complaining of nausea and vomiting for 2 days, along with severe continuous midabdominal pain. She has a low-grade fever and the ER physician finds that she has a slightly elevated WBC count (12,000) and an elevated serum amylase. The most likely diagnosis is (2.0分) A.Ruptured abdominal aortic aneurysm
B.Hepatitis
C.Peptic ulcer disease
D.Early phase of acute appendicitis
E.Acute pancreatitis
E.Acute pancreatitis
6 3 2008 33. Laparoscopic cholecystectomy is indicated for symptomatic gallstones in which of the following conditions? (2.0分) A.Cirrhosis
B.Prior upper abdominal surgery
C.Suspected carcinoma of the gallbladder
D.Morbid obesity
E.Coagulopathy
D.Morbid obesity
6 3 2008 34. 以下哪项不一定是婴幼儿急性腹泻病 (2.0分) A.3岁幼儿大便呈水样,>3次/日,病程3日
B.2月龄母乳喂养儿大便5次/日
C.幼儿大便呈粘液脓血样,病程7日
D.便秘幼儿近3天日解2次稀水便
E.人工喂养儿大便突呈稀水样
B.2月龄母乳喂养儿大便5次/日
6 3 2008 35. 我国,引起婴幼儿腹泻的最常见的病原体是 A(2.0分) A.轮状病毒
B.空肠弯曲菌
C.沙门氏菌
D.志贺氏菌
E.诺如病毒
A.轮状病毒
6 3 2008 36. 哪项说法是错误的 (2.0分) A.在2周内接触腹泻病人是病毒性腹泻主要风险因素
B.医院获得性腹泻往往比社区获得的腹泻更严重
C.出国、接触腹泻病人、住院、失业、父母文化水平低是引起腹泻的风险增加的因素
D.年龄小、免疫缺陷、营养不良、医学知识教育差的患儿更加易致轮状病毒感染
E.母乳喂养的婴儿对轮状病毒感染有较大的保护性
B.医院获得性腹泻往往比社区获得的腹泻更严重
6 3 2008 37. 关于腹泻的症状与疾病严重性,正确的是 (2.0分) A.腹泻患儿出现血便或便中带血,基本肯定是细菌感染
B.腹泻病人体温≥39℃,大多是细菌感染
C.秋冬季节,患儿出现呼吸道感染症状后再有腹泻,大多是病毒性腹泻
D.腹泻病人一旦伴有中枢神经系统症状,提示细菌感染
E.水样泻病人出现腹胀,可能伴有坏死性小肠结肠炎
C.秋冬季节,患儿出现呼吸道感染症状后再有腹泻,大多是病毒性腹泻
6 3 2008 38. 对脱水的评估,最为有效的体征是(2.0分) A.眼窝凹陷、少尿、末梢循环
B.精神状况、心率、前囟凹陷
C.尿量减少、血压、心率
D.皮肤弹性、血压、眼窝凹陷
E.皮肤弹性、毛细血管充盈时间、呼吸方式
E.皮肤弹性、毛细血管充盈时间、呼吸方式
6 3 2008 39. 低渗性脱水时,血钠浓度为 (2.0分) A.>140mmol/L
B.>145mmol/L
C.>150mmol/L
D.>155mmol/L
E.<130mmol/L
E.<130mmol/L
6 3 2008 40. 低钾血症时,血钾浓度为 (2.0分) A. >4.0mmol/L
B.>5.0mmol/L
C.>5.5mmol/L
D.<3.5 mmol/L
E.>6.0mmol/L
D.<3.5 mmol/L
6 3 2008 41. 以下说法正确的是 (2.0分) A.一旦明确致病病原体,急性腹泻病人才能得到合理的诊治
B.粪便培养是腹泻的常规检查
C.粪便中一旦出现白细胞,则提示细菌感染
D.不管病原体检查结果,腹泻患儿均可补液治疗
E.细菌性腹泻均应该应用抗菌素的治疗
D.不管病原体检查结果,腹泻患儿均可补液治疗
6 3 2008 42. 关于腹泻病人的水电解质紊乱,以下说法哪项是不合理 (2.0分) A.所有腹泻病人均应该行血气及电解质的分析
B.急性腹泻的脱水大多为等渗性脱水
C.pH、BE、电解质并不能预测脱水的严重性
D.中度以上脱水、病史体征与腹泻程度不一致、需静脉补液者需行电解质的测定
E.怀疑高钠血症、12小时以上无尿者需要血气电解质检测
A.所有腹泻病人均应该行血气及电解质的分析
6 3 2008 43. 下列哪项是脱水的风险因素 E(2.0分) A.<1岁,尤其6月以下
B.低体重或营养不良婴儿
C.前24小时腹泻次数>5次
D.患病期间停止母乳喂养者
E.以上都是
E.以上都是
6 3 2008 44. 关于口服补液,不正确的是 (2.0分) A.口服补液应作为儿童急性胃肠炎的一线治疗手段
B.患儿无法进行口服补液时,可采用鼻胃管途径补液
C.轻中度脱水口服补液盐8小时内服完,再重新评估
D.1小时内无ORS补充或脱水状况加重则进行鼻胃管补液,如果症状仍无改善则静脉补液
E.对于能接受口服补液的患者不推荐静脉补液
C.轻中度脱水口服补液盐8小时内服完,再重新评估
6 3 2008 45. 有关低渗ORS在腹泻中的治疗,哪项说法不对 (2.0分) A.推荐低渗ORS为急性腹泻的一线治疗
B.低渗ORS对非霍乱腹泻的治疗更有效
C.低渗ORS与标准ORS比较可增加低钠血症的发生
D.ESPGHAN推荐的ORS(低渗)配方对急性腹泻的治疗同样有效
E.治疗霍乱性腹泻低渗ORS也有效
C.低渗ORS与标准ORS比较可增加低钠血症的发生
6 3 2008 46. 饮食治疗是婴幼儿腹泻治疗中的关键因素之一,以下说法不合理的是(2.0分) A.无脱水症状的腹泻儿应该喂食与患儿年龄相当的正常饮食
B.接受补液的患儿可暂停饮食,停止正常饮食时间可超过6小时
C.多数幼儿在急性腹泻期间均能耐受含乳糖的配方奶粉
D.急性腹泻患儿应该继续接受母乳喂养
E.配方奶稀释喂养或者逐渐增加浓度法喂养,均没有必要
B.接受补液的患儿可暂停饮食,停止正常饮食时间可超过6小时
6 3 2008 47. 婴儿腹泻的家庭治疗 (2.0分) A.预防脱水:给患儿口服足够的ORS;
B.锌的补充;
C.持续喂养;
D.及时就医;
E.以上均是
E.以上均是
6 3 2008 48. 有关腹泻的药物治疗,下列哪项不正确? (2.0分) A.不推荐止吐剂常规应用急性腹泻患儿
B.止泻药易蒙停可应用于急性腹泻患儿
C.吸附剂思密达考虑用于急性腹泻患儿的治疗
D.抗分泌药物消旋卡多曲可用于急性腹泻的治疗
E.锌制剂可作为婴幼儿腹泻的常规添加剂
B.止泻药易蒙停可应用于急性腹泻患儿
6 3 2008 49. 治疗股疝一般选用的手术是:(2.0分) A.Bassini法
B. Halsted法
C. Ferguson法
D. Mcvay法
E.Shouldice法
D. Mcvay法
6 3 2008 50. 嵌顿疝和绞窄性疝的区别是: (2.0分) A.疝囊有无压痛
B.疝内容不能还纳的时间
C.疝内容有无动脉血运障碍
D.是否有休克
E.是否有肠梗阻表现
C.疝内容有无动脉血运障碍
6 3 2009 1. 男性,15岁,突发心窝部阵发性钻顶样疼痛6小时,疼痛时大汗淋漓,辗转不安,痛止时又平息如常,体检:剑突偏右方有深在压痛,无腹肌紧张及反跳痛,为明确诊断应采取简单安全的方法是(2.0分) A.BUS
B.右上腹X线平片
C.测定血清淀粉酶
D.十二指肠引流液检查
E.ERCP
A.BUS
6 3 2009 2. 女性,42岁,3年来经常夜间上腹部不适,2日前进油腻食,突然右上腹部阵发性绞痛伴恶心,入院时体温38℃,巩膜轻度黄染,右上腹肌紧张,压痛明显,肠鸣音减弱,WBC16×109/L,血清淀粉酶为128温氏单位,应首先考虑诊断为何种疾病(2.0分) A.高位急性阑尾炎
B.急性胰腺炎
C.溃疡病穿孔
D.急性化脓性胆囊炎
E.胆道蛔虫症
D.急性化脓性胆囊炎
6 3 2009 3. 女性,45岁,突发右上腹及心窝部刀割样绞痛伴阵发性加剧一天,发病后12小时寒战、高热,巩膜黄染,剑突偏右侧深压痛,右上腹轻度肌紧张,体温38℃,WBC 14×109/L血清总胆红素30μmol/L,尿胆原(-),尿胆素(++),应诊断为(2.0分) A.溃疡病穿孔
B.急性胰腺炎
C.急性胆囊炎
D.胆总管结石
E.高位阑尾炎
D.胆总管结石
6 3 2009 4. 女性43岁,突发右上腹剧痛并阵发性加剧3天,寒战高热,恶心呕吐。体检:全身黄染,体温39℃,脉搏120次/分,血压80/60mmHg,谵妄,神志不清,剑突偏左腹肌紧张,肝肋下2cm,WBC15×109/L,中性80%血清总胆红素50μmol/L,本例治疗首选(2.0分) A.抗休克,抗感染治疗
B.胆囊切除,腹腔引流术
C.胆囊造口术
D.胆总管减压引流
E.溃疡病穿孔缝合
D.胆总管减压引流
6 3 2009 5. 女性,43岁,因右上腹阵发性绞痛,伴恶心呕吐4小时来院。检查:体温37℃,右上腹深在轻度压痛,无腹肌紧张,Murphy征阴性,为确诊进一步检查应首选(2.0分) A.白细胞计数和分类
B.腹部X线平片
C.BUS
D.测血清淀粉酶
E.PTC
C.BUS
6 3 2009 6. 胆囊动脉多源于(2.0分) A.肝左动脉
B.肝固有动脉
C.胃十二指肠动脉
D.肝右动脉
E.胃右动脉
D.肝右动脉
6 3 2009 7. 梗阻性黄疸时,BUS显示肝内胆管扩张,胆总管直径2cm,进一步明确梗阻部位的检查是(2.0分) A.放射性核素胰腺扫描
B.低张十二指肠造影
C.PTC
D.ERCP
E.MRI
C.PTC
6 3 2009 8. 梗阻性黄疸时,BUS显示胆总管和肝内胆管均不扩张,为明确诊断应选择哪项检查(2.0分) A.放射性核素胰腺扫描
B.PTC
C.低张十二指肠造影
D.ERCP
E.CT
D.ERCP
6 3 2009 9. 胆囊结石临床表现各异,主要取决于(2.0分) A.结石的大小、部位、梗阻与否、有无感染
B.结石的大小及部位、嵌顿与否
C.体位的改变或静卧状态
D.进油腻食物后
E.结石嵌顿于胆囊颈部
A.结石的大小、部位、梗阻与否、有无感染
6 3 2009 10. Murphy征阳性则提示(2.0分) A.细菌性肝脓肿
B.急性胆管炎
C.肝总管结石
D.左肝管结石
E.急性胆囊炎
E.急性胆囊炎
6 3 2009 11. 胆道手术时,下述哪项不是胆总管探查的指征(2.0分) A.胆总管,肝总管、肝胆管结石
B.黄疸或黄疸病史
C.胰头肿大、变硬
D.胆总管增厚变粗
E.胆囊积液
E.胆囊积液
6 3 2009 12. 急性胆囊炎最严重的并发症是(2.0分) A.细菌性肝脓肿
B.胆囊积脓
C.胆囊坏疸穿死致胆汁性腹膜炎
D.急性胰腺炎
E.胆囊十二指肠内瘘
C.胆囊坏疸穿死致胆汁性腹膜炎
6 3 2009 13. 急性梗阻性化脓性胆管炎最常见的原因是(2.0分) A.胆总管结石
B.胆总管末端狭窄
C.胆道出血继发感染
D.胆总管癌
E.先天性胆总管扩张症
A.胆总管结石
6 3 2009 14. 急性梗阻性化脓性胆管炎,最关键的治疗是(2.0分) A.输液、输血维持有效血容量
B.纠正代谢性酸中毒
C.静脉输入大量抗生素
D.胆道减压引流解除梗阻
E.急诊行胆囊切除术
D.胆道减压引流解除梗阻
6 3 2009 15. 急性梗阻性化脓性胆管炎的治疗一般不宜采取(2.0分) A.胆囊造瘘术
B.纠正水电解质和酸碱平衡失调
C.胆总管探查取石去除病灶
D.使用有效足量的抗生素
E.及时合理使用多巴胺
A.胆囊造瘘术
6 3 2009 16. A 45-year-old obese woman with cholelithiasis presents to the emergency room complaining of nausea and vomiting for 2 days, along with severe continuous midabdominal pain. She has a low-grade fever and the ER physician finds that she has a slightly elevated WBC count (12,000) and an elevated serum amylase. The most likely diagnosis is(2.0分) A.Ruptured abdominal aortic aneurysm
B.Hepatitis
C.Peptic ulcer disease
D.Early phase of acute appendicitis
E.Acute pancreatitis
E.Acute pancreatitis
6 3 2009 17. Laparoscopic cholecystectomy is indicated for symptomatic gallstones in which of the following conditions? (2.0分) A.Cirrhosis
B.Prior upper abdominal surgery
C.Suspected carcinoma of the gallbladder
D.Morbid obesity
E.Coagulopathy
D.Morbid obesity
6 3 2009 18. 男性,52岁,右上腹痛已半年,近半个月出现黄疸,体重下降,食欲减退。既往患胆囊结石已5年。体查:巩膜及全身皮肤黄染,右上腹扪及6cm×6cm肿块,无移动性,BUS显示胆囊部位有一实质性占位病变,并可见强光团及声影。可能诊断: (2.0分) A.胆囊息肉
B.高位胆管癌
C.胆囊癌并胆囊结石
D.充满型胆囊结石
E.幽门管癌并肝十二指肠韧带淋巴结转移
C.胆囊癌并胆囊结石
6 3 2009 19. 男,65岁。皮肤巩膜黄染进行性加重1个月,自述尿色深黄,粪便呈灰白色,查体:胆囊无肿大,murphy征阴性,腹部未触及肿块,首先考虑(2.0分) A.胰头癌
B.胆总管下端癌
C.乏特壶腹癌
D.肝门部胆管癌
E.十二指肠腺癌
D.肝门部胆管癌
6 3 2009 20. 男,57岁。皮肤黄染,尿色变深伴皮肤瘙痒3周。查体:皮肤巩膜变黄,右上腹可触及无痛性圆形肿块,随呼吸上下活动。该肿块可能为: (2.0分) A.胆总管囊肿
B.肝脏囊肿
C.肿大胆囊
D.胰头癌
E.胆管癌
C.肿大胆囊
6 3 2009 21. 肝外胆道解剖特点,下列哪项是错误的: (2.0分) A.胆囊管常有变异
B.胆囊动脉常有变异
C.胆总管末端多与主胰管汇合
D.ODDi括约肌由胆胰管壶腹部括约肌构成
E.胆囊分为颈体底三部分
D.ODDi括约肌由胆胰管壶腹部括约肌构成
6 3 2009 22. 胆管壶腹部癌最重要的症状是: (2.0分) A.波动性黄疸
B.上腹痛及要背疼
C.寒战,发热
D.消化道症状
E.贫血,消瘦
A.波动性黄疸
6 3 2009 23. 黄疸病人合并肿大而无触痛的胆囊时,最有可能是: (2.0分) A.急性胆囊炎
B.慢性胆囊炎,胆囊积水
C.胆囊颈部结石嵌顿
D.中下段胆管癌
E.胆总管下段结石
D.中下段胆管癌
6 3 2009 24. 肝外胆管癌发生率最高的部位是: (2.0分) A.左肝管
B.下段胆管
C.中段胆管
D.上段胆管
E.胆囊管
D.上段胆管
6 3 2009 25. 下面关于GIST的描述哪项不正确(2.0分) A.常见淋巴结转移
B.腔内、外肿块
C.临床表现多见消化道出血
D.肿瘤可液化、坏死及囊变
E.肿瘤可均匀的强化
A.常见淋巴结转移
6 3 2009 26. 关于胃肠淋巴瘤的描述,下列哪项不正确(2.0分) A.病理上以非霍奇金淋巴瘤多见
B.以胃淋巴瘤最多见,其次是小肠,结肠较少见
C.病变肠管可呈动脉瘤样扩张
D.特征性的"三明治征"
E.肿块常有出血、坏死及钙化
E.肿块常有出血、坏死及钙化
6 3 2009 27. 下面哪项描述是错误的(2.0分) A.克罗恩病多见于年轻人
B.是一种缓解与复发交替发生的慢性疾病
C.MRI小肠造影可以帮助明确病变范围及炎性活动情况
D.CT小肠造影可以帮助明确病变范围及炎性活动情况
E.由于CT较MRI检查时间短,推荐使用CT小肠造影检查
E.由于CT较MRI检查时间短,推荐使用CT小肠造影检查
6 3 2009 28. Crohn's diseased 的影像表现,哪项描述是错误的(2.0分) A.好发于回肠末端
B.多节段分部,呈跳跃性
C.炎症活动期的影像特征是肠壁增厚强化明显、"梳征"明显、腹腔脓肿
D.疾病缓解期不出现肠梗阻
E.可并发蜂窝组织炎和瘘管形成
D.疾病缓解期不出现肠梗阻
6 3 2009 29. 有关肠梗阻CT表现的描述不正确的是(2.0分) A.CT也可作为肠梗阻的首选检查方法
B.梗阻部位越低,时间越长时肠腔扩张越明显
C.梗阻以下肠腔萎陷、空虚或存少量粪便
D.梗阻以下肠腔可明显扩张
E.低位肠梗阻时,上段空肠可不扩张
D.梗阻以下肠腔可明显扩张
6 3 2009 30. 肝内肿块,螺旋CT多期扫描表现"快进快出"现象,最可能的疾病是: (2.0分) A.肝细胞癌
B.肝海绵状血管瘤
C.肝囊肿
D.肝脓肿
E.肝转移瘤
A.肝细胞癌
6 3 2009 31. 男性,66岁,3年前结肠癌手术。近来消瘦、纳差,右上腹部痛。CT检查:肝实质多发大小不等结节状病灶,多数病灶中央坏死液化,对比增强结节边缘强化,少数外周有低密度环,形成"牛眼征"应诊断为: (2.0分) A.肝结节性硬化
B.转移性肝癌
C.多发性肝脓肿
D.原发性肝癌
E.肝多发性囊肿
B.转移性肝癌
6 3 2009 32. 男性,65岁,腹痛4个月,黄疸、陶土样大便半个月。检查巩膜、皮肤黄染,心肺未见异常,上腹部轻度压痛,未及包块。CT检查肝内胆管扩张,胆总管中上段扩张,下端狭窄,未见阳性胆结石;MRCP肝内外胆管软藤状扩张,下端狭窄,局部胆管壁增厚。应该考虑为: (2.0分) A.胆囊结石
B.胆管肿瘤
C.胆管结石
D.胆管炎症
E.胆囊炎症
B.胆管肿瘤
6 3 2009 33. 胰腺癌CT表现的直接征象是:(2.0分) A.胰腺肿块
B.胰周脂肪消失
C.肿瘤侵犯血管
D.淋巴结转移
E.胰管和胆管扩张
A.胰腺肿块
6 3 2009 34. 与急性胰腺炎的CT表现不符的是(2.0分) A.胰腺肿大
B.吉氏筋膜增厚
C.蜂窝织炎和假囊肿形成
D.胰腺及胰管钙化
E.可合并脓肿、出血
D.胰腺及胰管钙化
6 3 2009 35. 胰腺钩突前面,CT显示二个血管断面,应是(2.0分) A. (右)门静脉 (左)脾静脉
B. (右)肠系膜上静脉 (左)脾静脉
C. (右)肠系膜上动脉 (左)肠系膜上静脉
D. (右)腹腔动脉 (左)门静脉
E. (右)肠系膜上静脉 (左)肠系膜上动脉
E. (右)肠系膜上静脉 (左)肠系膜上动脉
6 3 2009 36. 以下哪一个上腹部器官不位于腹膜腔:(2.0分) A.肝脏
B.胆囊
C.胰腺
D.脾脏
C.胰腺
6 3 2009 37. 男性,60岁,黄胆,进行性加重,CT显示胰头部肿块,强化不明显,边界不清,主胰管及胆总管扩张,最可能的诊断是(2.0分) A.胰腺导管内乳头状黏液瘤
B.胰腺粘液性囊腺瘤
C.胰腺癌
D.慢性胰腺炎
E.胰腺实性假乳头状瘤
C.胰腺癌
6 3 2009 38. 关于胰腺导管内乳头状黏液性肿瘤,以下认识中错误的是(2.0分) A.是一种胰腺的外分泌性肿瘤,来源于胰腺的导管上皮,呈乳头状生长,分泌黏液
B.好发于老年男性
C.为良性肿瘤
D.病理分为主胰管型、分支胰管型及混合型
E.因黏液栓塞,常引发胰腺炎
C.为良性肿瘤
6 3 2009 39. 急性胆囊炎常见的体征是:(2.0分) A.Murphy 征阳性
B.麦氏点压痛
C.板状腹
D.Courvoisier征阳性
E.Courvoisier征阴性
A.Murphy 征阳性
6 3 2009 40. 常常表现为转移性腹痛的急腹症是:(2.0分) A.急性阑尾炎
B.急性胰腺炎
C.急性胆囊炎
D.急性胆管炎
E.急性胃炎
A.急性阑尾炎
6 3 2009 41. 对于诊断空腔脏器穿孔最有价值的辅助检查是(2.0分) A.B超
B.心电图
C.血常规
D.淀粉酶
E.立位腹部X片
E.立位腹部X片
6 3 2009 42. 高位肠梗阻除腹痛以外最明显的表现是(2.0分) A.停止排气排便
B.腹胀明显
C.频繁呕吐
D.可扪及腹部肿块
E.肠鸣音减弱
C.频繁呕吐
6 3 2009 43. 男,42岁,因右上腹疼痛伴高热5小时入院。查体:神智清楚,体温39.3摄氏度,心率:95次/分,血压119/75mmHg。皮肤巩膜黄染,右上腹压痛,肌紧张,肝区叩痛。最可能的诊断是(2.0分) A.急性胆管炎
B.AOSC
C.急性胰腺炎
D.急性胃穿孔
E.急性胆囊炎
A.急性胆管炎
6 3 2009 44. Which is one of 'tension-free hernioblasty'?(2.0分) A.Bassini repair
B.Shouldice repair
C.McVay repair
D.Lichtenstein repair
E.None of above
D.Lichtenstein repair
6 3 2009 45. Which kind of hernias are the most prone to incarcerated ?(2.0分) A.Incisional hernia.
B.indirect inguinal hernia
C.direct inguinal hernia
D.sliding hernia.
E.femoral hernia
E.femoral hernia
6 3 2009 46. 左侧腹股沟滑动性疝,下列哪项是正确的(2.0分) A.属于可复性疝
B.疝内容物没有小肠
C.乙状结肠是疝囊的一部分
D.最易嵌顿
E.疝块很小
C.乙状结肠是疝囊的一部分
6 3 2009 47. 嵌顿疝与绞窄疝鉴别要点是:(2.0分) A.疝块不能回纳
B.疝块是否有压痛
C.绞窄疝出现呕吐
D.绞窄疝出现休克
E.绞窄疝内容物发生血循环障碍
E.绞窄疝内容物发生血循环障碍
6 3 2009 48. 以下哪项不是胰腺癌高危因素: (2.0分) A.胰腺癌家族史
B.突发糖尿病者
C.慢性胰腺炎
D.吸烟、大量饮酒
E.胆道结石
E.胆道结石
6 3 2009 49. 以下哪项是胰腺癌手术切除的禁忌症: (2.0分) A.肿瘤大于4cm
B.后腹膜淋巴结肿大
C.肠系膜上动脉肿瘤包绕侵犯
D.肠系膜上静脉肿瘤包绕侵犯
E.肿瘤侵犯十二指肠、胆总管
C.肠系膜上动脉肿瘤包绕侵犯
6 3 2009 50. 胰腺癌切除率低的主要原因为(2.0分) A.癌直接浸润和转移
B.癌的恶性程度高
C.并发胆道疾病
D.年老,体弱
E.手术复杂
A.癌直接浸润和转移
6 3 2010 1. 女性,47岁,胆囊结石病史4年,曾先后发作性胆绞痛4次,BUS显示胆囊内充满型结石。首选的治疗方法是下列哪一种(2.0分) A.口服排石饮液
B.口服熊去氧胆酸片
C.口服利胆素片
D.胆囊切除术
E.ERCP取石术
D.胆囊切除术
6 3 2010 2. 女性,42岁,3年来经常夜间上腹部不适,2日前进油腻食,突然右上腹部阵发性绞痛伴恶心,入院时体温38℃,巩膜轻度黄染,右上腹肌紧张,压痛明显,肠鸣音减弱,WBC16×109/L,血清淀粉酶为128温氏单位,应首先考虑诊断为何种疾病(2.0分) A.高位急性阑尾炎
B.急性胰腺炎
C.溃疡病穿孔
D.急性化脓性胆囊炎
E.胆道蛔虫症
D.急性化脓性胆囊炎
6 3 2010 3. 女性43岁,突发右上腹剧痛并阵发性加剧3天,寒战高热,恶心呕吐。体检:全身黄染,体温39℃,脉搏120次/分,血压80/60mmHg,谵妄,神志不清,剑突偏左腹肌紧张,肝肋下2cm,WBC15×109/L,中性80%血清总胆红素50μmol/L,本例治疗首选(2.0分) A.ENBD
B.胆囊切除,腹腔引流术
C.胆囊造口术
D.开腹胆总管探查术
E.溃疡病穿孔缝合
A.ENBD
6 3 2010 4. 男性,50岁,3年前曾行胆总管切开取石术,现持续黄疸一个月,肝大肋下3cm,血清胆红素为70μmol/L,BUS显示胆总管直径2cm,下述何种检查方法是最优(2.0分) A.口服法胆囊造影术
B.静脉法胆道造影术
C.低张性十二指肠造影
D.MRCP
E.CT
D.MRCP
6 3 2010 5. 男性,52岁,右上腹痛已半年,近半个月出现黄疸,体重下降,食欲减退。既往患胆囊结石已5年。检查:巩膜及全身皮肤黄染,右上腹扪及的6cm×6cm肿块,无移动性,BUS显示胆囊部位有一实质性占位病变,并可见强光团及声影。此病人可能的诊断是(2.0分) A.胆囊息肉
B.高位胆管癌
C.胆囊癌并胆囊结石
D.充满型胆囊结石
C.胆囊癌并胆囊结石
6 3 2010 6. 放射性核素胰腺扫描(2.0分) A.低张十二指肠造影
B.PTC
C.ERCP
D.MRI
C.ERCP
6 3 2010 7. 结石的大小、部位、梗阻与否、有无感染(2.0分) A.结石的大小及部位、嵌顿与否
B.体位的改变或静卧状态
C.进油腻食物后
D.结石嵌顿于胆囊颈部
A.结石的大小及部位、嵌顿与否
6 3 2010 8. BUS(2.0分) A.ERCP
B.PTC
C.口服法胆囊造影
D.十二指肠引流术
A.ERCP
6 3 2010 9. 腹痛、呕吐、发热(2.0分) A.腹痛、呕吐、黄疸
B.腹痛、黄疸、腹泻
C.腹痛、腹胀、昏迷
D.腹痛、黄疸、发热
NaN
6 3 2010 10. 细菌性肝脓肿(2.0分) A.胆囊积脓
B.胆囊坏疸穿死致胆汁性腹膜炎
C.急性胰腺炎
D.胆囊十二指肠内瘘
C.急性胰腺炎
6 3 2010 11. 输液、输血维持有效血容量(2.0分) A.纠正代谢性酸中毒
B.静脉输入大量抗生素
C.胆道减压引流解除梗阻
D.急诊行胆囊切除术
D.急诊行胆囊切除术
6 3 2010 12. severe, constant epigastric pain that radiates to his midback. Further evaluation finds fever, steatorrhea, and discoloration around his flank and umbilicus. Laboratory tests find elevated serum levels of amylase and lipase. What is the most likely cause of these findings? (2.0分) A.Acute appendicitis
B.Acute cholangitis
C.Acute cholecystitis
D.Acute diverticulitis
E.Acute pancreatitis
E.Acute pancreatitis
6 3 2010 13. Laparoscopic cholecystectomy is indicated for symptomatic gallstones in which of the following conditions?(2.0分) A.Cirrhosis
B.Prior upper abdominal surgery
C.Suspected carcinoma of the gallbladder
D.Morbid obesity
E.Coagulopathy
D.Morbid obesity
6 3 2010 14. An 88-year-old man with a history of end-stage renal failure, severe coronary artery disease, and brain metastases from lung cancer presents with acute cholecystitis. His family wants "everything done." The best management option in this patient would be (2.0分) A.Tube cholecystostomy
B.Open cholecystectomy
C.Laparoscopic cholecystectomy
D.Intravenous antibiotics followed by elective cholecystectomy
E.Lithotripsy followed by long-term bile acid therapy
A.Tube cholecystostomy
6 3 2010 15. 临床上壶腹癌最重要的症状是(2.0分) A.寒战、发热
B.上腹疼及腰背疼
C.黄疸
D.消化道症状
E.贫血、消瘦
C.黄疸
6 3 2010 16. 壶腹部癌的切除率和5年生存率都高于胰头癌,主要原因是(2.0分) A.恶性程度低
B.转移较晚
C.症状岀现早,能作岀早期诊断和治疗
D.手术方式简单
E.病人年龄较轻
C.症状岀现早,能作岀早期诊断和治疗
6 3 2010 17. 胰腺癌切除率低的主要原因为(2.0分) A.癌直接浸润和转移
B.癌的恶性程度高
C.并发胆道疾病
D.年老,体弱
E.手术复杂
A.癌直接浸润和转移
6 3 2010 18. 诊断胰岛素瘤的最准确方法是(2.0分) A.Wipple三联征
B.血糖空腹时2.2mmol.L以下
C.葡萄糖耐量试验
D.饥饿试验
E.血胰岛素测定
E.血胰岛素测定
6 3 2010 19. 下列CT影像中,最支持胰腺癌诊断的是(2.0分) A.胰周及腹腔渗出,胰腺实质内有不规则低密度区,强化后低密度区增强不明显
B.胰头有3cmx4cm不均匀低密度区,强化后有不均匀增强
C.胰头部增大,密度与其他部位胰腺组织密度一致,胰周界限模糊
D.胰头均匀低密度区,CT值10 Hu,不被强化,胰体尾萎缩,胰管扩张
E.胆管扩张明显,胆总管下端可见2 cmx1 cm的极强密度区
B.胰头有3cmx4cm不均匀低密度区,强化后有不均匀增强
6 3 2010 20. 关于肝脏分段下列哪些是错误的(2.0分) A.肝尾状叶为IV段
B.肝左叶内、外侧段以肝左静脉为界
C.肝VIII段与肝IV相邻
D.肝VII段位于肝VI上方
E.门静脉水平区分肝上下段
A.肝尾状叶为IV段
6 3 2010 21. 肝内肿块,螺旋CT多期扫描表现"快进快出"现象,最可能的疾病是(2.0分) A.肝细胞癌
B.肝海绵状血管瘤
C.肝囊肿
D.肝脓肿
E.肝转移瘤
A.肝细胞癌
6 3 2010 22. 下列有关肝硬化的CT表现的描述哪项不确切? (2.0分) A.中晚期肝叶比例失调
B.肝脏边缘轮廓凹凸不平
C.门脉高压可见门脉侧支循环形成
D.肝脏密度均匀
E.常合并有恶变
D.肝脏密度均匀
6 3 2010 23. 女,60岁,皮肤巩膜黄染17天,无明显发热,CA199明显高于正常值,MRCP提示肝门部胆管梗阻,胆总管显示正常,CT增强提示肝门部胆管壁增厚伴强化明显,肝门区见17mm淋巴结,强化密度不均,后腹膜区亦见多个强化密度不均、肿大的淋巴结。最可能的诊断是(2.0分) A.肝门部胆管结石
B.化脓性胆管炎
C.肝门部胆管癌
D.胆管结核
E.硬化性胆管炎
C.肝门部胆管癌
6 3 2010 24. 与急性胰腺炎的CT表现不符的是(2.0分) A.胰腺肿大
B.吉氏筋膜增厚
C.蜂窝织炎和假性囊肿形成
D.胰腺及胰管钙化
E.可合并脓肿、出血
D.胰腺及胰管钙化
6 3 2010 25. 男性,60岁,黄胆,进行性加重,CT显示胰头部肿块,强化不明显,边界不清,主胰管及胆总管扩张,最可能的诊断是(2.0分) A.胰腺导管内乳头状黏液瘤
B.胰腺粘液性囊腺瘤
C.胰腺癌
D.慢性胰腺炎
E.胰腺实性假乳头状瘤
C.胰腺癌
6 3 2010 26. 下面哪项描述是错误的(2.0分) A.克罗恩病多见于年轻人
B.是一种缓解与复发交替发生的慢性疾病
C.MRI小肠造影可以帮助明确病变范围及炎性活动情况
D.CT小肠造影可以帮助明确病变范围及炎性活动情况
E.由于CT较MRI检查时间短,推荐使用CT小肠造影检查
E.由于CT较MRI检查时间短,推荐使用CT小肠造影检查
6 3 2010 27. 男性,69岁,腹水,乏力,CT提示大量腹水,腹膜结节样增厚、大网膜弥漫结节影,下面描述哪项可能性最小(2.0分) A.肿瘤腹腔种植转移可能
B.发脓性腹膜炎可能
C.结核性腹膜炎可能
D.腹膜间皮瘤可能
E.结节病可能
B.发脓性腹膜炎可能
6 3 2010 28. 男性,48岁,贫血,大便隐性(+++),肠镜提示直肠癌,下面描述错误的是(2.0分) A.对肿瘤分期评估,MRI优于CT
B.直肠系膜内见6mm环形强化淋巴结,提示淋巴结转移
C.T3肿瘤距离直肠系膜筋膜小于1mm,提示筋膜受侵
D.高位直肠癌侵犯腹膜返折,为T4b
E.低位直肠癌需评估肛管是否受侵犯
E.低位直肠癌需评估肛管是否受侵犯
6 3 2010 29. 胃肠道穿孔的主要X线征象是(2.0分) A.胃泡增大,胃膈间距增大
B.膈下游离气体
C.麻痹性肠梗阻
D.肠管充气扩张
E.气液平出现
B.膈下游离气体
6 3 2010 30. Which is one of 'tension-free hernioblasty'?(2.0分) A.Bassini repair
B.Shouldice repair
C.McVay repair
D.Lichtenstein repair
E.None of above
D.Lichtenstein repair
6 3 2010 31. 左侧腹股沟滑动性疝,下列哪项是正确的(2.0分) A.属于可复性疝
B.疝内容物没有小肠
C.乙状结肠是疝囊的一部分
D.最易嵌顿
E.疝块很小
C.乙状结肠是疝囊的一部分
6 3 2010 32. 急腹症最重要的体格检查方法是(2.0分) A.望诊
B.触诊
C.叩诊
D.听诊
E.直肠指检
B.触诊
6 3 2010 33. 常伴有感染性休克的急腹症是(2.0分) A.急性阑尾炎
B.急性胆囊炎
C.急性化脓性胆管炎
D.急性水肿型胰腺炎
E.急性粘连性肠梗阻
C.急性化脓性胆管炎
6 3 2010 34. 下列关于腹痛的描述错误的是(2.0分) A.机械性肠梗阻可表现为阵发性疼痛
B.急性阑尾炎早期可表现为上腹痛
C.急性胆囊炎可表现为剑突下痛
D.输尿管下段结石可出现会阴部放射痛
E.急性胰腺炎上腹痛可伴右肩痛
E.急性胰腺炎上腹痛可伴右肩痛
6 3 2010 35. 男,45岁。腹部剧烈疼痛2小时入院。既往有剑突下疼痛病史5年,夜间疼痛加重。查体:心率107次/分,血压97/59mmHg。腹平,全腹压痛反跳痛,板状腹,肠鸣音消失。该患者最有意义的检查是 (2.0分) A.血清淀粉酶
B.尿淀粉酶
C.腹部B超
D.立位腹平片
E.血常规
D.立位腹平片
6 3 2010 36. 男,55岁。腹部剧烈疼痛3小时入院。尿少色黄。查体:心率107次/分,血压80/59mmHg。口唇干燥,精神萎靡。腹平,全腹压痛反跳痛,板状腹,肠鸣音消失。WBC 20×109/L,N97%,腹平片提示膈下游离气体。首要的的治疗是 (2.0分) A.腹部CT扫描,明确病因
B.应用大剂量广谱抗生素
C.立即剖腹探查
D.抗休克的同时准备手术
E.利尿
D.抗休克的同时准备手术
6 3 2010 37. 以下哪项考虑慢性腹泻病 (2.0分) A.3岁幼儿大便呈水样,>3次/日,病程18日
B.3月龄母乳喂养儿生后持续稀糊状大便,5次/日
C.幼儿大便呈粘液脓血样,病程7日
D.便秘幼儿近3天日解2次稀水便
E.人工喂养儿大便突呈稀水样,每天4-6次,持续5日
B.3月龄母乳喂养儿生后持续稀糊状大便,5次/日
6 3 2010 38. 下列哪项不是儿童腹泻病的易感因素? (2.0分) A.消化系统发育尚未成熟
B.胃肠道负担重
C.机体防御功能较差
D.肠道菌群失调
E.母乳喂养
E.母乳喂养
6 3 2010 39. 下列哪项不符合重型腹泻的特点? (2.0分) A.多由肠道内感染引起,常急性起病
B.常由饮食因素或肠道外感染引起
C.胃肠道症状较重
D.明显的脱水和电解质、酸碱平衡紊乱
E.全身感染中毒症状明显
B.常由饮食因素或肠道外感染引起
6 3 2010 40. 低渗性脱水时,血钠浓度为(2.0分) A.<140mmol/L
B.<145mmol/L
C.<125mmol/L
D.<130mmol/L
E.<135mmol/L
D.<130mmol/L
6 3 2010 41. 以下说法正确的是(2.0分) A.一旦明确致病病原体,急性腹泻病人才能得到合理的诊治
B.粪便培养是腹泻的常规检查
C.粪便中一旦出现白细胞,则提示细菌感染
D.不管病原体检查结果,腹泻患儿均可补液治疗
E.细菌性腹泻均应该应用抗菌素的治疗
D.不管病原体检查结果,腹泻患儿均可补液治疗
6 3 2010 42. 有关抗生素相关性腹泻,下列哪项不正确? (2.0分) A.使用抗生素史
B.可表现为发热、呕吐、腹泻
C.可伴脱水、电解质紊乱
D.粪培养可有大肠埃希氏菌生长
E.粪培养可有金黄色葡萄球菌生长
D.粪培养可有大肠埃希氏菌生长
6 3 2010 43. 有关WHO低渗ORS配方与第二代标准配方相比,下列哪项说法不对(2.0分) A.钠浓度降低
B.钾浓度降低
C.葡萄糖浓度降低
D. 氯浓度降低
E.渗透压降低
B.钾浓度降低
6 3 2010 44. 饮食治疗是婴幼儿腹泻治疗中的关键因素之一,以下说法不合理的是(2.0分) A.无脱水症状的腹泻儿应该喂食与患儿年龄相当的正常饮食
B.接受补液的患儿可暂停饮食,停止正常饮食时间不超过4小时
C.多数幼儿在急性腹泻期间均能耐受含乳糖的配方奶粉
D.急性腹泻患儿应该继续接受母乳喂养
E.配方奶稀释喂养或者逐渐增加浓度法喂养,均有必要
E.配方奶稀释喂养或者逐渐增加浓度法喂养,均有必要
6 3 2010 45. 下列疾病没有抗感染治疗指征的是(2.0分) A.艰难梭菌感染
B.志贺氏菌感染
C.产志贺毒素大肠埃希氏菌感染
D.伤寒沙门菌感染
E.弯曲杆菌感染
C.产志贺毒素大肠埃希氏菌感染
6 3 2010 46. 有关静脉补液原则,下列哪项符合? (2.0分) A.三定(定量、定性、定速)
B.三先(先盐后糖、先浓后淡、先快后慢)
C.两补(见尿补钾、惊跳补钙)
D.强调个体化补液
E.以上均是
E.以上均是
6 3 2010 47. 有关液体的张力,下列哪项不正确(2.0分) A.5%葡萄糖液,不产生张力
B.10%葡萄糖液,产生1个张力
C.10%氯化钠溶液,产生11个张力
D.0.9%氯化钠溶液,产生1个张力
E.5%碳酸氢钠溶液,产生3.5个张力
B.10%葡萄糖液,产生1个张力
6 3 2010 48. 有关鼻饲补液,下列哪项不正确? (2.0分) A.重度脱水如无输液条件,立即转运到就近医院进行静脉补液,转运途中可以用鼻饲点滴方法进行补液。
B.液体采用ORS液,以20ml/(kg.h)的速度补充。
C.如患儿反复呕吐或腹胀,放慢鼻饲点滴速度,总量不超过120ml/kg。
D.如有肠梗阻时也可以鼻饲补液。
E. 每1-2h评估1次患者脱水情况
D.如有肠梗阻时也可以鼻饲补液。
6 3 2010 49. 微生态制剂治疗急性腹泻,下列哪项不正确? (2.0分) A.微生态制剂对急性腹泻具有辅助治疗作用
B.微生态制剂治疗腹泻的疗效有菌种依赖性
C.微生态制剂治疗腹泻的疗效没有剂量依赖性
D.鼠李乳杆菌和布拉氏酵母菌证明有效
E.不推荐益生元制剂用于治疗婴幼儿急性腹泻
C.微生态制剂治疗腹泻的疗效没有剂量依赖性
6 3 2010 50. 胆囊动脉多源于(2.0分) A.肝左动脉
B.肝固有动脉
C.胃十二指肠动脉
D.肝右动脉
E.胃右动脉
D.肝右动脉
6 3 2011 1. 男性,50岁,3年前曾行胆总管切开取石术,现持续黄疸一个月,肝大肋下3cm,血清胆红素为70μmol/L,BUS显示胆总管直径2cm,下述何种检查方法是最优(2.5分) A.口服法胆囊造影术
B.静脉法胆道造影术
C.低张性十二指肠造影
D.MRCP
E.CT
D.MRCP
6 3 2011 2. 女性,42岁,3年来经常夜间上腹部不适,2日前进油腻食,突然右上腹部阵发性绞痛伴恶心,入院时体温38℃,巩膜轻度黄染,右上腹肌紧张,压痛明显,肠鸣音减弱,WBC16×109/L,血清淀粉酶为128温氏单位,应首先考虑诊断为何种疾病(2.5分) A.高位急性阑尾炎
B.急性胰腺炎
C.溃疡病穿孔
D.急性化脓性胆囊炎
E.胆道蛔虫症
D.急性化脓性胆囊炎
6 3 2011 3. 女性43岁,突发右上腹剧痛并阵发性加剧3天,寒战高热,恶心呕吐。体检:全身黄染,体温39℃,脉搏120次/分,血压80/60mmHg,谵妄,神志不清,剑突偏左腹肌紧张,肝肋下2cm,WBC15×109/L,中性80%血清总胆红素50μmol/L,本例治疗首选(2.5分) A.ENBD
B.胆囊切除,腹腔引流术
C.胆囊造口术
D.开腹胆总管探查术
E.溃疡病穿孔缝合
A.ENBD
6 3 2011 4. 男性,43岁,突然右上腹绞痛,并出现寒战高热,于16小时后出现明显黄疸。检查:意识尚清楚,巩膜及周身黄染,BP130/70mmHg,体温38.5℃,右上腹轻度压痛,此病最可能为(2.5分) A.Courvoisier征
B.Murphy征
C.Charcot征
D.Grey-Turner征
E.Reynold征
C.Charcot征
6 3 2011 5. 梗阻性黄疸时,BUS显示肝内胆管扩张,胆总管直径2cm,进一步明确梗阻部位的检查是(2.5分) A.放射性核素胰腺扫描
B.低张十二指肠造影
C.PTC
D.ERCP
E.MRI
C.PTC
6 3 2011 6. Murphy征阳性则提示(2.5分) A.细菌性肝脓肿
B.急性胆管炎
C.肝总管结石
D.左肝管结石
E.急性胆囊炎
E.急性胆囊炎
6 3 2011 7. 胆管结石和急性胆管炎急性发作的典型症状是(2.5分) A.腹痛、呕吐、发热
B.腹痛、呕吐、黄疸
C.腹痛、黄疸、腹泻
D.腹痛、腹胀、昏迷
E.腹痛、黄疸、发热
E.腹痛、黄疸、发热
6 3 2011 8. 急性梗阻性化脓性胆管炎最常见的原因是(2.5分) A.胆总管结石
B.胆总管末端狭窄
C.胆道出血继发感染
D.胆总管癌
E.先天性胆总管扩张症
A.胆总管结石
6 3 2011 9. A 54-year-old male alcoholic presents with the sudden onset of severe, constant epigastric pain that radiates to his midback. Further evaluation finds fever, steatorrhea, and discoloration around his flank and umbilicus. Laboratory tests find elevated serum levels of amylase and lipase. What is the most likely cause of these findings? (2.5分) A.Acute appendicitis
B.Acute cholangitis
C.Acute cholecystitis
D.Acute diverticulitis
E.Acute pancreatitis
E.Acute pancreatitis
6 3 2011 10. Laparoscopic cholecystectomy is indicated for symptomatic gallstones in which of the following conditions? (2.5分) A.Cirrhosis
B.Prior upper abdominal surgery
C.Suspected carcinoma of the gallbladder
D.Morbid obesity
E.Coagulopathy
D.Morbid obesity
6 3 2011 11. 临床上壶腹癌最重要的症状是 (2.5分) A.寒战、发热
B.上腹疼及腰背疼
C.黄疸
D.消化道症状
E.贫血、消瘦
C.黄疸
6 3 2011 12. 壶腹部癌的切除率和5年生存率都高于胰头癌,主要原因是(2.5分) A.恶性程度低
B.转移较晚
C.症状岀现早,能作岀早期诊断和治疗
D.手术方式简单
E.病人年龄较轻
C.症状岀现早,能作岀早期诊断和治疗
6 3 2011 13. 胰腺癌切除率低的主要原因为(2.5分) A.癌直接浸润和转移
B.癌的恶性程度高
C.并发胆道疾病
D.年老,体弱
E.手术复杂
A.癌直接浸润和转移
6 3 2011 14. 诊断胰岛素瘤的最准确方法是(2.5分) A.Wipple三联征
B.血糖空腹时2.2mmol.L以下
C.葡萄糖耐量试验
D.饥饿试验
E.血胰岛素测定
E.血胰岛素测定
6 3 2011 15. 下列CT影像中,最支持胰腺癌诊断的是(2.5分) A.胰周及腹腔渗出,胰腺实质内有不规则低密度区,强化后低密度区增强不明显
B.胰头有3cmx4cm不均匀低密度区,强化后有不均匀增强
C.胰头部增大,密度与其他部位胰腺组织密度一致,胰周界限模糊
D.胰头均匀低密度区,CT值10 Hu,不被强化,胰体尾萎缩,胰管扩张
E.胆管扩张明显,胆总管下端可见2 cmx1 cm的极强密度区
B.胰头有3cmx4cm不均匀低密度区,强化后有不均匀增强
6 3 2011 16. Which kind of hernias are the most prone to incarcerated ? (2.5分) A.Incisional hernia.
B.indirect inguinal hernia
C.direct inguinal hernia
D.sliding hernia.
E.femoral hernia
E.femoral hernia
6 3 2011 17. 左侧腹股沟滑动性疝,下列哪项是正确的(2.5分) A.属于可复性疝
B.疝内容物没有小肠
C.乙状结肠是疝囊的一部分
D.最易嵌顿
E.疝块很小
C.乙状结肠是疝囊的一部分
6 3 2011 18. 急腹症最重要的体格检查方法是(2.5分) A.望诊
B.触诊
C.叩诊
D.听诊
E.直肠指检
B.触诊
6 3 2011 19. 常伴有感染性休克的急腹症是(2.5分) A.急性阑尾炎
B.急性胆囊炎
C.急性化脓性胆管炎
D.急性水肿型胰腺炎
E.急性粘连性肠梗阻
C.急性化脓性胆管炎
6 3 2011 20. 下列关于腹痛的描述错误的是(2.5分) A.机械性肠梗阻可表现为阵发性疼痛
B.急性阑尾炎早期可表现为上腹痛
C.急性胆囊炎可表现为剑突下痛
D.输尿管下段结石可出现会阴部放射痛
E.急性胰腺炎上腹痛可伴右肩痛
E.急性胰腺炎上腹痛可伴右肩痛
6 3 2011 21. 男,45岁。腹部剧烈疼痛2小时入院。既往有剑突下疼痛病史5年,夜间疼痛加重。查体:心率107次/分,血压97/59mmHg。腹平,全腹压痛反跳痛,板状腹,肠鸣音消失。该患者最有意义的检查是(2.5分) A.血清淀粉酶
B.尿淀粉酶
C.腹部B超
D.立位腹平片
E.血常规
D.立位腹平片
6 3 2011 22. 男,55岁。腹部剧烈疼痛3小时入院。尿少色黄。查体:心率107次/分,血压80/59mmHg。口唇干燥,精神萎靡。腹平,全腹压痛反跳痛,板状腹,肠鸣音消失。WBC 20×109/L,N97%,腹平片提示膈下游离气体。首要的的治疗是(2.5分) A.腹部CT扫描,明确病因
B.应用大剂量广谱抗生素
C.立即剖腹探查
D.抗休克的同时准备手术
E.利尿
D.抗休克的同时准备手术
6 3 2011 23. 下列哪种为非感染性腹泻?(2.5分) A.病毒
B.细菌
C.真菌
D.症状性
E.原虫
D.症状性
6 3 2011 24. 下列哪种细菌不属于大肠埃希氏菌?(2.5分) A.EPEC
B.CD
C.EAEC
D.STEC
E.EITC
B.CD
6 3 2011 25. 轮状病毒感染引起腹泻的机制主要是(2.5分) A.渗出性
B.肠动力性
C.侵袭性
D.渗透性
E.菌群失调
D.渗透性
6 3 2011 26. 高渗性脱水时,血钠浓度为(2.5分) A.>130mmol/L
B.>135mmol/L
C.>140mmol/L
D.>145mmol/L
E.>150mmol/L
E.>150mmol/L
6 3 2011 27. 以下说法正确的是(2.5分) A.一旦明确致病病原体,急性腹泻病人才能得到合理的诊治
B.粪便培养是腹泻的常规检查
C.粪便中一旦出现白细胞,则提示细菌感染
D.不管病原体检查结果,腹泻患儿均可补液治疗
E.细菌性腹泻均应该应用抗菌素的治疗
D.不管病原体检查结果,腹泻患儿均可补液治疗
6 3 2011 28. 有关抗生素相关性腹泻,下列哪项不正确?(2.5分) A.使用抗生素史
B.可表现为发热、呕吐、腹泻
C.可伴脱水、电解质紊乱
D.粪培养可有大肠埃希氏菌生长
E.粪培养可有金黄色葡萄球菌生长
D.粪培养可有大肠埃希氏菌生长
6 3 2011 29. 有关WHO低渗ORS配方与第二代标准配方相比,下列哪项说法不对(2.5分) A.钠浓度降低
B.钾浓度降低
C.葡萄糖浓度降低
D.氯浓度降低
E.渗透压降低
B.钾浓度降低
6 3 2011 30. 有关腹泻的药物治疗,下列哪项不正确?(2.5分) A.不推荐止吐剂常规应用急性腹泻患儿
B.止泻药易蒙停可应用于急性腹泻患儿
C.吸附剂思密达考虑用于急性腹泻患儿的治疗
D.抗分泌药物消旋卡多曲可用于急性腹泻的治疗
E.锌制剂可作为婴幼儿腹泻的常规添加剂
B.止泻药易蒙停可应用于急性腹泻患儿
6 3 2011 31. 有关液体的张力,下列哪项不正确(2.5分) A.5%葡萄糖液,不产生张力
B.10%葡萄糖液,产生1个张力
C.10%氯化钠溶液,产生11个张力
D.0.9%氯化钠溶液,产生1个张力
E.5%碳酸氢钠溶液,产生3.5个张力
B.10%葡萄糖液,产生1个张力
6 3 2011 32. 关于肝脏分段下列哪些是错误的(2.5分) A.肝尾状叶为IV段
B.肝左叶内、外侧段以肝左静脉为界
C.肝VIII段与肝IV相邻
D.肝VII段位于肝VI上方
E.门静脉水平区分肝上下段
A.肝尾状叶为IV段
6 3 2011 33. 肝内肿块,螺旋CT多期扫描表现"快进快出"现象,最可能的疾病是(2.5分) A.肝细胞癌
B.肝海绵状血管瘤
C.肝囊肿
D.肝脓肿
E.肝转移瘤
A.肝细胞癌
6 3 2011 34. 下列有关肝硬化的CT表现的描述哪项不确切(2.5分) A.中晚期肝叶比例失调
B.肝脏边缘轮廓凹凸不平
C.门脉高压可见门脉侧支循环形成
D.肝脏密度均匀
E.常合并有恶变
D.肝脏密度均匀
6 3 2011 35. 女,60岁,皮肤巩膜黄染17天,无明显发热,CA199明显高于正常值,MRCP提示肝门部胆管梗阻,胆总管显示正常,CT增强提示肝门部胆管壁增厚伴强化明显,肝门区见17mm淋巴结,强化密度不均,后腹膜区亦见多个强化密度不均、肿大的淋巴结。最可能的诊断是(2.5分) A.肝门部胆管结石
B.化脓性胆管炎
C.肝门部胆管癌
D.胆管结核
E.硬化性胆管炎
C.肝门部胆管癌
6 3 2011 36. 与急性胰腺炎的CT表现不符的是(2.5分) A.胰腺肿大
B.吉氏筋膜增厚
C.蜂窝织炎和假性囊肿形成
D.胰腺及胰管钙化
E.可合并脓肿、出血
D.胰腺及胰管钙化
6 3 2011 37. 男性,60岁,黄胆,进行性加重,CT显示胰头部肿块,强化不明显,边界不清,主胰管及胆总管扩张,最可能的诊断是(2.5分) A.胰腺导管内乳头状黏液瘤
B.胰腺粘液性囊腺瘤
C.胰腺癌
D.慢性胰腺炎
E.胰腺实性假乳头状瘤
C.胰腺癌
6 3 2011 38. Crohn's disease 的影像表现,哪项描述是错误的(2.5分) A.好发于回肠末端
B.常表现多节段、跳跃性
C.炎症活动期的影像特征是肠壁增厚、水肿、强化明显、肠壁溃疡、"梳征"明显、腹腔脓肿
D.疾病缓解期不出现肠梗阻
E.可并发蜂窝组织炎和瘘管形成
D.疾病缓解期不出现肠梗阻
6 3 2011 39. 有关肠梗阻CT表现的描述不正确的是(2.5分) A.CT可作为肠梗阻的首选检查方法
B.梗阻点表现为其近段肠管扩张,远段肠管塌陷
C.肠壁强化减弱或无强化,提示肠壁血运障碍
D.都可发现梗阻点
E.CT检查需回答有无肠梗阻、梗阻点、梗阻原因、有无绞窄
D.都可发现梗阻点
6 3 2011 40. 男性,69岁,血便,下面叙述错误的是(2.5分) A.增强CT扫描,见对比剂外渗,提示活动性出血
B.增强CT扫描,未见对比剂外渗,排除活动性出血
C.增强CT扫描未见明显异常,可行DSA检查
D.肠壁血管发育不良性出血,可自愈
E.AVM常有静脉早显
B.增强CT扫描,未见对比剂外渗,排除活动性出血
6 3 2012 1. 男性,15岁,突发心窝部阵发性钻顶样疼痛6小时,疼痛时大汗淋漓,辗转不安,痛止时又平息如常,体检:剑突偏右方有深在压痛,无腹肌紧张及反跳痛,为明确诊断应采取简单安全的方法是(2.5分) A.BUS
B.右上腹X线平片
C.测定血清淀粉酶
D.十二指肠引流液检查
E.ERCP
A.BUS
6 3 2012 2. 女性,45岁,突发右上腹及心窝部刀割样绞痛伴阵发性加剧一天,发病后12小时寒战、高热,巩膜黄染,剑突偏右侧深压痛,右上腹轻度肌紧张,体温38℃,WBC 14×109/L血清总胆红素30μmol/L,尿胆原(-),尿胆素(++),应诊断为(2.5分) A.溃疡病穿孔
B.急性胰腺炎
C.急性胆囊炎
D.胆总管结石
E.高位阑尾炎
D.胆总管结石
6 3 2012 3. 女性,43岁,因右上腹阵发性绞痛,伴恶心呕吐4小时来院。检查:体温37℃,右上腹深在轻度压痛,无腹肌紧张,Murphy征阴性,为确诊进一步检查应首选(2.5分) A.白细胞计数和分类
B.腹部X线平片
C.BUS
D.测血清淀粉酶
E.PTC
C.BUS
6 3 2012 4. 男性,43岁,突然右上腹绞痛,并出现寒战高热,于16小时后出现明显黄疸。检查:意识尚清楚,巩膜及周身黄染,BP130/70mmHg,体温38.5℃,右上腹轻度压痛,此病最可能为(2.5分) A.Courvoisier征
B.Murphy征
C.Charcot征
D.Grey-Turner征
E.Reynold征
C.Charcot征
6 3 2012 5. 胆囊动脉多源于(2.5分) A.肝左动脉
B.肝固有动脉
C.胃十二指肠动脉
D.肝右动脉
E.胃右动脉
D.肝右动脉
6 3 2012 6. 梗阻性黄疸时,BUS显示胆总管和肝内胆管均不扩张,为明确诊断应选择哪项检查(2.5分) A.放射性核素胰腺扫描
B.PTC
C.低张十二指肠造影
D.ERCP
E.CT
D.ERCP
6 3 2012 7. Murphy征阳性则提示(2.5分) A.细菌性肝脓肿
B.急性胆管炎
C.肝总管结石
D.左肝管结石
E.急性胆囊炎
E.急性胆囊炎
6 3 2012 8. Charcot三联征间歇发作最大的可能是(2.5分) A.壶腹部癌
B.肝细胞癌
C.胆总管结石
D.黄疸型肝炎
E.细菌性肝脓肿
C.胆总管结石
6 3 2012 9. 胆道手术时,下述哪项不是胆总管探查的指征(2.5分) A.胆总管,肝总管、肝胆管结石
B.黄疸或黄疸病史
C.胰头肿大、变硬
D.胆总管增厚变粗
E.胆囊积液
E.胆囊积液
6 3 2012 10. Laparoscopic cholecystectomy is indicated for symptomatic gallstones in which of the following conditions? (2.5分) A.Cirrhosis
B.Prior upper abdominal surgery
C.Suspected carcinoma of the gallbladder
D.Morbid obesity
E.Coagulopathy
D.Morbid obesity
6 3 2012 11. An 88-year-old man with a history of end-stage renal failure, severe coronary artery disease, and brain metastases from lung cancer presents with acute cholecystitis. His family wants "everything done." The best management option in this patient would be (2.5分) A.Tube cholecystostomy
B.Open cholecystectomy
C.Laparoscopic cholecystectomy
D.Intravenous antibiotics followed by elective cholecystectomy
E.Lithotripsy followed by long-term bile acid therapy
A.Tube cholecystostomy
6 3 2012 12. 下列哪项是胆囊癌分型(2.5分) A.Bismuth分型;
B.Gazzaniga分型;
C.FAB分型;
D.S1-S3分型;
E.Nevin分型
E.Nevin分型
6 3 2012 13. 胆囊癌80%为腺癌,其病理类型不包括(2.5分) A.乳头状腺瘤;
B.粘液腺癌;
C.印戒细胞癌;
D.多核巨细胞癌;
E.低分化癌
E.低分化癌
6 3 2012 14. 胆囊癌的主要治疗方法是(2.5分) A.胆囊切除+肝右叶切除;
B.单纯胆囊切除术;
C.胆肠吻合术;
D.PTCD;
E.ERCP
A.胆囊切除+肝右叶切除;
6 3 2012 15. 胆管癌最常见好发部位是胆管哪一段(2.5分) A.上1/3段.
B.中1/3段;
C.下1/3段;
D.Vater壶腹部;
E.十二指肠后段
A.上1/3段.
6 3 2012 16. 胆管癌的临床表现(2.5分) A.腹痛腹胀、寒战高热、黄疸、休克、精神症状;
B.厌油腻、腹胀、嗳气、肩背部隐痛、但稍有畏寒、高热和黄疸者;
C.右上腹阵发性绞痛,放射至右肩背部,伴恶心呕吐,有畏寒和发热;
D.上腹部不适、呃逆、腹痛、寒战高热和黄疸;
E.上腹部隐痛,向腰背部放射,伴消瘦,梗阻性黄疸,排白陶土色粪便
E.上腹部隐痛,向腰背部放射,伴消瘦,梗阻性黄疸,排白陶土色粪便
6 3 2012 17. 男性,65岁,无明显诱因出现皮肤巩膜黄染、尿色变深、皮肤瘙痒20天,右上腹扪及肿大的胆囊,无触痛。最可能的诊断是(2.5分) A.胆囊结石
B.急性肝炎
C.胆囊炎
D.胆总管结石
E.胰头癌
E.胰头癌
6 3 2012 18. 胰腺癌最多见的组织学类型是 (2.5分) A.腺泡细胞癌
B.导管细胞癌
C.多形性腺癌
D.纤毛细胞癌
E.鳞状上皮癌
B.导管细胞癌
6 3 2012 19. 下列除哪条外均与胰岛细胞瘤有因果关系(2.5分) A.精神病发作
B.精神的衰退
C.顽固性胃十二指肠溃疡
D.库欣(Cushing)综合征
E.发作性低血糖
D.库欣(Cushing)综合征
6 3 2012 20. 下列CT影像中,最支持胰腺癌诊断的是(2.5分) A.胰周及腹腔渗出,胰腺实质内有不规则低密度区,强化后低密度区增强不明显
B.胰头有3cmx4cm不均匀低密度区,强化后有不均匀增强
C.胰头部增大,密度与其他部位胰腺组织密度一致,胰周界限模糊
D.胰头均匀低密度区,CT值10 Hu,不被强化,胰体尾萎缩,胰管扩张
E.胆管扩张明显,胆总管下端可见2 cmx1 cm的极强密度区
B.胰头有3cmx4cm不均匀低密度区,强化后有不均匀增强
6 3 2012 21. 老年糖尿病患者,体温正常,WBC及中性粒细胞比例较正常值稍增高,肝脏MRI增强扫描发现肝VI段48*50mm病灶, T2WI高信号,环形强化,其中病灶无强化区DWI为高信号、ADC均匀低信号,动脉期病灶周围肝脏组织强化相对明显,最可能的诊断是(2.5分) A.肝细胞肝癌
B.肝脓肿
C.肝转移瘤
D.肝内胆管细胞癌
E.肝囊肿
B.肝脓肿
6 3 2012 22. 肝硬化患者,乙肝病史,AFP1045ng/ml,那一项叙述是错误的(2.5分) A.多期增强CT扫描未见明显异常,需做MRI增强扫描
B.肝硬化背景,T2WI稍高信号小结节,动脉期强化明显,静脉期强化消退,需考虑肝癌结节
C.T2WI明显高信号结节,ADC为高信号,动脉期明显强化,静脉期及延迟期强化未见消退,小血管瘤可能
D.T2WI稍高信号小结节,动脉期病灶显示不明显,静脉期强化消退显示为低信号伴周边包膜样强化,需考虑肝癌结节
E.T2WI稍高信号小结节,动脉期病灶显示不明显,静脉期强化消退显示为低信号伴周边包膜样强化,可排除肝癌结节
E.T2WI稍高信号小结节,动脉期病灶显示不明显,静脉期强化消退显示为低信号伴周边包膜样强化,可排除肝癌结节
6 3 2012 23. 男性,66岁,3年前结肠癌手术。近来消瘦、纳差,右上腹部痛。CT检查:肝实质多发大小不等结节状病灶,多数病灶中央坏死液化,对比增强结节边缘强化,少数外周有低密度环,形成"牛眼征"应诊断为(2.5分) A.肝结节性硬化
B.转移性肝癌
C.多发性肝脓肿
D.原发性肝癌
E.肝多发性囊肿
B.转移性肝癌
6 3 2012 24. 女,58岁,右下腹痛一周,CT增强扫描发现胆囊壁稍增厚伴强化稍明显,腔内见8*9mm附壁结节,平扫为高密度,CT值为73HU,增强后CT值约为75HU。最可能的诊断是(2.5分) A.胆囊结石,胆囊炎
B.胆囊息肉
C.胆囊癌
D.胆囊腺肌症
E.黄色肉芽肿性胆囊炎
A.胆囊结石,胆囊炎
6 3 2012 25. 女性,56岁,CT见胰头部分叶状囊实性肿块,增强后肿块呈边界清楚、蜂窝状结构,中心见小块钙化,最可能的诊断是(2.5分) A.胰腺浆液性囊腺瘤
B.胰腺粘液性囊腺瘤
C.胰腺实性假乳头状肿瘤
D.胰腺神经内分泌肿瘤
E.胰腺癌
A.胰腺浆液性囊腺瘤
6 3 2012 26. 关于胰腺导管内乳头状黏液性肿瘤,以下认识中错误的是(2.5分) A.是一种胰腺的外分泌性肿瘤,来源于胰腺的导管上皮,呈乳头状生长,分泌黏液
B.好发于老年男性
C.为良性肿瘤
D.病理分为主胰管型、分支胰管型及混合型
E.因黏液栓塞,常引发胰腺炎
C.为良性肿瘤
6 3 2012 27. 胃肠道穿孔的主要X线征象是(2.5分) A.胃泡增大,胃膈间距增大
B.膈下游离气体
C.麻痹性肠梗阻
D.肠管充气扩张
E.气液平出现
B.膈下游离气体
6 3 2012 28. 男性,69岁,血便,下面叙述错误的是(2.5分) A.增强CT扫描,见对比剂外渗,提示活动性出血
B.增强CT扫描,未见对比剂外渗,排除活动性出血
C.增强CT扫描未见明显异常,可行DSA检查
D.肠壁血管发育不良性出血,可自愈
E.AVM常有静脉早显
B.增强CT扫描,未见对比剂外渗,排除活动性出血
6 3 2012 29. 关于门静脉高压的描述错误的是(2.5分) A.常伴有食道胃底静脉曲张
B.不与肾静脉交通
C.可继发腹壁静脉曲张
D.常引起消化道大出血
E.可不伴有肝硬化
B.不与肾静脉交通
6 3 2012 30. Which kind of hernias are the most prone to incarcerated ? (2.5分) A.Incisional hernia.
B.indirect inguinal hernia
C.direct inguinal hernia
D.sliding hernia.
E.femoral hernia
E.femoral hernia
6 3 2012 31. 嵌顿疝与绞窄疝鉴别要点是(2.5分) A.疝块不能回纳
B.疝块是否有压痛
C.绞窄疝出现呕吐
D.绞窄疝出现休克
E.绞窄疝内容物发生血循环障碍
E.绞窄疝内容物发生血循环障碍
6 3 2012 32. 对于诊断空腔脏器穿孔最有价值的辅助检查是(2.5分) A.B超
B.心电图
C.血常规
D.淀粉酶
E.立位腹部X片
E.立位腹部X片
6 3 2012 33. 急性胆囊炎常见的体征是(2.5分) A.Murphy 征阳性
B.麦氏点压痛
C.板状腹
D.Courvoisier征阳性
E.Courvoisier征阴性
A.Murphy 征阳性
6 3 2012 34. 高位肠梗阻除腹痛以外最明显的表现是(2.5分) A.停止排气排便
B.腹胀明显
C.频繁呕吐
D.可扪及腹部肿块
E.肠鸣音减弱
C.频繁呕吐
6 3 2012 35. 下列急腹症中,.常常出现严重的腹痛但体征轻微的是(2.5分) A.急性阑尾炎
B.胃穿孔
C.急性胰腺炎
D.机械性肠梗阻
E.肠系膜上动脉栓塞
E.肠系膜上动脉栓塞
6 3 2012 36. 我国,引起婴幼儿腹泻的最常见的病原体是(2.5分) A.沙门氏菌
B.志贺氏菌
C.轮状病毒
D.空肠弯曲菌
E.诺如病毒
C.轮状病毒
6 3 2012 37. 高渗性脱水时,血钠浓度为(2.5分) A.>135mmol/L
B.>140mmol/L
C.>145mmol/L
D.>150mmol/L
E.>155mmol/L
D.>150mmol/L
6 3 2012 38. 关于口服补液,不正确的是(2.5分) A.口服补液应作为儿童急性腹泻的一线治疗手段
B.患儿无法进行口服补液时,可采用鼻胃管途径补液
C.轻中度脱水口服补液盐8小时内服完,再重新评估
D.1小时内无ORS补充或脱水状况加重则进行鼻胃管补液,如果症状仍无改善则静脉补液
E.对于能接受口服补液的患者不推荐静脉补液
C.轻中度脱水口服补液盐8小时内服完,再重新评估
6 3 2012 39. 有关腹泻的药物治疗,下列哪项不正确? (2.5分) A.不推荐止吐剂常规应用急性腹泻患儿
B.止泻药易蒙停可应用于急性腹泻患儿
C.吸附剂思密达考虑用于急性腹泻患儿的治疗
D.抗分泌药物消旋卡多曲可用于急性腹泻的治疗
E.锌制剂可作为婴幼儿腹泻的常规添加剂
B.止泻药易蒙停可应用于急性腹泻患儿
6 3 2012 40. 有关鼻饲补液,下列哪项不正确? (2.5分) A.重度脱水如无输液条件,立即转运到就近医院进行静脉补液,转运途中可以用鼻饲点滴方法进行补液。
B.液体采用ORS液,以20ml/(kg.h)的速度补充。
C.如患儿反复呕吐或腹胀,放慢鼻饲点滴速度,总量不超过120ml/kg。
D.如有肠梗阻时也可以鼻饲补液。
E.每1-2h评估1次患者脱水情况。
D.如有肠梗阻时也可以鼻饲补液。
6 3 2013 1. 男性,15岁,突发心窝部阵发性钻顶样疼痛6小时,疼痛时大汗淋漓,辗转不安,痛止时又平息如常,体检:剑突偏右方有深在压痛,无腹肌紧张及反跳痛,为明确诊断应采取简单安全的方法是(2.5分) A.BUS
B.右上腹X线平片
C.测定血清淀粉酶
D.十二指肠引流液检查
E.ERCP
A.BUS
6 3 2013 2. 女性,45岁,突发右上腹及心窝部刀割样绞痛伴阵发性加剧一天,发病后12小时寒战、高热,巩膜黄染,剑突偏右侧深压痛,右上腹轻度肌紧张,体温38℃,WBC 14×109/L血清总胆红素30μmol/L,尿胆原(-),尿胆素(++),应诊断为(2.5分) A.溃疡病穿孔
B.急性胰腺炎
C.急性胆囊炎
D.胆总管结石
E.高位阑尾炎
D.胆总管结石
6 3 2013 3. 女性,43岁,因右上腹阵发性绞痛,伴恶心呕吐4小时来院。检查:体温37℃,右上腹深在轻度压痛,无腹肌紧张,Murphy征阴性,为确诊进一步检查应首选(2.5分) A.白细胞计数和分类
B.腹部X线平片
C.BUS
D.测血清淀粉酶
E.PTC
C.BUS
6 3 2013 4. 男性,43岁,突然右上腹绞痛,并出现寒战高热,于16小时后出现明显黄疸。检查:意识尚清楚,巩膜及周身黄染,BP130/70mmHg,体温38.5℃,右上腹轻度压痛,此病最可能为(2.5分) A.Courvoisier征
B.Murphy征
C.Charcot征
D.Grey-Turner征
E.Reynold征
C.Charcot征
6 3 2013 5. 胆囊动脉多源于(2.5分) A.肝左动脉
B.肝固有动脉
C.胃十二指肠动脉
D.肝右动脉
E.胃右动脉
E.胃右动脉
6 3 2013 6. 梗阻性黄疸时,BUS显示胆总管和肝内胆管均不扩张,为明确诊断应选择哪项检查(2.5分) A.放射性核素胰腺扫描
B.PTC
C.低张十二指肠造影
D.ERCP
E.CT
D.ERCP
6 3 2013 7. Murphy征阳性则提示(2.5分) A.细菌性肝脓肿
B.急性胆管炎
C.肝总管结石
D.左肝管结石
E.急性胆囊炎
E.急性胆囊炎
6 3 2013 8. Charcot三联征间歇发作最大的可能是(2.5分) A.壶腹部癌
B.肝细胞癌
C.胆总管结石
D.黄疸型肝炎
E.细菌性肝脓肿
C.胆总管结石
6 3 2013 9. 胆道手术时,下述哪项不是胆总管探查的指征(2.5分) A.胆总管,肝总管、肝胆管结石
B.黄疸或黄疸病史
C.胰头肿大、变硬
D.胆总管增厚变粗
E.胆囊积液
E.胆囊积液
6 3 2013 10. Laparoscopic cholecystectomy is indicated for symptomatic gallstones in which of the following conditions? (2.5分) A.Cirrhosis
B.Prior upper abdominal surgery
C.Suspected carcinoma of the gallbladder
D.Morbid obesity
E.Coagulopathy
D.Morbid obesity
6 3 2013 11. An 88-year-old man with a history of end-stage renal failure, severe coronary artery disease, and brain metastases from lung cancer presents with acute cholecystitis. His family wants "everything done." The best management option in this patient would be (2.5分) A.Tube cholecystostomy
B.Open cholecystectomy
C.Laparoscopic cholecystectomy
D.Intravenous antibiotics followed by elective cholecystectomy
E.Lithotripsy followed by long-term bile acid therapy
A.Tube cholecystostomy
6 3 2013 12. 下列哪项是胆囊癌分型(2.5分) A.Bismuth分型;
B.Gazzaniga分型;
C.FAB分型;
D.S1-S3分型;
E.Nevin分型
E.Nevin分型
6 3 2013 13. 胆囊癌80%为腺癌,其病理类型不包括(2.5分) A.乳头状腺瘤;
B.粘液腺癌;
C.印戒细胞癌;
D.多核巨细胞癌;
E.低分化癌
E.低分化癌
6 3 2013 14. 胆囊癌的主要治疗方法是(2.5分) A.胆囊切除+肝右叶切除;
B.单纯胆囊切除术;
C.胆肠吻合术;
D.PTCD;
E.ERCP
A.胆囊切除+肝右叶切除;
6 3 2013 15. 胆管癌最常见好发部位是胆管哪一段(2.5分) A.上1/3段.
B.中1/3段;
C.下1/3段;
D.Vater壶腹部;
E.十二指肠后段
A.上1/3段.
6 3 2013 16. 胆管癌的临床表现(2.5分) A.腹痛腹胀、寒战高热、黄疸、休克、精神症状;
B.厌油腻、腹胀、嗳气、肩背部隐痛、但稍有畏寒、高热和黄疸者;
C.右上腹阵发性绞痛,放射至右肩背部,伴恶心呕吐,有畏寒和发热;
D.上腹部不适、呃逆、腹痛、寒战高热和黄疸;
E.上腹部隐痛,向腰背部放射,伴消瘦,梗阻性黄疸,排白陶土色粪便
E.上腹部隐痛,向腰背部放射,伴消瘦,梗阻性黄疸,排白陶土色粪便
6 3 2013 17. 男性,65岁,无明显诱因出现皮肤巩膜黄染、尿色变深、皮肤瘙痒20天,右上腹扪及肿大的胆囊,无触痛。最可能的诊断是(2.5分) A.胆囊结石
B.急性肝炎
C.胆囊炎
D.胆总管结石
E.胰头癌
E.胰头癌
6 3 2013 18. 胰腺癌最多见的组织学类型是 (2.5分) A.腺泡细胞癌
B.导管细胞癌
C.多形性腺癌
D.纤毛细胞癌
E.鳞状上皮癌
B.导管细胞癌
6 3 2013 19. 下列除哪条外均与胰岛细胞瘤有因果关系(2.5分) A.精神病发作
B.精神的衰退
C.顽固性胃十二指肠溃疡
D.库欣(Cushing)综合征
E.发作性低血糖
D.库欣(Cushing)综合征
6 3 2013 20. 下列CT影像中,最支持胰腺癌诊断的是(2.5分) A.胰周及腹腔渗出,胰腺实质内有不规则低密度区,强化后低密度区增强不明显
B.胰头有3cmx4cm不均匀低密度区,强化后有不均匀增强
C.胰头部增大,密度与其他部位胰腺组织密度一致,胰周界限模糊
D.胰头均匀低密度区,CT值10 Hu,不被强化,胰体尾萎缩,胰管扩张
E.胆管扩张明显,胆总管下端可见2 cmx1 cm的极强密度区
B.胰头有3cmx4cm不均匀低密度区,强化后有不均匀增强
6 3 2013 21. 老年糖尿病患者,体温正常,WBC及中性粒细胞比例较正常值稍增高,肝脏MRI增强扫描发现肝VI段48*50mm病灶, T2WI高信号,环形强化,其中病灶无强化区DWI为高信号、ADC均匀低信号,动脉期病灶周围肝脏组织强化相对明显,最可能的诊断是(2.5分) A.肝细胞肝癌
B.肝脓肿
C.肝转移瘤
D.肝内胆管细胞癌
E.肝囊肿
B.肝脓肿
6 3 2013 22. 肝硬化患者,乙肝病史,AFP1045ng/ml,那一项叙述是错误的(2.5分) A.多期增强CT扫描未见明显异常,需做MRI增强扫描
B.肝硬化背景,T2WI稍高信号小结节,动脉期强化明显,静脉期强化消退,需考虑肝癌结节
C.T2WI明显高信号结节,ADC为高信号,动脉期明显强化,静脉期及延迟期强化未见消退,小血管瘤可能
D.T2WI稍高信号小结节,动脉期病灶显示不明显,静脉期强化消退显示为低信号伴周边包膜样强化,需考虑肝癌结节
E.T2WI稍高信号小结节,动脉期病灶显示不明显,静脉期强化消退显示为低信号伴周边包膜样强化,可排除肝癌结节
E.T2WI稍高信号小结节,动脉期病灶显示不明显,静脉期强化消退显示为低信号伴周边包膜样强化,可排除肝癌结节
6 3 2013 23. 男性,66岁,3年前结肠癌手术。近来消瘦、纳差,右上腹部痛。CT检查:肝实质多发大小不等结节状病灶,多数病灶中央坏死液化,对比增强结节边缘强化,少数外周有低密度环,形成"牛眼征"应诊断为: (2.5分) A.肝结节性硬化
B.转移性肝癌
C.多发性肝脓肿
D.原发性肝癌
E.肝多发性囊肿
B.转移性肝癌
6 3 2013 24. 女,58岁,右下腹痛一周,CT增强扫描发现胆囊壁稍增厚伴强化稍明显,腔内见8*9mm附壁结节,平扫为高密度,CT值为73HU,增强后CT值约为75HU。最可能的诊断是(2.5分) A.胆囊结石,胆囊炎
B.胆囊息肉
C.胆囊癌
D.胆囊腺肌症
E.黄色肉芽肿性胆囊炎
A.胆囊结石,胆囊炎
6 3 2013 25. 女性,56岁,CT见胰头部分叶状囊实性肿块,增强后肿块呈边界清楚、蜂窝状结构,中心见小块钙化,最可能的诊断是(2.5分) A.胰腺浆液性囊腺瘤
B.胰腺粘液性囊腺瘤
C.胰腺实性假乳头状肿瘤
D.胰腺神经内分泌肿瘤
E.胰腺癌
A.胰腺浆液性囊腺瘤
6 3 2013 26. 关于胰腺导管内乳头状黏液性肿瘤,以下认识中错误的是(2.5分) A.是一种胰腺的外分泌性肿瘤,来源于胰腺的导管上皮,呈乳头状生长,分泌黏液
B.好发于老年男性
C.为良性肿瘤
D.病理分为主胰管型、分支胰管型及混合型
E.因黏液栓塞,常引发胰腺炎
C.为良性肿瘤
6 3 2013 27. 胃肠道穿孔的主要X线征象是(2.5分) A.胃泡增大,胃膈间距增大
B.膈下游离气体
C.麻痹性肠梗阻
D.肠管充气扩张
E.气液平出现
B.膈下游离气体
6 3 2013 28. 男性,69岁,血便,下面叙述错误的是(2.5分) A.增强CT扫描,见对比剂外渗,提示活动性出血
B.增强CT扫描,未见对比剂外渗,排除活动性出血
C.增强CT扫描未见明显异常,可行DSA检查
D.肠壁血管发育不良性出血,可自愈
E.AVM常有静脉早显
B.增强CT扫描,未见对比剂外渗,排除活动性出血
6 3 2013 29. 关于门静脉高压的描述错误的是(2.5分) A.常伴有食道胃底静脉曲张
B.不与肾静脉交通
C.可继发腹壁静脉曲张
D.常引起消化道大出血
E.可不伴有肝硬化
B.不与肾静脉交通
6 3 2013 30. Which kind of hernias are the most prone to incarcerated ?(2.5分) A.Incisional hernia.
B.indirect inguinal hernia
C.direct inguinal hernia
D.sliding hernia.
E.femoral hernia
E.femoral hernia
6 3 2013 31. 左侧腹股沟滑动性疝,下列哪项是正确的(2.5分) A.属于可复性疝
B.疝内容物没有小肠
C.乙状结肠是疝囊的一部分
D.最易嵌顿
E.疝块很小
C.乙状结肠是疝囊的一部分
6 3 2013 32. 对于诊断空腔脏器穿孔最有价值的辅助检查是(2.5分) A.B超
B.心电图
C.血常规
D.淀粉酶
E.立位腹部X片
E.立位腹部X片
6 3 2013 33. 急性胆囊炎常见的体征是(2.5分) A.Murphy 征阳性
B.麦氏点压痛
C.板状腹
D.Courvoisier征阳性
E.Courvoisier征阴性
A.Murphy 征阳性
6 3 2013 34. 高位肠梗阻除腹痛以外最明显的表现是(2.5分) A.停止排气排便
B.腹胀明显
C.频繁呕吐
D.可扪及腹部肿块
E.肠鸣音减弱
C.频繁呕吐
6 3 2013 35. 下列急腹症中,.常常出现严重的腹痛但体征轻微的是(2.5分) A.急性阑尾炎
B.胃穿孔
C.急性胰腺炎
D.机械性肠梗阻
E.肠系膜上动脉栓塞
E.肠系膜上动脉栓塞
6 3 2013 36. 我国,引起婴幼儿腹泻的最常见的病原体是(2.5分) A.沙门氏菌
B.志贺氏菌
C.轮状病毒
D.空肠弯曲菌
E.诺如病毒
C.轮状病毒
6 3 2013 37. 低渗性脱水时,血钠浓度为(2.5分) A.<125mmol/L
B.<130mmol/L
C.<135mmol/L
D.<140mmol/L
E.<145mmol/L
B.<130mmol/L
6 3 2013 38. 有关抗生素相关性腹泻,下列哪项不正确? (2.5分) A.使用抗生素史
B.可表现为发热、呕吐、腹泻
C.可伴脱水、电解质紊乱
D.粪培养可有大肠埃希氏菌生长
E.粪培养可有金黄色葡萄球菌生长
D.粪培养可有大肠埃希氏菌生长
6 3 2013 39. 有关腹泻的药物治疗,下列哪项不正确? (2.5分) A.不推荐止吐剂常规应用急性腹泻患儿
B.止泻药易蒙停可应用于急性腹泻患儿
C.吸附剂思密达考虑用于急性腹泻患儿的治疗
D.抗分泌药物消旋卡多曲可用于急性腹泻的治疗
E.锌制剂可作为婴幼儿腹泻的常规添加剂
B.止泻药易蒙停可应用于急性腹泻患儿
6 3 2013 40. 微生态制剂治疗急性腹泻,下列哪项不正确? (2.5分) A.微生态制剂对急性腹泻具有辅助治疗作用
B.微生态制剂治疗腹泻的疗效有菌种依赖性
C.微生态制剂治疗腹泻的疗效没有剂量依赖性
D.鼠李乳杆菌和布拉氏酵母菌证明有效
E.不推荐益生元制剂用于治疗婴幼儿急性腹泻
C.微生态制剂治疗腹泻的疗效没有剂量依赖性
6 3 2014 1. 胆总管的血液供应主要来(2.5分) A.肝固有动脉
B.胆囊动脉
C.胃十二指肠动脉
D.肝右动脉
E.肝左动脉
C.胃十二指肠动脉
6 3 2014 2. 肝外胆道的解剖特点中,下列哪项是错误的(2.5分) A.胆囊管常有变异
B.胆囊动脉常有变异
C.胆总管末端多与主胰管汇合
D.Oddi括约肌由胆胰管壶腹部括约肌构成
E.胆囊分为颈体底三部
D.Oddi括约肌由胆胰管壶腹部括约肌构成
6 3 2014 3. 对疑有胆结石所致梗阻性黄疸较深者,应选择哪种检查法较合适? (2.5分) A.口服胆囊造影
B.静脉胆道造影
C.ERCP
D.PTC
E.气钡对比十二指肠造影
D.PTC
6 3 2014 4. 梗阻性黄疸时, BUS显示胆总管和肝内胆管均不扩张,为明确诊断应选择哪项检查(2.5分) A.放射性核素胰腺扫描
B.PTC
C.低张十二指肠造影
D.ERCP
E.C T
D.ERCP
6 3 2014 5. Murphy征阳性则提示(2.5分) A.细菌性肝脓肿
B.急性胆管炎
C.肝总管结石
D.左肝管结石
E.急性胆囊炎
E.急性胆囊炎
6 3 2014 6. 引起右上腹胆绞痛及黄疸的最常见的原因是(2.5分) A.胆道虫回虫症
B.急性胆囊炎
C.胆总管结石
D.先天性胆总管扩张
E.复发性慢性胰腺炎
C.胆总管结石
6 3 2014 7. 胆管结石和急性胆管炎急性发作的典型症状是(2.5分) A.腹痛、呕吐、寒热
B.腹痛、呕吐、黄疸
C.腹痛、黄疸、腹泻
D.腹痛、腹胀、昏迷
E.腹痛、黄疸、寒热
E.腹痛、黄疸、寒热
6 3 2014 8. 急性胆囊炎致病菌的主要来源于(2.5分) A.肠道逆行入侵胆囊
B.淋巴管道
C.邻近脏器
D.经门静脉
E.经胃十二指肠动脉
A.肠道逆行入侵胆囊
6 3 2014 9. 急性梗阻性化脓性胆管炎最常见的原因是(2.5分) A.胆总管结石
B.胆总管末端狭窄
C.胆道出血继发感染
D.胆总管癌
E.先天性胆总管扩张症
A.胆总管结石
6 3 2014 10. 急性梗阻性化脓性胆管炎的治疗一般不宜采取(2.5分) A.胆囊透口术
B.纠正水电解质和酸碱平衡失调
C.胆总管探查取石去除病灶
D.使用有效足量的抗生素
E.及时合理使用多巴胺
A.胆囊透口术
6 3 2014 11. Crohn's disease 的影像表现,哪项描述是错误的(2.5分) A.好发于回肠末端
B.常表现多节段、跳跃性
C.炎症活动期的影像特征是肠壁增厚、水肿、强化明显、肠壁溃疡、"梳征"明显、腹腔脓肿
D.疾病缓解期不出现肠梗阻
E.可并发蜂窝组织炎和瘘管形成
D.疾病缓解期不出现肠梗阻
6 3 2014 12. 老年糖尿病患者,体温正常,WBC及中性粒细胞比例较正常值稍增高,肝脏MRI增强扫描发现肝VI段48*50mm病灶, T2WI高信号,环形强化,其中病灶无强化区DWI为高信号、ADC均匀低信号,动脉期病灶周围肝脏组织强化相对明显,最可能的诊断是(2.5分) A.肝细胞肝癌
B.肝脓肿
C.肝转移瘤
D.肝内胆管细胞癌
E.肝囊肿
B.肝脓肿
6 3 2014 13. 肝硬化患者,乙肝病史,AFP1045ng/ml,那一项叙述是错误的(2.5分) A.多期增强CT扫描未见明显异常,需做MRI增强扫描
B.肝硬化背景,T2WI稍高信号小结节,动脉期强化明显,静脉期强化消退,需考虑肝癌结节
C.T2WI明显高信号结节,ADC为高信号,动脉期明显强化,静脉期及延迟期强化未见消退,小血管瘤可能
D.T2WI稍高信号小结节,动脉期病灶显示不明显,静脉期强化消退显示为低信号伴周边包膜样强化,需考虑肝癌结节
E.T2WI稍高信号小结节,动脉期病灶显示不明显,静脉期强化消退显示为低信号伴周边包膜样强化,可排除肝癌结节
E.T2WI稍高信号小结节,动脉期病灶显示不明显,静脉期强化消退显示为低信号伴周边包膜样强化,可排除肝癌结节
6 3 2014 14. 男性,66岁,3年前结肠癌手术。近来消瘦、纳差,右上腹部痛。CT检查:肝实质多发大小不等结节状病灶,多数病灶中央坏死液化,对比增强结节边缘强化,少数外周有低密度环,形成"牛眼征"应诊断为(2.5分) A.肝结节性硬化
B.转移性肝癌
C.多发性肝脓肿
D.原发性肝癌
E.肝多发性囊肿
B.转移性肝癌
6 3 2014 15. 女,58岁,右下腹痛一周,CT增强扫描发现胆囊壁稍增厚伴强化稍明显,腔内见8*9mm附壁结节,平扫为高密度,CT值为73HU,增强后CT值约为75HU。最可能的诊断是(2.5分) A.胆囊结石,胆囊炎
B.胆囊息肉
C.胆囊癌
D.胆囊腺肌症
E.黄色肉芽肿性胆囊炎
A.胆囊结石,胆囊炎
6 3 2014 16. 女性,56岁,CT见胰头部分叶状囊实性肿块,增强后肿块呈边界清楚、蜂窝状结构,中心见小块钙化,最可能的诊断是(2.5分) A.胰腺浆液性囊腺瘤
B.胰腺粘液性囊腺瘤
C.胰腺实性假乳头状肿瘤
D.胰腺神经内分泌肿瘤
E.胰腺癌
A.胰腺浆液性囊腺瘤
6 3 2014 17. 关于胰腺导管内乳头状黏液性肿瘤,以下认识中错误的是(2.5分) A.是一种胰腺的外分泌性肿瘤,来源于胰腺的导管上皮,呈乳头状生长,分泌黏液
B.好发于老年男性
C.为良性肿瘤
D.病理分为主胰管型、分支胰管型及混合型
E.因黏液栓塞,常引发胰腺炎
C.为良性肿瘤
6 3 2014 18. 胃肠道穿孔的主要X线征象是(2.5分) A.胃泡增大,胃膈间距增大
B.膈下游离气体
C.麻痹性肠梗阻
D.肠管充气扩张
E.气液平出现
B.膈下游离气体
6 3 2014 19. 男性,69岁,血便,下面叙述错误的是(2.5分) A.增强CT扫描,见对比剂外渗,提示活动性出血
B.增强CT扫描,未见对比剂外渗,排除活动性出血
C.增强CT扫描未见明显异常,可行DSA检查
D.肠壁血管发育不良性出血,可自愈
E.AVM常有静脉早显
B.增强CT扫描,未见对比剂外渗,排除活动性出血
6 3 2014 20. 关于门静脉高压的描述错误的是(2.5分) A.常伴有食道胃底静脉曲张
B.不与肾静脉交通
C.可继发腹壁静脉曲张
D.常引起消化道大出血
E.可不伴有肝硬化
B.不与肾静脉交通
6 3 2014 21. Which kind of hernias are the most prone to incarcerated ?(2.5分) A.Incisional hernia.
B.indirect inguinal hernia
C.direct inguinal hernia
D.sliding hernia.
E.femoral hernia
E.femoral hernia
6 3 2014 22. 左侧腹股沟滑动性疝,下列哪项是正确的(2.5分) A.属于可复性疝
B.疝内容物没有小肠
C.乙状结肠是疝囊的一部分
D.最易嵌顿
E.疝块很小
C.乙状结肠是疝囊的一部分
6 3 2014 23. 嵌顿疝与绞窄疝鉴别要点是(2.5分) A.疝块不能回纳
B.疝块是否有压痛
C.绞窄疝出现呕吐
D.绞窄疝出现休克
E.绞窄疝内容物发生血循环障碍
E.绞窄疝内容物发生血循环障碍
6 3 2014 24. 最常出现发冷、发热的外科急腹症是 (2.5分) A.急性胰腺炎
B.急性化脓性阑尾炎
C.急性化脓性梗阻性胆管炎
D.绞窄性肠梗阻
E.溃疡病穿孔
C.急性化脓性梗阻性胆管炎
6 3 2014 25. 女性,27岁,已婚,突发性脐周痛,1小时后局限于右下腹伴呕吐。体检:右下腹压痛,血常规:WBC11×109/L,中性粒细胞88%,尿常规:WBC0~1个/HP,红细胞0~2个/HP。月经过期10天。该病例最合适的处理是(2.5分) A.腹腔穿刺
B.查妊娠试验,请妇科会诊
C.腹部X线平片检查
D.静脉肾盂造影检查
E.进行血沉检查及OT试验
B.查妊娠试验,请妇科会诊
6 3 2014 26. 1岁男孩,突发哭闹不休,面色苍白,片刻后如常,症状多次出现,近1天来解果酱样大便,首先应考虑: (2.5分) A.肠套叠
B.肠扭转
C.肠炎
D.肠蛔虫
E.肠系膜淋巴结炎
A.肠套叠
6 3 2014 27. Charcot三联症对下列哪一种疾病有诊断意义(2.5分) A.急性憩室炎
B.急性出血性胰腺炎
C.急性胆管炎
D.十二指肠憩室
E.胃溃疡
C.急性胆管炎
6 3 2014 28. 男性,75岁,腹痛腹胀、呕吐、肛门停止排气排便2天。全腹压痛明显,腹部平片示肠管可见多处液平面。首先考虑(2.5分) A.急性阑尾炎
B.胃溃疡穿孔
C.急性肠梗阻
D.急性胰腺炎
E.急性胆囊炎
C.急性肠梗阻
6 3 2014 29. 男性,28岁,突感上腹部剧痛45分钟就诊。体检:血压110/80mmHg,心率100次/分,板状腹,肠鸣音消失,血常规:血红蛋白120g/L ,WBC7×109/L,N0.72。尿淀粉酶288u/L(正常:80-300 u/L)。最有意义的进一步检查是: (2.5分) A.腹部CT检查
B.腹立位平片
C.腹部B超检查
D.腹腔滴注灌洗
E.生化检查
B.腹立位平片
6 3 2014 30. 引起病毒性腹泻最常见的原因(2.5分) A.轮状病毒
B.腺病毒
C.杯状病毒
D.诺如病毒
E.星状病毒
A.轮状病毒
6 3 2014 31. 下列哪项不符合分泌性腹泻的特点? (2.5分) A.大便量大于200ml/天
B.暂时禁食后腹泻持续
C.大便钠离子浓度大于70mEq/L
D.大便还原糖阴性
E.大便pH值小于6
E.大便pH值小于6
6 3 2014 32. 轮状病毒感染引起腹泻的机制主要是:(2.5分) A.渗出性
B.肠动力性
C.侵袭性
D.渗透性
E.菌群失调
D.渗透性
6 3 2014 33. 高渗性脱水时,血钠浓度为(2.5分) A.>130mmol/L
B.>135mmol/L
C.>140mmol/L
D.>145mmol/L
E.>150mmol/L
E.>150mmol/L
6 3 2014 34. 以下说法正确的是(2.5分) A.一旦明确致病病原体,急性腹泻病人才能得到合理的诊治
B.粪便培养是腹泻的常规检查
C.粪便中一旦出现白细胞,则提示细菌感染
D.不管病原体检查结果,腹泻患儿均可补液治疗
E.细菌性腹泻均应该应用抗菌素的治疗
D.不管病原体检查结果,腹泻患儿均可补液治疗
6 3 2014 35. 有关抗生素相关性腹泻,下列哪项不正确? (2.5分) A.使用抗生素史
B.可表现为发热、呕吐、腹泻
C.可伴脱水、电解质紊乱
D.粪培养可有大肠埃希氏菌生长
E.粪培养可有金黄色葡萄球菌生长
D.粪培养可有大肠埃希氏菌生长
6 3 2014 36. 关于口服补液,不正确的是(2.5分) A.口服补液应作为儿童急性腹泻的一线治疗手段
B.患儿无法进行口服补液时,可采用鼻胃管途径补液
C.轻中度脱水口服补液盐8小时内服完,再重新评估
D.1小时内无ORS补充或脱水状况加重则进行鼻胃管补液,如果症状仍无改善则静脉补液
E.对于能接受口服补液的患者不推荐静脉补液
C.轻中度脱水口服补液盐8小时内服完,再重新评估
6 3 2014 37. 饮食治疗是婴幼儿腹泻治疗中的关键因素之一,以下说法不合理的是(2.5分) A.无脱水症状的腹泻儿应该喂食与患儿年龄相当的正常饮食
B.接受补液的患儿可暂停饮食,停止正常饮食时间不超过4小时
C.多数幼儿在急性腹泻期间均能耐受含乳糖的配方奶粉
D.急性腹泻患儿应该继续接受母乳喂养
E.配方奶稀释喂养或者逐渐增加浓度法喂养,均有必要
E.配方奶稀释喂养或者逐渐增加浓度法喂养,均有必要
6 3 2014 38. 有关鼻饲补液,下列哪项不正确? (2.5分) A.重度脱水如无输液条件,立即转运到就近医院进行静脉补液,转运途中可以用鼻饲点滴方法进行补液。
B.液体采用ORS液,以20ml/(kg.h)的速度补充。
C.如患儿反复呕吐或腹胀,放慢鼻饲点滴速度,总量不超过120ml/kg。
D.如有肠梗阻时也可以鼻饲补液。
E.每1-2h评估1次患者脱水情况。
D.如有肠梗阻时也可以鼻饲补液。
6 3 2014 39. 有关液体的张力,下列哪项不正确(2.5分) A.5%葡萄糖液,不产生张力
B.10%葡萄糖液,产生1个张力
C.10%氯化钠溶液,产生11个张力
D.0.9%氯化钠溶液,产生1个张力
E.5%碳酸氢钠溶液,产生3.5个张力
B.10%葡萄糖液,产生1个张力
6 3 2014 40. 有关低渗口服补液盐,正确的是(2.5分) A.osmolality (<200 mOsm/l) and sodium (60-70 mmol/l)
B.osmolality (200-250 mOsm/l) and sodium (<60 mmol/l)
C.osmolality (260-280 mOsm/l) and sodium (60-70 mmol/l)
D.osmolality (200-250 mOsm/l) and sodium (60-70 mmol/l)
E.osmolality (200-250 mOsm/l) and sodium (80-90 mmol/l)
D.osmolality (200-250 mOsm/l) and sodium (60-70 mmol/l)
6 3 2015 1. 男性,50岁,3年前曾行胆总管切开取石术,现持续黄疸一个月,肝大肋下3cm,血清胆红素为70μmol/L,BUS显示胆总管直径2cm,下述何种检查方法是最优(2.5分) A.口服法胆囊造影术
B.静脉法胆道造影术
C.低张性十二指肠造影
D.MRCP
E.CT
D.MRCP
6 3 2015 2. 女性,42岁,3年来经常夜间上腹部不适,2日前进油腻食,突然右上腹部阵发性绞痛伴恶心,入院时体温38℃,巩膜轻度黄染,右上腹肌紧张,压痛明显,肠鸣音减弱,WBC16×109/L,血清淀粉酶为128温氏单位,应首先考虑诊断为何种疾病(2.5分) A.高位急性阑尾炎
B.急性胰腺炎
C.溃疡病穿孔
D.急性化脓性胆囊炎
E.胆道蛔虫症
D.急性化脓性胆囊炎
6 3 2015 3. 女性,43岁,因右上腹阵发性绞痛,伴恶心呕吐4小时来院。检查:体温37℃,右上腹深在轻度压痛,无腹肌紧张,Murphy征阴性,为确诊进一步检查应首选(2.5分) A.白细胞计数和分类
B.腹部X线平片
C.BUS
D.测血清淀粉酶
E.PTC
C.BUS
6 3 2015 4. 男性,43岁,突然右上腹绞痛,并出现寒战高热,于16小时后出现明显黄疸。检查:意识尚清楚,巩膜及周身黄染,BP130/70mmHg,体温38.5℃,右上腹轻度压痛,此病最可能为(2.5分) A.Courvoisier征
B.Murphy征
C.Charcot征
D.Grey-Turner征
E.Reynolds征
C.Charcot征
6 3 2015 5. 胆囊动脉多源于(2.5分) A.肝左动脉
B.肝固有动脉
C.胃十二指肠动脉
D.肝右动脉
E.胃右动脉
D.肝右动脉
6 3 2015 6. Murphy征阳性则提示(2.5分) A.细菌性肝脓肿
B.急性胆管炎
C.肝总管结石
D.左肝管结石
E.急性胆囊炎
E.急性胆囊炎
6 3 2015 7. 诊断胆囊结石的简单而可靠的方法是(2.5分) A.BUS
B.ERCP
C.PTC
D.口服法胆囊造影
E.十二指肠引流术
A.BUS
6 3 2015 8. 胆管结石和急性胆管炎急性发作的典型症状是(2.5分) A.腹痛、呕吐、发热
B.腹痛、呕吐、黄疸
C.腹痛、黄疸、腹泻
D.腹痛、腹胀、昏迷
E.腹痛、黄疸、发热
E.腹痛、黄疸、发热
6 3 2015 9. 急性梗阻性化脓性胆管炎最常见的原因是(2.5分) A.胆总管结石
B.胆总管末端狭窄
C.胆道出血继发感染
D.胆总管癌
E.先天性胆总管扩张症
A.胆总管结石
6 3 2015 10. Laparoscopic cholecystectomy is indicated for symptomatic gallstones in which of the following conditions?(2.5分) A.Cirrhosis
B.Prior upper abdominal surgery
C.Suspected carcinoma of the gallbladder
D.Morbid obesity
E.Coagulopathy
D.Morbid obesity
6 3 2015 11. 胆囊癌在组织学上最多见的是一下那种类型?(2.5分) A.未分化癌
B.腺鳞癌
C.腺癌
D.鳞癌
E.上皮细胞癌
C.腺癌
6 3 2015 12. 下列哪像是胆管癌的主要症状?(2.5分) A.上腹胀痛
B.发热
C.无痛性进行性加重的黄疸
D.黑便
E.呕血
C.无痛性进行性加重的黄疸
6 3 2015 13. 关于胆管癌,下列哪项叙述不正确(2.5分) A.上段胆管癌比下段发病率高
B.病理组织大多是腺癌
C.局限型生长较多
D.主要转移方式是血行转移
E.胆管癌预后较差
D.主要转移方式是血行转移
6 3 2015 14. 胆管癌最常见的肿瘤形态为(2.5分) A.结节型
B.囊变型
C.硬化型
D.浸润型
E.肿块型
D.浸润型
6 3 2015 15. 胰腺恶性肿瘤最常见的是(2.5分) A.导管腺癌
B.腺泡细胞癌
C.黏液性癌
D.神经内分泌癌
E.转移癌
A.导管腺癌
6 3 2015 16. 下面那个不是胰岛素瘤的临床表现(2.5分) A.水样腹泻
B.空腹或劳累状态下出现低血糖症状
C.进食或输糖后低血糖症状缓解
D.四肢抽搐等神经精神症状
E.低血糖发作时血糖<50mg/dl
A.水样腹泻
6 3 2015 17. 下面哪项不是胰腺癌术前影像学评估的常用手段(2.5分) A.腹部增强CT
B.腹部增强MRI
C.超声内镜
D.骨骼ECT
E.腹部彩超
D.骨骼ECT
6 3 2015 18. 关于肝脏分段下列哪些是错误的(2.5分) A.肝尾状叶为IV段
B.肝左叶内、外侧段以肝左静脉为界
C.肝VIII段与肝IV相邻
D.肝VII段位于肝VI上方
E.门静脉水平区分肝上下段
A.肝尾状叶为IV段
6 3 2015 19. 肝内肿块,螺旋CT多期扫描表现"快进快出"现象,最可能的疾病是(2.5分) A.肝细胞癌
B.肝海绵状血管瘤
C.肝囊肿
D.肝脓肿
E.肝转移瘤
A.肝细胞癌
6 3 2015 20. 下列有关肝硬化的CT表现的描述哪项不确切? (2.5分) A.中晚期肝叶比例失调
B.肝脏边缘轮廓凹凸不平
C.门脉高压可见门脉侧支循环形成
D.肝脏密度均匀
E.常合并有恶变
D.肝脏密度均匀
6 3 2015 21. 与急性胰腺炎的CT表现不符的是(2.5分) A.胰腺肿大
B.吉氏筋膜增厚
C.蜂窝织炎和假性囊肿形成
D.胰腺及胰管钙化
E.可合并脓肿、出血
D.胰腺及胰管钙化
6 3 2015 22. 关于胰腺导管内乳头状黏液性肿瘤,以下认识中错误的是(2.5分) A.是一种胰腺的外分泌性肿瘤,来源于胰腺的导管上皮,呈乳头状生长,分泌黏液
B.好发于老年男性
C.为良性肿瘤
D.病理分为主胰管型、分支胰管型及混合型
E.因黏液栓塞,常引发胰腺炎
C.为良性肿瘤
6 3 2015 23. Crohn's disease 的影像表现,哪项描述是错误的(2.5分) A.好发于回肠末端
B.常表现多节段、跳跃性
C.炎症活动期的影像特征是肠壁增厚、水肿、强化明显、肠壁溃疡、"梳征"明显、腹腔脓肿
D.疾病缓解期不出现肠梗阻
E.可并发蜂窝组织炎和瘘管形成
D.疾病缓解期不出现肠梗阻
6 3 2015 24. 有关肠梗阻CT表现的描述不正确的是(2.5分) A.CT可作为肠梗阻的首选检查方法
B.梗阻点表现为其近段肠管扩张,远段肠管塌陷
C.肠壁强化减弱或无强化,提示肠壁血运障碍
D.都可发现梗阻点
E.CT检查需回答有无肠梗阻、梗阻点、梗阻原因、有无绞窄
D.都可发现梗阻点
6 3 2015 25. 男性,69岁,腹水,乏力,CT提示大量腹水,腹膜结节样增厚、大网膜弥漫结节影,下面描述哪项可能性最小(2.5分) A.肿瘤腹腔种植转移可能
B.发脓性腹膜炎可能
C.结核性腹膜炎可能
D.腹膜间皮瘤可能
E.结节病可能
B.发脓性腹膜炎可能
6 3 2015 26. Which kind of hernias are the most prone to incarcerated ? (2.5分) A.Incisional hernia.
B.indirect inguinal hernia
C.direct inguinal hernia
D.sliding hernia.
E.femoral hernia
E.femoral hernia
6 3 2015 27. 左侧腹股沟滑动性疝,下列哪项是正确的(2.5分) A.属于可复性疝
B.疝内容物没有小肠
C.乙状结肠是疝囊的一部分
D.最易嵌顿
E.疝块很小
C.乙状结肠是疝囊的一部分
6 3 2015 28. 右上腹痛病人伴有黄疸、寒战高热,提示下列哪一种疾病(2.5分) A.胆总管结石
B.胆囊积水
C.高胆固醇血症
D.病毒性肝炎
E.急性胰腺炎
A.胆总管结石
6 3 2015 29. 1岁男孩,突发哭闹不休,面色苍白,片刻后如常,症状多次出现,近1天来解果酱样大便,首先应考虑(2.5分) A.肠套叠
B.肠扭转
C.肠炎
D.肠蛔虫
E.肠系膜淋巴结炎
A.肠套叠
6 3 2015 30. Charcot三联症对下列哪一种疾病有诊断意义 (2.5分) A.急性憩室炎
B.急性出血性胰腺炎
C.急性胆管炎
D.十二指肠憩室
E.胃溃疡
C.急性胆管炎
6 3 2015 31. 男性,28岁,突感上腹部剧痛45分钟就诊。体检:血压110/80mmHg,心率100次/分,板状腹,肠鸣音消失,血常规:血红蛋白120g/L ,WBC7×109/L,N0.72。尿淀粉酶288u/L(正常:80-300 u/L)。最有意义的进一步检查是(2.5分) A.腹部CT检查
B.腹立位平片
C.腹部B超检查
D.腹腔滴注灌洗
E.生化检查
B.腹立位平片
6 3 2015 32. 女性,27岁,已婚,突发性脐周痛,1小时后局限于右下腹伴呕吐。体检:右下腹压痛,血常规:WBC11×109/L,中性粒细胞88%,尿常规:WBC0~1个/HP,红细胞0~2个/HP。月经过期10天。该病例最合适的处理是(2.5分) A.腹腔穿刺
B.查妊娠试验,请妇科会诊
C.腹部X线平片检查
D.静脉肾盂造影检查
E.进行血沉检查及OT试验
B.查妊娠试验,请妇科会诊
6 3 2015 33. A 28-years-old male patient presents at the emergency department because of a severe epigastrical pain over the past 4 hours. The pain began after drinking wine. The patient has 5-year history of mid-epigastric pain, that could be relieved by food and antacid. Physical examination shows that his blood pressure is 90/60 mmHg, pulse is 78/min and regular, respirations are 34/min, and her temperature is 37.9℃ Physical examination reveals generalized abdominal tenderness with diminished bowel sounds. Which of the following is the most likely diagnosis? (2.5分) A.Free perforation of peptic ulcer
B.Penetrating peptic ulcer
C.Acute pancreatitis
D.Acute appendicitis with free perforation
E.Acute cholecystitis with free perforation
A.Free perforation of peptic ulcer
6 3 2015 34. 以下说法正确的是(2.5分) A.一旦明确致病病原体,急性腹泻病人才能得到合理的诊治
B.粪便培养是腹泻的常规检查
C.粪便中一旦出现白细胞,则提示细菌感染
D.不管病原体检查结果,腹泻患儿均可补液治疗
E.细菌性腹泻均应该应用抗菌素的治疗
D.不管病原体检查结果,腹泻患儿均可补液治疗
6 3 2015 35. 有关抗生素相关性腹泻,下列哪项不正确? (2.5分) A.使用抗生素史
B.可表现为发热、呕吐、腹泻
C.可伴脱水、电解质紊乱
D.粪培养可有大肠埃希氏菌生长
E.粪培养可有金黄色葡萄球菌生长
D.粪培养可有大肠埃希氏菌生长
6 3 2015 36. 关于口服补液,不正确的是(2.5分) A.口服补液应作为儿童急性腹泻的一线治疗手段
B.患儿无法进行口服补液时,可采用鼻胃管途径补液
C.轻中度脱水口服补液盐8小时内服完,再重新评估
D.1小时内无ORS补充或脱水状况加重则进行鼻胃管补液,如果症状仍无改善则静脉补液
E.对于能接受口服补液的患者不推荐静脉补液
C.轻中度脱水口服补液盐8小时内服完,再重新评估
6 3 2015 37. 饮食治疗是婴幼儿腹泻治疗中的关键因素之一,以下说法不合理的是(2.5分) A.无脱水症状的腹泻儿应该喂食与患儿年龄相当的正常饮食
B.接受补液的患儿可暂停饮食,停止正常饮食时间不超过4小时
C.多数幼儿在急性腹泻期间均能耐受含乳糖的配方奶粉
D.急性腹泻患儿应该继续接受母乳喂养
E.配方奶稀释喂养或者逐渐增加浓度法喂养,均有必要
E.配方奶稀释喂养或者逐渐增加浓度法喂养,均有必要
6 3 2015 38. 对脱水的评估,哪项不是较为客观的指标? (2.5分) A.一般情况
B.前囟、眼窝凹陷程度
C.血压
D.眼泪
E.黏膜干燥程度
C.血压
6 3 2015 39. 下列疾病没有抗感染治疗指征的是(2.5分) A.艰难梭菌感染
B.志贺氏菌感染
C.产志贺毒素大肠埃希氏菌感染
D.伤寒沙门菌感染
E.弯曲杆菌感染
C.产志贺毒素大肠埃希氏菌感染
6 3 2015 40. 有关液体的张力,下列哪项不正确(2.5分) A.5%葡萄糖液,不产生张力
B.10%葡萄糖液,产生1个张力
C.10%氯化钠溶液,产生11个张力
D.0.9%氯化钠溶液,产生1个张力
E.5%碳酸氢钠溶液,产生3.5个张力
B.10%葡萄糖液,产生1个张力
6 4 2006 1. 男性,50岁,右侧甲状腺肿大,硬,韧,声音嘶哑,心率90次/分,应首先考虑为:(2.5分) A.甲状腺癌
B.甲状腺腺瘤
C.结节性甲状腺肿
D.甲状腺功能亢进
E.慢性甲状腺炎
A.甲状腺癌
6 4 2006 2. Graves病甲亢药物治疗停药的指征是:(2.5分) A.TRH兴奋试验、T3抑制试验正常
B.T3、T4、TSH恢复正常
C.甲亢症状消失
D.TRAb 阴转
E.以上均是
E.以上均是
6 4 2006 3. A 35-year-old woman presents with a 2-year history of the gradual onset of nervousness and fatigue. She has lost 8 Kgs in the past year despite an increase in appetite. Her menses have become scant with frequent intermenstrual spotting and she feels that (2.5分) A.Peptic ulcer disease
B.Interstitial pneumonia
C.Infiltrative ophthalmopathy
D.Hepatitis
E.Hemolytic anemia
C.Infiltrative ophthalmopathy
6 4 2006 4. 诊断甲亢最敏感的指标是(2.5分) A.TRAb
B.FT3
C.FT4
D.TSH
E.TPO-Ab
D.TSH
6 4 2006 5. 女性,30岁,一月前无意中发现颈前肿块,无不适,检查时发现颈前偏右有3×2厘米园形肿块,光滑、硬、随吞咽上下活动,同位素扫描为冷结节,边缘较清楚,其诊断为(2.5分) A.甲状舌骨囊肿
B.右侧甲状腺瘤囊性变
C.颈淋巴结核
D.右侧高功能腺瘤
E.结节性甲状腺瘤肿
B.右侧甲状腺瘤囊性变
6 4 2006 6. 糖尿病饮食治疗中制定每日总热量应根据下列因素,但不包括:(2.5分) A.工作性质
B.理想体重
C.实际体重
D.参考原来的生活习惯
E.肥胖及消瘦程度
C.实际体重
6 4 2006 7. 有关甲亢危象,以下说法中不正确的是(2.5分) A.甲亢危象治疗首选药物是PTU。
B.甲亢危象常见的病因是感染和甲亢手术前准备不足
C.甲亢危象的患者体温可不升高,休克时甚至体温低于正常。
D.甲亢危象的患者多死于心肺循环衰竭
E.甲亢危象的预防相对于治疗更重要。
C.甲亢危象的患者体温可不升高,休克时甚至体温低于正常。
6 4 2006 8. 下列哪项不是甲状腺手术的并发症? (2.5分) A.甲状旁腺功能减退
B.甲状腺功能减退
C.声音嘶哑
D.甲亢复发
E.出血、感染
D.甲亢复发
6 4 2006 9. 甲状腺术后12小时内,病人出现呼吸困难,首先考虑的原因是(2.5分) A.声带麻痹
B.喉头水肿
C.术后出血
D.严重低钙血症
C.术后出血
6 4 2006 10. 貌似健康而自发性空腹或运动后低血糖症的最常见原因是(2.5分) A.反应性低血糖
B.胰岛素瘤
C.胰岛素自身免疫综合征
D.药源性低血糖症
E.伴瘤的低血糖
B.胰岛素瘤
6 4 2006 11. 女性,25岁,1个月前患上呼吸道感染后多汗、怕热、心慌、焦虑。体检:甲状腺弥漫性肿大,无压痛,右侧稍大些。血清TT3、TT4升高。TPOAb和TgAb正常。甲状腺放射性碘24小时摄碘率为15%,最可能的诊断是(2.5分) A.桥本氏甲状腺炎
B.毒性弥漫性甲状腺肿
C.单纯甲状腺肿
D.亚急性甲状腺炎
D.亚急性甲状腺炎
6 4 2006 12. 喉上神经外支支配(2.5分) A.环甲肌运动
B.喉粘膜感觉
C.声带运动
D.带状肌运动
A.环甲肌运动
6 4 2006 13. 甲状腺危象一般发生在甲亢术后(2.5分) A.8-12小时
B.8小时内
C.12-36小时
D.24-48小时
C.12-36小时
6 4 2006 14. 有关Graves病的描述,不正确的是(2.5分) A.Graves病甲亢多发生于生育期女性
B.甲亢浸润性突眼必须发生于甲亢程度明显的Graves病患者身上。
C.甲亢病因中,Graves病占85%。
D.Graves病与桥本甲状腺炎可同时发生于同一患者
E.突眼、胫前粘液性水肿是Graves病的特征性临床表现。
B.甲亢浸润性突眼必须发生于甲亢程度明显的Graves病患者身上。
6 4 2006 15. 糖尿病饮食治疗中,糖类摄入量应占总热量的(2.5分) A.50~60%
B.20~30%
C.30~40%
D.10~20%
E.80~90%
A.50~60%
6 4 2006 16. 甲状腺术后最凶险的并发症是(2.5分) A.甲状旁腺损伤
B.喉返神经损伤
C.喉上神经损伤
D.呼吸困难和窒息
E.甲亢危象
D.呼吸困难和窒息
6 4 2006 17. 女性,30岁,因右侧甲状腺单个肿物,直径1.5cm,颈部淋巴结肿大,作了甲状腺肿物摘除术,病理报告为甲状腺乳头状腺癌,术后五天拆线,拆线后还应作什么处理:(2.5分) A.再次手术清除患侧淋巴结
B.口服甲状腺素
C.口服碘剂
D.再次手术将患侧腺体连同峡部全切除加长期口服甲状腺素
E.颈部采用放射治疗
D.再次手术将患侧腺体连同峡部全切除加长期口服甲状腺素
6 4 2006 18. 糖尿病分型不包括:(2.5分) A.2型糖尿病
B.其他特殊类型糖尿病
C.儿童糖尿病
D.1型糖尿病
E.妊娠糖尿病
C.儿童糖尿病
6 4 2006 19. 甲状腺癌患者,临床上同时出现腹泻,心悸、脸面潮红,血钙降低,应考虑:(2.5分) A.未分化癌
B.滤泡状腺癌
C..乳头状腺瘤
D.髓样癌
E.结节型甲状腺机能亢进症伴癌变
D.髓样癌
6 4 2006 20. 口服降糖药补包括(2.5分) A.磺脲类
B. -葡萄糖苷酶抑制剂
C.噻唑烷二酮类
D.双胍类
E.硫脲类
E.硫脲类
6 4 2006 21. Which of the following is the single most useful and cost-effective test for screening for thyroid disease? (2.5分) A.Radioactive iodine uptake and scan
B.Free T4
C.Thyroid stimulating hormone level
D.Free thyroxine index
E.T3 uptake
C.Thyroid stimulating hormone level
6 4 2006 22. 糖尿病肾病分几期(2.5分) A.4期
B.3期
C.6期
D.5期
E.2期
D.5期
6 4 2006 23. 下列关于慢性淋巴细胞性甲状腺炎的描述,错误的是(2.5分) A.甲状腺肿大为其突出表现
B.易合并颈部淋巴结肿大
C.发病以30-50岁女性为主
D.甲状腺炎中最常见的类型
E.有发展为甲减的趋势
B.易合并颈部淋巴结肿大
6 4 2006 24. 男性,40岁,甲状腺左侧叶有一肿块4年,未作特殊检查,术中发现为单发实质性结节,应作何处理:(2.5分) A.结节摘除术
B.左侧腺叶切除术加冰冻切片检查
C.全甲状腺叶切除
D.腺体大部切除术
E.颈部淋巴结扩清术
B.左侧腺叶切除术加冰冻切片检查
6 4 2006 25. 女性,29岁,产后4个月,出现心悸、怕热、多汗、失眠,查血清FT3、FT4升高,甲状腺摄碘率降低,本病最可能的诊断是(2.5分) A.产后甲状腺炎
B.亚急性甲状腺炎
C.毒性弥漫性甲状腺肿
D.慢性淋巴细胞性甲状腺炎
E.急性化脓性甲状腺炎
A.产后甲状腺炎
6 4 2006 26. 慢性淋巴细胞性甲状腺炎最具诊断意义的指标是(2.5分) A.TRAb
B.HTG
C.TgAb
D.TPOAb
E.TSBAb
D.TPOAb
6 4 2006 27. 脑细胞活动的主要能源是 (2.5分) A.葡萄糖
B.糖原
C.乙酰乙酸
D.β-羟丁酸
E.乳酸
A.葡萄糖
6 4 2006 28. 糖尿病肾病Ⅲ期尿白蛋白排泄量为(2.5分) A.25~250 mg/24h
B.20~200 mg/24h
C.30~300 mg/24h
D.10~100 mg/24h
E.35~350 mg/24h
C.30~300 mg/24h
6 4 2006 29. 对甲亢治疗方法的选择和评价中,哪项是错误的。(2.5分) A.妊娠早中期的甲亢妇女可接受手术治疗
B.病情较重、病程长患者多采用同位素或手术治疗
C.甲状腺巨大的患者首选手术治疗
D.口服抗甲状腺药物多用于年龄小、病情轻和轻中度甲状腺肿大患者
E.儿童患者首先考虑药物治疗
A.妊娠早中期的甲亢妇女可接受手术治疗
6 4 2006 30. 糖尿病诊断标准中空腹血糖应(2.5分) A.≥6.1 mmol/L
B.≥11.1 mmol/L
C.≥7.8 mmol/L
D.≥5.6 mmol/L
E.≥7.0 mmol/L
E.≥7.0 mmol/L
6 4 2006 31. 糖尿病诊断标准中75g葡萄糖负荷后2小时血糖应(2.5分) A.≥7.0 mmol/L
B.≥7.8 mmol/L
C.≥11.1 mmol/L
D.≥10.0 mmol/L
E.≥12.0 mmol/L
C.≥11.1 mmol/L
6 4 2006 32. A 45-year-old man presents to the hospital complaining of a 4-week history of weakness of his shoulder muscles. He also reports that he has been losing weight without dieting and losing his temper a lot lately. He is not taking any medications. He works a(2.5分) A.Hyperthyroidism
B.Cushing's disease
C.Alcoholism
D.Hypothyroidism
E.Systemic Lupus Erythematosus
A.Hyperthyroidism
6 4 2006 33. 最常见的甲状腺癌病理类型是(2.5分) A.未分化癌
B.髓样癌
C.乳头状癌
D.滤泡状癌
E.恶性淋巴瘤
C.乳头状癌
6 4 2006 34. 抗甲状腺药物如他巴唑, PTU常见的副反应有白细胞减少, 严重者可出现粒细胞缺乏,故治疗时应监测白细胞, 停药的指征为: (2.5分) A.白细胞总数低于 3500, 中性粒细胞低于1000.
B.白细胞总数低于4000, 中性粒细胞低于2000.
C.白细胞总数低于3000, 中性粒细胞低于2000.
D.白细胞总数低于3000, 中性粒细胞低于 1500.
E.白细胞总数低于4000, 中性粒细胞低于 1500.
D.白细胞总数低于3000, 中性粒细胞低于 1500.
6 4 2006 35. 非胰岛素介导的低血糖症时,血浆胰岛素水平一般为(2.5分) A.≤30pmol/L
B.≥30pmol/L
C.≥20pmol/L
D.≤20pmol/L
E.≥36pmol/L
A.≤30pmol/L
6 4 2006 36. 1型糖尿病有关的自身抗体不包括(2.5分) A.IAA
B.ICA
C.GADA
D.TPOAb
E.IA-2
D.TPOAb
6 4 2006 37. 女性,35岁,甲状腺部位疼痛,并出现结节,红细胞沉降率加速,甲状腺摄碘率明显下降,经泼尼松治疗后,临床症状迅速消失,诊断考虑:(2.5分) A.亚急性甲状腺炎
B.甲状腺囊肿囊内出血
C.甲状腺癌
D.桥本甲状腺炎
E.甲状腺腺瘤内出血
A.亚急性甲状腺炎
6 4 2006 38. 一般糖尿病患者(无肾病及特殊需要者)每日蛋白质摄入量应占总热量的(2.5分) A.10~15%
B.5~10%
C.15~20%
D.20~25%
E.25~30%
A.10~15%
6 4 2006 39. 下列哪种情况同位素检查可有明显的甲状腺摄锝率增高: (2.5分) A.亚急性甲状腺炎
B.桥本氏甲状腺炎
C.弥漫性毒性甲状腺肿
D.碘甲亢
E.多结节性甲状腺肿
C.弥漫性毒性甲状腺肿
6 4 2006 40. 女,35岁,患有1型糖尿病1年,3小时前注射胰岛素12U后因生气而未进食,20分钟前家人发现患者意识不清而将其送来急诊,查体:BP 150/85mmHg,面色苍白,皮肤多汗,心率110/min,四肢肌张力减弱,病理征(-)。则正确的处理是(2.5分) A.立即静脉滴注5%葡萄糖盐水
B.立即抽血查血糖,待结果出来后再作处理
C.快速静脉滴注生理盐水,同时静脉注射胰岛素首次负荷量后,继续以每小时每公斤体重0.1u的速率静滴
D.立即经口给予糖水
E.立即抽血查血糖,但不必等待结果,立即给予50%葡萄糖液60-100ml,静脉注射
E.立即抽血查血糖,但不必等待结果,立即给予50%葡萄糖液60-100ml,静脉注射
6 4 2006 1. 有关甲亢危象,以下说法中不正确的是(2.5分) A.甲亢危象治疗首选药物是PTU。
B.甲亢危象的患者多死于心肺循环衰竭
C.甲亢危象常见的病因是感染和甲亢手术前准备不足
D.甲亢危象的患者体温可不升高,休克时甚至体温低于正常。
E.甲亢危象的预防相对于治疗更重要。
NaN
6 4 2006 2. Graves病甲亢药物治疗停药的指征是:(2.5分) A.甲亢症状消失
B.TRH兴奋试验、T3抑制试验正常
C.T3、T4、TSH恢复正常
D.TRAb 阴转
E.以上均是
NaN
6 4 2006 3. 下列哪种情况同位素检查可有明显的甲状腺摄锝率增高: (2.5分) A.弥漫性毒性甲状腺肿
B.桥本氏甲状腺炎
C.碘甲亢
D.亚急性甲状腺炎
E.多结节性甲状腺肿
NaN
6 4 2006 4. 诊断甲亢最敏感的指标是(2.5分) A.FT3
B.TSH
C.FT4
D.TRAb
E.TPO-Ab
NaN
6 4 2006 5. 糖尿病肾病Ⅲ期尿白蛋白排泄量为(2.5分) A.30~300 mg/24h
B.10~100 mg/24h
C.25~250 mg/24h
D.20~200 mg/24h
E.35~350 mg/24h
NaN
6 4 2006 6. 下列哪项不是甲状腺手术的并发症? (2.5分) A.声音嘶哑
B.甲状腺功能减退
C.甲亢复发
D.甲状旁腺功能减退
E.出血、感染
NaN
6 4 2006 7. 1型糖尿病有关的自身抗体不包括(2.5分) A.IAA
B.ICA
C.GADA
D.TPOAb
E.IA-2
NaN
6 4 2006 8. 最常见的甲状腺癌病理类型是(2.5分) A.滤泡状癌
B.髓样癌
C.未分化癌
D.乳头状癌
E.恶性淋巴瘤
NaN
6 4 2006 9. 有关Graves病的描述,不正确的是(2.5分) A.甲亢病因中,Graves病占85%。
B.Graves病甲亢多发生于生育期女性
C.甲亢浸润性突眼必须发生于甲亢程度明显的Graves病患者身上。
D.Graves病与桥本甲状腺炎可同时发生于同一患者
E.突眼、胫前粘液性水肿是Graves病的特征性临床表现。
NaN
6 4 2006 10. A 35-year-old woman presents with a 2-year history of the gradual onset of nervousness and fatigue. She has lost 8 Kgs in the past year despite an increase in appetite. Her menses have become scant with frequent intermenstrual spotting and she feels that (2.5分) A.Infiltrative ophthalmopathy
B.Hepatitis
C.Peptic ulcer disease
D.Interstitial pneumonia
E.Hemolytic anemia
NaN
6 4 2006 11. 男性,50岁,右侧甲状腺肿大,硬,韧,声音嘶哑,心率90次/分,应首先考虑为:(2.5分) A.甲状腺功能亢进
B.结节性甲状腺肿
C.甲状腺腺瘤
D.甲状腺癌
E.慢性甲状腺炎
NaN
6 4 2006 12. 口服降糖药补包括(2.5分) A.双胍类
B. -葡萄糖苷酶抑制剂
C.磺脲类
D.噻唑烷二酮类
E.硫脲类
NaN
6 4 2006 13. 甲状腺危象一般发生在甲亢术后(2.5分) A.12-36小时
B.8-12小时
C.8小时内
D.24-48小时
NaN
6 4 2006 14. 抗甲状腺药物如他巴唑, PTU常见的副反应有白细胞减少, 严重者可出现粒细胞缺乏,故治疗时应监测白细胞, 停药的指征为: (2.5分) A.白细胞总数低于3000, 中性粒细胞低于2000.
B.白细胞总数低于4000, 中性粒细胞低于2000.
C.白细胞总数低于3000, 中性粒细胞低于 1500.
D.白细胞总数低于 3500, 中性粒细胞低于1000.
E.白细胞总数低于4000, 中性粒细胞低于 1500.
NaN
6 4 2006 15. 糖尿病分型不包括:(2.5分) A.1型糖尿病
B.2型糖尿病
C.儿童糖尿病
D.其他特殊类型糖尿病
E.妊娠糖尿病
NaN
6 4 2006 16. 慢性淋巴细胞性甲状腺炎最具诊断意义的指标是(2.5分) A.TRAb
B.HTG
C.TgAb
D.TPOAb
E.TSBAb
NaN
6 4 2006 17. 喉上神经外支支配(2.5分) A.环甲肌运动
B.喉粘膜感觉
C.声带运动
D.带状肌运动
NaN
6 4 2006 18. 甲状腺术后最凶险的并发症是(2.5分) A.甲状旁腺损伤
B.喉返神经损伤
C.喉上神经损伤
D.呼吸困难和窒息
E.甲亢危象
NaN
6 4 2006 19. 脑细胞活动的主要能源是 (2.5分) A.β-羟丁酸
B.葡萄糖
C.乙酰乙酸
D.糖原
E.乳酸
NaN
6 4 2006 20. 貌似健康而自发性空腹或运动后低血糖症的最常见原因是(2.5分) A.胰岛素自身免疫综合征
B.药源性低血糖症
C.反应性低血糖
D.胰岛素瘤
E.伴瘤的低血糖
NaN
6 4 2006 21. 女性,30岁,因右侧甲状腺单个肿物,直径1.5cm,颈部淋巴结肿大,作了甲状腺肿物摘除术,病理报告为甲状腺乳头状腺癌,术后五天拆线,拆线后还应作什么处理:(2.5分) A.口服碘剂
B.再次手术将患侧腺体连同峡部全切除加长期口服甲状腺素
C.再次手术清除患侧淋巴结
D.口服甲状腺素
E.颈部采用放射治疗
NaN
6 4 2006 22. 女性,35岁,甲状腺部位疼痛,并出现结节,红细胞沉降率加速,甲状腺摄碘率明显下降,经泼尼松治疗后,临床症状迅速消失,诊断考虑:(2.5分) A.亚急性甲状腺炎
B.甲状腺囊肿囊内出血
C.甲状腺癌
D.桥本甲状腺炎
E.甲状腺腺瘤内出血
NaN
6 4 2006 23. 下列关于慢性淋巴细胞性甲状腺炎的描述,错误的是(2.5分) A.甲状腺肿大为其突出表现
B.易合并颈部淋巴结肿大
C.发病以30-50岁女性为主
D.甲状腺炎中最常见的类型
E.有发展为甲减的趋势
NaN
6 4 2006 24. A 45-year-old man presents to the hospital complaining of a 4-week history of weakness of his shoulder muscles. He also reports that he has been losing weight without dieting and losing his temper a lot lately. He is not taking any medications. He works a(2.5分) A.Alcoholism
B.Hypothyroidism
C.Hyperthyroidism
D.Cushing's disease
E.Systemic Lupus Erythematosus
NaN
6 4 2006 25. 女性,29岁,产后4个月,出现心悸、怕热、多汗、失眠,查血清FT3、FT4升高,甲状腺摄碘率降低,本病最可能的诊断是(2.5分) A.产后甲状腺炎
B.毒性弥漫性甲状腺肿
C.慢性淋巴细胞性甲状腺炎
D.亚急性甲状腺炎
E.急性化脓性甲状腺炎
NaN
6 4 2006 26. 男性,40岁,甲状腺左侧叶有一肿块4年,未作特殊检查,术中发现为单发实质性结节,应作何处理:(2.5分) A.左侧腺叶切除术加冰冻切片检查
B.结节摘除术
C.腺体大部切除术
D.全甲状腺叶切除
E.颈部淋巴结扩清术
NaN
6 4 2006 27. 甲状腺癌患者,临床上同时出现腹泻,心悸、脸面潮红,血钙降低,应考虑:(2.5分) A.滤泡状腺癌
B.髓样癌
C..乳头状腺瘤
D.未分化癌
E.结节型甲状腺机能亢进症伴癌变
NaN
6 4 2006 28. 女性,25岁,1个月前患上呼吸道感染后多汗、怕热、心慌、焦虑。体检:甲状腺弥漫性肿大,无压痛,右侧稍大些。血清TT3、TT4升高。TPOAb和TgAb正常。甲状腺放射性碘24小时摄碘率为15%,最可能的诊断是(2.5分) A.桥本氏甲状腺炎
B.毒性弥漫性甲状腺肿
C.单纯甲状腺肿
D.亚急性甲状腺炎
NaN
6 4 2006 29. 糖尿病肾病分几期(2.5分) A.6期
B.5期
C.4期
D.3期
E.2期
NaN
6 4 2006 30. 糖尿病诊断标准中空腹血糖应(2.5分) A.≥11.1 mmol/L
B.≥5.6 mmol/L
C.≥7.8 mmol/L
D.≥6.1 mmol/L
E.≥7.0 mmol/L
NaN
6 4 2006 31. 糖尿病诊断标准中75g葡萄糖负荷后2小时血糖应(2.5分) A.≥11.1 mmol/L
B.≥7.0 mmol/L
C.≥7.8 mmol/L
D.≥10.0 mmol/L
E.≥12.0 mmol/L
NaN
6 4 2006 32. 糖尿病饮食治疗中,糖类摄入量应占总热量的(2.5分) A.10~20%
B.30~40%
C.20~30%
D.50~60%
E.80~90%
NaN
6 4 2006 33. 糖尿病饮食治疗中制定每日总热量应根据下列因素,但不包括:(2.5分) A.理想体重
B.参考原来的生活习惯
C.实际体重
D.工作性质
E.肥胖及消瘦程度
NaN
6 4 2006 34. 女性,30岁,一月前无意中发现颈前肿块,无不适,检查时发现颈前偏右有3×2厘米园形肿块,光滑、硬、随吞咽上下活动,同位素扫描为冷结节,边缘较清楚,其诊断为(2.5分) A.右侧甲状腺瘤囊性变
B.右侧高功能腺瘤
C.颈淋巴结核
D.甲状舌骨囊肿
E.结节性甲状腺瘤肿
NaN
6 4 2006 35. 一般糖尿病患者(无肾病及特殊需要者)每日蛋白质摄入量应占总热量的(2.5分) A.5~10%
B.10~15%
C.20~25%
D.15~20%
E.25~30%
NaN
6 4 2006 36. 非胰岛素介导的低血糖症时,血浆胰岛素水平一般为(2.5分) A.≤20pmol/L
B.≤30pmol/L
C.≥20pmol/L
D.≥30pmol/L
E.≥36pmol/L
NaN
6 4 2006 37. 甲状腺术后12小时内,病人出现呼吸困难,首先考虑的原因是(2.5分) A.喉头水肿
B.术后出血
C.声带麻痹
D.严重低钙血症
NaN
6 4 2006 38. Which of the following is the single most useful and cost-effective test for screening for thyroid disease? (2.5分) A.Radioactive iodine uptake and scan
B.Free thyroxine index
C.Free T4
D.Thyroid stimulating hormone level
E.T3 uptake
NaN
6 4 2006 39. 对甲亢治疗方法的选择和评价中,哪项是错误的。(2.5分) A.口服抗甲状腺药物多用于年龄小、病情轻和轻中度甲状腺肿大患者
B.妊娠早中期的甲亢妇女可接受手术治疗
C.甲状腺巨大的患者首选手术治疗
D.病情较重、病程长患者多采用同位素或手术治疗
E.儿童患者首先考虑药物治疗
NaN
6 4 2006 40. 女,35岁,患有1型糖尿病1年,3小时前注射胰岛素12U后因生气而未进食,20分钟前家人发现患者意识不清而将其送来急诊,查体:BP 150/85mmHg,面色苍白,皮肤多汗,心率110/min,四肢肌张力减弱,病理征(-)。则正确的处理是(2.5分) A.立即抽血查血糖,待结果出来后再作处理
B.快速静脉滴注生理盐水,同时静脉注射胰岛素首次负荷量后,继续以每小时每公斤体重0.1u的速率静滴
C.立即静脉滴注5%葡萄糖盐水
D.立即经口给予糖水
E.立即抽血查血糖,但不必等待结果,立即给予50%葡萄糖液60-100ml,静脉注射
NaN
6 4 2007 1. 抗甲状腺药物如他巴唑, PTU常见的副反应有白细胞减少, 严重者可出现粒细胞缺乏,故治疗时应监测白细胞, 停药的指征为: (2.5分) A.白细胞总数低于4000, 中性粒细胞低于2000.
B.白细胞总数低于 3500, 中性粒细胞低于1000.
C.白细胞总数低于3000, 中性粒细胞低于2000.
D.白细胞总数低于3000, 中性粒细胞低于 1500.
E.白细胞总数低于4000, 中性粒细胞低于 1500.
D.白细胞总数低于3000, 中性粒细胞低于 1500.
6 4 2007 2. 下列哪种情况同位素检查可有明显的甲状腺摄锝率增高: (2.5分) A.桥本氏甲状腺炎
B.亚急性甲状腺炎
C.弥漫性毒性甲状腺肿
D.碘甲亢
E.多结节性甲状腺肿
C.弥漫性毒性甲状腺肿
6 4 2007 3. 下列哪项不属于淡漠型甲亢的特点(2.5分) A.老年甲亢
B.常有多食、多汗等表现
C.消瘦明显
D.常并发甲亢性心脏病
E.可有长期腹泻
B.常有多食、多汗等表现
6 4 2007 4. 诊断甲亢最敏感的指标是(2.5分) A.TSH
B.FT3
C.FT4
D.TRAb
E.TPO-Ab
A.TSH
6 4 2007 5. 有关Graves病的描述,不正确的是(2.5分) A.Graves病甲亢多发生于生育期女性
B.甲亢病因中,Graves病占85%。
C.甲亢浸润性突眼必须发生于甲亢程度明显的Graves病患者身上。
D.Graves病与桥本甲状腺炎可同时发生于同一患者
E.突眼、胫前粘液性水肿是Graves病的特征性临床表现。
C.甲亢浸润性突眼必须发生于甲亢程度明显的Graves病患者身上。
6 4 2007 6. 下列那项不是放射性碘治疗Graves病的适应症(2.5分) A.抗甲状腺药物治疗后复发
B.伴重症浸润性突眼
C.曾经在使用抗甲状腺药物后出现粒细胞减少
D.某些高功能结节的甲亢患者
E.甲亢手术后复发
B.伴重症浸润性突眼
6 4 2007 7. 有关甲亢危象,以下说法中不正确的是(2.5分) A.甲亢危象治疗首选药物是PTU。
B.甲亢危象常见的病因是感染和甲亢手术前准备不足
C.甲亢危象的患者多死于心肺循环衰竭
D.甲亢危象的患者体温可不升高,休克时甚至体温低于正常。
E.甲亢危象的预防相对于治疗更重要。
D.甲亢危象的患者体温可不升高,休克时甚至体温低于正常。
6 4 2007 8. Which of the following is the single most useful and cost-effective test for screening for thyroid disease?(2.5分) A.Radioactive iodine uptake and scan
B.Free T4
C.Free thyroxine index
D.Thyroid stimulating hormone level
E.T3 uptake
D.Thyroid stimulating hormone level
6 4 2007 9. A 40-year-old woman presents with mild anterior neck pain that developed 1 week after an episode of flu associated with sore throat. She is otherwise healthy and denies current use of medications. On physical examination, her blood pressure is 140/82 mmHg, and pulse is 102 bpm. Her left thyroid lobe is increased in size, tender, and has increased temperature. Laboratory findings show TSH of 0.1 mIU/ml (normal range= 0.5-4.5 mIU/ml), free T4 of 1.8 ng/dl (normal range =0.7-1.6 ng/dl), and erythrocyte sedimentation rate of 112 mm/hr. What would be the best initial treatment for this patient? (2.5分) A.Propylthiouracil
B.Propranolol
C.Iodine
D.Non-steroidal anti-inflammatory drugs
E.Prednisone
D.Non-steroidal anti-inflammatory drugs
6 4 2007 10. A 56-year-old man presents to the office with signs and symptoms of hyperthyroidism in the presence of a palpable goiter. Laboratory studies are borderline suggestive, but do not confirm thyrotoxicosis. Further testing includes a RadioActive I131 Uptake (RAIU) and scan of the thyroid, which reveals a high uptake in a diffuse, uniform pattern throughout the gland. What is the most likely diagnosis? (2.5分) A.Thyroiditis
B.Grave's disease
C.Toxic multinodular goiter (TMNG)
D.Toxic Thyroid Adenoma
E.Thyroid cancer
B.Grave's disease
6 4 2007 11. Which antibody is the etiology of Graves' disease(2.5分) A.Thyroglobulin antibodies;
B.Thyroid peroxidase antibodies;
C.Thyroid hormone antibody;
D.Thyrotropin (TSH) Receptor antibody;
E.Thyroid antibody
D.Thyrotropin (TSH) Receptor antibody;
6 4 2007 12. Which one is NOT the clinical sign of Graves' ophthalmopathy?(2.5分) A.Glaucoma
B.Exophthalmos
C.Impaired eye muscle movement (Diplopia)
D.Periorbital and conjunctival edema
E.Corneal ulceration
A.Glaucoma
6 4 2007 13. 成人甲减最常见的原因是: (2.5分) A.自身免疫损伤
B.甲状腺破坏
C.碘缺乏
D.碘过量
E.抗甲状腺药物
A.自身免疫损伤
6 4 2007 14. 与甲状腺功能减退症表现不相符的是: (2.5分) A.少汗、嗜睡、畏寒
B.记忆力减退、反应迟钝、腱反射迟钝
C.食欲亢进、体重减轻
D.声音低哑、语言缓慢
E.便秘、月经过多
C.食欲亢进、体重减轻
6 4 2007 15. 鉴别甲状腺性、垂体性和下丘脑性甲状腺功能减退最可靠的检查是 (2.5分) A.血清TSH测定
B.TSH刺激试验
C.TRH刺激试验
D.甲状腺自身抗体测定
E.血清TSH、TT3、TT4、FT3、FT4同时测定,综合分析
C.TRH刺激试验
6 4 2007 16. 关于甲减的心血管系统表现,下列说法不正确的是: (2.5分) A.心肌黏液性水肿可导致心肌收缩力损伤、心动过缓、心排血量下降
B.ECG显示低电压
C.由于心肌间质水肿、非特异性心肌纤维肿胀、左心室扩张和心包积液导致心脏扩大
D.心绞痛患者经甲状腺激素替代治疗后可缓解
E.约有10%的患者伴发高血压
D.心绞痛患者经甲状腺激素替代治疗后可缓解
6 4 2007 17. 下列有关黏液性水肿昏迷的说法哪项不正确?(2.5分) A.常见于病情严重的患者,多在夏季天气炎热时发病
B.常见诱因为严重的全身疾病、甲状腺激素替代治疗中断和手术等
C.可表现为嗜睡,低体温,心动过缓,四肢肌肉松弛,甚至昏迷
D.对这些患者需注意保暖
E.治疗上予补充甲状腺激素和糖皮质激素
A.常见于病情严重的患者,多在夏季天气炎热时发病
6 4 2007 18. 有关甲减治疗的说法下列哪项不正确?(2.5分) A.首选左甲状腺素片,小剂量开始
B.有缺血性心脏病患者起始剂量宜小,调整剂量宜慢
C.治疗后复查甲状腺功能正常即可停药
D.不能与氢氧化铝和碳酸钙片同服
E.妊娠时治疗剂量需要增加30-50%
C.治疗后复查甲状腺功能正常即可停药
6 4 2007 19. 患者,女性,因"双侧甲状腺肿大"接受甲状腺穿刺活检,其结果是弥漫性淋巴细胞浸润,未见嗜酸性粒细胞及纤维化。该患者诊断是(2.5分) A.慢性淋巴细胞性甲状腺炎
B.亚急性淋巴细胞性甲状腺炎
C.亚急性肉芽肿性甲状腺炎
D.毒性弥漫性甲状腺肿
E.多结节性甲状腺肿
A.慢性淋巴细胞性甲状腺炎
6 4 2007 20. 下列关于慢性淋巴细胞性甲状腺炎的描述,错误的是(2.5分) A.甲状腺炎中最常见的类型
B.易合并颈部淋巴结肿大
C.发病以30-50岁女性为主
D.甲状腺肿大为其突出表现
E.有发展为甲减的趋势
B.易合并颈部淋巴结肿大
6 4 2007 21. 女性,42岁,甲状腺部位疼痛,放射至下颌、耳部及枕部,伴甲状腺毒症。体格检查:左甲状腺肿大,可扪及一1.5cm*1.5cm结节,结节质地中等,伴压痛。诊断考虑亚急性甲状腺炎,下列哪项检查有助于诊断:(2.5分) A.FT3、FT4升高,TSH下降
B.FT3下降,TSH升高
C.FT3升高,甲状腺摄碘率明显下降,呈所谓"分离现象"
D.FT3、FT4升高,甲状腺摄碘率升高,但可被T3抑制
E.甲状腺穿刺细胞学检查
C.FT3升高,甲状腺摄碘率明显下降,呈所谓"分离现象"
6 4 2007 22. 女性,35岁,甲状腺部位疼痛,并出现结节,红细胞沉降率加速,甲状腺摄碘率明显下降,经泼尼松治疗后,临床症状迅速消失,诊断考虑:(2.5分) A.亚急性甲状腺炎
B.桥本甲状腺炎
C.甲状腺癌
D.甲状腺囊肿囊内出血
E.甲状腺腺瘤内出血
A.亚急性甲状腺炎
6 4 2007 23. 下列哪项不属于亚急性甲状腺炎的临床表现(2.5分) A.起病前1-3周常有病毒感染的症状
B.甲状腺区有明显触痛,可放射至耳部
C.体格检查发现双侧甲状腺弥漫性肿大,可闻及血管杂音
D.可有全身不适、食欲减退、肌肉疼痛、发热、心动过速、多汗等
E.少数有颈部淋巴结肿大
C.体格检查发现双侧甲状腺弥漫性肿大,可闻及血管杂音
6 4 2007 24. 女性,25岁,1个月前患上呼吸道感染后多汗、怕热、心慌、焦虑。体检:甲状腺弥漫性肿大,无压痛,右侧稍大些。血清TT3、TT4升高。TPOAb和TgAb正常。甲状腺放射性碘24小时摄碘率为15%,最可能的诊断是(2.5分) A.毒性弥漫性甲状腺肿
B.桥本氏甲状腺炎
C.单纯甲状腺肿
D.亚急性甲状腺炎
D.亚急性甲状腺炎
6 4 2007 25. 糖尿病肾病Ⅲ期尿白蛋白排泄量为(2.5分) A.30~300 mg/24h
B.20~200 mg/24h
C.25~250 mg/24h
D.10~100 mg/24h
E.35~350 mg/24h
A.30~300 mg/24h
6 4 2007 26. 糖尿病肾病分几期(2.5分) A.3期
B.5期
C.4期
D.6期
E.2期
B.5期
6 4 2007 27. 糖尿病诊断标准中75g葡萄糖负荷后2小时血糖应(2.5分) A.≥7.8 mmol/L
B.≥7.2 mmol/L
C.≥11.1 mmol/L
D.≥10.1 mmol/L
E.≥12.1 mmol/L
C.≥11.1 mmol/L
6 4 2007 28. 糖耐量减低(IGT)是指OGTT 2小时血糖(2.5分) A.7.0 ~ <10.0 mmol/L
B.6.1 ~ <7.8 mmol/L
C.6.1 ~ <7.0 mmol/L
D.7.0 ~ <7.8 mmol/L
E.7.8 ~ <11.1 mmol/L
E.7.8 ~ <11.1 mmol/L
6 4 2007 29. 妊娠糖尿病是指(2.5分) A.妊娠期间发生或发现的糖尿病
B.妊娠前发生或发现的糖尿病
C.妊娠前3个月发生或发现的糖尿病
D.妊娠后3个月发生或发现的糖尿病
E.妊娠中期发生或发现的糖尿病
A.妊娠期间发生或发现的糖尿病
6 4 2007 30. 糖尿病饮食治疗中,糖类摄入量应占总热量的(2.5分) A.10~20%
B.20~30%
C.30~40%
D.50~60%
E.80~90%
D.50~60%
6 4 2007 31. 肥胖的2型糖尿病降糖药首选(2.5分) A.磺脲类
B.双胍类
C.噻唑烷二酮类
D. -葡萄糖苷酶抑制剂
E.胰岛素
B.双胍类
6 4 2007 32. 1型糖尿病有关的自身抗体不包括(2.5分) A.ICA
B.GADA
C.TPOAb
D.IAA
E.IA-2
C.TPOAb
6 4 2007 33. 喉上神经外支支配(2.5分) A.声带运动
B.喉粘膜感觉
C.环甲肌运动
D.带状肌运动
C.环甲肌运动
6 4 2007 34. 甲状腺危象一般发生在甲亢术后(2.5分) A.8小时内
B.8-12小时
C.12-36小时
D.24-48小时
C.12-36小时
6 4 2007 35. 原发甲亢手术治疗时为了防止患者术后严重甲低,建议保留甲状腺组织为(2.5分) A.10%甲状腺组织
B.3-4g
C. 4-6g
D. 6-8g
E.保留一侧甲状腺
D. 6-8g
6 4 2007 36. 根据最新的甲状腺癌AJCC分期T1期指的肿瘤直径为(2.5分) A.<1cm
B. <2cm
C.<3cm
D.<4cm
B. <2cm
6 4 2007 37. 喉上神经外支损伤时:(2.5分) A.易引起误咽
B.音调降低
C.饮水呛咳
D.呼吸困难
E.失音
B.音调降低
6 4 2007 38. 男性,50岁,右侧甲状腺肿大,硬,韧,声音嘶哑,心率90次/分,应首先考虑为:(2.5分) A.甲状腺功能亢进
B.甲状腺腺瘤
C.结节性甲状腺肿
D.甲状腺癌
E.慢性甲状腺炎
D.甲状腺癌
6 4 2007 39. Iodine deficiency can result in (2.5分) A.endemic goiter
B. nodular goiter
C.Hypothyroidism
D.Cretinism
E.Hashimoto's disease
E.Hashimoto's disease
6 4 2007 40. The Size of Papillary Thyroid Microcarcinoma(PTMC)is (2.5分) A.<3mm
B.<5mm
C.<10mm
D.<20mm
E.10mm-20mm
C.<10mm
6 4 2008 1. 下列关于慢性淋巴细胞性甲状腺炎的描述,错误的是(2.0分) A.甲状腺炎中最常见的类型
B.其发病与碘摄入无关
C.发病以30-50岁女性为主
D.甲状腺肿大为其突出表现
E.有发展为甲减的趋势
B.其发病与碘摄入无关
6 4 2008 2. 自身免疫性甲状腺病不包括(2.0分) A.桥本甲状腺炎
B.结节性甲状腺肿
C.Graves病
D.Graves眼病
E.产后甲状腺炎
B.结节性甲状腺肿
6 4 2008 3. 患者,女性,55岁。因"双侧甲状腺肿大"接受甲状腺穿刺活检,其结果是弥漫性淋巴细胞浸润,未见嗜酸性粒细胞及纤维化。该患者诊断是(2.0分) A.亚急性肉芽肿性甲状腺炎
B.慢性淋巴细胞性甲状腺炎
C.亚急性淋巴细胞性甲状腺炎
D.毒性弥漫性甲状腺肿
E.多结节性甲状腺肿
B.慢性淋巴细胞性甲状腺炎
6 4 2008 4. 患者,男性,36岁。2周前感冒伴咽痛,1周前已痊愈。近5天颈前疼痛明显,有低热,来门诊就诊。查体T 37.8℃,皮肤无汗,甲状腺Ⅱ度肿大,右叶硬,明显触痛拒按,WBC 7.8*10 9/L。临床诊断可能是(2.0分) A.甲状腺右叶囊肿出血
B.甲状腺癌伴出血
C.慢性淋巴细胞性甲状腺炎
D.急性化脓性甲状腺炎
E.亚急性甲状腺炎
E.亚急性甲状腺炎
6 4 2008 5. 女性,61岁,乏力怕冷数年。体检:甲状腺结节性肿大,质韧。诊断考虑慢性淋巴细胞性甲状腺炎。下列哪项检查有确诊价值(2.0分) A.FT3、FT4正常,TSH升高
B.TPOAb和TgAb滴度明显增高
C.甲状腺摄碘率升高,但可被T3抑制
D.甲状腺呈均匀弥漫性摄碘率下降
E.甲状腺针刺活组织检查
E.甲状腺针刺活组织检查
6 4 2008 6. 患者女性,42岁,颈部疼痛,伴心悸、怕热、多汗一周。体格检查:左甲状腺肿大,可扪及一1.5cm*1.5cm结节,结节质地中等,伴压痛。诊断考虑亚急性甲状腺炎,下列哪项检查或表现最常用于诊断:(2.0分) A.FT3、FT4升高,TSH下降
B.FT3下降,TSH升高
C.FT3升高,甲状腺摄碘率明显下降
D.FT3、FT4升高,甲状腺摄碘率升高,但可被T3抑制
E.甲状腺穿刺细胞学检查
C.FT3升高,甲状腺摄碘率明显下降
6 4 2008 7. 成人甲减最常见的原因是: (2.0分) A.自身免疫损伤
B.甲状腺破坏
C.碘缺乏
D.碘过量
E.抗甲状腺药物
A.自身免疫损伤
6 4 2008 8. 以下哪项甲减不属于病因分类?(2.0分) A.药物性甲减
B.手术后甲减
C.131碘治疗后甲减
D.特发性甲减
E.幼年甲减
E.幼年甲减
6 4 2008 9. 与甲状腺功能减退症表现不相符的是: (2.0分) A.少汗、嗜睡、畏寒
B.记忆力减退、反应迟钝、腱反射迟钝
C.食欲亢进、体重减轻
D.声音低哑、语言缓慢
E.便秘、月经过多
C.食欲亢进、体重减轻
6 4 2008 10. 以下哪种检查结果不符合原发性甲状腺功能减退症: (2.0分) A.TT3、TT4低于正常
B.FT3、FT4低于正常
C.血胆固醇增加
D.血清TSH增高
E.膝腱反射亢进
E.膝腱反射亢进
6 4 2008 11. 下列指标中哪一个对亚临床型甲减的诊断具有重要意义:(2.0分) A.TT4
B.TT3
C.s-TSH
D.FT3
E.FT4
C.s-TSH
6 4 2008 12. 患者女性,60岁。畏寒、嗜睡、纳差、便秘伴毛发脱落3年。体格检查:浮肿,皮肤粗糙,反应迟钝,心率56次/分,律齐。实验室检查:FT3↓,FT4↓,TSH↑。诊断为原发性甲减。治疗主要采用:(2.0分) A.支持治疗
B.对症治疗
C.病因治疗
D.替代治疗
E.滋补治疗
D.替代治疗
6 4 2008 13. 下列有关黏液性水肿昏迷的说法哪项不正确?(2.0分) A.常见于病情严重的患者,多在夏季天气炎热时发病
B.常见诱因为严重的全身疾病、甲状腺激素替代治疗中断和手术等
C.可表现为嗜睡,低体温,心动过缓,四肢肌肉松弛,甚至昏迷
D.对这些患者需注意保暖
E.治疗上予补充甲状腺激素和糖皮质激素
A.常见于病情严重的患者,多在夏季天气炎热时发病
6 4 2008 14. 有关甲减治疗的说法下列哪项不正确?(2.0分) A.首选左甲状腺素片,小剂量开始
B.有缺血性心脏病患者起始剂量宜小,调整剂量宜慢
C.治疗后复查甲状腺功能正常即可停药
D.不能与氢氧化铝和碳酸钙片同服
E.妊娠时治疗剂量需要增加30-50%
C.治疗后复查甲状腺功能正常即可停药
6 4 2008 15. 喉上神经外支支配(2.0分) A.声带运动
B.喉粘膜感觉
C. 环甲肌运动
D.带状肌运动
C. 环甲肌运动
6 4 2008 16. 甲状腺危象一般发生在甲亢术后(2.0分) A.8小时内
B.8-12小时
C. 12-36小时
D. 24-48小时
C. 12-36小时
6 4 2008 17. 最常见的甲状腺癌病理类型是(2.0分) A.乳头状癌
B.滤泡状癌
C.髓样癌
D.未分化癌
E.恶性淋巴瘤
A.乳头状癌
6 4 2008 18. 甲状腺术后12小时内,病人出现呼吸困难,首先考虑的原因是(2.0分) A.喉头水肿
B.声带麻痹
C.术后出血
D.严重低钙血症
E.严重的甲状腺功能低下
C.术后出血
6 4 2008 19. 甲状腺术后最凶险的并发症是(2.0分) A.喉返神经损伤
B. 甲状旁腺损伤
C.喉上神经损伤
D. 呼吸困难和窒息
E. 甲亢危象
D. 呼吸困难和窒息
6 4 2008 20. 甲状腺结节性质判断,术前诊断的最佳方法是(2.0分) A.B超
B.CT
C.FNAC
D.ECT
C.FNAC
6 4 2008 21. 甲状腺髓样癌的特异性指标是(2.0分) A.CT CEA
B.CEA AFP
C.AFP、CT
D. CA199 AFP
A.CT CEA
6 4 2008 22. 以下起源于甲状腺滤泡旁细胞的肿瘤是(2.0分) A.乳头状癌
B.滤泡状癌
C. 髓样癌
D. 未分化癌
C. 髓样癌
6 4 2008 23. 男性,50岁,右侧甲状腺肿大,硬,韧,声音嘶哑,心率90次/分,应首先考虑为:(2.0分) A.甲状腺功能亢进
B.甲状腺腺瘤
C.结节性甲状腺肿
D.甲状腺癌
E.慢性甲状腺炎
D.甲状腺癌
6 4 2008 24. 男孩,10岁,颈前部硬结半年,无不适,检查颈前偏右有1.5cm直径硬块,随吞咽运动, 扫描为冷结节,边缘模糊,诊断最可能为:(2.0分) A.甲状腺腺瘤
B.甲状腺癌
C.甲状腺囊肿
D.青春期甲状腺增大
E.甲状腺舌管囊肿
B.甲状腺癌
6 4 2008 25. The half-life of T4 is about(B)(2.0分) A.3 days
B.7 days
C.2 weeks
D.3 weeks
E.2 Months
B.7 days
6 4 2008 26. The half-life of T3 is about(2.0分) A.3-5hours
B.6-8 hours
C. 8-12 hours
D.3 days
E.1-2 days
C. 8-12 hours
6 4 2008 27. Iodine deficiency can result in (2.0分) A.endemic goiter
B.nodular goiter
C.Hypothyroidism
D.Cretinism
E.Hashimoto's disease
E.Hashimoto's disease
6 4 2008 28. The Size of Papillary Thyroid Microcarcinoma(PTMC)is (C)(2.0分) A.<3mm
B.<5mm
C.<10mm
D.<20mm
E.10mm-20mm
C.<10mm
6 4 2008 29. 脑细胞活动的主要能源是 (2.0分) A.葡萄糖
B.β-羟丁酸
C.乙酰乙酸
D.糖原
E.乳酸
A.葡萄糖
6 4 2008 30. 非胰岛素介导的低血糖症时,血浆胰岛素水平一般为(2.0分) A.≤30pmol/L
B.≤20pmol/L
C.≥20pmol/L
D.≥30pmol/L
E.≥36pmol/L
A.≤30pmol/L
6 4 2008 31. 貌似健康而自发性空腹或运动后低血糖症的最常见原因是(2.0分) A.药源性低血糖症
B.胰岛素自身免疫综合征
C.反应性低血糖
D.胰岛素瘤
E.伴瘤的低血糖
D.胰岛素瘤
6 4 2008 32. 女,35岁,患有1型糖尿病1年,3小时前注射胰岛素12U后因生气而未进食,20分钟前家人发现患者意识不清而将其送来急诊,查体:BP 150/85mmHg,面色苍白,皮肤多汗,心率110/min,四肢肌张力减弱,病理征(-)。则正确的处理是(2.0分) A.立即抽血查血糖,待结果出来后再作处理
B.快速静脉滴注生理盐水,同时静脉注射胰岛素首次负荷量后,继续以每小时每公斤体重0.1u的速率静滴
C.立即经口给予糖水
D.立即静脉滴注5%葡萄糖盐水
E.立即抽血查血糖,但不必等待结果,立即给予50%葡萄糖液60-100ml,静脉注射
E.立即抽血查血糖,但不必等待结果,立即给予50%葡萄糖液60-100ml,静脉注射
6 4 2008 33. 纠正低血糖后,脑功能恢复的顺序是 (2.0分) A.延脑-下丘脑-皮质下中枢-自主神经中枢-大脑皮质
B.延脑-自主神经中枢-下丘脑-皮质下中枢-大脑皮质
C.大脑皮质-下丘脑-皮质下中枢-自主神经中枢-延脑
D.大脑皮质-皮质下中枢-延脑-下丘脑-自主神经中枢
E.下丘脑-皮质下中枢-延脑-自主神经中枢-大脑皮质
B.延脑-自主神经中枢-下丘脑-皮质下中枢-大脑皮质
6 4 2008 34. 男性,36岁,1型糖尿病,今日因感冒,食欲减退,少食,餐前按常规注射胰岛素,近午时突然心悸、出汗、手抖,继而头晕,视物模糊。应如何处理 (2.0分) A.胰岛素皮下注射
B.静滴生理盐水
C.葡萄糖静滴
D.碳酸氢钠注射
E.急查血糖,见结果后再行处理
C.葡萄糖静滴
6 4 2008 35. 关于胰岛素瘤,下列哪项不是该病的特点(2.0分) A.低血糖经常出现在空腹或活动后
B.胰岛素释放指数增加
C.血糖降至1.6mmol/L,胰岛素停止释放
D.禁食后多在48小时出现低血糖
E.胰高血糖素可诱发低血糖
C.血糖降至1.6mmol/L,胰岛素停止释放
6 4 2008 36. 下列哪项不符合低血糖的症状或表现(2.0分) A.手抖
B.心悸
C.饥饿感
D.便频
E.皮肤多汗
D.便频
6 4 2008 37. 糖尿病饮食治疗中制定每日总热量应根据下列因素,但不包括:(2.0分) A.理想体重
B.工作性质
C.实际体重
D.参考原来的生活习惯
E.肥胖及消瘦程度
C.实际体重
6 4 2008 38. 口服降糖药不包括(2.0分) A.磺脲类
B.双胍类
C.噻唑烷二酮类
D. -葡萄糖苷酶抑制剂
E.硫脲类
E.硫脲类
6 4 2008 39. 一般糖尿病患者(无肾病及特殊需要者)每日蛋白质摄入量应占总热量的(2.0分) A.5~10%
B.10~15%
C.15~20%
D.20~25%
E.25~30%
B.10~15%
6 4 2008 40. 肥胖的2型糖尿病降糖药首选(2.0分) A.磺脲类
B.双胍类
C.噻唑烷二酮类
D. -葡萄糖苷酶抑制剂
E.胰岛素
B.双胍类
6 4 2008 41. 妊娠期糖尿病诊断标准中75gOGTT2小时血糖是(2.0分) A.≥7.0 mmol/L
B.≥7.6 mmol/L
C.≥8.0 mmol/L
D.≥8.5 mmol/L
E.≥10.0 mmol/L
D.≥8.5 mmol/L
6 4 2008 42. 妊娠期糖尿病诊断标准中空腹血糖是(2.0分) A.≥5.1 mmol/L
B.≥5.3 mmol/L
C.≥5.5 mmol/L
D.≥6.1 mmol/L
E.≥7.0 mmol/L
A.≥5.1 mmol/L
6 4 2008 43. Gout is definitively diagnosed based on(2.0分) A.Serum uric acid
B.the demonstration of urate crystals in aspirated synovial fluid.
C.only 1 or 2 joints are usually involved.
B.the demonstration of urate crystals in aspirated synovial fluid.
6 4 2008 44. In men, the peak age of onset of gout in men is -() of life. (2.0分) A.in the sixth to eighth decade
B.in the fourth to sixth decade
C.in their early 30s
B.in the fourth to sixth decade
6 4 2008 45. As a rule, uric acid levels are elevated for---years before the onset of gout. (2.0分) A.10
B.20
C.30
B.20
6 4 2008 46. A gout attack may be triggered by (2.0分) A.precipitation of crystals in a supersaturated microenvironment (eg, release of urate due to cellular damage).
B.a release of uncoated crystals (eg, due to partial dissolution of a microtophus caused by changing serum urate levels
C.either a release of uncoated crystals (eg, due to partial dissolution of a microtophus caused by changing serum urate levels) or precipitation of crystals in a supersaturated microenvironment (eg, release of urate due to cellular damage).
C.either a release of uncoated crystals (eg, due to partial dissolution of a microtophus caused by changing serum urate levels) or precipitation of crystals in a supersaturated microenvironment (eg, release of urate due to cellular damage).
6 4 2008 47. Colchicinesin acute gout is now a ------approach 。(2.0分) A.first-line
B.second-line
C.no longer advocated
A.first-line
6 4 2008 48. 抗甲状腺药物如他巴唑, PTU常见的副反应有白细胞减少, 严重者可出现粒细胞缺乏,故治疗时应监测白细胞, 停药的指征为: (2.0分) A.白细胞总数低于4000, 中性粒细胞低于2000.
B.白细胞总数低于 3500, 中性粒细胞低于1000.
C.白细胞总数低于3000, 中性粒细胞低于2000.
D.白细胞总数低于3000, 中性粒细胞低于 1500.
E.白细胞总数低于4000, 中性粒细胞低于 1500.
D.白细胞总数低于3000, 中性粒细胞低于 1500.
6 4 2008 49. Graves病甲亢药物治疗停药的指征是:(2.0分) A.甲亢症状消失
B.T3、T4、TSH恢复正常
C.TRH兴奋试验、T3抑制试验正常
D.TRAb阴转
E.以上均是
E.以上均是
6 4 2008 50. 有关甲亢性心脏病,下列哪一项是不正确的: (2.0分) A.甲亢性心脏病多发生于有多年甲亢病史且控制不好的患者身上。
B.甲亢患者合并心律失常、心脏增大、心衰可诊断甲亢性心脏病。
C.甲亢性心脏病患者可选择同位素治疗。
D.甲亢性心脏病大多在甲亢控制后消失。
E.甲亢合并的心衰多为舒张性心衰。
B.甲亢患者合并心律失常、心脏增大、心衰可诊断甲亢性心脏病。
6 4 2009 1. Which antibody is the etiology of Graves' disease (2.5分) A.Thyroglobulin antibodies;
B.Thyroid peroxidase antibodies;
C.Thyroid hormone antibody;
D.Thyrotropin (TSH) Receptor antibody;
E.Thyroid antibody
D.Thyrotropin (TSH) Receptor antibody;
6 4 2009 2. 下列哪种情况同位素检查可有明显的甲状腺摄锝率增高: (2.5分) A.桥本氏甲状腺炎
B.亚急性甲状腺炎
C.弥漫性毒性甲状腺肿
D.碘甲亢
E.多结节性甲状腺肿
C.弥漫性毒性甲状腺肿
6 4 2009 3. 有关甲亢性心脏病,下列哪一项是不正确的: (2.5分) A.甲亢性心脏病多发生于有多年甲亢病史且控制不好的患者身上。
B.甲亢患者合并心律失常、心脏增大、心衰可诊断甲亢性心脏病。
C.甲亢性心脏病患者可选择同位素治疗。
D.甲亢性心脏病大多在甲亢控制后消失。
E.甲亢合并的心衰多为舒张性心衰。
B.甲亢患者合并心律失常、心脏增大、心衰可诊断甲亢性心脏病。
6 4 2009 4. 诊断甲亢最敏感的指标是 (2.5分) A.TSH
B.FT3
C.FT4
D.TRAb (TSH-receptor Antibody)
E.TPO-Ab
A.TSH
6 4 2009 5. 下列几项临床表现中,对Graves 病最具诊断意义的是: (2.5分) A.甲状腺肿大
B.甲状腺可及血管杂音
C.心房颤动
D.高代谢症群
E.高血糖
B.甲状腺可及血管杂音
6 4 2009 6. 对甲亢治疗方法的选择和评价中,哪项是错误的。 (2.5分) A.口服抗甲状腺药物多用于年龄小、病情轻和轻中度甲状腺肿大患者
B.病情较重、病程长患者多采用同位素碘或手术治疗
C.甲状腺巨大的患者首选手术治疗
D.妊娠早中期的甲亢妇女可接受手术治疗
E.儿童患者首先考虑药物治疗
D.妊娠早中期的甲亢妇女可接受手术治疗
6 4 2009 7. Which of the following is the single most useful and cost-effective test for screening for thyroid disease? (2.5分) A.Radioactive iodine uptake and scan
B.Free T4
C.Free thyroxine index
D.Thyroid stimulating hormone level
E. T3 uptake
D.Thyroid stimulating hormone level
6 4 2009 8. A 40-year-old woman presents with mild anterior neck pain that developed 1 week after an episode of flu associated with sore throat. She is otherwise healthy and denies current use of medications. On physical examination, her blood pressure is 140/82 mmHg, and pulse is 102 bpm. Her left thyroid lobe is increased in size, tender, and has increased temperature. Laboratory findings show TSH of 0.1 mIU/ml (normal range= 0.5-4.5 mIU/ml), free T4 of 1.8 ng/dl (normal range =0.7-1.6 ng/dl), and erythrocyte sedimentation rate of 112 mm/hr. What would be the best initial treatment for this patient? (2.5分) A.Propylthiouracil
B.Propranolol
C.Iodine
D.Non-steroidal anti-inflammatory drugs
E.Prednisone
D.Non-steroidal anti-inflammatory drugs
6 4 2009 9. A 45-year-old man presents to the hospital complaining of a 4-week history of weakness of his shoulder muscles. He also reports that he has been losing weight without dieting and losing his temper a lot lately. He is not taking any medications. He works as a teacher and recently separated from his wife. On examination, he is not pale or jaundiced. His BP is 120/72mm/Hg and PR 120/min. He has a tremor when he extends his fingers. A muscle biopsy is normal. What is the most likely cause of his muscle weakness? (2.5分) A.Alcoholism
B.Cushing's disease
C.Hyperthyroidism
D.Hypothyroidism
E.Systemic Lupus Erythematosus
C.Hyperthyroidism
6 4 2009 10. One 40-year-old man came to the clinic of cardiology complaining of palpitation, short of breathe and lower limbs edema. On examination, he is wasted, and the heart beats irregularly and fast. EKG showed atrial fibrillation with fast ventricular rate, echocardiography found dilated right atrium and left ventricle. Laboratory results: FT3/FT4 were elevated markedly and TSH suppressed. Which of the following is NOT typical feature of thyrotoxic heart disease? (2.5分) A.Long-term history of uncontrolled hyperthyroidism
B.Composed of arrhythmia, heart hypertrophy, and heart failure
C.Heart disease may be the only manifestation of Graves' disease in old patients
D.Radioactive iodine is the first-line therapy
E.Symptoms of heart disease always persist after hyperthyroidism is controlled
E.Symptoms of heart disease always persist after hyperthyroidism is controlled
6 4 2009 11. 抗甲状腺药物如他巴唑, PTU常见的副反应有白细胞减少, 严重者可出现粒细胞缺乏,故治疗时应监测白细胞, 停药的指征为: (2.5分) A.白细胞总数低于4000, 中性粒细胞低于2000.
B.白细胞总数低于 3500, 中性粒细胞低于1000.
C.白细胞总数低于3000, 中性粒细胞低于2000.
D.白细胞总数低于3000, 中性粒细胞低于 1500.
E.白细胞总数低于4000, 中性粒细胞低于 1500.
D.白细胞总数低于3000, 中性粒细胞低于 1500.
6 4 2009 12. 有关甲减治疗的说法下列哪项不正确?(2.5分) A.首选左甲状腺素片,小剂量开始
B.有缺血性心脏病患者起始剂量宜小,调整剂量宜慢
C.治疗后复查甲状腺功能正常即可停药
D.不能与氢氧化铝和碳酸钙片同服
E.妊娠时治疗剂量需要增加30-50%
C.治疗后复查甲状腺功能正常即可停药
6 4 2009 13. 鉴别甲状腺性、垂体性和下丘脑性甲状腺功能减退最可靠的检查是 (2.5分) A.血清TSH测定
B.TSH刺激试验
C.TRH刺激试验
D.甲状腺自身抗体测定
E.血清TSH、TT3、TT4、FT3、FT4同时测定,综合分析
C.TRH刺激试验
6 4 2009 14. 关于甲减的心血管系统表现,下列说法不正确的是: (2.5分) A.心肌黏液性水肿可导致心肌收缩力损伤、心动过缓、心排血量下降
B.ECG显示低电压
C.由于心肌间质水肿、非特异性心肌纤维肿胀、左心室扩张和心包积液导致心脏扩大
D.心绞痛患者经甲状腺激素替代治疗后可缓解
E.约有10%的患者伴发高血压
D.心绞痛患者经甲状腺激素替代治疗后可缓解
6 4 2009 15. 患者女性,60岁。畏寒、嗜睡、纳差、便秘伴毛发脱落3年。体格检查:浮肿,皮肤粗糙,反应迟钝,心率56次/分,律齐。实验室检查:FT3↓,FT4↓,TSH↑。诊断为原发性甲减。治疗主要采用:(2.5分) A.支持治疗
B.对症治疗
C.病因治疗
D.替代治疗
E.滋补治疗
D.替代治疗
6 4 2009 16. 成人甲减最常见的原因是: (2.5分) A.自身免疫损伤
B.甲状腺破坏
C.碘缺乏
D.碘过量
E.抗甲状腺药物
A.自身免疫损伤
6 4 2009 17. 以下哪项甲减不属于病因分类?(2.5分) A.药物性甲减
B.手术后甲减
C.131碘治疗后甲减
D.特发性甲减
E.幼年甲减
E.幼年甲减
6 4 2009 18. 与甲状腺功能减退症表现不相符的是: (2.5分) A.少汗、嗜睡、畏寒
B.记忆力减退、反应迟钝、腱反射迟钝
C.食欲亢进、体重减轻
D.声音低哑、语言缓慢
E.便秘、月经过多
C.食欲亢进、体重减轻
6 4 2009 19. 慢性淋巴细胞性甲状腺炎最具特征的指标是(2.5分) A.TgAb
B.TPOAb
C.TRAb
D.TSAb
E.TSBAb
B.TPOAb
6 4 2009 20. 患者,女性,因"双侧甲状腺肿大"接受甲状腺穿刺活检,其结果是弥漫性淋巴细胞浸润,未见嗜酸性粒细胞及纤维化。该患者诊断是(2.5分) A.慢性淋巴细胞性甲状腺炎
B.亚急性淋巴细胞性甲状腺炎
C.亚急性肉芽肿性甲状腺炎
D.毒性弥漫性甲状腺肿
E.多结节性甲状腺肿
A.慢性淋巴细胞性甲状腺炎
6 4 2009 21. 35岁女性,3周前感冒伴咽痛,2周前已痊愈。近5天颈前疼痛明显,有低热,来门诊就诊。查体T 37.8℃,皮肤无汗,甲状腺Ⅱ度肿大,右叶硬,明显触痛拒按,WBC 7.8*10 9/L。临床诊断可能是(2.5分) A.甲状腺右叶囊肿出血
B.甲状腺癌伴出血
C.慢性淋巴细胞性甲状腺炎
D.急性化脓性甲状腺炎
E.亚急性甲状腺炎
E.亚急性甲状腺炎
6 4 2009 22. 女性,42岁,甲状腺部位疼痛,放射至下颌、耳部及枕部,伴甲状腺毒症。体格检查:左甲状腺肿大,可扪及一1.5cm*1.5cm结节,结节质地中等,伴压痛。诊断考虑亚急性甲状腺炎,下列哪项检查有助于诊断:(2.5分) A.FT3、FT4升高,TSH下降
B.FT3下降,TSH升高
C.FT3升高,甲状腺摄碘率明显下降,呈所谓"分离现象"
D.FT3、FT4升高,甲状腺摄碘率升高,但可被T3抑制
E.甲状腺穿刺细胞学检查
C.FT3升高,甲状腺摄碘率明显下降,呈所谓"分离现象"
6 4 2009 23. 女性,29岁,产后4个月,出现心悸、怕热、多汗、失眠,查血清FT3、FT4升高,TSH低,甲状腺摄碘率降低,本病最可能的诊断是(2.5分) A.慢性淋巴细胞性甲状腺炎
B.亚急性甲状腺炎
C.毒性弥漫性甲状腺肿
D.产后甲状腺炎
E.急性化脓性甲状腺炎
D.产后甲状腺炎
6 4 2009 24. 下列哪项不属于亚急性甲状腺炎的临床表现(2.5分) A.起病前1-3周常有病毒感染的症状
B.甲状腺区有明显触痛,可放射至耳部
C.少数有颈部淋巴结肿大
D.可有全身不适、食欲减退、肌肉疼痛、发热、心动过速、多汗等
E.体格检查发现双侧甲状腺弥漫性肿大,可闻及血管杂音
E.体格检查发现双侧甲状腺弥漫性肿大,可闻及血管杂音
6 4 2009 25. 糖耐量减低(IGT)是指OGTT 2小时血糖(2.5分) A.7.0~10.0 mmol/L
B.6.1~7.8 mmol/L
C.6.1~7.0 mmol/L
D.7.0~7.8 mmol/L
E.7.8~11.1 mmol/L
E.7.8~11.1 mmol/L
6 4 2009 26. 糖尿病肾病Ⅲ期尿白蛋白排泄量为(2.5分) A.30~300 mg/24h
B.20~200 mg/24h
C.25~250 mg/24h
D.10~100 mg/24h
E.35~350 mg/24h
A.30~300 mg/24h
6 4 2009 27. 糖尿病诊断标准中空腹血糖应(2.5分) A.≥6.1 mmol/L
B.≥7.8 mmol/L
C.≥11.1 mmol/L
D.≥5.6 mmol/L
E.≥7.0 mmol/L
E.≥7.0 mmol/L
6 4 2009 28. 口服降糖药不包括(2.5分) A.磺脲类
B.双胍类
C.噻唑烷二酮类
D.a-葡萄糖苷酶抑制剂
E.硫脲类
E.硫脲类
6 4 2009 29. 喉上神经外支支配(2.5分) A.声带运动
B.喉粘膜感觉
C.环甲肌运动
D. 带状肌运动
C.环甲肌运动
6 4 2009 30. 甲状腺危象一般发生在甲亢术后(2.5分) A.8小时内
B.8-12小时
C.12-36小时
D. 24-48小时
C.12-36小时
6 4 2009 31. 最常见的甲状腺癌病理类型是(2.5分) A.乳头状癌
B.滤泡状癌
C.髓样癌
D. 未分化癌
E.恶性淋巴瘤
A.乳头状癌
6 4 2009 32. 甲状腺术后12小时内,病人出现呼吸困难,首先考虑的原因是(2.5分) A.喉头水肿
B.声带麻痹
C.术后出血
D.严重低钙血症
E.严重的甲状腺功能低下
C.术后出血
6 4 2009 33. 分化型甲状腺癌指的是(2.5分) A.乳头状癌和滤泡状癌
B.乳头状癌和髓样癌
C.滤泡状癌和髓样癌
D.乳头状癌、滤泡状癌和髓样癌
E.以上都不是
A.乳头状癌和滤泡状癌
6 4 2009 34. 原发甲亢手术治疗时为了防止患者术后严重甲低,建议保留甲状腺组织为(2.5分) A.10%甲状腺组织
B.3-4g
C.4-6g
D. 6-8g
E.保留一侧甲状腺
D. 6-8g
6 4 2009 35. 根据最新的甲状腺癌AJCC分期T1期指的肿瘤直径为(2.5分) A.<1cm
B. <2cm
C. <3cm
D.<4cm
B. <2cm
6 4 2009 36. 甲状腺结节性质判断,术前诊断的最佳方法是(2.5分) A.B超
B.CT
C.FNAC
D.ECT
C.FNAC
6 4 2009 37. 男性,50岁,右侧甲状腺肿大,硬,韧,声音嘶哑,心率90次/分,应首先考虑为:(2.5分) A.甲状腺功能亢进
B.甲状腺腺瘤
C.结节性甲状腺肿
D.甲状腺癌
E.慢性甲状腺炎
D.甲状腺癌
6 4 2009 38. 男孩,10岁,颈前部硬结半年,无不适,检查颈前偏右有1.5cm直径硬块,随吞咽运动, 扫描为冷结节,边缘模糊,诊断最可能为:(2.5分) A.甲状腺腺瘤
B.甲状腺癌
C.甲状腺囊肿
D.青春期甲状腺增大
E.甲状腺舌管囊肿
B.甲状腺癌
6 4 2009 39. The half-life of T3 is about(2.5分) A.3-5hours
B.6-8 hours
C.8-12 hours
D.3 days
E.1-2 days
C.8-12 hours
6 4 2009 40. Iodine deficiency can result in (2.5分) A.endemic goiter
B.nodular goiter
C.Hypothyroidism
D.Cretinism
E.Hashimoto's disease
E.Hashimoto's disease
6 4 2010 1. 抗甲状腺药物如他巴唑, PTU常见的副反应有白细胞减少, 严重者可出现粒细胞缺乏,故治疗时应监测白细胞, 停药的指征为: (2.5分) A.白细胞总数低于4000, 中性粒细胞低于2000.
B.白细胞总数低于 3500, 中性粒细胞低于1000.
C.白细胞总数低于3000, 中性粒细胞低于2000.
D.白细胞总数低于3000, 中性粒细胞低于 1500.
E.白细胞总数低于4000, 中性粒细胞低于 1500.
D.白细胞总数低于3000, 中性粒细胞低于 1500.
6 4 2010 2. 有关甲亢性心脏病,下列哪一项是不正确的(2.5分) A.甲亢性心脏病多发生于有多年甲亢病史且控制不好的患者身上。
B.甲亢患者合并心律失常、心脏增大、心衰可诊断甲亢性心脏病。
C.甲亢性心脏病患者可选择同位素治疗。
D.甲亢性心脏病大多在甲亢控制后消失。
E.甲亢合并的心衰多为舒张性心衰。
B.甲亢患者合并心律失常、心脏增大、心衰可诊断甲亢性心脏病。
6 4 2010 3. 有关妊娠合并甲亢,错误的是(2.5分) A.所有妊娠妇女出现甲亢的表现时必须用抗甲状腺药物。
B.妊娠妇女合并甲亢禁用β受体阻滞剂和同位素碘治疗
C.正常妊娠妇女TSH也可低于正常,但FT3、FT4正常。
D.抗甲状腺药物较大剂量使用可通过胎盘影响胎儿的甲状腺功能
E.母体的甲状腺刺激性免疫球蛋白可通过胎盘,导致新生儿甲亢。
A.所有妊娠妇女出现甲亢的表现时必须用抗甲状腺药物。
6 4 2010 4. 下列那项不是放射性碘治疗Graves病的适应症(2.5分) A.抗甲状腺药物治疗后复发
B.伴重症浸润性突眼
C.使用抗甲状腺药物后出现粒细胞减少
D.某些高功能结节的甲亢患者
E.甲亢手术治疗后复发
B.伴重症浸润性突眼
6 4 2010 5. 下列哪项不是甲状腺手术的并发症? (2.5分) A.甲亢复发
B.甲状腺功能减退
C.声音嘶哑
D.甲状旁腺功能减退
E.出血、感染
A.甲亢复发
6 4 2010 6. Which of the following is the single most useful and cost-effective test for screening for thyroid disease?(2.5分) A.Radioactive iodine uptake and scan
B.Free T4
C.Free thyroxine index
D.Thyroid stimulating hormone level
E.T3 uptake
D.Thyroid stimulating hormone level
6 4 2010 7. A 56-year-old man presents to the office with signs and symptoms of hyperthyroidism in the presence of a palpable goiter. Laboratory studies are borderline suggestive, but do not confirm thyrotoxicosis. Further testing includes a RadioActive I131 Uptake (RAIU) and scan of the thyroid, which reveals a high uptake in a diffuse, uniform pattern throughout the gland. What is the most likely diagnosis?(2.5分) A.Thyroiditis
B.Grave's disease
C.Toxic multinodular goiter (TMNG)
D.Toxic Thyroid Adenoma
E.Thyroid cancer
B.Grave's disease
6 4 2010 8. One 40-year-old man came to the clinic of cardiology complaining of palpitation, short of breathe and lower limbs edema. On examination, he is wasted, and the heart beats irregularly and fast. EKG showed atrial fibrillation with fast ventricular rate, echocardiography found dilated right atrium and left ventricle. Laboratory results: FT3/FT4 were elevated markedly and TSH suppressed. Which of the following is NOT typical feature of thyrotoxic heart disease?(2.5分) A.Long-term history of uncontrolled hyperthyroidism
B.Composed of arrhythmia, heart hypertrophy, and heart failure
C.Heart disease may be the only manifestation of Graves' disease in old patients
D.Radioactive iodine is the first-line therapy
E.Symptoms of heart disease always persist after hyperthyroidism is controlled
E.Symptoms of heart disease always persist after hyperthyroidism is controlled
6 4 2010 9. Which one is NOT the clinical sign of Graves' ophthalmopathy?(2.5分) A.Glaucoma
B.Exophthalmos
C.Impaired eye muscle movement (Diplopia)
D.Periorbital and conjunctival edema
E.Corneal ulceration
A.Glaucoma
6 4 2010 10. 成人甲减最常见的原因是(2.5分) A.自身免疫损伤
B.甲状腺破坏
C.碘缺乏
D.碘过量
E.抗甲状腺药物
A.自身免疫损伤
6 4 2010 11. 以下哪种检查结果不符合原发性甲状腺功能减退症(2.5分) A.TT3、TT4低于正常
B.FT3、FT4低于正常
C.血胆固醇增加
D.血清TSH增高
E.膝腱反射亢进
E.膝腱反射亢进
6 4 2010 12. 下列有关黏液性水肿昏迷的说法哪项不正确?(2.5分) A.常见于病情严重的患者,多在夏季天气炎热时发病
B.常见诱因为严重的全身疾病、甲状腺激素替代治疗中断和手术等
C.可表现为嗜睡,低体温,心动过缓,四肢肌肉松弛,甚至昏迷
D.对这些患者需注意保暖
E.治疗上予补充甲状腺激素和糖皮质激素
A.常见于病情严重的患者,多在夏季天气炎热时发病
6 4 2010 13. 有关甲减治疗的说法下列哪项不正确?(2.5分) A.首选左甲状腺素片,小剂量开始
B.有缺血性心脏病患者起始剂量宜小,调整剂量宜慢
C.治疗后复查甲状腺功能正常即可停药
D.不能与氢氧化铝和碳酸钙片同服
E.妊娠时治疗剂量需要增加30-50%
C.治疗后复查甲状腺功能正常即可停药
6 4 2010 14. 患者,女性,因"双侧甲状腺肿大"接受甲状腺穿刺活检,其结果是弥漫性淋巴细胞浸润,未见嗜酸性粒细胞及纤维化。该患者诊断是(2.5分) A.慢性淋巴细胞性甲状腺炎
B.亚急性淋巴细胞性甲状腺炎
C.亚急性肉芽肿性甲状腺炎
D.毒性弥漫性甲状腺肿
E.多结节性甲状腺肿
A.慢性淋巴细胞性甲状腺炎
6 4 2010 15. 女性,51岁,乏力数年。体检:甲状腺结节性肿大,质韧。诊断考虑慢性淋巴细胞性甲状腺炎。下列哪项检查有确诊价值(2.5分) A.甲状腺针刺活组织检查
B.FT3、FT4正常,TSH升高
C.甲状腺摄碘率升高,但可被T3抑制
D.甲状腺呈均匀弥漫性摄碘率下降
E.TPOAb和TgAb滴度明显增高
A.甲状腺针刺活组织检查
6 4 2010 16. 女性,29岁,产后4个月,出现心悸、怕热、多汗、失眠,查血清FT3、FT4升高,TSH低,甲状腺摄碘率降低,本病最可能的诊断是(2.5分) A.慢性淋巴细胞性甲状腺炎
B.亚急性甲状腺炎
C.毒性弥漫性甲状腺肿
D.产后甲状腺炎
E.急性化脓性甲状腺炎
D.产后甲状腺炎
6 4 2010 17. 下列哪项不属于亚急性甲状腺炎的临床表现(2.5分) A.起病前1-3周常有病毒感染的症状
B.甲状腺区有明显触痛,可放射至耳部
C.少数有颈部淋巴结肿大
D.可有全身不适、食欲减退、肌肉疼痛、发热、心动过速、多汗等
E.体格检查发现双侧甲状腺弥漫性肿大,可闻及血管杂音
E.体格检查发现双侧甲状腺弥漫性肿大,可闻及血管杂音
6 4 2010 18. 糖尿病分型不包括(2.5分) A.2型糖尿病
B.1型糖尿病
C.儿童糖尿病
D.其他特殊类型糖尿病
E.妊娠糖尿病
C.儿童糖尿病
6 4 2010 19. 糖尿病肾病分几期(2.5分) A.3期
B.5期
C.4期
D.6期
E.2期
B.5期
6 4 2010 20. 糖尿病诊断标准中空腹血糖应(2.5分) A.≥6.1 mmol/L
B.≥7.8 mmol/L
C.≥11.1 mmol/L
D.≥5.6 mmol/L
E.≥7.0 mmol/L
E.≥7.0 mmol/L
6 4 2010 21. 糖耐量减低(IGT)是指OGTT 2小时血糖(2.5分) A.7.0~10.0 mmol/L
B.6.1~7.8 mmol/L
C.6.1~7.0 mmol/L
D.7.0~7.8 mmol/L
E.7.8~11.1 mmol/L
E.7.8~11.1 mmol/L
6 4 2010 22. 口服降糖药不包括(2.5分) A.磺脲类
B.双胍类
C.噻唑烷二酮类
D. -葡萄糖苷酶抑制剂
E.硫脲类
E.硫脲类
6 4 2010 23. 肥胖的2型糖尿病降糖药首选(2.5分) A.磺脲类
B.双胍类
C.噻唑烷二酮类
D. -葡萄糖苷酶抑制剂
E.胰岛素
B.双胍类
6 4 2010 24. 1型糖尿病有关的自身抗体不包括(2.5分) A.ICA
B.GADA
C.TPOAb
D.IAA
E.IA-2
C.TPOAb
6 4 2010 25. 妊娠糖尿病诊断标准中75g葡萄糖耐量试验1小时餐后血糖是(2.5分) A.≥5.1.mmol/L
B.≥6.1 mmol/L
C.≥7.0 mmol/L
D.≥7.8 mmol/L
E.≥10.0 mmol/L
E.≥10.0 mmol/L
6 4 2010 26. Gout is definitively diagnosed based on (2.5分) A.Serum uric acid
B.the demonstration of urate crystals in aspirated synovial fluid.
C.only 1 or 2 joints are usually involved.
B.the demonstration of urate crystals in aspirated synovial fluid.
6 4 2010 27. A gout attack may be triggered by (2.5分) A.precipitation of crystals in a supersaturated microenvironment (eg, release of urate due to cellular damage).
B.a release of uncoated crystals (eg, due to partial dissolution of a microtophus caused by changing serum urate levels
C.either a release of uncoated crystals (eg, due to partial dissolution of a microtophus caused by changing serum urate levels) or precipitation of crystals in a supersaturated microenvironment (eg, release of urate due to cellular damage).
C.either a release of uncoated crystals (eg, due to partial dissolution of a microtophus caused by changing serum urate levels) or precipitation of crystals in a supersaturated microenvironment (eg, release of urate due to cellular damage).
6 4 2010 28. 喉上神经外支支配(2.5分) A.声带运动
B.喉粘膜感觉
C.环甲肌运动
D.带状肌运动
C.环甲肌运动
6 4 2010 29. 甲状腺危象一般发生在甲亢术后(2.5分) A.8小时内
B.8-12小时
C.12-36小时
D. 24-48小时
C.12-36小时
6 4 2010 30. 最常见的甲状腺癌病理类型是(2.5分) A.乳头状癌
B.滤泡状癌
C.髓样癌
D.未分化癌
E.恶性淋巴瘤
A.乳头状癌
6 4 2010 31. 原发甲亢手术治疗时为了防止患者术后严重甲低,建议保留甲状腺组织为(2.5分) A.10%甲状腺组织
B.3-4g
C. 4-6g
D. 6-8g
E.保留一侧甲状腺
D. 6-8g
6 4 2010 32. 根据最新的甲状腺癌AJCC分期T1期指的肿瘤直径为(2.5分) A.<1cm
B. <2cm
C. <3cm
D.<4cm
B. <2cm
6 4 2010 33. 喉上神经外支损伤时(2.5分) A.易引起误咽
B.音调降低
C.饮水呛咳
D.呼吸困难
E.失音
B.音调降低
6 4 2010 34. 男孩,10岁,颈前部硬结半年,无不适,检查颈前偏右有1.5cm直径硬块,随吞咽运动, 扫描为冷结节,边缘模糊,诊断最可能为(2.5分) A.甲状腺腺瘤
B.甲状腺癌
C.甲状腺囊肿
D.青春期甲状腺增大
E.甲状腺舌管囊肿
B.甲状腺癌
6 4 2010 35. Iodine deficiency can result in(2.5分) A.endemic goiter
B.nodular goiter
C.Hypothyroidism
D.Cretinism
E.Hashimoto's disease
E.Hashimoto's disease
6 4 2010 36. The thyroid glandstoringof the total-body iodide (2.5分) A.80%
B.90%
C.95%
D.99%
E.100%
B.90%
6 4 2010 37. 正常人对血糖下降的反应没有(2.5分) A.胰岛素分泌减少或完全抑制
B.糖原合成增加
C.升糖激素分泌增加
D.下丘脑-肾上腺素能神经兴奋反应
E.认知障碍
B.糖原合成增加
6 4 2010 38. 貌似健康而自发性空腹或运动后低血糖症的最常见原因是(2.5分) A.药源性低血糖症
B.胰岛素瘤
C.反应性低血糖
D.胰岛素自身免疫综合征
E.伴瘤的低血糖
B.胰岛素瘤
6 4 2010 39. 评价低血糖症时,常用的诱发试验是(2.5分) A.C肽释放试验
B.胰岛素释放指数
C.葡萄糖耐量试验
D.C肽抑制试验
E.饥饿试验
E.饥饿试验
6 4 2010 40. 下列检查能排除胰岛素瘤的是(2.5分) A.禁食24小时不出现低血糖
B.禁食36小时不出现低血糖
C.禁食48小时不出现低血糖
D.禁食60小时不出现低血糖
E.禁食72小时不出现低血糖
E.禁食72小时不出现低血糖
6 4 2011 1. 抗甲状腺药物如他巴唑, PTU常见的副反应有白细胞减少, 严重者可出现粒细胞缺乏,故治疗时应监测白细胞, 停药的指征为: (2.5分) A.白细胞总数低于4000, 中性粒细胞低于2000.
B.白细胞总数低于 3500, 中性粒细胞低于1000.
C.白细胞总数低于3000, 中性粒细胞低于2000.
D.白细胞总数低于3000, 中性粒细胞低于 1500.
E.白细胞总数低于4000, 中性粒细胞低于 1500.
D.白细胞总数低于3000, 中性粒细胞低于 1500.
6 4 2011 2. 下列哪种情况同位素检查可有明显的甲状腺摄锝率增高(2.5分) A.桥本氏甲状腺炎
B.亚急性甲状腺炎
C.弥漫性毒性甲状腺肿
D.碘甲亢
E.多结节性甲状腺肿
C.弥漫性毒性甲状腺肿
6 4 2011 3. 下列哪项不属于淡漠型甲亢的特点(2.5分) A.老年甲亢
B.常有多食、多汗等表现
C.消瘦明显
D.常并发甲亢性心脏病
E.可有长期厌食、腹泻
B.常有多食、多汗等表现
6 4 2011 4. 诊断甲亢最敏感的指标是 (2.5分) A.TSH
B.FT3
C.FT4
D.TRAb (TSH-receptor Antibody)
E.TPO-Ab
A.TSH
6 4 2011 5. 下列几项临床表现中,对Graves 病最具诊断意义的是(2.5分) A.甲状腺肿大
B.甲状腺可及血管杂音
C.心房颤动
D.高代谢症群
E.高血糖
B.甲状腺可及血管杂音
6 4 2011 6. 对甲亢治疗方法的选择和评价中,哪项是错误的(2.5分) A.口服抗甲状腺药物多用于年龄小、病情轻和轻中度甲状腺肿大患者
B.病情较重、病程长患者多采用同位素碘或手术治疗
C.甲状腺巨大的患者首选手术治疗
D.妊娠早中期的甲亢妇女可接受手术治疗
E.儿童患者首先考虑药物治疗
D.妊娠早中期的甲亢妇女可接受手术治疗
6 4 2011 7. 下列哪项不是甲状腺手术的并发症? (2.5分) A.甲亢复发
B.甲状腺功能减退
C.声音嘶哑
D.甲状旁腺功能减退
E.出血、感染
A.甲亢复发
6 4 2011 8. A 40-year-old woman presents with mild anterior neck pain that developed 1 week after an episode of flu associated with sore throat. She is otherwise healthy and denies current use of medications. On physical examination, her blood pressure is 140/82 mmHg, and pulse is 102 bpm. Her left thyroid lobe is increased in size, tender, and has increased temperature. Laboratory findings show TSH of 0.1 mIU/ml (normal range= 0.5-4.5 mIU/ml), free T4 of 1.8 ng/dl (normal range =0.7-1.6 ng/dl), and erythrocyte sedimentation rate of 112 mm/hr. What would be the best initial treatment for this patient? (2.5分) A.Propylthiouracil
B.Propranolol
C.Iodine
D.Non-steroidal anti-inflammatory drugs
E.Prednisone
D.Non-steroidal anti-inflammatory drugs
6 4 2011 9. A 56-year-old man presents to the office with signs and symptoms of hyperthyroidism in the presence of a palpable goiter. Laboratory studies are borderline suggestive, but do not confirm thyrotoxicosis. Further testing includes a RadioActive I131 Uptake (RAIU) and scan of the thyroid, which reveals a high uptake in a diffuse, uniform pattern throughout the gland. What is the most likely diagnosis? (2.5分) A.Thyroiditis
B.Grave's disease
C.Toxic multinodular goiter (TMNG)
D.Toxic Thyroid Adenoma
E.Thyroid cancer
B.Grave's disease
6 4 2011 10. A 45-year-old man presents to the hospital complaining of a 4-week history of weakness of his shoulder muscles. He also reports that he has been losing weight without dieting and losing his temper a lot lately. He is not taking any medications. He works as a teacher and recently separated from his wife. On examination, he is not pale or jaundiced. His BP is 120/72mm/Hg and PR 120/min. He has a tremor when he extends his fingers. A muscle biopsy is normal. What is the most likely cause of his muscle weakness? (2.5分) A.Alcoholism
B.Cushing's disease
C.Hyperthyroidism
D.Hypothyroidism
E.Systemic Lupus Erythematosus
C.Hyperthyroidism
6 4 2011 11. 成人甲减最常见的原因是 (2.5分) A.自身免疫损伤
B.甲状腺破坏
C.碘缺乏
D.碘过量
E.抗甲状腺药物
A.自身免疫损伤
6 4 2011 12. 与甲状腺功能减退症表现不相符的是(2.5分) A.少汗、嗜睡、畏寒
B.记忆力减退、反应迟钝、腱反射迟钝
C.食欲亢进、体重减轻
D.声音低哑、语言缓慢
E.溢乳、月经过多
C.食欲亢进、体重减轻
6 4 2011 13. 鉴别甲状腺性、垂体性和下丘脑性甲状腺功能减退最可靠的检查是(2.5分) A.血清TSH测定
B.TSH刺激试验
C.TRH刺激试验
D.甲状腺自身抗体测定
E.血清TSH、TT3、TT4、FT3、FT4同时测定,综合分析
C.TRH刺激试验
6 4 2011 14. 下列指标中哪一个对亚临床型甲减的诊断具有重要意义(2.5分) A.TT4
B.TT3
C.s-TSH
D.FT3
E.FT4
C.s-TSH
6 4 2011 15. 下列有关黏液性水肿昏迷的说法哪项不正确? (2.5分) A.常见于病情严重的患者,多在夏季天气炎热时发病
B.常见诱因为严重的全身疾病、甲状腺激素替代治疗中断和手术等
C.可表现为嗜睡,低体温,心动过缓,四肢肌肉松弛,甚至昏迷
D.对这些患者需注意保暖
E.治疗上予补充甲状腺激素和糖皮质激素
A.常见于病情严重的患者,多在夏季天气炎热时发病
6 4 2011 16. 糖尿病分型不包括(2.5分) A.2型糖尿病
B.1型糖尿病
C.儿童糖尿病
D.其他特殊类型糖尿病
E.妊娠糖尿病
C.儿童糖尿病
6 4 2011 17. 糖尿病肾病分几期(2.5分) A.3期
B.5期
C.4期
D.6期
E.2期
B.5期
6 4 2011 18. 糖尿病诊断标准中空腹血糖应 (2.5分) A.≥6.1 mmol/L
B.≥7.8 mmol/L
C.≥11.1 mmol/L
D.≥5.6 mmol/L
E.≥7.0 mmol/L
E.≥7.0 mmol/L
6 4 2011 19. 空腹血糖受损(IFG)是指空腹血糖 (2.5分) A.5.6~7.0 mmol/L
B.7.0~7.8 mmol/L
C.7.8~11.1 mmol/L
D.6.1~7.0 mmol/L
E.7.0~10.0 mmol/L
D.6.1~7.0 mmol/L
6 4 2011 20. 糖尿病饮食治疗中制定每日总热量应根据下列因素,但不包括(2.5分) A.理想体重
B.工作性质
C.实际体重
D.参考原来的生活习惯
E.肥胖及消瘦程度
C.实际体重
6 4 2011 21. 妊娠糖尿病诊断标准空腹血糖是(2.5分) A.≥3.9.mmol/L
B.≥5.1 mmol/L
C.≥5.6 mmol/L
D.≥6.1 mmol/L
E.≥7.0 mmol/L
B.≥5.1 mmol/L
6 4 2011 22. 一般糖尿病患者(无肾病及特殊需要者)每日蛋白质摄入量应占总热量的(2.5分) A.5~10%
B.10~15%
C.15~20%
D.20~25%
E.25~30%
B.10~15%
6 4 2011 23. I型糖尿病有关的自身抗体不包括(2.5分) A.ICA
B.GADA
C.TPOAb
D.IAA
E.IA-2
C.TPOAb
6 4 2011 24. 妊娠糖尿病诊断标准中75g葡萄糖耐量试验2小时餐后血糖是(2.5分) A.≥6.1.mmol/L
B.≥7.0 mmol/L
C.≥7.8 mmol/L
D.≥8.5 mmol/L
E.≥10.0 mmol/L
D.≥8.5 mmol/L
6 4 2011 25. 合并冠心病的2型糖尿病病人LDL-c应控制小于(2.5分) A.3.6 mmol/L
B.3.2 mmol/L
C.2.8 mmol/L
D.2.6 mmol/L
E.1.8 mmol/L
E.1.8 mmol/L
6 4 2011 26. 喉上神经外支支配(2.5分) A.声带运动
B.喉粘膜感觉
C.环甲肌运动
D.带状肌运动
C.环甲肌运动
6 4 2011 27. 最常见的甲状腺癌病理类型是(2.5分) A.乳头状癌
B.滤泡状癌
C.髓样癌
D.未分化癌
E.恶性淋巴瘤
A.乳头状癌
6 4 2011 28. 甲状腺术后最凶险的并发症是(2.5分) A.喉返神经损伤
B.甲状旁腺损伤
C.喉上神经损伤
D.呼吸困难和窒息
E.甲亢危象
D.呼吸困难和窒息
6 4 2011 29. 原发甲亢手术治疗时为了防止患者术后严重甲低,建议保留甲状腺组织为(2.5分) A.10%甲状腺组织
B.3-4g
C.4-6g
D.6-8g
E.保留一侧甲状腺
D.6-8g
6 4 2011 30. 甲状腺结节性质判断,术前诊断的最佳方法是(2.5分) A.B超
B.CT
C.FNAC
D.ECT
C.FNAC
6 4 2011 31. 以下起源于甲状腺滤泡旁细胞的肿瘤是(2.5分) A.PTC
B.FTC
C.MTC
D.ATC
C.MTC
6 4 2011 32. 男性,50岁,右侧甲状腺肿大,硬,韧,声音嘶哑,心率90次/分,应首先考虑为(2.5分) A.甲状腺功能亢进
B.甲状腺腺瘤
C.结节性甲状腺肿
D.甲状腺癌
E.慢性甲状腺炎
D.甲状腺癌
6 4 2011 33. The half-life of T4 is about(2.5分) A.3 days
B.7 days
C.2 weeks
D.3 weeks
E.2 Months
B.7 days
6 4 2011 34. The half-life of T3 is about(2.5分) A.3-5hours
B.6-8 hours
C.8-12 hours
D.3 days
E.1-2 days
C.8-12 hours
6 4 2011 35. The Size of Papillary Thyroid Microcarcinoma(PTMC)is(2.5分) A.<3mm
B.<5mm
C.<10mm
D.<20mm
E.10mm-20mm
C.<10mm
6 4 2011 36. 评价低血糖症时,常用的诱发试验是(2.5分) A.C肽释放试验
B.胰岛素释放指数
C.葡萄糖耐量试验
D.C肽抑制试验
E.48-72小时饥饿试验
E.48-72小时饥饿试验
6 4 2011 37. 对于特发性功能性低血糖症,下列几项中错误的是 (2.5分) A.病程较长,可呈进行性发展
B.是餐后低血糖症的常见类型
C.可自行缓解
D.血浆胰岛素水平、胰岛素释放指数均在正常范围
E.多见于女性
A.病程较长,可呈进行性发展
6 4 2011 38. 女,35岁,患有1型糖尿病1年,3小时前注射胰岛素12U后因生气而未进食,20分钟前家人发现患者意识不清而将其送来急诊,查体:BP 150/85mmHg,面色苍白,皮肤多汗,心率110/min,四肢肌张力减弱,病理征(-)。则正确的处理是(2.5分) A.立即抽血查血糖,待结果出来后再作处理
B.快速静脉滴注生理盐水,同时静脉注射胰岛素首次负荷量后,继续以每小时每公斤体重0.1u的速率静滴
C.立即经口给予糖水
D.立即静脉滴注5%葡萄糖盐水
E.立即抽血查血糖,但不必等待结果,立即给予50%葡萄糖液60-100ml,静脉注射
E.立即抽血查血糖,但不必等待结果,立即给予50%葡萄糖液60-100ml,静脉注射
6 4 2011 39. 正常人对血糖下降的反应没有(2.5分) A.胰岛素分泌减少或完全抑制
B.糖原合成增加
C.升糖激素分泌增加
D.下丘脑-肾上腺素能神经兴奋反应
E.认知障碍
B.糖原合成增加
6 4 2011 40. 下列检查能排除胰岛素瘤的是(2.5分) A.禁食24小时不出现低血糖
B.禁食36小时不出现低血糖
C.禁食48小时不出现低血糖
D.禁食60小时不出现低血糖
E.禁食72小时不出现低血糖
E.禁食72小时不出现低血糖
6 4 2012 1. 抗甲状腺药物如他巴唑, PTU常见的副反应有白细胞减少, 严重者可出现粒细胞缺乏,故治疗时应监测白细胞, 停药的指征为: (2.5分) A.白细胞总数低于4000, 中性粒细胞低于2000.
B.白细胞总数低于 3500, 中性粒细胞低于1000.
C.白细胞总数低于3000, 中性粒细胞低于2000.
D.白细胞总数低于3000, 中性粒细胞低于 1500.
E.白细胞总数低于4000, 中性粒细胞低于 1500.
D.白细胞总数低于3000, 中性粒细胞低于 1500.
6 4 2012 2. Graves病甲亢药物治疗停药的指征是(2.5分) A.甲亢症状消失
B.T3、T4、TSH恢复正常
C.TRH兴奋试验、T3抑制试验正常
D.TRAb 阴转
E.以上均是
E.以上均是
6 4 2012 3. 有关甲亢性心脏病,下列哪一项是不正确的(2.5分) A.甲亢性心脏病多发生于有多年甲亢病史且控制不好的患者身上。
B.甲亢患者合并心律失常、心脏增大、心衰可诊断甲亢性心脏病。
C.甲亢性心脏病患者可选择同位素治疗。
D.甲亢性心脏病大多在甲亢控制后消失。
E.甲亢合并的心衰多为舒张性心衰。
B.甲亢患者合并心律失常、心脏增大、心衰可诊断甲亢性心脏病。
6 4 2012 4. 有关妊娠合并甲亢,错误的是(2.5分) A.所有妊娠妇女出现甲亢的表现时必须用抗甲状腺药物。
B.妊娠妇女合并甲亢禁用β受体阻滞剂和同位素碘治疗
C.正常妊娠妇女TSH也可低于正常,但FT3、FT4正常。
D.抗甲状腺药物较大剂量使用可通过胎盘影响胎儿的甲状腺功能
E.母体的甲状腺刺激性免疫球蛋白可通过胎盘,导致新生儿甲亢。
A.所有妊娠妇女出现甲亢的表现时必须用抗甲状腺药物。
6 4 2012 5. 有关Graves病的描述,不正确的是(2.5分) A.Graves病甲亢多发生于生育期女性
B.甲亢病因中,Graves病占85%。
C.甲亢浸润性突眼必须发生于甲亢程度明显的Graves病患者身上。
D.Graves病与桥本甲状腺炎可同时发生于同一患者
E.突眼、胫前粘液性水肿是Graves病的特征性临床表现。
C.甲亢浸润性突眼必须发生于甲亢程度明显的Graves病患者身上。
6 4 2012 6. 下列那项不是放射性碘治疗Graves病的适应症(2.5分) A.抗甲状腺药物治疗后复发
B.伴重症浸润性突眼
C.使用抗甲状腺药物后出现粒细胞减少
D.某些高功能结节的甲亢患者
E.甲亢手术治疗后复发
B.伴重症浸润性突眼
6 4 2012 7. Which of the following is the single most useful and cost-effective test for screening for thyroid disease? (2.5分) A.Radioactive iodine uptake and scan
B.Free T4
C.Free thyroxine index
D.Thyroid stimulating hormone level
E.T3 uptake
D.Thyroid stimulating hormone level
6 4 2012 8. A 35-year-old woman presents with a 2-year history of the gradual onset of nervousness and fatigue. She has lost 8 Kgs in the past year despite an increase in appetite. Her menses have become scant with frequent intermenstrual spotting and she feels that she constantly prefers a cooler environment than those around her. On physical examination, she has a heart rate of 100/min with normal blood pressure and a fine tremor of the fingers is noted. The skin is warm, moist, and has a smooth texture. The thyroid is diffusely enlarged to palpation. A lid-lag sign is easily demonstrable. A common complication of this disease is:(2.5分) A.Hepatitis
B.Interstitial pneumonia
C.Infiltrative ophthalmopathy
D.Peptic ulcer disease
E.Hemolytic anemia
C.Infiltrative ophthalmopathy
6 4 2012 9. A 45-year-old man presents to the hospital complaining of a 4-week history of weakness of his shoulder muscles. He also reports that he has been losing weight without dieting and losing his temper a lot lately. He is not taking any medications. He works as a teacher and recently separated from his wife. On examination, he is not pale or jaundiced. His BP is 120/72mm/Hg and PR 120/min. He has a tremor when he extends his fingers. A muscle biopsy is normal. What is the most likely cause of his muscle weakness? (2.5分) A.Alcoholism
B.Cushing's disease
C.Hyperthyroidism
D.Hypothyroidism
E.Systemic Lupus Erythematosus
C.Hyperthyroidism
6 4 2012 10. 最常见的甲状腺癌病理类型是(2.5分) A.乳头状癌
B.滤泡状癌
C.髓样癌
D.未分化癌
E.恶性淋巴瘤
A.乳头状癌
6 4 2012 11. 原发甲亢手术治疗时为了防止患者术后严重甲低,建议保留甲状腺组织为(2.5分) A.10%甲状腺组织
B.3-4g
C.4-6g
D.6-8g
E.保留一侧甲状腺
D.6-8g
6 4 2012 12. 甲状腺髓样癌的特异性指标是(2.5分) A.Ct、 CEA
B.CEA、 AFP
C.AFP、CT
D.CA199 AFP
A.Ct、 CEA
6 4 2012 13. 男性,50岁,右侧甲状腺肿大,硬,韧,声音嘶哑,心率90次/分,应首先考虑为(2.5分) A.甲状腺功能亢进
B.甲状腺腺瘤
C.结节性甲状腺肿
D.甲状腺癌
E.慢性甲状腺炎
D.甲状腺癌
6 4 2012 14. Iodine deficiency can not result in(2.5分) A.endemic goiter
B.nodular goiter
C.Hypothyroidism
D.Cretinism
E.Hashimoto's disease
E.Hashimoto's disease
6 4 2012 15. The Size of Papillary Thyroid Microcarcinoma(PTMC)is (2.5分) A.<3mm
B.<5mm
C.<10mm
D.<20mm
E.10mm-20mm
C.<10mm
6 4 2012 16. 原发性甲状腺功能减退症替代治疗的原则(2.5分) A.由大剂量开始,逐渐递减到合适剂量
B.从小剂量开始,逐渐递增到合适剂量
C.甲减越严重,起始剂量应越大
D.间断用药有利于预防心绞痛
E.待甲状腺功能正常后可停用
B.从小剂量开始,逐渐递增到合适剂量
6 4 2012 17. 甲减合并妊娠治疗时,TSH应控制在(2.5分) A.<2.5miu/L
B.>2.5miu/L
C.<5miu/L
D.<10miu/L
E.控制在正常范围上限
A.<2.5miu/L
6 4 2012 18. 甲减的临床表现中,下列哪项是错误的(2.5分) A.心动过缓
B.皮肤粗糙
C.大便次数增多或腹泻
D.疲劳、嗜睡
E.贫血
C.大便次数增多或腹泻
6 4 2012 19. A 60-year-old woman comes to the emergency room in a coma. The patient's temperature is 35.5℃. She is bradycardic. Her thyroid gland is enlarged. There is bilateral hyporeflexia. The next step in management is (2.5分) A.Await results of T4, TSH
B.Obtain T4, TSH; begin thyroid hormone and glucocorticoids
C.Begin rapid rewarming
D.Obtain CT scan of the head
E.Urgent echocardiography
B.Obtain T4, TSH; begin thyroid hormone and glucocorticoids
6 4 2012 20. 慢性淋巴细胞性甲状腺炎最具诊断意义的指标是(2.5分) A.TgAb
B.ESR
C.TRAb
D.TPOAb
E.TSBAb
D.TPOAb
6 4 2012 21. 患者 女性,44岁。3周前感冒伴咽痛,2周前已痊愈。近5天颈前疼痛明显,有低热。查体T 37.8℃,皮肤无汗,甲状腺Ⅱ度肿大,右叶硬,明显触痛拒按,WBC 7.8*10 9/L。临床诊断可能是(2.5分) A.甲状腺右叶囊肿出血
B.甲状腺癌伴出血
C.慢性淋巴细胞性甲状腺炎
D.急性化脓性甲状腺炎
E.亚急性甲状腺炎
E.亚急性甲状腺炎
6 4 2012 22. 女性,51岁,乏力数年。体检:甲状腺结节性肿大,质韧。诊断考虑慢性淋巴细胞性甲状腺炎。下列哪项检查有确诊价值(2.5分) A.甲状腺针刺活组织检查
B.FT3、FT4正常,TSH升高
C.甲状腺摄碘率升高,但可被T3抑制
D.甲状腺呈均匀弥漫性摄碘率下降
E.TPOAb和TgAb滴度明显增高
A.甲状腺针刺活组织检查
6 4 2012 23. 女性,29岁,产后4个月,出现心悸、怕热、多汗、失眠,查血清FT3、FT4升高,TSH降低,血沉正常,TRAB(-)。本病最可能的诊断是(2.5分) A.慢性淋巴细胞性甲状腺炎
B.亚急性甲状腺炎
C.毒性弥漫性甲状腺肿
D.产后甲状腺炎
E.急性化脓性甲状腺炎
D.产后甲状腺炎
6 4 2012 24. 下列关于亚急性甲状腺炎的描述,错误的是(2.5分) A.甲状腺弥漫性肿大、质地韧,表面不光滑
B.可合并颈部淋巴结肿大
C.发病以女性多见
D.甲状腺疼痛可从一叶转移至另一叶
E.为自限性疾病,不会发展为甲减
A.甲状腺弥漫性肿大、质地韧,表面不光滑
6 4 2012 25. 糖尿病肾病根据尿白蛋白排泄量分期A2期尿白蛋白为(2.5分) A.30~300 mg/g
B.20~200 mg/g
C.25~250 mg/g
D.10~100 mg/g
E.35~350 mg/g
A.30~300 mg/g
6 4 2012 26. HbA1c能反映取血前几周的平均血糖水平(2.5分) A.2~4周
B.4~6周
C.6~8周
D.8~12周
E.12~16周
D.8~12周
6 4 2012 27. 糖尿病诊断标准中空腹血糖应(2.5分) A.≥6.1 mmol/L
B.≥7.8 mmol/L
C.≥11.1 mmol/L
D.≥5.6 mmol/L
E.≥7.0 mmol/L
E.≥7.0 mmol/L
6 4 2012 28. 糖耐量减低(IGT)是指OGTT 2小时血糖(2.5分) A.7.0~10.0 mmol/L
B.6.1~7.8 mmol/L
C.6.1~7.0 mmol/L
D.7.0~7.8 mmol/L
E.7.8~11.1 mmol/L
E.7.8~11.1 mmol/L
6 4 2012 29. 口服降糖药不包括(2.5分) A.磺脲类
B.双胍类
C.噻唑烷二酮类
D.a-葡萄糖苷酶抑制剂
E.硫脲类
E.硫脲类
6 4 2012 30. 肥胖的2型糖尿病降糖药首选(2.5分) A.磺脲类
B.双胍类
C.噻唑烷二酮类
D.a-葡萄糖苷酶抑制剂
E.胰岛素
B.双胍类
6 4 2012 31. 妊娠糖尿病诊断标准中75g葡萄糖耐量试验1小时餐后血糖是(2.5分) A.≥5.1.mmol/L
B.≥6.1 mmol/L
C.≥7.0 mmol/L
D.≥7.8 mmol/L
E.≥10.0 mmol/L
E.≥10.0 mmol/L
6 4 2012 32. 下列哪项不是糖尿病慢性并发症(2.5分) A.低血糖
B.大血管病变
C.肾脏病变
D.视网膜病变
E.神经病变
A.低血糖
6 4 2012 33. 纠正低血糖后,脑功能恢复的顺序是(2.5分) A.延脑-下丘脑-皮质下中枢-自主神经中枢-大脑皮质
B.延脑-自主神经中枢-下丘脑-皮质下中枢-大脑皮质
C.大脑皮质-下丘脑-皮质下中枢-自主神经中枢-延脑
D.大脑皮质-皮质下中枢-延脑-下丘脑-自主神经中枢
E.下丘脑-皮质下中枢-延脑-自主神经中枢-大脑皮质
B.延脑-自主神经中枢-下丘脑-皮质下中枢-大脑皮质
6 4 2012 34. T评价低血糖症时,常用的诱发试验是(2.5分) A.C肽释放试验
B.胰岛素释放指数
C.葡萄糖耐量试验
D.C肽抑制试验
E.48-72小时饥饿试验
E.48-72小时饥饿试验
6 4 2012 35. 男性,36岁,1型糖尿病,今日因感冒,食欲减退,少食,餐前按常规注射胰岛素,近午时突然心悸、出汗、手抖,继而头晕,视物模糊。应如何处理 (2.5分) A.胰岛素皮下注射
B.静滴生理盐水
C.葡萄糖静滴
D.碳酸氢钠注射
E.急查血糖,见结果后再行处理
C.葡萄糖静滴
6 4 2012 36. 患者女性,34岁,既往无特殊病史,营养状况良好,无特殊服药史,近1个月常于空腹时出现心悸、出汗、手抖,进食后好转,院外查FT4、FT3、TSH正常。最可能的诊断是 (2.5分) A.心律失常
B.低血糖症
C.神经官能症
D.糖尿病
E.嗜铬细胞瘤
B.低血糖症
6 4 2012 37. 针对持续性亚临床甲亢(3-6月内2次甲状腺功能检查),下列哪种情况需要治疗? (2.5分) A.TSH < 0.1年龄大于65岁
B.TSH < 0.1,年龄45-65岁,有心脏疾患。
C.TSH 0.1-0.4,年龄大于65岁
D.TSH 0.1-0.4,年龄45-65岁,有骨代谢疾患
E.以上所有
E.以上所有
6 4 2012 38. One 40-year-old man came to the clinic of cardiology complaining of palpitation, short of breathe and lower limbs edema. On examination, he is wasted, and the heart beats irregularly and fast. EKG showed atrial fibrillation with fast ventricular rate, echocardiography found dilated right atrium and left ventricle. Laboratory results: FT3/FT4 were elevated markedly and TSH suppressed. Which of the following is NOT typical feature of thyrotoxic heart disease? (2.5分) A.Long-term history of uncontrolled hyperthyroidism
B.Composed of arrhythmia, heart hypertrophy, and heart failure
C.Heart disease may be the only manifestation of Graves' disease in old patients
D.Radioactive iodine is the first-line therapy
E.Symptoms of heart disease always persist after hyperthyroidism is controlled
E.Symptoms of heart disease always persist after hyperthyroidism is controlled
6 4 2012 39. 2型糖尿病病人每周主动有氧运动时间应(2.5分) A.≥60分钟
B.≥90分钟
C.≥110分钟
D.≥130分钟
E.≥150分钟
E.≥150分钟
6 4 2012 40. The half-life of T4 is about(2.5分) A.3 days
B.7 days
C.2 weeks
D.3 weeks
E.2 Months
B.7 days
6 4 2013 1. Graves病甲亢药物治疗停药的指征是(2.5分) A.甲亢症状消失
B.T3、T4、TSH恢复正常
C.TRH兴奋试验、T3抑制试验正常
D.TRAb 阴转
E.以上均是
E.以上均是
6 4 2013 2. 下列几项临床表现中,对Graves 病最具诊断意义的是(2.5分) A.甲状腺肿大
B.甲状腺可及血管杂音
C.心房颤动
D.高代谢症群
E.高血糖
B.甲状腺可及血管杂音
6 4 2013 3. 有关甲亢性心脏病,下列哪一项是不正确的(2.5分) A.甲亢性心脏病多发生于有多年甲亢病史且控制不好的患者身上。
B.甲亢患者合并心律失常、心脏增大、心衰可诊断甲亢性心脏病。
C.甲亢性心脏病患者可选择同位素治疗。
D.甲亢性心脏病大多在甲亢控制后消失。
E.甲亢合并的心衰多为舒张性心衰。
B.甲亢患者合并心律失常、心脏增大、心衰可诊断甲亢性心脏病。
6 4 2013 4. 有关妊娠合并甲亢,错误的是(2.5分) A.所有妊娠妇女出现甲亢的表现时必须用抗甲状腺药物。
B.妊娠妇女合并甲亢禁用β受体阻滞剂和同位素碘治疗
C.正常妊娠妇女TSH也可低于正常,但FT3、FT4正常。
D.抗甲状腺药物较大剂量使用可通过胎盘影响胎儿的甲状腺功能
E.母体的甲状腺刺激性免疫球蛋白可通过胎盘,导致新生儿甲亢。
A.所有妊娠妇女出现甲亢的表现时必须用抗甲状腺药物。
6 4 2013 5. 有关Graves病的描述,不正确的是(2.5分) A.Graves病甲亢多发生于生育期女性
B.甲亢病因中,Graves病占85%。
C.甲亢浸润性突眼必须发生于甲亢程度明显的Graves病患者身上。
D.Graves病与桥本甲状腺炎可同时发生于同一患者
E.突眼、胫前粘液性水肿是Graves病的特征性临床表现。
C.甲亢浸润性突眼必须发生于甲亢程度明显的Graves病患者身上。
6 4 2013 6. 下列那项不是放射性碘治疗Graves病的适应症(2.5分) A.抗甲状腺药物治疗后复发
B.伴重症浸润性突眼
C.使用抗甲状腺药物后出现粒细胞减少
D.某些高功能结节的甲亢患者
E.甲亢手术治疗后复发
B.伴重症浸润性突眼
6 4 2013 7. Which of the following is the single most useful and cost-effective test for screening for thyroid disease? (2.5分) A.Radioactive iodine uptake and scan
B.Free T4
C.Free thyroxine index
D.Thyroid stimulating hormone level
E.T3 uptake
D.Thyroid stimulating hormone level
6 4 2013 8. A 35-year-old woman presents with a 2-year history of the gradual onset of nervousness and fatigue. She has lost 8 Kgs in the past year despite an increase in appetite. Her menses have become scant with frequent intermenstrual spotting and she feels that she constantly prefers a cooler environment than those around her. On physical examination, she has a heart rate of 100/min with normal blood pressure and a fine tremor of the fingers is noted. The skin is warm, moist, and has a smooth texture. The thyroid is diffusely enlarged to palpation. A lid-lag sign is easily demonstrable. A common complication of this disease is:(2.5分) A.Hepatitis
B.Interstitial pneumonia
C.Infiltrative ophthalmopathy
D.Peptic ulcer disease
E.Hemolytic anemia
C.Infiltrative ophthalmopathy
6 4 2013 9. A 45-year-old man presents to the hospital complaining of a 4-week history of weakness of his shoulder muscles. He also reports that he has been losing weight without dieting and losing his temper a lot lately. He is not taking any medications. He works as a teacher and recently separated from his wife. On examination, he is not pale or jaundiced. His BP is 120/72mm/Hg and PR 120/min. He has a tremor when he extends his fingers. A muscle biopsy is normal. What is the most likely cause of his muscle weakness? (2.5分) A.Alcoholism
B.Cushing's disease
C.Hyperthyroidism
D.Hypothyroidism
E.Systemic Lupus Erythematosus
C.Hyperthyroidism
6 4 2013 10. One 40-year-old man came to the clinic of cardiology complaining of palpitation, short of breathe and lower limbs edema. On examination, he is wasted, and the heart beats irregularly and fast. EKG showed atrial fibrillation with fast ventricular rate, echocardiography found dilated right atrium and left ventricle. Laboratory results: FT3/FT4 were elevated markedly and TSH suppressed. Which of the following is NOT typical feature of thyrotoxic heart disease? (2.5分) A.Long-term history of uncontrolled hyperthyroidism
B.Composed of arrhythmia, heart hypertrophy, and heart failure
C.Heart disease may be the only manifestation of Graves' disease in old patients
D.Radioactive iodine is the first-line therapy
E.Symptoms of heart disease always persist after hyperthyroidism is controlled
E.Symptoms of heart disease always persist after hyperthyroidism is controlled
6 4 2013 11. 下列关于慢性淋巴细胞性甲状腺炎的描述,错误的是(2.5分) A.甲状腺炎中最常见的类型
B.发病以30-50岁女性为主
C.甲状腺肿大为其突出表现
D.易合并颈部淋巴结肿大
E.有发展为甲减的趋势
D.易合并颈部淋巴结肿大
6 4 2013 12. 患者,女性,36岁,因"发现双侧甲状腺肿大伴结节",行甲状腺穿刺活检,结果可见多淋巴细胞浸润,未见嗜酸性粒细胞及纤维化。该患者诊断是(2.5分) A.急性淋巴细胞性甲状腺炎
B.毒性弥漫性甲状腺肿
C.慢性淋巴细胞性甲状腺炎
D.亚急性肉芽肿性甲状腺炎
E.结节性甲状腺肿
C.慢性淋巴细胞性甲状腺炎
6 4 2013 13. 患者,女性,42岁,甲状腺部位疼痛,放射至下颌、耳部及枕部,伴甲状腺毒症。体格检查:左甲状腺肿大,可扪及一1.5cm*1.5cm结节,结节质地中等,伴压痛。诊断考虑亚急性甲状腺炎,下列哪项检查有助于诊断(2.5分) A.FT3、FT4升高,TSH下降
B.FT3下降,TSH升高
C.FT3升高,甲状腺摄碘率明显下降,呈所谓"分离现象"
D.FT3、FT4升高,甲状腺摄碘率升高,但可被T3抑制
E.甲状腺穿刺细胞学检查
C.FT3升高,甲状腺摄碘率明显下降,呈所谓"分离现象"
6 4 2013 14. 女性,29岁,产后4个月,哺乳。近期诉颈部增粗,伴心悸、怕热、多汗、失眠,查血清FT3、FT4升高,TSH<0.01,APOAb明显增高。首先考虑以下哪种疾病(2.5分) A.慢性淋巴细胞性甲状腺炎
B.亚急性甲状腺炎
C.毒性弥漫性甲状腺肿
D.产后甲状腺炎
E.急性化脓性甲状腺炎
D.产后甲状腺炎
6 4 2013 15. 女性,35岁,甲状腺部位疼痛,并出现结节,红细胞沉降率加速,甲状腺摄碘率明显下降,经双氯芬酸钠治疗后,临床症状迅速消失,诊断考虑(2.5分) A.亚急性甲状腺炎
B.桥本甲状腺炎
C.甲状腺癌
D.甲状腺囊肿囊内出血
E.甲状腺腺瘤内出血
A.亚急性甲状腺炎
6 4 2013 16. 下列关于亚急性甲状腺炎的描述,错误的是(2.5分) A.甲状腺弥漫性肿大、质地韧
B.可合并颈部淋巴结肿大
C.发病以女性多见
D.甲状腺疼痛可从一叶转移至另一叶
E.为自限性疾病,一般不会发展为甲减
A.甲状腺弥漫性肿大、质地韧
6 4 2013 17. 成人甲减最常见的原因是 (2.5分) A.自身免疫损伤
B.甲状腺破坏
C.碘缺乏
D.碘过量
E.抗甲状腺药物
A.自身免疫损伤
6 4 2013 18. 与甲状腺功能减退症表现不相符的是 (2.5分) A.少汗、嗜睡、畏寒
B.记忆力减退、反应迟钝、腱反射迟钝
C.食欲亢进、体重减轻
D.声音低哑、语言缓慢
E.便秘、月经过多
C.食欲亢进、体重减轻
6 4 2013 19. 鉴别甲状腺性、垂体性和下丘脑性甲状腺功能减退最可靠的检查是(2.5分) A.血清TSH测定
B.TSH刺激试验
C.TRH刺激试验
D.甲状腺自身抗体测定
E.血清TSH、TT3、TT4、FT3、FT4同时测定,综合分析
C.TRH刺激试验
6 4 2013 20. 患者女性,60岁。畏寒、嗜睡、纳差、便秘伴毛发脱落3年。体格检查:浮肿,皮肤粗糙,反应迟钝,心率56次/分,律齐。实验室检查:FT3↓,FT4↓,TSH↑。诊断为原发性甲减。治疗主要采用(2.5分) A.支持治疗
B.对症治疗
C.病因治疗
D.替代治疗
E.滋补治疗
D.替代治疗
6 4 2013 21. 下列有关黏液性水肿昏迷的说法哪项不正确(2.5分) A.常见于病情严重的患者,多在夏季天气炎热时发病
B.常见诱因为严重的全身疾病、甲状腺激素替代治疗中断和手术等
C.可表现为嗜睡,低体温,心动过缓,四肢肌肉松弛,甚至昏迷
D.对这些患者需注意保暖
E.治疗上予补充甲状腺激素和糖皮质激素
A.常见于病情严重的患者,多在夏季天气炎热时发病
6 4 2013 22. 男性,48岁。体检发现多发肾结石入院。实验室检查:血钙3.1mmol/L,血磷0.7mmol/L。24小时尿钙210mg。胃镜检查:胃窦前壁溃疡。根据患者情况,诊断首先要考虑:(2.5分) A.原发性肾结石
B.继发性肾结石
C.高钙血症
D.甲状旁腺功能亢进
E.以上都不是
D.甲状旁腺功能亢进
6 4 2013 23. 女性,43岁。因甲状腺功能亢进行手术治疗,术后第2天患者突然出现面部及四肢抽搐。应立即做出的处置措施是(2.5分) A.静脉滴注地塞米松
B.口服抗甲状腺药物
C.肌内注射安定
D.口服甲状腺素片
E.静脉推注10%葡萄糖酸钙
E.静脉推注10%葡萄糖酸钙
6 4 2013 24. 女性,56岁,颈前肿大20余年就诊。查体:甲状腺III度肿大,质地硬,多个结节,最大结节直径5.0cm,随吞咽活动,气管轻度左移。 血T3,T4,TSH正常,TgAb(-),TPOAb(-).气管正侧位X线片示气管局部受压,向左侧移位。最佳的处理措施是(2.5分) A.长期服用甲状腺素
B.定期检测甲状腺功能
C.手术治疗
D.多食含碘丰富的食物
E.禁用含碘药物
C.手术治疗
6 4 2013 25. 女性,42岁,右侧颈部肿块2月余,如蚕豆大小,可活动,无压痛,无发热咳嗽,鼻咽部无异常。甲状腺峡部可触及直径1cm大小结节。最可能的诊断是(2.5分) A.慢性淋巴结炎
B.甲状腺癌转移
C.淋巴结结核
D.肺癌转移
E.淋巴瘤
B.甲状腺癌转移
6 4 2013 26. HbA1c能反映取血前几周的平均血糖水平(2.5分) A.2~4周
B.4~6周
C.6~8周
D.8~12周
E.12~16周
D.8~12周
6 4 2013 27. 糖尿病诊断标准中空腹血糖应(2.5分) A.≥6.1 mmol/L
B.≥7.8 mmol/L
C.≥11.1 mmol/L
D.≥5.6 mmol/L
E.≥7.0 mmol/L
E.≥7.0 mmol/L
6 4 2013 28. 糖耐量减低(IGT)是指OGTT 2小时血糖(2.5分) A.7.0~10.0 mmol/L
B.6.1~7.8 mmol/L
C.6.1~7.0 mmol/L
D.7.0~7.8 mmol/L
E.7.8~11.1 mmol/L
E.7.8~11.1 mmol/L
6 4 2013 29. 口服降糖药不包括(2.5分) A.磺脲类
B.双胍类
C.噻唑烷二酮类
D.a-葡萄糖苷酶抑制剂
E.硫脲类
E.硫脲类
6 4 2013 30. 肥胖的2型糖尿病降糖药首选(2.5分) A.磺脲类
B.双胍类
C.噻唑烷二酮类
D.a-葡萄糖苷酶抑制剂
E.胰岛素
B.双胍类
6 4 2013 31. 对甲状腺肿瘤性质诊断正确率可达80%以上的检查方法是(2.5分) A.B超
B.CT
C.ECT
D.MRI
E.FNA
E.FNA
6 4 2013 32. 下列哪项不是糖尿病慢性并发症(2.5分) A.低血糖
B.大血管病变
C.肾脏病变
D.视网膜病变
E.神经病变
A.低血糖
6 4 2013 33. 纠正低血糖后,脑功能恢复的顺序是(2.5分) A.延脑-下丘脑-皮质下中枢-自主神经中枢-大脑皮质
B.延脑-自主神经中枢-下丘脑-皮质下中枢-大脑皮质
C.大脑皮质-下丘脑-皮质下中枢-自主神经中枢-延脑
D.大脑皮质-皮质下中枢-延脑-下丘脑-自主神经中枢
E.下丘脑-皮质下中枢-延脑-自主神经中枢-大脑皮质
B.延脑-自主神经中枢-下丘脑-皮质下中枢-大脑皮质
6 4 2013 34. 评价低血糖症时,常用的诱发试验是 (2.5分) A.C肽释放试验
B.胰岛素释放指数
C.葡萄糖耐量试验
D.C肽抑制试验
E.48-72小时饥饿试验
E.48-72小时饥饿试验
6 4 2013 35. 男性,36岁,1型糖尿病,今日因感冒,食欲减退,少食,餐前按常规注射胰岛素,近午时突然心悸、出汗、手抖,继而头晕,视物模糊。应如何处理(2.5分) A.胰岛素皮下注射
B.静滴生理盐水
C.葡萄糖静滴
D.碳酸氢钠注射
E.急查血糖,见结果后再行处理
C.葡萄糖静滴
6 4 2013 36. 患者女性,34岁,既往无特殊病史,营养状况良好,无特殊服药史,近1个月常于空腹时出现心悸、出汗、手抖,进食后好转,院外查FT4、FT3、TSH正常。最可能的诊断是 (2.5分) A.心律失常
B.低血糖症
C.神经官能症
D.糖尿病
E.嗜铬细胞瘤
B.低血糖症
6 4 2013 37. 针对持续性亚临床甲亢(3-6月内2次甲状腺功能检查),下列哪种情况需要治疗? (2.5分) A.TSH < 0.1年龄大于65岁
B.TSH < 0.1,年龄45-65岁,有心脏疾患。
C.TSH 0.1-0.4,年龄大于65岁
D.TSH 0.1-0.4,年龄45-65岁,有骨代谢疾患
E.以上所有
E.以上所有
6 4 2013 38. 与甲状腺髓样癌有关的激素是(2.5分) A.甲状腺素
B.促甲状腺素
C.降钙素
D.促甲状腺激素释放激素
E.甲状旁腺激素
C.降钙素
6 4 2013 39. 关于甲状腺癌,下面说法正确的是(2.5分) A.甲状腺乳头状癌好发于中年女性,恶性程度低,早期可发生血行转移
B.甲状腺髓样癌可分泌PTH等,引起类癌综合征
C.甲状腺滤泡癌早期可发生淋巴结转移,预后较甲状腺乳头状癌差
D.甲状腺乳头状癌是甲状腺癌肿预后最好的病理类型
E.甲状腺滤泡状腺瘤和甲状腺滤泡状癌可通过FNA进行鉴别
D.甲状腺乳头状癌是甲状腺癌肿预后最好的病理类型
6 4 2013 40. 女性,44岁,健康体检发现甲状腺肿大就诊。查体甲状腺对称性Ⅱ度肿大,表面不平,中等硬度,无触痛,无血管杂音,心率78次/分。拟诊为慢性淋巴细胞性甲状腺炎,有助于确诊的首选检查是(2.5分) A.TSH,FT3,FT4
B.抗甲状腺抗体
C.甲状腺B超
D.甲状腺摄碘131率
E.甲状腺CT
B.抗甲状腺抗体
6 4 2014 1. A 48-year-old woman returns for a follow-up visit for management of type 1 diabetes mellitus. She reports doing well since the last visit. Overall, she believes that most of her blood glucose levels are at goal, but is concerned about occasional episodes of hyperglycemia occurring in the morning before breakfast. She eats a bedtime snack every night that is not covered with mealtime insulin. Review of her blood glucose log demonstrates morning fasting blood glucose values from 80 to 190 mg/dL (4.4-10.5 mmol/L). Her other premeal and bedtime values range from 100 to 120 mg/dL (5.5-6.7 mmol/L). She exercises two to three times per week in the evening. Medical history is significant for hypertension and hyperlipidemia.Medications are insulin glargine, insulin lispro, ramipril, simvastatin, and aspirin. On physical examination, blood pressure is 130/72 mm Hg and pulse rate is 67/min. BMI is 24. The remainder of the examination is unremarkable. Results of laboratory studies show a hemoglobin A1c level of 6.9% and serum creatinine level of 1.0 mg/dL (88.4 ?mol/L). Serum electrolytes are normal.Which of the following is the most appropriate management of this patient's occasional fasting hyperglycemia?(4.0分) A.add insulin lisrpo at bedtime
B.add metformin
C.increase insulin glargine
D.Measure 3 AM blood glucose level
E.Continue current regimen
D.Measure 3 AM blood glucose level
6 4 2014 2. A 74-year-old woman is evaluated in the emergency department for several hours of altered mental status. She is from out-of-state and is visiting with relatives. One of her young relatives was recently ill with gastrointestinal symptoms. The patient developed anorexia 3 days ago and vomiting 2 days ago. She has been unable to tolerate any liquid or solid foods for the last 24 hours. Medical history is significant for type 2 diabetes mellitus, hypertension, hyperlipidemia, and hypothyroidism. Medications are aspirin, lisinopril, glimepiride, levothyroxine, and atorvastatin. Her last dose of medications was 48 hours ago.On physical examination, her temperature is 37.5 °C (99.5 °F), blood pressure is 115/65 mm Hg, and pulse rate is 95/min. She is arousable but confused. Mucous membranes are dry. Her neck is supple. Cardiac examination reveals no murmurs. Her chest is clear to auscultation. Bowel sounds are present, and mild tenderness to palpation is noted throughout the abdomen. There is no rebound or guarding. There are no focal neurologic deficits.Laboratory studies are pending.Which of the following is the most likely cause of this patient's altered mental status?(4.0分) A.cerebrovascular accident
B.hypoglycemia
C.hypothyroidism
D.statin toxicity
B.hypoglycemia
6 4 2014 3. A 52-year-old woman presents for follow-up evaluation after being diagnosed with type 2 diabetes mellitus 6 weeks ago. Her initial hemoglobin A1c level was 8.0%. Management at this time is with lifestyle modifications. She has worked closely with a diabetes educator and a nutritionist since her diagnosis. She has lost 3.2 kg (7 lb) by making changes to her diet and activity level. Review of her blood glucose log for the past 2 weeks shows preprandial blood glucose values in the 150 to 160 mg/dL (8.3-8.9 mmol/L) range and several 2-hour postprandial blood glucose values of 190 to 200 mg/dL (10.5-11.1 mmol/L). Her only other medical problem is hypertension for which she takes lisinopril.On physical examination, blood pressure is 125/70 mm Hg and pulse rate is 74/min. BMI is 28. There is no evidence of diabetic retinopathy. She has normal monofilament and vibratory sensation in her extremities.Except for her blood glucose parameters, basic laboratory studies obtained at the time of her initial diagnosis were normal.In addition to continuing lifestyle modifications, which of the following is the most appropriate management for this patient's diabetes?(4.0分) A.initiate dapagliflozin
B.initiate glipizide
C.initiate metformin
D.initiate sitagliptin
C.initiate metformin
6 4 2014 4. A 66-year-old man is evaluated in the office after being treated in the emergency department for an exacerbation of chronic obstructive pulmonary disease. While in the emergency department, he was noted to have a random blood glucose level of 211 mg/dL (11.7 mmol/L). His hemoglobin A1c was 7.8% at the time. A repeat random fingerstick blood glucose level in office is 204 mg/dL (11.3 mmol/L).The patient reports recent polyuria and polydipsia. He has lost 6 kg (13.2 lb) over the last 3 months. He has chronic epigastric pain associated with loose, oily stools due to chronic pancreatitis.He has a 20-pack-year history of tobacco use and prior alcohol use, however, he does not currently use alcohol. Current medications are enteric-coated pancreatic enzymes, vitamins, tiotropium inhaler, and an albuterol inhaler as needed.On physical examination, temperature is 37.1 °C (98.8 °F), blood pressure is 130/75 mm Hg, and pulse rate is 90/min. BMI is 22. He has mild epigastric pain on palpation without rebound tenderness or guarding. The rest of his examination is unremarkable.Which of the following is the most appropriate treatment for his diabetes?(4.0分) A.exenatide
B.glipizide
C.insulin
D.metformin
C.insulin
6 4 2014 5. A 72-year-old man presents to the emergency department overnight. His wife noted that he was diaphoretic and restless during the night, and his initial blood glucose level measured by emergency medical services was 41 mg/dL (2.3 mmol/L). He was given a single dose of intravenous glucose, which increased his blood glucose level to 85 mg/dL (4.7 mmol/L). His symptoms recurred en route to the hospital, and he responded to a second dose of intravenous glucose. While in the emergency department, he was provided food and his glucose level has remained above 80 mg/dL (4.4 mmol/L) without further treatment. The patient's wife reports several recent episodes of overnight diaphoresis over the last several weeks. His last meal prior to this episode was approximately 7 hours before symptom onset.His medical history is significant for hypertension and osteoarthritis. Medications are lisinopril and as-needed ibuprofen.On physical examination, the patient is alert and oriented. Blood pressure is 135/84 mm Hg, and pulse rate is 82/min. BMI is 23. The general physical and neurologic examinations are normal.Laboratory studies are significant for a normal complete metabolic profile and blood count. An insulin secretagogue screen is obtained, and hypoglycemic studies consisting of measurement of insulin, C-peptide, proinsulin, and β-hydroxybutyrate levels are planned.Which of the following is the most appropriate diagnostic strategy for this patient?(4.0分) A.hypoglycemic studies now
B.mixed meal testing with hypoglycemic studies at the time of symptomatic hypoglycemia
C.oral glucose tolerance testing with hypoglycemia studies at the time of symptomatic hypoglycemia
D.72-hour fasting with hypoglycemic studies at the time of symptomatic hypoglycemia
D.72-hour fasting with hypoglycemic studies at the time of symptomatic hypoglycemia
6 4 2014 6. A 21-year-old woman is evaluated for management of type 1 diabetes. She was diagnosed 3 months ago after presenting to the emergency department with diabetic ketoacidosis. Her hemoglobin A1c level at the time of diagnosis was 15.2%. She was discharged from the hospital on a basal and prandial insulin regimen with minor adjustments required as an outpatient. Her blood glucose log demonstrates a recent change manifesting as frequent symptomatic episodes of both fasting and postprandial hypoglycemia with blood glucose levels ranging from 50 to 65 mg/dL (2.8-3.6 mmol/L). Medications are insulin glargine and insulin aspart.On physical examination, blood pressure is 110/70 mm Hg and pulse rate is 70/min. BMI is 19. The remainder of the examination is unremarkable.Hemoglobin A1c level is 6.2%.Which of the following is the most appropriate management for this patient's hypoglycemia?(4.0分) A.Decrease insulin glargine, decrease insulin aspart
B.Decrease insulin glargine, decrease insulin aspart, add pramlintide
C.Discontinue insulin glargine, change insulin aspart to a sliding-scale regimen
D.Discontinue insulin glargine, discontinue insulin aspart
A.Decrease insulin glargine, decrease insulin aspart
6 4 2014 7. A 23-year-old woman is evaluated because of a 1-week history of palpitations. She also reports some heat intolerance and mild anxiety during the last several weeks, but she otherwise feels well. She is in the first trimester of an otherwise uncomplicated first pregnancy. Her only medication is a prenatal vitamin.On physical examination, she is afebrile, blood pressure is 110/72 mm Hg, pulse rate is 105/min, and respiration rate is 13/min. BMI is 20. The skin is warm and moist. There is no proptosis or lid lag. Examination of the neck shows a diffusely enlarged thyroid with an audible bruit over both lobes. Cardiopulmonary and abdominal examinations are unremarkable. Neurologic examination reveals a fine resting tremor of the hands and brisk reflexes.Laboratory studies: Thyroid-stimulating hormone(4.0分) A.Methimazole
B.Propylthiouracil
C.Radioactive iodine
D.Thyroidectomy
B.Propylthiouracil
6 4 2014 8. A 64-year-old man in the ICU is evaluated because of abnormal thyroid function tests. He was admitted 3 days ago for community-acquired pneumonia requiring intubation, mechanical ventilation, intravenous fluids, and dopamine support for his blood pressure.On physical examination, temperature is 38.8 °C (101.8 °F), blood pressure is 95/60 mm Hg, and pulse rate is 130/min. The skin is warm and dry. There is no proptosis. Examination of the neck shows a normal-sized thyroid without nodules. Cardiovascular examination reveals regular tachycardia. On neurologic examination, reflexes are slightly delayed.The serum thyroid-stimulating hormone (TSH) level is 0.1 ?U/mL (0.1 mU/L), the serum free thyroxine level (T4) is 0.9 ng/dL (11.6 pmol/L), and the serum total triiodothyronine (T3) level is 50 ng/dL (0.8 nmol/L).Which of the following is the most likely cause of this patient's abnormal thyroid function?(4.0分) A.Euthyroid sick syndrome
B.Graves' disease
C.Hashimoto thyroiditis
D.Subacute thyroiditis
A.Euthyroid sick syndrome
6 4 2014 9. A 74-year-old woman is evaluated because of new-onset anxiety and insomnia. For the last 6 weeks, she has been waking up multiple times each night. She does not have heat intolerance, change in bowel habits, palpitations, or dyspnea on exertion. She takes no medications.On physical examination, blood pressure is 125/68 mm Hg and pulse rate is 89/min. BMI is 18. There is no proptosis or lid lag. Examination of the thyroid reveals a 1.5-cm firm nodule in the left lobe that moves upward with swallowing. A fine resting hand tremor is present bilaterally.Laboratory studies reveal a serum thyroid-stimulating hormone level of 0.05 ?U/mL (0.05 mU/L), a serum free thyroxine (T4) level of 2.9 ng/dL (37.4 pmol/L), and a serum total triiodothyronine (T3) level of 250 ng/dL (3.8 nmol/L).Ultrasound of the neck shows two thyroid nodules, a 1.5-cm nodule in the right lobe and a 2.0-cm nodule in the left lobe.Which of the following is the most appropriate next step in management?(4.0分) A.fine needle aspiration of both thyroid nodules
B.initiation of methimazole
C.radioactive iodine (I123) uptake and scan of the thyroid
D.total thyroidectomy
C.radioactive iodine (I123) uptake and scan of the thyroid
6 4 2014 10. A 29-year-old woman is evaluated during her first prenatal visit. She feels well. Medical history is significant only for hypothyroidism, for which she has taken levothyroxine, 100 ?g/d, for the last 3 years. Her only other medication is a prenatal vitamin.On physical examination, she is afebrile. Blood pressure is 98/72 mm Hg, pulse rate is 88/min, and respiration rate is 12/min. The thyroid is of normal size and without nodules. There is no cervical lymphadenopathy. Cardiovascular and pulmonary examinations are unremarkable. Abdominal examination reveals normal bowel sounds. The uterus is not palpable.Laboratory studies show a serum thyroid-stimulating hormone level of 3.6 ?U/mL (3.6 mU/L) and a serum total thyroxine (T4) level of 4.5 ?g/dL (58 nmol/L).Which of the following is the most appropriate treatment of this patient's hypothyroidism?(4.0分) A.continue levothyroxine dose
B.decrease levothyroxine dose
C.discontinue levothyroxine
D.increase levothyroxine dose
D.increase levothyroxine dose
6 4 2014 11. A 32-year-old woman is evaluated for a 2-week history of foreign body sensation in her eyes. Despite flushing the eyes numerous times with saline, the sensation remains. She also feels pressure behind her eyes. She is not pregnant. Medical history is unremarkable.On physical examination, scleral injection and periorbital edema are present bilaterally, right worse than left. Diplopia is noted on lateral gaze. There is no lid lag or proptosis. The thyroid is diffusely enlarged, and a bruit is noted. Neurologic examination discloses brisk reflexes and a bilateral hand tremor.Laboratory studies reveal a serum thyroid-stimulating hormone level of less than 0.008 ?U/mL (0.008 mU/L), a serum free thyroxine (T4) level of 4.5 ng/dL (58.0 pmol/L), and a serum total triiodothyronine (T3) level of 365 ng/dL (5.6 nmol/L).Which of the following is the most appropriate next step in treatment?(4.0分) A.external-beam radiotherapy to the orbits
B.methimazole
C.radioactive iodine
D.total thyroidectomy
B.methimazole
6 4 2014 12. A 78-year-old woman is evaluated for a rapidly enlarging neck mass that has been present for 4 weeks and is associated with neck discomfort, dysphagia, and hoarseness. The patient has had Hashimoto thyroiditis and hypothyroidism since age 24 years and has been taking levothyroxine since that time.Physical examination reveals an older woman in severe distress. Temperature is 39.4 °C (102.9 °F), blood pressure is 145/75 mm Hg, pulse rate is 110/min, and respiration rate is 16/min; BMI is 23. Pulmonary examination reveals dyspnea with bilateral basilar rhonchi, and cardiac examination shows tachycardia without a murmur. The thyroid gland is enlarged and firm without nodules. Facial plethora and distended bilateral cervical neck veins are noted. The patient is hoarse. Bilateral cervical lymphadenopathy is palpated. Although neurologically intact, she finds it difficult to concentrate when asked questions.A thyroid ultrasound shows an enlarged thyroid gland with heterogeneous echotexture but no specific nodularity and multiple bilateral cervical lymph nodes measuring 1 to 3 cm in diameter.Which of the following is the most likely diagnosis?(4.0分) A.Bleeding into the thyroid gland
B.Medullary thyroid cancer
C.Papillary thyroid cancer
D.Primary thyroid lymphoma
D.Primary thyroid lymphoma
6 4 2014 13. A 62 year old farmer is evaluated as a new patient. He has a 20-year history of hypothyroidism and hypogonadism. He takes levothyroxine (88 mcg daily) and testosterone gel 1% (2.5 g daily) and fells well. He weights 75Kg and has a small thyroid. Physical examination findings are unremarkable. Laboratory test results include the following: thyrotropin 0.2 mIU/L (reference range 0.3 - 5.0 mIU/L) and free thyroxine 1.3 ng/dL (reference range 0.8 - 1.8 ng/dl). You decided to decrease the levothyroxine to 75 mcg daily. After 2 months, thyrotropin was 0.1 mIU/L, free thyroxine was 0.9 ng/dL, and he feels tired and cold. Which of the following should you do now?(4.0分) A.Decrease the levothyroxine dosage to 50 mcg daily and recheck in 6 weeks.
B.Stop levothyroxine altogether and evaluate thyroid uptake for hyperthyroidism
C.Discontinue the testosterone gel since it increases the thyroxine-binding globulins.
D.Check his thyroglobulin to distinguish between primary and secondary hypothyroidism.
E.Continue levothyroxine at 125 mcg daily, get his old records, and consider cortisol testing.
E.Continue levothyroxine at 125 mcg daily, get his old records, and consider cortisol testing.
6 4 2014 14. A 70-year-old man was hospitalized because of anterior chest pain. His vital signs were normal. An urgent coronary angiogram led to angioplasty, which was successfully carried out, and the patient was discharged home 2 days later. During the next month, he had some nervousness, anxiety and a 10 kg weight loss while eating his usual diet. His cardiologist recently determined that the patient had an atrial fibrillation and the prescriber a calcium channel blocker. When you see him a few days later in the emergency department, he has a ventricular rate of 130 beats per minutes and shortness of breath. On examination, he is in no acute distress at rest, but he is in atrial fibrillation with a rate of 120-130 bpm. He has no tremors and no signs of grave's ophthalmopathy or dermopathy. His thyroid is low-lying, multinodular, nontender, and slightly enlarged. You learn of a long-standing history of nontoxic multi nodule goiters. Because of the atrial fibrillation, thyroid test was performed, and the result was as follows: Thyrotropin less than 0.01 mIU/L, free thyroxine 3.7 ng/dL, and negative for thyroid peroxidase antibodies. What is the most likely cause of this patient's thyrotoxicosis?(4.0分) A.Lymphocytic thyroiditis
B.Autonomous thyroid nodule
C.Iodine induced hyperthyroidism.
D.Graves' disease
E.TSH producing pituitary adenoma.
C.Iodine induced hyperthyroidism.
6 4 2014 15. 貌似健康而自发性空腹或运动后低血糖症的最常见原因是: (4.0分) A.药源性低血糖症
B.胰岛素瘤
C.反应性低血糖
D.胰岛素自身免疫综合征
E.伴瘤的低血糖
B.胰岛素瘤
6 4 2014 16. 纠正低血糖后,脑功能恢复的顺序是:(4.0分) A.延脑-下丘脑-皮质下中枢-自主神经中枢-大脑皮质
B.延脑-自主神经中枢-下丘脑-皮质下中枢-大脑皮质
C.大脑皮质-下丘脑-皮质下中枢-自主神经中枢-延脑
D.大脑皮质-皮质下中枢-延脑-下丘脑-自主神经中枢
E.下丘脑-皮质下中枢-延脑-自主神经中枢-大脑皮质
B.延脑-自主神经中枢-下丘脑-皮质下中枢-大脑皮质
6 4 2014 17. 对于特发性功能性低血糖症,下列几项中错误的是: (4.0分) A.病程较长,可呈进行性发展
B.是餐后低血糖症的常见类型
C.可自行缓解
D.血浆胰岛素水平、胰岛素释放指数均在正常范围
E.多见于女性
A.病程较长,可呈进行性发展
6 4 2014 18. 评价低血糖症时,常用的诱发试验是:(4.0分) A.C肽释放试验
B.胰岛素释放指数
C.葡萄糖耐量试验
D.C肽抑制试验
E.48-72小时饥饿试验
E.48-72小时饥饿试验
6 4 2014 19. 男性,36岁,1型糖尿病,今日因感冒,食欲减退,少食,餐前按常规注射胰岛素,近午时突然心悸、出汗、手抖,继而头晕,视物模糊。应如何处理:(4.0分) A.胰岛素皮下注射
B.静滴生理盐水
C.葡萄糖静滴
D.碳酸氢钠注射
E.急查血糖,见结果后再行处理
C.葡萄糖静滴
6 4 2014 20. 患者女性,34岁,既往无特殊病史,营养状况良好,无特殊服药史,近1个月常于空腹时出现心悸、出汗、手抖,进食后好转,院外查FT4、FT3、TSH正常。最可能的诊断是:(4.0分) A.心律失常
B.低血糖症
C.神经官能症
D.糖尿病
E.嗜铬细胞瘤
B.低血糖症
6 4 2014 21. 男性,48岁。体检发现多发肾结石入院。实验室检查:血钙3.1mmol/L,血磷0.7mmol/L。24小时尿钙210mg。胃镜检查:胃窦前壁溃疡。根据患者情况,诊断首先要考虑: (4.0分) A.原发性肾结石
B.继发性肾结石
C.高钙血症
D.甲状旁腺功能亢进
E.以上都不是
D.甲状旁腺功能亢进
6 4 2014 22. 女性,43岁。因甲状腺功能亢进行手术治疗,术后第2天患者突然出现面部及四肢抽搐。应立即做出的处置措施是:(4.0分) A.静脉滴注地塞米松
B.口服抗甲状腺药物
C.肌内注射安定
D.口服甲状腺素片
E.静脉推注10%葡萄糖酸钙
E.静脉推注10%葡萄糖酸钙
6 4 2014 23. 女性,56岁,颈前肿大20余年就诊。查体:甲状腺III度肿大,质地硬,多个结节,最大结节直径5.0cm,随吞咽活动,气管轻度左移。血T3,T4,TSH正常,TgAb(-),TPOAb(-),气管正侧位X线片示气管局部受压,向左侧移位。最佳的处理措施是:(4.0分) A.长期服用甲状腺素
B.定期检测甲状腺功能
C.手术治疗
D.多食含碘丰富的食物
E.禁用含碘药物
C.手术治疗
6 4 2014 24. 女,20岁,左颈前部肿块2个月来诊。查体:甲状腺右叶上级触及3×2cm肿块,表面光滑,颈部未触及肿大的淋巴结。该病人首选的检查是:(4.0分) A.甲状腺B超
B.颈部CT
C.甲状腺核素扫描
D.肿块针吸细胞学检查
E.颈部正侧位X线片
A.甲状腺B超
6 4 2014 25. 对甲状腺肿瘤性质诊断正确率可达80%以上的检查方法是:(4.0分) A.B超
B.CT
C.ECT
D.MRI
E.FNA
E.FNA
6 4 2015 1. 抗甲状腺药物如他巴唑,PTU常见的副反应有白细胞减少,严重者可出现粒细胞缺乏,故治疗时应监测白细胞,停药的指征为(2.5分) A.白细胞总数低于4000, 中性粒细胞低于2000.
B.白细胞总数低于 3500, 中性粒细胞低于1000.
C.白细胞总数低于3000, 中性粒细胞低于2000.
D.白细胞总数低于3000, 中性粒细胞低于 1500.
E.白细胞总数低于4000, 中性粒细胞低于 1500.
D.白细胞总数低于3000, 中性粒细胞低于 1500.
6 4 2015 2. Graves病甲亢药物治疗停药的指征是(2.5分) A.甲亢症状消失
B.3、T4、TSH恢复正常
C.TRH兴奋试验、T3抑制试验正常
D.TRAb 阴转
E.以上均是
E.以上均是
6 4 2015 3. 有关甲亢性心脏病,下列哪一项是不正确的(2.5分) A.甲亢性心脏病多发生于有多年甲亢病史且控制不好的患者身上。
B.甲亢患者合并心律失常、心脏增大、心衰可诊断甲亢性心脏病。
C.甲亢性心脏病患者可选择同位素治疗。
D.甲亢性心脏病大多在甲亢控制后消失。
E.甲亢合并的心衰多为舒张性心衰。
B.甲亢患者合并心律失常、心脏增大、心衰可诊断甲亢性心脏病。
6 4 2015 4. 有关妊娠合并甲亢,错误的是(2.5分) A.所有妊娠妇女出现甲亢的表现时必须用抗甲状腺药物。
B.妊娠妇女合并甲亢禁用β受体阻滞剂和同位素碘治疗
C.正常妊娠妇女TSH也可低于正常,但FT3、FT4正常。
D.抗甲状腺药物较大剂量使用可通过胎盘影响胎儿的甲状腺功能
E.母体的甲状腺刺激性免疫球蛋白可通过胎盘,导致新生儿甲亢。
A.所有妊娠妇女出现甲亢的表现时必须用抗甲状腺药物。
6 4 2015 5. 下列几项临床表现中,对Graves 病最具诊断意义的是(2.5分) A.甲状腺肿大
B.甲状腺可及血管杂音
C.心房颤动
D.高代谢症群
E.高血糖
B.甲状腺可及血管杂音
6 4 2015 6. 下列哪项不是甲状腺手术的并发症? (2.5分) A.甲亢复发
B.甲状腺功能减退
C.声音嘶哑
D.甲状旁腺功能减退
E.出血、感染
A.甲亢复发
6 4 2015 7. A 40-year-old woman presents with mild anterior neck pain that developed 1 week after an episode of flu associated with sore throat. She is otherwise healthy and denies current use of medications. On physical examination, her blood pressure is 140/82 mmHg, and pulse is 102 bpm. Her left thyroid lobe is increased in size, tender, and has increased temperature. Laboratory findings show TSH of 0.1 mIU/ml (normal range= 0.5-4.5 mIU/ml), free T4 of 1.8 ng/dl (normal range =0.7-1.6 ng/dl), and erythrocyte sedimentation rate of 112 mm/hr. What would be the best initial treatment for this patient? (2.5分) A.Propylthiouracil
B.Propranolol
C.Iodine
D.Non-steroidal anti-inflammatory drugs
E.Prednisone
D.Non-steroidal anti-inflammatory drugs
6 4 2015 8. Which antibody is the etiology of Graves' disease(2.5分) A.Thyroglobulin antibodies;
B.Thyroid peroxidase antibodies;
C.Thyroid hormone antibody;
D.Thyrotropin (TSH) Receptor antibody;
E.Thyroid antibody
D.Thyrotropin (TSH) Receptor antibody;
6 4 2015 9. 男性,48岁。体检发现多发肾结石入院。实验室检查:血钙3.1mmol/L,血磷0.7mmol/L。24小时尿钙210mg。胃镜检查:胃窦前壁溃疡。根据患者情况,诊断首先要考虑(2.5分) A.原发性肾结石
B.继发性肾结石
C.高钙血症
D.甲状旁腺功能亢进
E.以上都不是
D.甲状旁腺功能亢进
6 4 2015 10. 女性,23岁,因原发性甲状腺功能亢进症在气管插管全麻下行甲状腺双侧次全切除术,术后清醒拔出气管插管后患者出现呼吸困难,伴有失音,无手足麻木。查体:T 37.3 ℃,P 92次/min, R28次/min,BP130/70mmHg,面红无发绀,颈部不肿,引流管通畅,有少许血液流出。引起该患者呼吸困难最可能的原因示:(2.5分) A.双侧喉返神经损伤
B.甲亢危象
C.喉上神经损伤
D.伤口出血
E.甲状旁腺损伤
A.双侧喉返神经损伤
6 4 2015 11. 女,60岁。食欲不振,恶心呕吐3个月。既往有多次骨折病史。实验室检查:血钙3.3mmol/L,血清甲状旁腺激素高于正常。骨密度检查示重度骨质疏松。为明确诊断,首选的检查是:(2.5分) A.血清CEA监测
B.胃镜
C.上消化道钡餐透视
D.头颅CT
E.颈部B超
E.颈部B超
6 4 2015 12. 女,20岁,左颈前部肿块2个月来诊。查体:甲状腺右叶上级触及3×2cm肿块,表面光滑,颈部未触及肿大的淋巴结。该病人首选的检查是:(2.5分) A.甲状腺B超
B.颈部CT
C.甲状腺核素扫描
D.肿块针吸细胞学检查
E.颈部正侧位X线片
A.甲状腺B超
6 4 2015 13. 与甲状腺髓样癌有关的激素是(2.5分) A.甲状腺素
B.促甲状腺素
C.降钙素
D.促甲状腺激素释放激素
E.甲状旁腺激素
C.降钙素
6 4 2015 14. 在病程的不同阶段,甲状腺功能可以分别出现亢进和减退的情况常见于(2.5分) A.亚急性甲状腺炎
B.结节性甲状腺肿
C.Graves病
D.甲状腺腺瘤
E.桥本甲状腺炎
E.桥本甲状腺炎
6 4 2015 15. 女性,44岁,健康体检发现甲状腺肿大就诊。查体甲状腺对称性Ⅱ度肿大,表面不平,中等硬度,无触痛,无血管杂音,心率78次/分。拟诊为慢性淋巴细胞性甲状腺炎,有助于确诊的首选检查是(2.5分) A.TSH,FT3,FT4
B.抗甲状腺抗体
C.甲状腺B超
D.甲状腺摄碘131率
E.甲状腺CT
B.抗甲状腺抗体
6 4 2015 16. 关于甲状腺癌,下面说法正确的是(2.5分) A.甲状腺乳头状癌好发于中年女性,恶性程度低,早期可发生血行转移
B.甲状腺髓样癌可分泌PTH等,引起类癌综合征
C.甲状腺滤泡癌早期可发生淋巴结转移,预后较甲状腺乳头状癌差
D.甲状腺乳头状癌是甲状腺癌肿预后最好的病理类型
E.甲状腺滤泡状腺瘤和甲状腺滤泡状癌可通过FNA进行鉴别
D.甲状腺乳头状癌是甲状腺癌肿预后最好的病理类型
6 4 2015 17. 对甲状腺肿瘤性质诊断正确率可达80%以上的检查方法是(2.5分) A.B超
B.CT
C.ECT
D.MRI
E.FNA
E.FNA
6 4 2015 18. 成人甲减最常见的原因是(2.5分) A.自身免疫损伤
B.甲状腺破坏
C.碘缺乏
D.碘过量
E.抗甲状腺药物
A.自身免疫损伤
6 4 2015 19. 与甲状腺功能减退症表现不相符的是(2.5分) A.少汗、嗜睡、畏寒
B.记忆力减退、反应迟钝、腱反射迟钝
C.食欲亢进、体重减轻
D.声音低哑、语言缓慢
E.便秘、月经过多
C.食欲亢进、体重减轻
6 4 2015 20. 鉴别甲状腺性、垂体性和下丘脑性甲状腺功能减退最可靠的检查是(2.5分) A.血清TSH测定
B.TSH刺激试验
C.TRH刺激试验
D.甲状腺自身抗体测定
E.血清TSH、TT3、TT4、FT3、FT4同时测定,综合分析
C.TRH刺激试验
6 4 2015 21. 患者女性,60岁。畏寒、嗜睡、纳差、便秘伴毛发脱落3年。体格检查:浮肿,皮肤粗糙,反应迟钝,心率56次/分,律齐。实验室检查:FT3↓,FT4↓,TSH↑。诊断为原发性甲减。治疗主要采用(2.5分) A.支持治疗
B.对症治疗
C.病因治疗
D.替代治疗
E.滋补治疗
D.替代治疗
6 4 2015 22. 慢性淋巴细胞性甲状腺炎最具诊断意义的指标是(2.5分) A.CRP、ESR
B.TPOAb、TgAb
C.TRAb
D.TSAb
E.TSBAb
B.TPOAb、TgAb
6 4 2015 23. 患者,女性,44岁。3周前感冒伴咽痛,2周前已痊愈。近5天颈前疼痛明显,有低热。查体T 37.8℃,皮肤无汗,甲状腺Ⅱ度肿大,右叶硬,明显触痛拒按,WBC 7.8*109/L。临床诊断可能是(2.5分) A.甲状腺右叶囊肿出血
B.甲状腺癌伴出血
C.慢性淋巴细胞性甲状腺炎
D.急性化脓性甲状腺炎
E.亚急性甲状腺炎
E.亚急性甲状腺炎
6 4 2015 24. 下列关于亚急性甲状腺炎的描述,错误的是(2.5分) A.甲状腺弥漫性肿大、质地韧
B.可合并颈部淋巴结肿大
C.发病以女性多见
D.甲状腺疼痛可从一叶转移至另一叶
E.为自限性疾病,一般不会发展为甲减
A.甲状腺弥漫性肿大、质地韧
6 4 2015 25. 女性,35岁,甲状腺部位疼痛,并出现结节,红细胞沉降率加速,甲状腺摄碘率明显下降,经双氯芬酸钠治疗后,临床症状迅速消失,诊断考虑(2.5分) A.亚急性甲状腺炎
B.桥本甲状腺炎
C.甲状腺癌
D.甲状腺囊肿囊内出血
E.甲状腺腺瘤内出血
A.亚急性甲状腺炎
6 4 2015 26. 糖尿病分型不包括(2.5分) A.2型糖尿病
B.1型糖尿病
C.儿童糖尿病
D.其他特殊类型糖尿病
E.妊娠期糖尿病
C.儿童糖尿病
6 4 2015 27. HbA1c能反映取血前几周的平均血糖水平(2.5分) A.2~4周
B.4~6周
C.6~8周
D.8~12周
E.12~16周
D.8~12周
6 4 2015 28. 糖尿病诊断标准中75g葡萄糖负荷后2小时血糖应(2.5分) A.≥7.8 mmol/L
B.≥7.0 mmol/L
C.≥11.1 mmol/L
D.≥10.0 mmol/L
E.≥12.0 mmol/L
C.≥11.1 mmol/L
6 4 2015 29. 糖耐量减低(IGT)是指OGTT 2小时血糖(2.5分) A.7.0~10.0 mmol/L
B.6.1~7.8 mmol/L
C.6.1~7.0 mmol/L
D.7.0~7.8 mmol/L
E.7.8~11.1 mmol/L
E.7.8~11.1 mmol/L
6 4 2015 30. 口服降糖药不包括(2.5分) A.磺脲类
B.双胍类
C.噻唑烷二酮类
D.a-葡萄糖苷酶抑制剂
E.硫脲类
E.硫脲类
6 4 2015 31. 妊娠期糖尿病诊断标准空腹血糖是(2.5分) A.≥3.9.mmol/L
B.≥5.1 mmol/L
C.≥5.6 mmol/L
D.≥6.1 mmol/L
E.≥7.0 mmol/L
B.≥5.1 mmol/L
6 4 2015 32. 肥胖的2型糖尿病降糖药首选(2.5分) A.磺脲类
B.双胍类
C.噻唑烷二酮类
D.a-葡萄糖苷酶抑制剂
E.胰岛素
B.双胍类
6 4 2015 33. 1型糖尿病有关的自身抗体不包括(2.5分) A.ICA
B.GADA
C.TPOAb
D.IAA
E.IA-2
C.TPOAb
6 4 2015 34. 下列哪项不是糖尿病慢性并发症(2.5分) A.低血糖
B.大血管病变
C.肾脏病变
D.视网膜病变
E.神经病变
A.低血糖
6 4 2015 35. 对于特发性功能性低血糖症,下列几项中错误的是(2.5分) A.病程较长,可呈进行性发展
B.是餐后低血糖症的常见类型
C.可自行缓解
D.血浆胰岛素水平、胰岛素释放指数均在正常范围
E.多见于女性
A.病程较长,可呈进行性发展
6 4 2015 36. 器质性低血糖最常见的原因是(2.5分) A.胰岛素及口服降糖药致药源性低血糖
B.胰岛素瘤
C.肝源性低血糖
D.特发性功能性低血糖
E.长期饥饿、慢性腹泻
B.胰岛素瘤
6 4 2015 37. 纠正低血糖后,脑功能恢复的顺序是(2.5分) A.延脑-下丘脑-皮质下中枢-自主神经中枢-大脑皮质
B.延脑-自主神经中枢-下丘脑-皮质下中枢-大脑皮质
C.大脑皮质-下丘脑-皮质下中枢-自主神经中枢-延脑
D.大脑皮质-皮质下中枢-延脑-下丘脑-自主神经中枢
E.下丘脑-皮质下中枢-延脑-自主神经中枢-大脑皮质
B.延脑-自主神经中枢-下丘脑-皮质下中枢-大脑皮质
6 4 2015 38. Whipple三联征,包括(2.5分) A.出汗、心悸、软弱无力
B.血糖低于3.9mmol/L
C.舌下含服硝酸甘油有效
D.嗜睡伴呕吐
E.给予呋塞米后症状缓解
A.出汗、心悸、软弱无力
6 4 2015 39. 导致低血糖症的发生(2.5分) A.抑制肝糖原分解
B.胰岛素分泌和作用过强
C.胰岛素分泌减少
D.交感神经兴奋
E.胸腺发育不全
B.胰岛素分泌和作用过强
6 4 2015 40. 难治性痛风以下哪条提法不正确(2.5分) A.单用或联用常规降尿酸药物足量、足疗程,但血尿酸仍≥360 μmol/L;
B.接受规范化治疗,痛风仍发作≥2次/年;
C.存在多发性和(或)进展性痛风石(2C)
D.可将聚乙二醇重组尿酸酶制剂用于难治性痛风的降尿酸治疗
E.必须手术治疗
E.必须手术治疗
6 5 2006 1. 下列哪项检查不是生长激素缺乏症的确诊试验 (2.0分) A.胰岛素刺激试验
B.左旋多巴试验
C.可乐定刺激试验
D.精氨酸刺激试验
E.睡眠试验
E.睡眠试验
6 5 2006 2. 女,42岁,乏力怕冷便秘伴声音嘶哑1年,体重增加8kg,经检查诊断为甲状腺功能减退症,拟左甲状腺素替代治疗。最适宜的起始剂量为(2.0分) A.125μg
B.75μg
C.100μg
D.50μg
E.25μg
E.25μg
6 5 2006 3. 预防甲状腺功能减退症粘液性水肿昏迷的关键是(2.0分) A.坚持甲状腺素替代治疗
B.增强免疫力
C.禁用镇静、安眠药
D.水摄入量不宜过多
E.避免过度劳累
A.坚持甲状腺素替代治疗
6 5 2006 4. 患儿,20天,出生体重4公斤,嗜睡,吃奶少,便秘,查体:皮肤黄染,前囟4*3厘米,心肺听诊无殊,腹略胀,脐疝,肝、脾肋下未及,神经系统(-)。该患儿可能的疾病是: (2.0分) A.新生儿败血症
B.粘多糖1型
C.先天性甲状腺功能低下症
D.新生儿溶血症
E.先天性巨结肠
C.先天性甲状腺功能低下症
6 5 2006 5. 对于生长激素缺乏症患儿,需用生长激素治疗,其剂量是: (2.0分) A.0.05U/kg/d
B.0.2U/kg/d
C.0.15U/kg/d
D.0.1U/kg/d
E.0.25U/kg/d
D.0.1U/kg/d
6 5 2006 6. 下列哪项不是先天性甲状腺功能减低症的临床表现: (2.0分) A.通关手
B.表情呆滞
C.便秘
D.生长落后
E.心音低钝
A.通关手
6 5 2006 7. 先天性甲状腺功能减低症的治疗,正确的是 (2.0分) A.应服药5年以上
B.持续大剂量冲击治疗
C.明确诊断后即应开始治疗
D.药物应大剂量开始
E.随访5年
C.明确诊断后即应开始治疗
6 5 2006 8. 确诊先天性甲状腺功能减低症主要依靠下列哪项检查(2.0分) A.血清蛋白结合碘测定
B.血清T3、 T4 TSH测定
C.骨骼X线片检查
D.染色体检查
E.特殊面容
B.血清T3、 T4 TSH测定
6 5 2006 9. 下面哪一项不是系统性红斑狼疮的诊断标准:(2.0分) A.多浆膜炎
B.关节痛
C.蝶形红斑
D.光敏感
E.SM抗体
B.关节痛
6 5 2006 10. 人体糖皮质激素如皮质醇、脱氧皮质酮等主要在 合成。(2.0分) A.肾上腺髓质
B.肾上腺皮质球状带
C.肾上腺皮质网状带
D.肾上腺皮质束状带
E.肾小球
D.肾上腺皮质束状带
6 5 2006 11. 17a-羟化酶缺乏临床表现不是的:(2.0分) A.高钠血症
B.同性性早熟
C.高血压
D.高孕酮水平
E.原发闭经
B.同性性早熟
6 5 2006 12. 女性性发育一般首先表现为:(2.0分) A.座疮
B.阴毛出现
C.乳房发育
D.声音变尖
E.月经
C.乳房发育
6 5 2006 13. 先天性甲状腺功能减低症的治疗,下列哪项是错误的(2.0分) A.治疗开始时间越早越好
B.间隔1~2周加量一次至症状改善
C.初始剂量以最小剂量开始
D.症状改善后停药
E.维持量个体差异比较大
D.症状改善后停药
6 5 2006 14. 生长激素替代疗法,下列哪项不对 (2.0分) A.治疗应持续至骨骺愈合为止
B.口服给药
C.治疗第一年效果最佳
D.开始治疗时年龄越小,效果越好
E.如同时伴有甲状腺素缺乏治疗时应补充甲状腺素
B.口服给药
6 5 2006 15. 下列卵巢癌MRI描述中,不正确的是 (2.0分) A.肿瘤的边缘不规则
B.囊壁厚簿不均
C.肿瘤内实性性成分多.
D.盆腔内软组织密度肿块。
E.增强扫描肿瘤无明显强化
E.增强扫描肿瘤无明显强化
6 5 2006 16. 甲减合并妊娠治疗时,TSH应控制在:(2.0分) A. <10miu/L
B.>2.5miu/L
C.<5miu/L
D.<2.5miu/L
E. 控制在正常范围上限
D.<2.5miu/L
6 5 2006 17. 肾上腺CT或MRI扫描图像上的形态不包括(2.0分) A.倒V形
B.倒Y形
C.线条形
D.类圆形
E.三角形
D.类圆形
6 5 2006 18. 下列疾病中,不会引起漏出性胸水的是(2.0分) A.肝硬化
B.低蛋白血症
C.心力衰竭
D.肾病综合征
E.系统性红斑狼疮
E.系统性红斑狼疮
6 5 2006 19. 川崎病最早出现的症状是:(2.0分) A.颈淋巴结肿大
B.指、趾端脱皮
C.持续高热
D.多形性皮疹
E.杨梅舌
C.持续高热
6 5 2006 20. 下列哪项对川崎病诊断具有特征意义:(2.0分) A.手足硬肿,指、趾端膜状脱皮
B.持续高热5天以上
C.颈淋巴结肿大
D.多形性皮疹
E.双球结膜充血,口唇皲裂,杨梅舌
A.手足硬肿,指、趾端膜状脱皮
6 5 2006 21. 决定过敏性紫癜预后的临床表现为:(2.0分) A.皮肤紫癜
B.便血
C.鼻出血
D.关节痛
E.肾脏病变
E.肾脏病变
6 5 2006 22. 过敏性紫癜特征性的表现:(2.0分) A.关节肿痛
B.腹痛
C.皮肤紫癜
D.蛋白尿、血尿
E.消化道出血
C.皮肤紫癜
6 5 2006 23. 下列哪项不是SLE患者的血液系统改变? (2.0分) A.自身免疫性溶血性贫血
B.正色素细胞性贫血
C.血小板减少
D.白细胞减少
E.类白血病样改变
E.类白血病样改变
6 5 2006 24. 下列哪项不是中枢性性早熟治疗的目的: (2.0分) A.改善患儿的最终身高
B.控制或减缓第二性征发育,延缓性成熟过程
C.抑制性激素引起的骨成熟,防止骨骺早闭
D.同步心理和行为指导,使生理、心理发展同步
E.控制第二性征发育以适合患者或监护者的需求
E.控制第二性征发育以适合患者或监护者的需求
6 5 2006 25. 典型先天性肾上腺皮质增生症的发病率约为: (2.0分) A.1/100 000
B.1/5000
C.1/1000
D.1/10 000
E.1/20 000
D.1/10 000
6 5 2006 26. 子宫内膜癌侵犯深肌层的主要MRI征象是(2.0分) A.结合带变薄
B.子宫内膜出血
C.子宫内膜轮廓不规整
D.结合带低信号中断
E.子宫内膜信号异常
D.结合带低信号中断
6 5 2006 27. 下列那一项不是外周性性早熟的病因 : (2.0分) A.摄入含外源性性激素的食物、补品等
B.卵巢畸胎瘤
C.原发性甲状腺功能减低症
D.肾上腺皮质增生症
E.误服避孕药等
C.原发性甲状腺功能减低症
6 5 2006 28. 风湿热关节炎的特点:(2.0分) A.经治疗后可完全痊愈,不留畸形
B.关节畸形
C.累及小关节为主
D.最终转为慢性关节炎
E.晨僵
A.经治疗后可完全痊愈,不留畸形
6 5 2006 29. 李娜,女,4岁,发现"左乳房增大2月"就诊。有大量服用中药以及补品史以及误服避孕药史。B超提示:左侧乳腺组织增生,子宫卵巢幼稚型,体检发现左乳B2期,乳晕颜色较深。该女童最可能的诊断是:(2.0分) A.单纯乳房早发育
B.周围性性早熟
C.中枢性性早熟
D.部分性性早熟
E. "小青春期"
B.周围性性早熟
6 5 2006 30. 幼年类风湿关节炎主要特征:(2.0分) A.慢性关节炎
B.不规则发热
C.皮疹
D.胸膜炎
E.淋巴结肿大
A.慢性关节炎
6 5 2006 31. 垂体微腺瘤的MR征象不包括:(2.0分) A.动态扫描可以显示部分难以发现的微腺瘤低信号
B.视交叉和视神经的受压移位
C.动态扫描后,肿瘤更加明显低信号
D.微腺瘤T1W呈低信号,T2W呈高信号
E.T1WI微腺瘤伴出血时为高信号
B.视交叉和视神经的受压移位
6 5 2006 32. 患儿 男 ,3个月, 孕43周娩出。 出生体重4200g。 生后少哭,少动,哭声低哑,喂奶量少,便秘,黄疸持续1个月消退,现不会抬头。查体:T35℃,嗜睡,反应迟钝,眼距稍宽,鼻梁宽平,舌大,皮肤干,毛发稀少,无中毒貌,腹胀,脐疝。心电图:各导联低电压,T波低平。该疾病最可能的诊断: (2.0分) A.佝偻病
B.败血症
C.先天性巨结肠
D.先天性甲状腺功能低下
E.先天愚型
D.先天性甲状腺功能低下
6 5 2006 33. 子宫内膜异位症常见于(2.0分) A.输卵管
B.腹膜
C.子宫直肠窝
D.双侧卵巢
E.膀胱子宫隐窝
D.双侧卵巢
6 5 2006 34. 原发性甲状腺功能减退症替代治疗的原(2.0分) A.由大剂量开始,逐渐递减到合适剂量
B.从小剂量开始,逐渐递增到合适剂量
C.间断用药有利于预防心绞痛
D.甲减越严重,起始剂量应越大
E.待甲状腺功能正常后可停用
B.从小剂量开始,逐渐递增到合适剂量
6 5 2006 35. 早期前列腺癌影像表现为:(2.0分) A.CT平扫前列腺局部结节样凸出
B.增强CT扫描前列腺周边结节样低增强
C.MR T2WI 高信号的外周带内低信号结节
D.超声前列腺横经大于5cm,内部回声不均
E.MR 脂肪抑制T1W序列扫描前列腺中心区高信号结节
C.MR T2WI 高信号的外周带内低信号结节
6 5 2006 36. 宫腔形态改变,子宫信号正常见于(2.0分) A.子宫先天畸形
B.子宫内膜癌
C.子宫肌瘤
D.畸胎瘤
E.卵巢囊腺瘤
A.子宫先天畸形
6 5 2006 37. 关于SLE患者的妊娠问题,下面哪项不正确? (2.0分) A.应病情稳定,心肾功能正常,方可妊娠
B.易发生流产、早产
C.可出现新生儿狼疮
D.妊娠时可使SLE病情恶化
E.妊娠头3个月内可应用免疫抑制剂
E.妊娠头3个月内可应用免疫抑制剂
6 5 2006 38. 目前显示垂体病变最好的影像学检查方法是(2.0分) A.头颅平片
B.MRI
C.CT
D. 核素显像
E.超声
B.MRI
6 5 2006 39. 下面哪个自身抗体特异性高,但与SLE活动性无关? (2.0分) A.抗SSA抗体
B.抗Sm抗体
C.抗双链DNA抗体
D.抗核抗体
E.抗SSB抗体
B.抗Sm抗体
6 5 2006 40. 怀疑前列腺癌病人CT检查可以(2.0分) A.发现早期病变
B.与其他前列腺病变鉴别
C.发现骨转移
D.为早期病变分期
E.除外前列腺被膜侵犯
C.发现骨转移
6 5 2006 41. 下面哪项是SLE的表现? (2.0分) A.颊部蝶形皮疹及蛋白尿
B.膝关节受累
C.腕、掌指、近指关节受累
D.第一趾较剧烈疼痛
E.大量龋齿提示
A.颊部蝶形皮疹及蛋白尿
6 5 2006 42. 关于SLE患者的关节损害,下面哪项是错误的? (2.0分) A.关节软骨破坏,关节畸形
B.呈多关节对称性损害
C.近端指间关节多受累
D.关节肿痛
E.大关节很少受累
A.关节软骨破坏,关节畸形
6 5 2006 43. 系统性红斑狼疮的狼疮,疮是指疮状免疫荧光出现于 (2.0分) A.肾小球壁膜
B.血管壁
C.关节囊内
D.真皮与表皮交界处
E.肺部血管
B.血管壁
6 5 2006 44. 下面哪个自身抗体是SLE的标准筛选试验,但特异性小? (2.0分) A.抗Sm抗体
B.抗核抗体
C.抗双链DNA抗体
D.抗SSA抗体
E.抗SSB抗体
B.抗核抗体
6 5 2006 45. 对子宫内膜癌的描述,下列哪项是错误的 (2.0分) A.仅次于宫颈癌
B.多为老年妇女
C.绝经后出血
D.限于子宫内膜T1WI或T2WI像可显示正常;
E.肿瘤最初位于子宫肌,后向内侵犯子宫内膜
E.肿瘤最初位于子宫肌,后向内侵犯子宫内膜
6 5 2006 46. 下面哪项不是SLE患者使用糖皮质激素的适应征? (2.0分) A.伴AIHA和ITP
B.发热
C.急性期或暴发性狼疮
D.有心,肾,脑,肺等重要脏器受累
E.妊娠期;
B.发热
6 5 2006 47. 下面哪项不是SLE活动的主要指征? (2.0分) A.发热
B.疲乏
C.泡沫尿
D.关节痛
E.腹痛
E.腹痛
6 5 2006 48. 引起先天性甲状腺功能减低症最常见的原因(2.0分) A.甲状腺靶器官反应低下
B.碘缺乏
C.甲状腺不发育或发育不全
D.酶缺乏
E.促甲状腺激素缺陷
C.甲状腺不发育或发育不全
6 5 2006 49. 哪一项不是应用激素的指征:(2.0分) A.系统性红斑狼疮
B.川崎病伴冠状动脉瘤
C.风湿性心脏炎
D.类风湿关节炎伴心包炎
E.过敏性紫癜伴有消化道症状
B.川崎病伴冠状动脉瘤
6 5 2006 50. 下列那一项不是21羟化酶缺乏症女童的临床表现: (2.0分) A.低钠、高钾
B.阴蒂肥大
C.性早熟
D.多毛症
E.高血压
C.性早熟
6 5 2007 1. 对于生长激素缺乏症患儿,需用生长激素治疗,其剂量是: (2.0分) A.0.05U/kg/d
B.0.1U/kg/d
C.0.15U/kg/d
D.0.2U/kg/d
E.0.25U/kg/d
B.0.1U/kg/d
6 5 2007 2. 患儿,20天,出生体重4公斤,嗜睡,吃奶少,便秘,查体:皮肤黄染,前囟4*3厘米,心肺听诊无殊,腹略胀,脐疝,肝、脾肋下未及,神经系统(-)。该患儿可能的疾病是: (2.0分) A.新生儿败血症
B.新生儿溶血症
C.先天性甲状腺功能低下症
D.粘多糖1型
E.先天性巨结肠
C.先天性甲状腺功能低下症
6 5 2007 3. 患儿,男,12岁,身高130厘米,学习成绩优秀,父亲身高175厘米,母亲身高160厘米,查体:反应灵活,四肢匀称,心肺听诊无殊,腹平软,肝、脾肋下未及,神经系统(-)。该患儿可能的疾病是: (2.0分) A.家族性矮小
B.21-三体综合症
C.生长激素缺乏症
D.甲状腺功能减低症
E.先天性软骨发育不良
C.生长激素缺乏症
6 5 2007 4. 引起中枢性甲状腺功能低下的病因是: (2.0分) A.甲状腺不发育或发育不全
B.甲状腺激素合成 途径障碍或母体存在抗甲状腺抗体
C.甲状腺激素合成途径障碍
D.促甲状腺素(TSH)缺乏
E.甲状腺或靶器官反应低下
D.促甲状腺素(TSH)缺乏
6 5 2007 5. 确诊先天性甲状腺功能减低症主要依靠下列哪项检查 (2.0分) A.骨骼X线片检查
B.血清T3、 T4 TSH测定
C.血清蛋白结合碘测定
D.染色体检查
E.特殊面容
B.血清T3、 T4 TSH测定
6 5 2007 6. 男,10岁,出生体重3200g,身材比例匀称,智力正常,身高110cm,生长速度3cm/年。最可能的诊断是(2.0分) A.性早熟
B.家族性身矮
C.糖尿病
D.生长激素缺乏
E.甲状腺功能低下
D.生长激素缺乏
6 5 2007 7. 下列哪项检查不是生长激素缺乏症的确诊试验 (2.0分) A.可乐定刺激试验
B.左旋多巴试验
C.胰岛素刺激试验
D.精氨酸刺激试验
E.睡眠试验
E.睡眠试验
6 5 2007 8. 下列哪项不是先天性甲状腺功能减低症的临床表现: (2.0分) A.表情呆滞
B.生长落后
C.通关手
D.便秘
E.心音低钝
C.通关手
6 5 2007 9. 下列哪一项不是风湿热的主要表现:(2.0分) A.血沉增快
B.心脏炎
C.皮下结节
D.环形红斑
E.舞蹈病
A.血沉增快
6 5 2007 10. 川崎病时皮肤粘膜表现中最具有特征性的改变是:(2.0分) A.皮疹
B.指(趾)端变化
C.眼球结膜充血
D.杨梅舌
E.唇红干燥、皲裂
B.指(趾)端变化
6 5 2007 11. 治疗川崎病首选的药物是:(2.0分) A.丙种球蛋白
B.阿司匹林
C.青霉素
D.肾上腺皮质激素
E.环磷酰胺
B.阿司匹林
6 5 2007 12. 哪一项不是应用激素的指征:(2.0分) A.类风湿关节炎伴心包炎
B.川崎病伴冠状动脉瘤
C.风湿性心脏炎
D.系统性红斑狼疮
E.过敏性紫癜伴有消化道症状
B.川崎病伴冠状动脉瘤
6 5 2007 13. 过敏性紫癜特征性的表现:(2.0分) A.腹痛
B.关节肿痛
C.皮肤紫癜
D.蛋白尿、血尿
E.消化道出血
C.皮肤紫癜
6 5 2007 14. 风湿热关节炎的特点:(2.0分) A.最终转为慢性关节炎
B.关节畸形
C.累及小关节为主
D.经治疗后可完全痊愈,不留畸形
E.晨僵
D.经治疗后可完全痊愈,不留畸形
6 5 2007 15. 幼年类风湿关节炎主要特征:(2.0分) A.不规则发热
B.慢性关节炎
C.皮疹
D.胸膜炎
E.淋巴结肿大
B.慢性关节炎
6 5 2007 16. 川崎病最早出现的症状是:(2.0分) A.持续高热
B.颈淋巴结肿大
C.多形性皮疹
D.指、趾端脱皮
E.杨梅舌
A.持续高热
6 5 2007 17. 下列哪项对川崎病诊断具有特征意义:(2.0分) A.持续高热5天以上
B.颈淋巴结肿大
C.多形性皮疹
D.手足硬肿,指、趾端膜状脱皮
E.双球结膜充血,口唇皲裂,杨梅舌
D.手足硬肿,指、趾端膜状脱皮
6 5 2007 18. 决定过敏性紫癜预后的临床表现为:(2.0分) A.皮肤紫癜
B.便血
C.关节痛
D.鼻出血
E.肾脏病变
E.肾脏病变
6 5 2007 19. 川崎病合并冠状动脉瘤服用阿司匹林的时间为:(2.0分) A.血清水杨酸浓度达到质量水平
B.血沉正常
C.血小板降至正常
D.长期服用直至动脉瘤消退
E.热退后
D.长期服用直至动脉瘤消退
6 5 2007 20. 过敏性紫癜的皮疹特点有:(2.0分) A.全身散在针尖大小紫癜,压之不退色
B.全身出现大块瘀斑,压不退色
C.下肢、臀部出现分布对称红疹,压不退色
D.全身出现风团样皮疹,压之退色
E.大关节出现大块瘀斑,伴功能障碍,压不退色
C.下肢、臀部出现分布对称红疹,压不退色
6 5 2007 21. 确诊风湿热的次要表现不包括:(2.0分) A.发热
B.关节疼痛
C.皮下结节
D.血沉加快
E.有风湿热既往史
C.皮下结节
6 5 2007 22. 川崎病的治疗中可使用丙种球蛋白,宜于几天内使用:(2.0分) A.10天
B.15天
C.4天
D.5天
E.7天
A.10天
6 5 2007 23. 子宫内膜异位症常见于 (2.0分) A.腹膜
B.双侧卵巢
C.输卵管
D.子宫直肠窝
E.膀胱子宫隐窝
B.双侧卵巢
6 5 2007 24. 女性生殖器结核最常见的 (2.0分) A.输卵管结核
B.子宫内膜结核
C.子宫颈结核
D.卵巢结核
E.盆腔结核
A.输卵管结核
6 5 2007 25. 子宫内膜癌侵犯深肌层的主要MRI征象是(2.0分) A.结合带低信号中断
B.结合带变薄
C.子宫内膜出血
D.子宫内膜轮廓不规整
E.子宫内膜信号异常
A.结合带低信号中断
6 5 2007 26. 患者女性,35岁,CT、扫描子宫增大呈分叶状,表面光滑,子宫肌壁内实性略低密度影,有钙化,官腔受压移位,考虑为(2.0分) A.葡萄胎
B.子宫内膜癌
C.子宫肌瘤
D.子宫肌腺瘤
E.妊娠
C.子宫肌瘤
6 5 2007 27. 对子宫内膜癌的描述,下列哪项是错误的 (2.0分) A.绝经后出血
B.多为老年妇女
C.仅次于宫颈癌
D.限于子宫内膜T1WI或T2WI像可显示正常;
E.肿瘤最初位于子宫肌,后向内侵犯子宫内膜
E.肿瘤最初位于子宫肌,后向内侵犯子宫内膜
6 5 2007 28. 关于子宫解剖的MRI表现,描述正确的是 (2.0分) A.子宫内膜正常厚度大于5mm
B.子宫内膜T2加权像呈低信号
C.结合带T2加权像呈低信号
D.子宫肌层T1加权像呈高信号
E.子宫峡部位于宫颈和阴道交界处
C.结合带T2加权像呈低信号
6 5 2007 29. 下列卵巢癌MRI描述中,不正确的是 (2.0分) A.盆腔内软组织密度肿块
B.囊壁厚簿不均
C.肿瘤内实性性成分多
D.肿瘤的边缘不规则
E.增强扫描肿瘤无明显强化
E.增强扫描肿瘤无明显强化
6 5 2007 30. 怀疑前列腺癌病人CT检查可以(2.0分) A.发现早期病变
B.与其他前列腺病变鉴别
C.为早期病变分期
D.发现骨转移
E.除外前列腺被膜侵犯
D.发现骨转移
6 5 2007 31. 前列腺良性增生的影像表现特点是:(2.0分) A.前列腺局部增大
B.CT平扫前列腺密度不均
C.MRI T2WI前列腺中央区增大,周边区变薄
D.膀胱造影示膀胱下壁乳头状充盈缺损
E.CT增强扫描前列腺中心区强化明显
C.MRI T2WI前列腺中央区增大,周边区变薄
6 5 2007 32. 正常前列腺的中央带与外周带在T2WI的信号特点为(2.0分) A.两部分均为高信号
B.两部分均为低信号
C.中央带为低信号,外周带为高信号
D.外周带为低信号,中央带为高信号
E.中央带为等信号,外周带为低信号
C.中央带为低信号,外周带为高信号
6 5 2007 33. 目前显示垂体病变最好的影像学检查方法是(2.0分) A.头颅平片
B.CT
C.MRI
D.核素显像
E.超声
C.MRI
6 5 2007 34. 垂体瘤不包括: (2.0分) A.生长激素(GH)腺瘤
B.生殖细胞瘤
C.泌乳素(PRL)腺瘤
D.促肾上腺皮质激素(ACTH)腺瘤
E.嫌色细胞腺瘤
B.生殖细胞瘤
6 5 2007 35. 关于垂体腺瘤的说法,以下哪种是错误的是:(2.0分) A.是从垂体前叶发生的良性肿瘤
B.当腺瘤的直径小于1厘米时,被称为微腺瘤
C.功能性腺瘤由于有内分泌症状,通常发现较早
D.大腺瘤是无功能性腺瘤
E.微腺瘤多为女性
D.大腺瘤是无功能性腺瘤
6 5 2007 36. 垂体瘤的CT表现不正确的是 (2.0分) A.蝶鞍扩大
B.鞍底下陷
C.瘤周水肿
D.肿瘤有强化
E.垂体柄移位
C.瘤周水肿
6 5 2007 37. 肾上腺髓质最常见的功能性肿瘤是:(2.0分) A.嗜铬细胞瘤
B.髓质脂肪瘤
C.皮质腺瘤
D.神经母细胞瘤
E.肾上腺瘤肾上腺癌
A.嗜铬细胞瘤
6 5 2007 38. 患者男,33岁,以阵发性高血压入院。生化检查提示尿中儿茶酚氨明显增高应选择的最佳影像学检查为:(2.0分) A.超声
B.血管造影
C.CT
D.腹部平片
E.PCT
C.CT
6 5 2007 39. 肾上腺皮质腺瘤的临床表现哪项不正确(2.0分) A.成人、女性多发
B.柯兴(Cushing)氏综合征
C.满月脸、多血质、向心性肥胖
D.停经泌乳
E.血皮质醇增高
D.停经泌乳
6 5 2007 40. 正常肾上腺的MRI信号与肝实质信号强度相比(2.0分) A.T1及T2加权像均为稍高信号
B.T1加权像低信号、T2加权像高信号
C.T1加权像高信号、T2加权像低信号
D.T1及T2加权像均为高信号
E.T1WI和T2WI上信号类似肝脏
E.T1WI和T2WI上信号类似肝脏
6 5 2007 41. 典型嗜铬细胞瘤在(2.0分) A.T1WI上信号强度类似肌肉,T2WI上呈明显高信号
B.1WI呈高信号,T2WI呈低信号
C.T1WI呈低信号,T2WI呈中等信号
D.T1WI及T2WI均呈高信号
E.T1WI及T2WI均呈低信号
A.T1WI上信号强度类似肌肉,T2WI上呈明显高信号
6 5 2007 42. Graves病的叙述不正确的是(2.0分) A.常见的自身免疫性疾病
B.以甲状腺肿大和突眼为特征性表现
C.常有甲状腺功能降低
D.CT增强检查不宜采用
E.MRI检查可显示眼外肌肌腹肥大
C.常有甲状腺功能降低
6 5 2007 43. 下面哪个自身抗体是结缔组织病的标准筛选试验? (2.0分) A.抗核抗体
B.抗Sm抗体
C.抗双链DNA抗体
D.抗SSA抗体
E.抗SSB抗体
A.抗核抗体
6 5 2007 44. SLE的标记性抗体是(2.0分) A.抗组蛋白抗体
B.抗Sm抗体
C.抗SSˉB抗体
D.抗Sclˉ70抗体
E.抗Joˉ1抗体
B.抗Sm抗体
6 5 2007 45. 关于SLE患者的关节损害,下面哪项是错误的?(2.0分) A.关节肿痛
B.呈多关节对称性损害
C.近端指间关节多受累
D.关节软骨破坏,关节畸形
E.大关节很少受累
D.关节软骨破坏,关节畸形
6 5 2007 46. 下列哪项不是SLE患者的血液系统改变?(2.0分) A.白细胞减少
B.血小板减少
C.自身免疫性溶血性贫血
D.正色素细胞性贫血
E.类白血病样改变
E.类白血病样改变
6 5 2007 47. 下面哪一项不是系统性红斑狼疮的诊断标准:(2.0分) A.光敏感
B.关节痛
C.蝶形红斑
D.多浆膜炎
E.SM抗体
B.关节痛
6 5 2007 48. 下列那一项不是外周性性早熟的病因 (2.0分) A.肾上腺皮质增生症
B.卵巢畸胎瘤
C.原发性甲状腺功能减低症
D.摄入含外源性性激素的食物、补品等
E.误服避孕药等
C.原发性甲状腺功能减低症
6 5 2007 49. 男性性发育一般首先表现为:(2.0分) A.胡须生长
B.睾丸增大,容积超过4ml;阴茎增粗
C.声音变低沉、喉结出现
D.遗精
E.阴毛腋毛出现
B.睾丸增大,容积超过4ml;阴茎增粗
6 5 2007 50. 李娜,女,4岁,发现"左乳房增大2月"就诊。有大量服用中药以及补品史以及误服避孕药史。B超提示:左侧乳腺组织增生,子宫卵巢幼稚型,体检发现左乳B2期,乳晕颜色较深。该女童最可能的诊断是:(2.0分) A.中枢性性早熟
B.部分性性早熟
C.周围性性早熟
D.单纯乳房早发育
E. "小青春期"
C.周围性性早熟
6 5 2008 1. 下列那一项不是外周性性早熟的病因 : (2.0分) A.肾上腺皮质增生症
B.卵巢畸胎瘤
C.原发性甲状腺功能减低症
D.摄入含外源性性激素的食物、补品等
E.误服避孕药等
C.原发性甲状腺功能减低症
6 5 2008 2. 男性性发育一般首先表现为:(2.0分) A.胡须生长
B.睾丸增大,容积超过4ml;阴茎增粗
C.声音变低沉、喉结出现
D.遗精
E.阴毛腋毛出现
B.睾丸增大,容积超过4ml;阴茎增粗
6 5 2008 3. 患儿,男,5岁3月,发现阴茎增粗3个月入院;查体:身高118.5cm,心肺腹无殊,阴茎增粗,长约5cm,睾丸容积2ml,未见阴毛;E2 77.9Pg/ml, T861.0ug/ml, HCG 29.9mIU/ml;血ACTH、皮质醇、 17a-类固醇、以及基础和继发后的促性腺激素均在正常范围内。脑脊液HCG 90.6mIU/ml 左手骨片:8/10枚骨化中心(骨龄5.5岁);头颅MRI检查:松果体占位病变,生殖细胞瘤可能。该患者最可能的诊断是:(2.0分) A.中枢性性早熟;松果体生殖细胞瘤
B.部分性性早熟;松果体生殖细胞瘤
C.周围性性早熟;松果体生殖细胞瘤
D.松果体生殖细胞瘤
E. "小青春期", 松果体生殖细胞瘤
C.周围性性早熟;松果体生殖细胞瘤
6 5 2008 4. 通常情况下,女性青春发育的顺序是:(2.0分) A.阴毛发育→乳房发育→外生殖器发育、腋毛发育→月经来潮
B.乳房发育→阴毛、外生殖器发育→月经来潮→腋毛发育
C.阴毛发育→外生殖器发育、腋毛发育→乳房发育→月经来潮
D.乳房发育→月经来潮→阴毛、外生殖器发育→腋毛发育
E.乳房发育→阴毛、外生殖器发育→腋毛发育→月经来潮
B.乳房发育→阴毛、外生殖器发育→月经来潮→腋毛发育
6 5 2008 5. 通常情况下,男性青春发育的顺序是:(2.0分) A.胡须生长、阴毛腋毛出现→声音变低沉-→睾丸增大,容积超过4ml;阴茎增粗→遗精
B.睾丸增大,容积超过4ml;阴茎增粗→声音变低沉→ 遗精→胡须生长、阴毛腋毛出现
C.睾丸增大,容积超过4ml;阴茎增粗→胡须生长、阴毛腋毛出现→声音变低沉→ 遗精
D.胡须生长、阴毛腋毛出现→睾丸增大,容积超过4ml;阴茎增粗→声音变低沉→ 遗精
E.胡须生长、阴毛腋毛出现→睾丸增大,容积超过4ml;阴茎增粗→遗精→声音变低沉
C.睾丸增大,容积超过4ml;阴茎增粗→胡须生长、阴毛腋毛出现→声音变低沉→ 遗精
6 5 2008 6. 下列哪项不是中枢性性早熟治疗的目的: (2.0分) A.控制或减缓第二性征发育,
B.延缓性成熟过程
C.抑制性激素引起的骨成熟,防止骨骺早闭
D.帮助儿童达到理想的最终身高
E.同步心理和行为指导,使生理、心理发展同步
D.帮助儿童达到理想的最终身高
6 5 2008 7. 典型先天性肾上腺皮质增生症的发病率约为: (2.0分) A.1/10 000
B.1/5000
C.1/1000
D.1/100 000
E.1/20 000
A.1/10 000
6 5 2008 8. 人体糖皮质激素如皮质醇、脱氧皮质酮等主要在 合成。(2.0分) A.肾上腺皮质球状带
B.肾上腺皮质网状带
C.肾上腺皮质束状带
D.肾上腺髓质
E.肾小球
C.肾上腺皮质束状带
6 5 2008 9. 下列那一项不是21羟化酶缺乏症女童的临床表现: (2.0分) A.阴蒂肥大
B.低钠、高钾
C.性早熟
D.多毛症
E. 高血压
C.性早熟
6 5 2008 10. 男婴5个月,因为"呕吐、嗜睡,精神萎靡1天"就诊。生后母乳喂养,但出生后至今体重仅增加600克,查体:重度脱水貌,血电解质示:Na+ 120 mmol/L, K+ 7.0 mmol/L, Cl+ 90 mmol/L. 考虑肾上腺皮质激素缺乏的可能性比较大,请问可能性最大的疾病是: (2.0分) A.Addison 病
B.18 -羟化酶缺乏症
C.11-羟化酶缺乏症
D.21-羟化酶缺乏症
E.3β-羟化酶缺乏症
D.21-羟化酶缺乏症
6 5 2008 11. 患儿,女,1月,因"流涕、呕吐2天"入院。其父母体健,有1兄,新生儿期呕吐后夭折(具体原因不详)。入院查体: T37.3℃,P180bpm,R40bpm,BP50/30mmHg,精神软,皮肤弹性差,前囟凹,口唇干,口唇、乳晕偏黑,心肺听诊无殊,腹软,肝脾不大,阴蒂偏大。血气电解质:PH7.45,PO268mmHg,PCO235mmHg,K+7.0mmol/L,Na+115mmol/L,Cl-85mmol/L,ABE:-7.8mmol/L。此患者最可能的诊断是:(2.0分) A.21羟化酶缺乏症(失盐型)
B.21羟化酶缺乏症(单纯性)
C.Addison 病
D.感染后继发肾上腺危象
E.感染性休克
B.21羟化酶缺乏症(单纯性)
6 5 2008 12. 以下哪项不是自身免疫性疾病的特点? (2.0分) A.体内存在高浓度的自身抗体
B.自身抗体的浓度与疾病严重程度相关
C.常有多系统和多器官损害
D.免疫球蛋白升高,补体降低
E.免疫抑制剂治疗有效
D.免疫球蛋白升高,补体降低
6 5 2008 13. 下面哪个自身抗体是结缔组织病的标准筛选试验? (2.0分) A.抗核抗体
B.抗Sm抗体
C.抗双链DNA抗体
D.抗SSA抗体
E.抗SSB抗体
A.抗核抗体
6 5 2008 14. 与SLE活动性相关的自身抗体特异性高? (2.0分) A.抗核抗体
B.抗Sm抗体
C.抗双链DNA抗体
D.抗SSA抗体
E.抗SSB抗体
C.抗双链DNA抗体
6 5 2008 15. 系统性红斑狼疮口腔溃疡的特点是(2.0分) A.如无合并感染,多无疼痛
B.伴有明显疼痛
C.多见于舌尖和舌体
D.溃疡的发作与疾病的活动多数无关
E.溃疡表面覆盖有黄色坏死组织
A.如无合并感染,多无疼痛
6 5 2008 16. 女性病人,28岁。双手掌指关节、近端指间关节疼痛伴月经量增多2年。曾在外院查RF(+),拟类风湿关节炎,治疗效果不佳。查体:面色苍白,浮肿,浅表淋巴结不肿大,心肺(-),肝脾(-)。双手小关节活动无障碍,无畸形,指端可见红斑。查血 WBC3.2×109/L, Hb6.5g/L, PLT54×109/L,网织红细胞1%。尿常规RBC5~8/HP,Pro(++),双手X片未见异常。最可能的诊断是(2.0分) A.风湿性关节炎
B.类风湿关节炎
C.系统性红斑狼疮
D.干燥综合征
E.强直性脊柱炎
C.系统性红斑狼疮
6 5 2008 17. 女性病人,18岁,半年前开始出现双侧颊部高于皮肤的红斑,日晒后加重。同时伴有脱发及口腔溃疡,无明显关节疼痛。查ESR36mm/h,WBC4.5×109/L, Hb14g/L, PLT21×109/L。ANA1:320(+), 抗Sm(+),尿蛋白(-)。目前病人的治疗首先考虑用(2.0分) A.大剂量糖皮质激素
B.羟基氯喹+中等剂量糖皮质激素
C.大剂量糖皮质激素+CTX冲击
D.NASIDS
E.以上都不是
B.羟基氯喹+中等剂量糖皮质激素
6 5 2008 18. 糖皮质激素通过胎盘时可被灭活,但下面哪种药物除外(2.0分) A.泼尼松
B.甲基泼尼松
C.氢化可的松
D.可的松
E.地塞米松和倍他米松
E.地塞米松和倍他米松
6 5 2008 19. 下列哪一项不是风湿热的主要表现:(2.0分) A.血沉增快
B.心脏炎
C.皮下结节
D.环形红斑
E.舞蹈病
A.血沉增快
6 5 2008 20. 川崎病时皮肤粘膜表现中最具有特征性的改变是:(2.0分) A.皮疹
B.指(趾)端变化
C. 眼球结膜充血
D.杨梅舌
E.唇红干燥、皲裂
B.指(趾)端变化
6 5 2008 21. 川崎病最常见的心血管并发症是:(2.0分) A.心包炎
B.冠状动脉损害
C.心律失常
D.心肌炎
E.心内膜炎
B.冠状动脉损害
6 5 2008 22. 哪一项不是应用激素的指征:(2.0分) A.类风湿关节炎伴心包炎
B.川崎病伴冠状动脉瘤
C. 风湿性心脏炎
D.系统性红斑狼疮
E.过敏性紫癜伴有消化道症状
B.川崎病伴冠状动脉瘤
6 5 2008 23. 过敏性紫癜特征性的表现:(2.0分) A.腹痛
B.关节肿痛
C.皮肤紫癜
D.蛋白尿、血尿
E.消化道出血
C.皮肤紫癜
6 5 2008 24. 风湿热关节炎的特点:(2.0分) A.最终转为慢性关节炎
B.关节畸形
C. 累及小关节为主
D.经治疗后可完全痊愈,不留畸形
E.晨僵
D.经治疗后可完全痊愈,不留畸形
6 5 2008 25. 幼年类风湿关节炎主要特征:(2.0分) A.不规则发热
B.慢性关节炎
C. 皮疹
D.胸膜炎
E.淋巴结肿大
B.慢性关节炎
6 5 2008 26. 川崎病最早出现的症状是:(2.0分) A.持续高热
B.颈淋巴结肿大
C.多形性皮疹
D.指、趾端脱皮
E.杨梅舌
A.持续高热
6 5 2008 27. 下列哪项对川崎病诊断具有特征意义:(2.0分) A.持续高热5天以上
B.颈淋巴结肿大
C.多形性皮疹
D.手足硬肿,指、趾端膜状脱皮
E.双球结膜充血,口唇皲裂,杨梅舌
D.手足硬肿,指、趾端膜状脱皮
6 5 2008 28. 决定过敏性紫癜预后的临床表现为:(2.0分) A.皮肤紫癜
B. 便血
C.关节痛
D.鼻出血
E.肾脏病变
E.肾脏病变
6 5 2008 29. 川崎病合并冠状动脉瘤服用阿司匹林的时间为:(2.0分) A.血清水杨酸浓度达到质量水平
B.血沉正常
C.血小板降至正常
D.长期服用直至动脉瘤消退
E.热退后
D.长期服用直至动脉瘤消退
6 5 2008 30. 过敏性紫癜的皮疹特点有:(2.0分) A.全身散在针尖大小紫癜,压之不退色
B.全身出现大块瘀斑,压不退色
C.下肢、臀部出现分布对称红疹,压不退色
D. 全身出现风团样皮疹,压之退色
E.大关节出现大块瘀斑,伴功能障碍,压不退色
C.下肢、臀部出现分布对称红疹,压不退色
6 5 2008 31. 确诊风湿热的次要表现不包括:(2.0分) A.发热
B.关节疼痛
C.皮下结节
D. 血沉加快
E.有风湿热既往史
C.皮下结节
6 5 2008 32. 川崎病的治疗中可使用丙种球蛋白,宜于几天内使用:(2.0分) A.10天
B.15天
C.4天
D. 5天
E.7天
A.10天
6 5 2008 33. 决定川崎病预后的临床表现为:(2.0分) A.心肌炎
B.心包炎
C.关节痛
D.无菌性脑膜炎
E.冠状动脉病变
E.冠状动脉病变
6 5 2008 34. 我国川崎病患者发热时阿司匹林的剂量通常是?(2.0分) A.30~50mg/Kg
B.3~5mg/Kg
C.80~100mg/Kg
D. 10~20mg/Kg
E.50~100mg/Kg
A.30~50mg/Kg
6 5 2008 35. 子宫内膜异位症常见于 (2.0分) A.腹膜
B.双侧卵巢
C.输卵管
D.子宫直肠窝
E.膀胱子宫隐窝
B.双侧卵巢
6 5 2008 36. 女性生殖器结核最常见的 (2.0分) A.输卵管结核
B.子宫内膜结核
C.子宫颈结核
D.卵巢结核
E.盆腔结核
A.输卵管结核
6 5 2008 37. 下列哪种检查方法常兼有治疗作用(2.0分) A.子宫输卵管碘油造影
B.盆腔充气造影
C.盆腔动脉造影
D.盆腔静脉造影
E.腹部平片
A.子宫输卵管碘油造影
6 5 2008 38. 子宫内膜癌侵犯深肌层的主要MRI征象是(2.0分) A.结合带低信号中断
B.结合带变薄
C.子宫内膜出血
D.子宫内膜轮廓不规整
E.子宫内膜信号异常
A.结合带低信号中断
6 5 2008 39. CT显示宫颈癌向外侵犯的确切根据是(2.0分) A.宫颈径线大于3mm
B.宫颈呈分叶状
C.周围脂肪层消失
D.直肠壁局限性增厚
E.宫颈与阴道壁分界不清
C.周围脂肪层消失
6 5 2008 40. 患者女性,35岁,CT、扫描子宫增大呈分叶状,表面光滑,子宫肌壁内实性略低密度影,有钙化,官腔受压移位,考虑为(2.0分) A.葡萄胎
B.子宫内膜癌
C.子宫肌瘤
D.子宫肌腺瘤
E.妊娠
C.子宫肌瘤
6 5 2008 41. 宫腔形态改变,子宫信号正常见于(2.0分) A.子宫肌瘤
B.子宫先天畸形
C.畸胎瘤
D.子宫内膜癌
E.卵巢囊腺瘤
B.子宫先天畸形
6 5 2008 42. 对子宫内膜癌的描述,下列哪项是错误的 (2.0分) A.绝经后出血
B.多为老年妇女
C.仅次于宫颈癌
D.限于子宫内膜T1WI或T2WI像可显示正常;
E.肿瘤最初位于子宫肌,后向内侵犯子宫内膜
E.肿瘤最初位于子宫肌,后向内侵犯子宫内膜
6 5 2008 43. 下列哪种疾病是引起月经过多的主要原因(2.0分) A.浆膜下子宫肌瘤
B.粘膜下子宫肌瘤
C.子宫颈囊肿
D.卵巢畸胎瘤
E.输卵管妊娠
B.粘膜下子宫肌瘤
6 5 2008 44. 关于子宫解剖的MRI表现,描述正确的是 (2.0分) A.子宫内膜正常厚度大于5mm
B.子宫内膜T2加权像呈低信号
C.结合带T2加权像呈低信号
D.子宫肌层T1加权像呈高信号
E.子宫峡部位于宫颈和阴道交界处
C.结合带T2加权像呈低信号
6 5 2008 45. 子宫颈癌的MRI表现是(2.0分) A.T2加权像为低信号
B.T2加权像为高信号
C.2加权像为等信号
D.A+B
E.A+B+C
B.T2加权像为高信号
6 5 2008 46. 下列卵巢癌MRI描述中,不正确的是 (2.0分) A.盆腔内软组织密度肿块。
B.囊壁厚簿不均
C.肿瘤内实性性成分多.
D.肿瘤的边缘不规则
E.增强扫描肿瘤无明显强化
E.增强扫描肿瘤无明显强化
6 5 2008 47. 前列腺病变最敏感的影像检查是(2.0分) A.X线腹平片
B.肛诊
C.CT平扫与增强扫描
D.经腹B型超声
E.MRI T2WI
E.MRI T2WI
6 5 2008 48. 怀疑前列腺癌病人CT检查可以(2.0分) A.发现早期病变
B.与其他前列腺病变鉴别
C.为早期病变分期
D.发现骨转移
E.除外前列腺被膜侵犯
D.发现骨转移
6 5 2008 49. 早期前列腺癌影像表现为:(2.0分) A.CT平扫前列腺局部结节样凸出
B.MR T2WI 高信号的外周带内低信号结节
C.增强CT扫描前列腺周边结节样低增强
D.超声前列腺横经大于5cm,内部回声不均
E.MR 脂肪抑制T1W序列扫描前列腺中心区高信号结节
B.MR T2WI 高信号的外周带内低信号结节
6 5 2008 50. 前列腺良性增生的影像表现特点是:(2.0分) A.前列腺局部增大
B.CT平扫前列腺密度不均
C.MRI T2WI前列腺中央区增大,周边区变薄
D.膀胱造影示膀胱下壁乳头状充盈缺损
E.CT增强扫描前列腺中心区强化明显
C.MRI T2WI前列腺中央区增大,周边区变薄
6 5 2009 1. 患儿,20天,出生体重4公斤,嗜睡,吃奶少,便秘,查体:皮肤黄染,前囟4*3厘米,心肺听诊无殊,腹略胀,脐疝,肝、脾肋下未及,神经系统(-)。该患儿可能的疾病是: (2.0分) A.新生儿败血症
B.新生儿溶血症
C.先天性甲状腺功能低下症
D.粘多糖1型
E.先天性巨结肠
C.先天性甲状腺功能低下症
6 5 2009 2. 患儿,男,7岁,身高100厘米,学习良好,查体:反应灵活,躯体长,四肢短,心肺听诊无殊,腹软,肝、脾肋下未及,神经系统(-)。该患儿可能的疾病是: (2.0分) A.21-三体综合症
B.甲状腺功能减低症
C.先天性软骨发育不良
D.生长激素缺乏症
E.粘多糖病
C.先天性软骨发育不良
6 5 2009 3. 患儿,女,17岁,身高130厘米,学习成绩中等,月经未来潮,父亲身高175厘米,母亲身高160厘米,查体:发际较低,脸上较多黑痣,四肢匀称,心肺听诊无殊,乳房B1,腹平软,肝、脾肋下未及,神经系统(-),外阴幼稚。该患儿可能的疾病是: (2.0分) A.先天性卵巢发育不全综合症
B.生长激素缺乏症
C.甲状腺功能减低症
D.家族性矮小
E.21-三体综合症
A.先天性卵巢发育不全综合症
6 5 2009 4. 列哪项检查结果可以诊断完全性生长激素缺乏症: (2.0分) A.<5ug/L
B.<6ug/L
C.>10ug/L
D.<8ug/L
E.>15ug/L
A.<5ug/L
6 5 2009 5. 确诊先天性甲状腺功能减低症主要依靠下列哪项检查(2.0分) A.骨骼X线片检查
B.血清T3、 T4 TSH测定
C.血清蛋白结合碘测定
D.染色体检查
E.特殊面容
B.血清T3、 T4 TSH测定
6 5 2009 6. 下列哪项不符合原发性生长激素缺乏症(2.0分) A.身材矮小,躯干长、四肢短
B.生长速率<4cm/年
C.智能正常
D.骨化中心发育落后
E.患儿出生时身高和体重均正常
A.身材矮小,躯干长、四肢短
6 5 2009 7. 在身材矮小的儿童中,哪种情况下其骨龄发育是正常的(2.0分) A.先天性肾上腺皮质增生症
B.体质性青春期延迟
C.生长激素缺乏症
D.甲状腺功能减低症
E.家族性矮身材
E.家族性矮身材
6 5 2009 8. 下列哪项检查不是生长激素缺乏症的确诊试验(2.0分) A.可乐定刺激试验
B.左旋多巴试验
C.胰岛素刺激试验
D.精氨酸刺激试验
E.睡眠试验
E.睡眠试验
6 5 2009 9. 关于生长激素缺乏症,错误的是(2.0分) A.特发性下丘脑,垂体功能障碍是致生长激素缺乏的主要原因
B.任何病变侵及下丘脑或垂体前叶时都可引起生长迟缓
C.生长激素是由垂体前叶分泌的
D.原发性生长激素缺乏症多见于女孩
E.骨龄落后
D.原发性生长激素缺乏症多见于女孩
6 5 2009 10. 先天性甲状腺功能减低症的治疗,下列哪项是错误的(2.0分) A.治疗开始时间越早越好
B.症状改善后停药
C.初始剂量以最小剂量开始
D.间隔1~2周加量一次至症状改善
E.维持量个体差异比较大
B.症状改善后停药
6 5 2009 11. 甲状腺激素的主要生理作用,下列哪项是错误的 (2.0分) A.产热
B.促进蛋白质合成
C.促进中枢神经系统发育
D.促进脂肪分解和利用
E.抑制钙磷在骨质中的合成代谢
E.抑制钙磷在骨质中的合成代谢
6 5 2009 12. 川崎病时皮肤粘膜表现中最具有特征性的改变是:(2.0分) A.皮疹
B.指(趾)端变化
C.眼球结膜充血
D.杨梅舌
E.唇红干燥、皲裂
B.指(趾)端变化
6 5 2009 13. 川崎病最常见的心血管并发症是:(2.0分) A.心包炎
B.冠状动脉损害
C.心律失常
D.心肌炎
E.心内膜炎
B.冠状动脉损害
6 5 2009 14. 川崎病的治疗中可使用丙种球蛋白,宜于几天内使用:(2.0分) A.10天
B.15天
C. 4天
D.5天
E. 7天
A.10天
6 5 2009 15. 决定川崎病预后的临床表现为:(2.0分) A.心肌炎
B. 心包炎
C.关节痛
D.无菌性脑膜炎
E.冠状动脉病变
E.冠状动脉病变
6 5 2009 16. 我国川崎病患者发热时阿司匹林的剂量通常是?(2.0分) A.30~50mg/Kg
B. 3~5mg/Kg
C.80~100mg/Kg
D.10~20mg/Kg
E.50~100mg/Kg
A.30~50mg/Kg
6 5 2009 17. 下列哪一项不是风湿热的主要表现:(2.0分) A.血沉增快
B.心脏炎
C.皮下结节
D. 环形红斑
E.舞蹈病
A.血沉增快
6 5 2009 18. 下列哪项不属于幼年特发性关节炎的亚型:(2.0分) A.多关节型
B.少关节型
C. 全身型
D. 幼年强直性脊柱炎
E. 未分化型
D. 幼年强直性脊柱炎
6 5 2009 19. 哪一项不是应用激素的指征:(2.0分) A.类风湿关节炎伴心包炎
B.川崎病伴冠状动脉瘤
C. 风湿性心脏炎
D.系统性红斑狼疮
E.过敏性紫癜伴有消化道症状
B.川崎病伴冠状动脉瘤
6 5 2009 20. 过敏性紫癜特征性的表现:(2.0分) A.腹痛
B.关节肿痛
C. 皮肤紫癜
D.蛋白尿、血尿
E.消化道出血
C. 皮肤紫癜
6 5 2009 21. 风湿热关节炎的特点:(2.0分) A.最终转为慢性关节炎
B.关节畸形
C.累及小关节为主
D.经治疗后可完全痊愈,不留畸形
E. 晨僵
D.经治疗后可完全痊愈,不留畸形
6 5 2009 22. 幼年特发性关节炎主要特征:(2.0分) A.不规则发热
B.慢性关节炎
C.皮疹
D.胸膜炎
E.淋巴结肿大
B.慢性关节炎
6 5 2009 23. 以下哪项不是自身免疫性疾病的特点? (2.0分) A.体内存在高浓度的自身抗体
B.自身抗体的浓度与疾病严重程度相关
C.常有多系统和多器官损害
D.免疫球蛋白升高,补体降低
E.免疫抑制剂治疗有效
D.免疫球蛋白升高,补体降低
6 5 2009 24. 系统性红斑狼疮属于(2.0分) A.Ⅰ型变态反应
B.Ⅱ型变态反应
C.Ⅲ型变态反应
D.Ⅳ型变态反应
E.以上都不是
C.Ⅲ型变态反应
6 5 2009 25. 下面哪个自身抗体是结缔组织病的标准筛选试验? (2.0分) A.抗核抗体
B.抗Sm抗体
C.抗双链DNA抗体
D.抗SSA抗体
E.抗SSB抗体
A.抗核抗体
6 5 2009 26. 系统性红斑狼疮口腔溃疡的特点是(2.0分) A.如无合并感染,多无疼痛
B.伴有明显疼痛
C.多见于舌尖和舌体
D.溃疡的发作与疾病的活动多数无关
E.溃疡表面覆盖有黄色坏死组织
A.如无合并感染,多无疼痛
6 5 2009 27. 以下哪项不是狼疮脑病的脑脊液特点? (2.0分) A.压力增高
B.细胞总数增加
C.蛋白轻度升高
D.糖和氯化物下降
E.以上都不是
D.糖和氯化物下降
6 5 2009 28. 目前显示垂体病变最好的影像学检查方法是(2.0分) A.头颅平片
B.CT
C.MRI
D.核素显像
E.超声
C.MRI
6 5 2009 29. 头颅侧位片,下列正常蝶鞍形态中,不正确的是(2.0分) A.圆形
B.椭圆形
C.扁圆形
D.双鞍底
E.桥形蝶鞍
D.双鞍底
6 5 2009 30. 垂体微腺瘤的MRI直接征象是:(2.0分) A.垂体上缘局部或偏侧膨隆
B.垂体高度的增加
C.垂体柄偏移
D.鞍底的破坏
E.病灶强化早期信号低于垂体,后期高于垂体
E.病灶强化早期信号低于垂体,后期高于垂体
6 5 2009 31. 鞍上池内不包括 (2.0分) A.垂体柄
B.视束
C.丘脑
D.颈内动脉
E.视交叉
C.丘脑
6 5 2009 32. 32.肾上腺最佳影像学成像技术是:(2.0分) A. B超
B. MRI检查
C. CT
D.血管造影
E.核素显像
C. CT
6 5 2009 33. 患者男,33岁,以阵发性高血压入院。CT检查发现肾上腺异常,可能的诊断中不包括:(2.0分) A.嗜铬细胞瘤
B.髓质脂肪瘤
C.皮质腺瘤
D.肾上腺增生
E.肾上腺癌
B.髓质脂肪瘤
6 5 2009 34. 关于嗜铬细胞瘤的描述,错误的是:(2.0分) A.多见于14岁以下儿童
B.为圆形或卵圆形肿块
C.即可有实质性肿块,也可有囊性肿块
D.增强扫描后肿瘤强化明显
E.肿瘤大小不能鉴别良恶性
A.多见于14岁以下儿童
6 5 2009 35. 肾上腺单发肿块,MRI检查在同相位、反相位上信号无改变,可能的诊断不包括:(2.0分) A.不含脂肪的肾上腺腺瘤
B.肾上腺转移瘤
C.肾上腺腺癌
D.肾上腺腺瘤
E.嗜铬细胞瘤
D.肾上腺腺瘤
6 5 2009 36. 肾上腺转移瘤最常见的原发肿瘤是(2.0分) A.乳腺癌
B.肝癌
C.胰腺癌
D.胆囊癌
E.肺癌
E.肺癌
6 5 2009 37. 可引起Cushing综合征的是(2.0分) A.肾上腺增生,嗜铬细胞瘤,肾上腺皮质腺瘤
B.肾上腺腺瘤,嗜铬细胞瘤,肾上腺皮质腺瘤
C.肾上腺增生, 肾上腺腺瘤,肾上腺皮质腺瘤
D.肾上腺腺瘤,肾上腺增生,肾上腺髓样脂肪瘤
E.肾上腺腺瘤,转移瘤,囊肿
C.肾上腺增生, 肾上腺腺瘤,肾上腺皮质腺瘤
6 5 2009 38. 女性生殖器官结核首先累及的是 (2.0分) A.子宫体
B.子宫内膜
C.子宫颈
D.子宫腔
E.输卵管
E.输卵管
6 5 2009 39. 关于子宫内膜异位症的描述,错误的是(2.0分) A.分外在性和内在性
B.子宫腺肌症是内在性子宫内膜异位
C.子宫内膜异位在卵巢表面,可形成巧克力囊肿
D.子宫内膜可异位在大肠
E.RI主要表现是T2WI上低信号联合带的变薄
E.RI主要表现是T2WI上低信号联合带的变薄
6 5 2009 40. 有关子宫内膜癌描述错误的是(2.0分) A.联合带的信号改变可作为肌层受侵的标志
B.联合带变薄
C.病变限于子宫内膜时,T1WI和T2WI可为正常
D.联合带于T2WI上的低信号中断
E.联合带于T2WI上的高信号中断
E.联合带于T2WI上的高信号中断
6 5 2009 41. 最常见的子宫肌瘤类型是(2.0分) A.肌壁间肌瘤
B.粘膜下肌瘤
C.浆膜下肌瘤
D.宫颈肌瘤
E.阔韧带肌瘤
A.肌壁间肌瘤
6 5 2009 42. 卵巢癌典型的大网膜种植表现为(2.0分) A.呈饼状软组织肿块
B.多发结节状改变
C.网状纤维条索
D.无一定特征
E.以上均不是
A.呈饼状软组织肿块
6 5 2009 43. 关于子宫肌瘤的描述,正确的是(2.0分) A.MRI T1WI上子宫肌瘤的信号高于子宫肌
B.MRI不能检出3mm以下的子宫肌瘤
C.CT增强后子宫肌瘤的密度略高于正常子宫肌的强化
D.MRI T2WI上,典型的子宫肌瘤为明显高信号
E.MRI T2WI上,典型的子宫肌瘤为明显低信号
E.MRI T2WI上,典型的子宫肌瘤为明显低信号
6 5 2009 44. 关于子宫联合带的描述,正确的是 (2.0分) A.MRI T2WI上为高信号
B.代表子宫肌内层
C.代表子宫肌外层
D.代表子宫内膜
E.代表子宫分泌物
B.代表子宫肌内层
6 5 2009 45. 我国女性生殖系统最常见的恶性肿瘤是(2.0分) A.子宫颈癌
B.子宫癌
C.盆腔癌
D.卵巢癌
E.阴道癌
A.子宫颈癌
6 5 2009 46. 卵巢转移癌最常见的原发性肿瘤是 (2.0分) A.肺癌
B.胶质瘤
C.肾癌
D.甲状腺癌
E.胃肠道肿瘤
E.胃肠道肿瘤
6 5 2009 47. 前列腺横径增大的标准是(2.0分) A.>2cm
B.3-4cm
C.>4cm
D.4-5cm
E.>5cm
E.>5cm
6 5 2009 48. 大多数前列腺癌发生在(2.0分) A.中央带
B.周边带
C.移行带
D.尿道周围腺体
E.无一定规律
B.周边带
6 5 2009 49. 典型先天性肾上腺皮质增生症的发病率约为: (2.0分) A.1/10 000
B.1/5000
C.1/1000
D.1/100 000
E.1/20 000
A.1/10 000
6 5 2009 50. 人体糖皮质激素如皮质醇、脱氧皮质酮等主要在 合成。(2.0分) A.肾上腺皮质球状带
B.肾上腺皮质网状带
C.肾上腺皮质束状带
D.肾上腺髓质
E.肾小球
C.肾上腺皮质束状带
6 5 2010 1. 对于特发性矮小患儿,需用生长激素治疗,其起始剂量是 (2.5分) A.0.05U/kg/d
B.0.1U/kg/d
C.0.15U/kg/d
D.0.2U/kg/d
E.0.25U/kg/d
C.0.15U/kg/d
6 5 2010 2. 患儿,20天,出生体重4公斤,嗜睡,吃奶少,便秘,查体:皮肤黄染,前囟4*3厘米,心肺听诊无殊,腹略胀,脐疝,肝、脾肋下未及,神经系统(-)。该患儿可能的疾病是: (2.5分) A.新生儿败血症
B.新生儿溶血症
C.先天性甲状腺功能低下症
D.粘多糖1型
E.先天性巨结肠
C.先天性甲状腺功能低下症
6 5 2010 3. 患儿,男,12岁,身高130厘米,学习成绩优秀,父亲身高175厘米,母亲身高160厘米,查体:反应灵活,四肢匀称,心肺听诊无殊,腹平软,肝、脾肋下未及,神经系统(-)。该患儿可能的疾病是 (3.5分) A.家族性矮小
B.21-三体综合症
C.生长激素缺乏症
D.甲状腺功能减低症
E.先天性软骨发育不良
C.生长激素缺乏症
6 5 2010 4. 引起中枢性甲状腺功能低下的病因是 (2.5分) A.甲状腺不发育或发育不全
B.甲状腺激素合成 途径障碍或母体存在抗甲状腺抗体
C.甲状腺激素合成途径障碍
D.促甲状腺素(TSH)缺乏
E.甲状腺或靶器官反应低下
D.促甲状腺素(TSH)缺乏
6 5 2010 5. 列哪项检查结果可以诊断完全性生长激素缺乏症 (2.5分) A.<5ug/L
B.<6ug/L
C.>10ug/L
D.<8ug/L
E.>15ug/L
A.<5ug/L
6 5 2010 6. 确诊先天性甲状腺功能减低症主要依靠下列哪项检查(2.5分) A.骨骼X线片检查
B.血清T3、 T4 TSH测定
C.血清蛋白结合碘测定
D.染色体检查
E.特殊面容
B.血清T3、 T4 TSH测定
6 5 2010 7. 地方性甲状腺功能减低症临床表现为"神经性"综合征的患儿下列哪项不符合(2.5分) A.甲状腺功能正常或轻度减低
B.智能低下,聋哑
C.共济失调
D.身材矮小
E.痉挛性瘫痪
D.身材矮小
6 5 2010 8. 在身材矮小的儿童中,哪种情况下其骨龄发育是正常的(2.5分) A.先天性肾上腺皮质增生症
B.体质性青春期延迟
C.生长激素缺乏症
D.甲状腺功能减低症
E.家族性矮身材
E.家族性矮身材
6 5 2010 9. 关于生长激素缺乏症,错误的是(3.5分) A.特发性下丘脑,垂体功能障碍是致生长激素缺乏的主要原因
B.任何病变侵及下丘脑或垂体前叶时都可引起生长迟缓
C.生长激素是由垂体前叶分泌的
D.原发性生长激素缺乏症多见于女孩
E.骨龄落后
D.原发性生长激素缺乏症多见于女孩
6 5 2010 10. 先天性甲状腺功能减低症的治疗,下列哪项是错误的(2.5分) A.治疗开始时间越早越好
B.症状改善后停药
C.初始剂量以最小剂量开始
D.间隔1~2周加量一次至症状改善
E.维持量个体差异比较大
B.症状改善后停药
6 5 2010 11. 先天性甲状腺功能减低症3岁患儿检查骨龄时,X线摄片的最佳部位应是(2.5分) A.膝部
B.腕部
C.肘部
D.髋部
E.踝部
B.腕部
6 5 2010 12. 垂体微腺瘤的MRI直接征象是(2.5分) A.垂体上缘局部或偏侧膨隆
B.垂体高度的增加
C.垂体柄偏移
D.鞍底的破坏
E.病灶强化早期信号低于垂体,后期高于垂体
E.病灶强化早期信号低于垂体,后期高于垂体
6 5 2010 13. 患者男,33岁,以阵发性高血压入院。CT检查发现肾上腺异常,可能的诊断中不包括(2.5分) A.嗜铬细胞瘤
B.髓质脂肪瘤
C.皮质腺瘤
D.肾上腺增生
E.肾上腺癌
B.髓质脂肪瘤
6 5 2010 14. 肾上腺单发肿块,MRI检查在同相位、反相位上信号无改变,可能的诊断不包括(2.5分) A.不含脂肪的肾上腺腺瘤
B.肾上腺转移瘤
C.肾上腺腺癌
D.肾上腺腺瘤
E.嗜铬细胞瘤
D.肾上腺腺瘤
6 5 2010 15. 对甲状腺结节的影像学检查,下列说法正确的是(2.5分) A.增强CT检查甲状腺结节最有价值
B.MRI检查甲状腺结节最有价值
C.核医学检查甲状腺结节最有价值
D.B超检查甲状腺结节最有价值
E.数字X线摄片(DR)检查甲状腺结节最有价值
D.B超检查甲状腺结节最有价值
6 5 2010 16. 肾上腺最佳影像学成像技术是(3.5分) A.B超
B.MRI检查
C.CT
D.血管造影
E.核素显像
C.CT
6 5 2010 17. 有关子宫内膜癌描述错误的是(2.5分) A.联合带的信号改变可作为肌层受侵的标志
B.联合带变薄
C.病变限于子宫内膜时,T1WI和T2WI可为正常
D.联合带于T2WI上的低信号中断
E.联合带于T2WI上的高信号中断
E.联合带于T2WI上的高信号中断
6 5 2010 18. 最常见的子宫肌瘤类型是(3.5分) A.肌壁间肌瘤
B.粘膜下肌瘤
C.浆膜下肌瘤
D.宫颈肌瘤
E.阔韧带肌瘤
A.肌壁间肌瘤
6 5 2010 19. 关于子宫联合带的描述,正确的是(2.5分) A.MRI T2WI上为高信号
B.代表子宫肌内层
C.代表子宫肌外层
D.代表子宫内膜
E.代表子宫分泌物
B.代表子宫肌内层
6 5 2010 20. 卵巢转移癌最常见的原发性肿瘤是(2.5分) A.肺癌
B.胶质瘤
C.肾癌
D.甲状腺癌
E.胃肠道肿瘤
E.胃肠道肿瘤
6 5 2010 21. 良性前列腺增生主要发生在(3.5分) A.移行带
B.中央带
C.周边带
D.前肌纤维质
E.各部位发生率无差别
A.移行带
6 5 2010 22. 川崎病时皮肤粘膜表现中最具有特征性的改变是(2.5分) A.皮疹
B.指(趾)端变化
C.眼球结膜充血
D.杨梅舌
E.唇红干燥、皲裂
B.指(趾)端变化
6 5 2010 23. 川崎病最早出现的症状是(2.5分) A.持续高热
B.颈淋巴结肿大
C.多形性皮疹
D.指、趾端脱皮
E.杨梅舌
A.持续高热
6 5 2010 24. 川崎病合并冠状动脉瘤服用阿司匹林的时间为(2.5分) A.血清水杨酸浓度达到质量水平
B.血沉正常
C.血小板降至正常
D.长期服用直至动脉瘤消退
E.热退后
D.长期服用直至动脉瘤消退
6 5 2010 25. 决定川崎病预后的临床表现为(2.5分) A.心肌炎
B.心包炎
C.关节痛
D.无菌性脑膜炎
E.冠状动脉病变
E.冠状动脉病变
6 5 2010 26. 如没有冠状动脉病变,川崎病患者阿司匹林的疗程(2.5分) A.8~12周
B.血沉正常
C. 6~8周
D.长期服用
E.热退后
C. 6~8周
6 5 2010 27. 以下哪项不是自身免疫性疾病的特点? (2.5分) A.体内存在高浓度的自身抗体
B.自身抗体的浓度与疾病严重程度相关
C.常有多系统和多器官损害
D.D.免疫球蛋白升高,补体降低
E.免疫抑制剂治疗有效
D.D.免疫球蛋白升高,补体降低
6 5 2010 28. 与SLE活动性相关的自身抗体特异性高? (2.5分) A.抗核抗体
B.抗Sm抗体
C.抗双链DNA抗体
D.抗SSA抗体
E.抗SSB抗体
C.抗双链DNA抗体
6 5 2010 29. 雷诺现象的皮肤颜色变化顺序为(2.5分) A.红→白→紫
B.紫→红→白
C.紫→白→红
D.红→紫→白
E.白→紫→红
E.白→紫→红
6 5 2010 30. 女性病人,18岁,半年前开始出现双侧颊部高于皮肤的红斑,日晒后加重。同时伴有脱发及口腔溃疡,无明显关节疼痛。查ESR36mm/h,WBC4.5×109/L, Hb14g/L, PLT21×109/L。ANA1:320(+), 抗Sm(+),尿蛋白(-)。目前病人的治疗首先考虑用(3.5分) A.大剂量糖皮质激素
B.羟基氯喹+中等剂量糖皮质激素
C.大剂量糖皮质激素+CTX冲击
D.NASIDS
E.以上都不是
B.羟基氯喹+中等剂量糖皮质激素
6 5 2010 31. 系统性红斑狼疮合并感染发热,与疾病活动所致发热的区别在于(3.5分) A.后者ANA滴度较高
B.前者ESR较高
C.后者抗ds-DNA抗体升高
D.后者抗Sm抗体阳性率较高
E.后者尿蛋白(+)
C.后者抗ds-DNA抗体升高
6 5 2010 32. 下列哪一项不是风湿热的主要表现(3.5分) A.血沉增快
B.心脏炎
C.皮下结节
D.环形红斑
E.舞蹈病
A.血沉增快
6 5 2010 33. 哪一项不是应用激素的指征(2.5分) A.类风湿关节炎伴心包炎
B.川崎病伴冠状动脉瘤
C.风湿性心脏炎
D.系统性红斑狼疮
E.过敏性紫癜伴有消化道症状
B.川崎病伴冠状动脉瘤
6 5 2010 34. 川崎病最早出现的症状是(3.5分) A.持续高热
B.颈淋巴结肿大
C.多形性皮疹
D.指、趾端脱皮
E.杨梅舌
A.持续高热
6 5 2010 35. 确诊风湿热的次要表现不包括(3.5分) A.发热
B.关节疼痛
C.皮下结节
D.血沉加快
E.有风湿热既往史
C.皮下结节
6 5 2010 36. 过敏性紫癜的皮疹特点有(2.5分) A.全身散在针尖大小紫癜,压之不退色
B.全身出现大块瘀斑,压不退色
C.下肢、臀部出现分布对称红疹,压不退色
D.全身出现风团样皮疹,压之退色
E.大关节出现大块瘀斑,伴功能障碍,压不退色
C.下肢、臀部出现分布对称红疹,压不退色
6 5 2011 1. 以下哪项不是自身免疫性疾病的特点?(2.5分) A.体内存在高浓度的自身抗体
B.自身抗体的浓度与疾病严重程度相关
C.常有多系统和多器官损害
D.免疫球蛋白升高,补体降低
E.免疫抑制剂治疗有效
D.免疫球蛋白升高,补体降低
6 5 2011 2. 与SLE活动性相关的自身抗体特异性高? (2.5分) A.抗核抗体
B.抗Sm抗体
C.抗双链DNA抗体
D.抗SSA抗体
E.抗SSB抗体
C.抗双链DNA抗体
6 5 2011 3. 雷诺现象的皮肤颜色变化顺序为(2.5分) A.红→白→紫
B.紫→红→白
C.紫→白→红
D.红→紫→白
E.白→紫→红
E.白→紫→红
6 5 2011 4. 女性病人,18岁,半年前开始出现双侧颊部高于皮肤的红斑,日晒后加重。同时伴有脱发及口腔溃疡,无明显关节疼痛。查ESR36mm/h,WBC4.5×109/L, Hb14g/L, PLT21×109/L。ANA1:320(+), 抗Sm(+),尿蛋白(-)。目前病人的治疗首先考虑用(2.5分) A.大剂量糖皮质激素
B.羟基氯喹+中等剂量糖皮质激素
C.大剂量糖皮质激素+CTX冲击
D.NASIDS
E.以上都不是
B.羟基氯喹+中等剂量糖皮质激素
6 5 2011 5. 系统性红斑狼疮合并感染发热,与疾病活动所致发热的区别在于(2.5分) A.后者ANA滴度较高
B.前者ESR较高
C.后者抗ds-DNA抗体升高
D.后者抗Sm抗体阳性率较高
E.后者尿蛋白(+)
C.后者抗ds-DNA抗体升高
6 5 2011 6. 川崎病时皮肤粘膜表现中最具有特征性的改变是: (2.5分) A.皮疹
B.指(趾)端变化
C.眼球结膜充血
D.杨梅舌
E.唇红干燥、皲裂
B.指(趾)端变化
6 5 2011 7. 川崎病最早出现的症状是(2.5分) A.持续高热
B.颈淋巴结肿大
C.形性皮疹
D.指、趾端脱皮
E.杨梅舌
A.持续高热
6 5 2011 8. 川崎病合并冠状动脉瘤服用阿司匹林的时间为(2.5分) A.血清水杨酸浓度达到质量水平
B.血沉正常
C.血小板降至正常
D.长期服用直至动脉瘤消退
E.热退后
D.长期服用直至动脉瘤消退
6 5 2011 9. 决定川崎病预后的临床表现为(2.5分) A.心肌炎
B.心包炎
C.关节痛
D.无菌性脑膜炎
E.冠状动脉病变
E.冠状动脉病变
6 5 2011 10. 如没有冠状动脉病变,川崎病患者阿司匹林的疗程(2.5分) A.8~12周
B.血沉正常
C.6~8周
D.长期服用
E.热退后
C.6~8周
6 5 2011 11. 目前显示垂体病变最好的影像学检查方法是(2.5分) A.头颅平片
B.CT
C.MRI
D.核素显像
E.超声
C.MRI
6 5 2011 12. 鞍上池内不包括(2.5分) A.垂体柄
B.视束
C.丘脑
D.颈内动脉
E.视交叉
C.丘脑
6 5 2011 13. 关于垂体腺瘤的说法,以下哪种是错误的: (2.5分) A.大腺瘤可突出到鞍上池
B.当腺瘤的直径小于1.5厘米时,被称为微腺瘤
C.腺瘤可以呈哑铃状
D.大腺瘤会导致脑积水
E.腺瘤可以侵犯海绵窦
C.腺瘤可以呈哑铃状
6 5 2011 14. 分泌抑制青春期激素的是(2.5分) A.垂体
B.松果体
C.丘脑
D.苍白球
E.海马
B.松果体
6 5 2011 15. 肾上腺皮质最常见的功能性肿瘤是(2.5分) A.嗜铬细胞瘤
B.皮质腺瘤
C.髓质脂肪瘤
D.神经母细胞瘤
E.肾上腺癌
B.皮质腺瘤
6 5 2011 16. 患者男,33岁,以阵发性高血压入院。CT检查发现肾上腺异常,可能的诊断中不包括:(2.5分) A.嗜铬细胞瘤
B.肾上腺癌
C.皮质腺瘤
D.肾上腺增生
E.髓质脂肪瘤
E.髓质脂肪瘤
6 5 2011 17. 关于肾上腺皮质醇增多症描述不正确的是(2.5分) A.糖皮质激素分泌过多
B.多为肾上腺增生所致,少数为肾上腺皮质腺瘤
C.肾上腺增生可为结节性和弥漫性
D.肾上腺皮质腺瘤多为双侧
E.肾上腺皮质腺瘤瘤体较小,直径多在2-3cm
D.肾上腺皮质腺瘤多为双侧
6 5 2011 18. 嗜铬细胞瘤被称为10%肿瘤,下列叙述中错误的是(2.5分) A.10%发生于肾上腺外
B.10%为双侧
C.10%为女性
D.10%为恶性
E.10%为多发
C.10%为女性
6 5 2011 19. 肾上腺转移瘤最常见的原发肿瘤是(2.5分) A.乳腺癌
B.肺癌
C.胰腺癌
D.胆囊癌
E.肝癌
B.肺癌
6 5 2011 20. 肾上腺肿瘤在反相位上信号无信号丢失时,下列错误的是(2.5分) A.转移瘤的典型表现
B.不含脂肪的腺瘤
C.嗜铬细胞瘤
D.嗜铬细胞瘤囊变
E.含脂肪的腺瘤
E.含脂肪的腺瘤
6 5 2011 21. 发现和诊断子宫肌瘤最敏感的影像学方法是(2.5分) A.CT
B.X线
C.超声
D.MRI
E.输卵管造影
D.MRI
6 5 2011 22. 关于子宫内膜异位症,错误的是(2.5分) A.多见于30-45岁妇女
B.功能性子宫内膜出现在宫腔以外任何部位
C.异位的子宫内膜发生在子宫以外任何其他部位
D.异位的子宫内膜出现在宫体的肌层时,称为外在性子宫内膜异位症,也称子宫腺肌病
E.常见于卵巢、子宫的韧带、直肠阴道隔、子宫直肠隐窝、输卵管等部位
D.异位的子宫内膜出现在宫体的肌层时,称为外在性子宫内膜异位症,也称子宫腺肌病
6 5 2011 23. 有关宫颈癌的表述,正确的是(2.5分) A.宫颈癌向外侵犯的确切根据是周围脂肪层存在
B.接触性出血是宫颈癌晚期的主要症状
C.CT对肿瘤范围的显示要优于MRI
D.宫颈癌早期诊断主要依靠影像学检查
E.影像学检查主要用于评估宫颈癌的范围及分期
E.影像学检查主要用于评估宫颈癌的范围及分期
6 5 2011 24. 卵巢癌典型的大网膜种植表现为(2.5分) A.网状纤维条索
B.多发结节状改变
C.呈饼状软组织肿块
D.无一定特征
E.以上均不是
C.呈饼状软组织肿块
6 5 2011 25. 关于子宫肌瘤的描述,正确的是(2.5分) A.MRI T1WI上子宫肌瘤的信号高于子宫肌
B.MRI不能检出3mm以下的子宫肌瘤
C.CT增强后子宫肌瘤的密度略低于正常子宫肌的强化
D.MRI T2WI上,典型的子宫肌瘤为明显高信号
E.MRI T2WI上,典型的子宫肌瘤为等信号
C.CT增强后子宫肌瘤的密度略低于正常子宫肌的强化
6 5 2011 26. 我国女性生殖系统最常见的恶性肿瘤是(2.5分) A.盆腔癌
B.子宫癌
C.子宫颈癌
D.卵巢癌
E.阴道癌
C.子宫颈癌
6 5 2011 27. 下列说法正确的是(2.5分) A.前列腺癌血行转移最常发生在肺部
B.前列腺癌血行转移最常发生在肝脏
C.前列腺癌血行转移最常发生在肾脏
D.前列腺癌血行转移最常发生在骨骼
E.前列腺癌血行转移最常发生在脑部
D.前列腺癌血行转移最常发生在骨骼
6 5 2011 28. 下列说法正确的是: (2.5分) A.大多数前列腺癌发生在中央带
B.大多数前列腺癌发生在移行带
C.大多数前列腺癌发生在周边带
D.大多数前列腺癌发生在尿道周围腺体
E.无一定规律
C.大多数前列腺癌发生在周边带
6 5 2011 29. 前列腺增生与早期前列腺癌鉴别诊断最有价值的是(2.5分) A.CT
B.MRI
C.超声
D.X线平片
E.膀胱造影
B.MRI
6 5 2011 30. 关于前列腺癌的描述,正确的是(2.5分) A.常规MRI能确定起源于中央腺体内的早期前列腺癌
B.MRI显示前列腺癌主要靠增强检查
C.前列腺癌主要发生在外周带
D.精囊受侵表现为T2加权像上呈高信号肿块
E.MRI在前列腺癌的诊断上与CT相当
C.前列腺癌主要发生在外周带
6 5 2011 31. 患儿,20天,出生体重4公斤,嗜睡,吃奶少,便秘,查体:皮肤黄染,前囟4*3厘米,心肺听诊无殊,腹略胀,脐疝,肝、脾肋下未及,神经系统(-)。该患儿可能的疾病是 (2.5分) A.新生儿败血症
B.新生儿溶血症
C.先天性甲状腺功能低下症
D.粘多糖1型
E.先天性巨结肠
C.先天性甲状腺功能低下症
6 5 2011 32. 患儿,男,12岁,身高130厘米,学习成绩优秀,父亲身高175厘米,母亲身高160厘米,查体:反应灵活,四肢匀称,心肺听诊无殊,腹平软,肝、脾肋下未及,神经系统(-)。该患儿可能的疾病是: (2.5分) A.家族性矮小
B.21-三体综合症
C.生长激素缺乏症
D.甲状腺功能减低症
E.先天性软骨发育不良
C.生长激素缺乏症
6 5 2011 33. 引起中枢性甲状腺功能低下的病因是 (2.5分) A.甲状腺不发育或发育不全
B.甲状腺激素合成 途径障碍或母体存在抗甲状腺抗体
C.甲状腺激素合成途径障碍
D.促甲状腺素(TSH)缺乏
E.甲状腺或靶器官反应低下
D.促甲状腺素(TSH)缺乏
6 5 2011 34. 确诊先天性甲状腺功能减低症主要依靠下列哪项检查 (2.5分) A.骨骼X线片检查
B.血清T3、 T4 TSH测定
C.血清蛋白结合碘测定
D.染色体检查
E.特殊面容
B.血清T3、 T4 TSH测定
6 5 2011 35. 下列哪项不符合原发性生长激素缺乏症 (2.5分) A.身材矮小,躯干长、四肢短
B.生长速率<4cm/年
C.智能正常
D.骨化中心发育落后
E.患儿出生时身高和体重均正常
A.身材矮小,躯干长、四肢短
6 5 2011 36. 在身材矮小的儿童中,哪种情况下其骨龄发育是正常的(2.5分) A.先天性肾上腺皮质增生症
B.体质性青春期延迟
C.生长激素缺乏症
D.甲状腺功能减低症
E.家族性矮身材
E.家族性矮身材
6 5 2011 37. 男,10岁,出生体重3200g,身材比例匀称,智力正常,身高110cm,生长速度3cm/年。最可能的诊断是(2.5分) A.性早熟
B.家族性身矮
C.糖尿病
D.生长激素缺乏
E.甲状腺功能低下
D.生长激素缺乏
6 5 2011 38. 先天性甲状腺功能减低症的治疗,正确的是 (2.5分) A.明确诊断后即应开始治疗
B.药物应大剂量开始
C.持续大剂量冲击治疗
D.应服药5年以上
E.随访5年
A.明确诊断后即应开始治疗
6 5 2011 39. 先天性甲状腺功能减低症的治疗,下列哪项是错误的(2.5分) A.治疗开始时间越早越好
B.症状改善后停药
C.初始剂量以最小剂量开始
D.间隔1~2周加量一次至症状改善
E.维持量个体差异比较大
B.症状改善后停药
6 5 2011 40. 生长激素替代疗法,下列哪项不对(2.5分) A.开始治疗时年龄越小,效果越好
B.口服给药
C.治疗第一年效果最佳
D.治疗应持续至骨骺愈合为止
E.如同时伴有甲状腺素缺乏治疗时应补充甲状腺素
B.口服给药
6 5 2012 1. 患儿,20天,出生体重4公斤,嗜睡,吃奶少,便秘,查体:皮肤黄染,前囟4*3厘米,心肺听诊无殊,腹略胀,脐疝,肝、脾肋下未及,神经系统(-)。该患儿可能的疾病是: (2.5分) A.新生儿败血症
B.新生儿溶血症
C.先天性甲状腺功能低下症
D.粘多糖1型
E.先天性巨结肠
C.先天性甲状腺功能低下症
6 5 2012 2. 患儿,男,12岁,身高130厘米,学习成绩优秀,父亲身高175厘米,母亲身高160厘米,查体:反应灵活,四肢匀称,心肺听诊无殊,腹平软,肝、脾肋下未及,神经系统(-)。该患儿可能的疾病是 (2.5分) A.家族性矮小
B.21-三体综合症
C.生长激素缺乏症
D.甲状腺功能减低症
E.先天性软骨发育不良
C.生长激素缺乏症
6 5 2012 3. 引起中枢性甲状腺功能低下的病因是 (2.5分) A.甲状腺不发育或发育不全
B.甲状腺激素合成 途径障碍或母体存在抗甲状腺抗体
C.甲状腺激素合成途径障碍
D.促甲状腺素(TSH)缺乏
E.甲状腺或靶器官反应低下
D.促甲状腺素(TSH)缺乏
6 5 2012 4. 确诊先天性甲状腺功能减低症主要依靠下列哪项检查 (2.5分) A.骨骼X线片检查
B.血清T3、 T4 TSH测定
C.血清蛋白结合碘测定
D.染色体检查
E.特殊面容
B.血清T3、 T4 TSH测定
6 5 2012 5. 患儿,男,7岁,身高100厘米,学习良好,查体:反应灵活,躯体长,四肢短,心肺听诊无殊,腹软,肝、脾肋下未及,神经系统(-)。该患儿可能的疾病是 (2.5分) A.21-三体综合症
B.甲状腺功能减低症
C.先天性软骨发育不良
D.生长激素缺乏症
E.粘多糖病
C.先天性软骨发育不良
6 5 2012 6. 引起先天性甲状腺功能减低症最常见的原因(2.5分) A.碘缺乏
B.甲状腺靶器官反应低下
C.甲状腺不发育或发育不全
D.酶缺乏
E.促甲状腺激素缺陷
C.甲状腺不发育或发育不全
6 5 2012 7. 男,10岁,出生体重3200g,身材比例匀称,智力正常,身高110cm,生长速度3cm/年。最可能的诊断是(2.5分) A.性早熟
B.家族性身矮
C.糖尿病
D.生长激素缺乏
E.甲状腺功能低下
D.生长激素缺乏
6 5 2012 8. 甲状腺激素的主要生理作用,下列哪项是错误的(2.5分) A.产热
B.促进蛋白质合成
C.促进中枢神经系统发育
D.促进脂肪分解和利用
E.抑制钙磷在骨质中的合成代谢
E.抑制钙磷在骨质中的合成代谢
6 5 2012 9. 女孩青春期发育顺序为(2.5分) A.乳房发育-腋毛-阴毛-月经-外生殖器改变
B.乳房发育-阴毛-月经-外生殖器改变-腋毛
C.乳房发育-月经-外生殖器改变-腋毛-阴毛
D.乳房发育-外生殖器改变-腋毛-阴毛-月经
E.乳房发育-阴毛-外生殖器改变-月经-腋毛
E.乳房发育-阴毛-外生殖器改变-月经-腋毛
6 5 2012 10. 下列哪一项不是风湿热的主要表现(2.5分) A.血沉增快
B.心脏炎
C.皮下结节
D.环形红斑
E.舞蹈病
A.血沉增快
6 5 2012 11. 川崎病最常见的心血管并发症是(2.5分) A.心包炎
B.冠状动脉损害
C.心律失常
D.心肌炎
E.心内膜炎
B.冠状动脉损害
6 5 2012 12. 下列哪项对川崎病诊断具有特征意义(2.5分) A.持续高热5天以上
B.颈淋巴结肿大
C.多形性皮疹
D.手足硬肿,指、趾端膜状脱皮
E.双球结膜充血,口唇皲裂,杨梅舌
D.手足硬肿,指、趾端膜状脱皮
6 5 2012 13. 川崎病合并冠状动脉瘤服用阿司匹林的时间为(2.5分) A.血清水杨酸浓度达到质量水平
B.血沉正常
C.血小板降至正常
D.长期服用直至动脉瘤消退
E.热退后
D.长期服用直至动脉瘤消退
6 5 2012 14. 可致关节畸形的常见小儿风湿性疾病是(2.5分) A.过敏性紫癜
B.川崎病
C.风湿热
D.幼年特发性关节炎
E.系统性红斑狼疮
D.幼年特发性关节炎
6 5 2012 15. 川崎病最早出现的症状是(2.5分) A.持续高热
B.颈淋巴结肿大
C.多形性皮疹
D.指、趾端脱皮
E.杨梅舌
A.持续高热
6 5 2012 16. 内分泌器官正确的是(2.5分) A.垂体、肾上腺、甲状腺、甲状旁腺、肾脏
B.垂体、肾上腺、甲状腺、子宫、卵巢
C.垂体、肾上腺、 脾脏、甲状旁腺、卵巢
D.垂体、 下丘脑、甲状腺、甲状旁腺、卵巢
E.垂体、肾上腺、甲状腺、甲状旁腺、卵巢
E.垂体、肾上腺、甲状腺、甲状旁腺、卵巢
6 5 2012 17. 有关垂体瘤的描述,错误的是(2.5分) A.属脑内肿瘤,包膜完整。
B.可向上生长突破鞍隔侵及鞍上池。
C.较大肿瘤常因缺血或出血发生中心坏死或囊变。
D.有压迫症状,如视力障碍、垂体功能低下、阳痿等。
E.内分泌亢进的症状
A.属脑内肿瘤,包膜完整。
6 5 2012 18. 肾上腺不常用的影像学成像技术是(2.5分) A.B超
B.MRI检查
C.CT
D.血管造影
E.血管造影+核素显像
E.血管造影+核素显像
6 5 2012 19. 常累及双侧肾上腺的病变是(2.5分) A.肾上腺腺瘤
B.肾上腺出血
C.肾上腺结核
D.神经源性肿瘤
E.髓质脂肪瘤
C.肾上腺结核
6 5 2012 20. 关于嗜铬细胞瘤的描述,错误的是(2.5分) A.多见于14岁以下儿童
B.为圆形或卵圆形肿块
C.即可有实质性肿块,也可有囊性肿块
D.增强扫描后肿瘤强化明显
E.肿瘤大小不能鉴别良恶性
A.多见于14岁以下儿童
6 5 2012 21. 关于肾上腺皮质醇增多症描述不正确的是(2.5分) A.糖皮质激素分泌过多
B.多为肾上腺增生所致,少数为肾上腺皮质腺瘤
C.肾上腺增生可为结节性和弥漫性
D.肾上腺皮质腺瘤多为单侧
E.肾上腺皮质腺瘤瘤体较大,直径多在5cm以上
E.肾上腺皮质腺瘤瘤体较大,直径多在5cm以上
6 5 2012 22. 肾上腺外的嗜铬细胞瘤发生的主要部位是(2.5分) A.肝门旁
B.肾门旁
C.腹主动脉旁
D.后纵膈
E.脊柱旁
C.腹主动脉旁
6 5 2012 23. 对甲状腺结节的影像学检查,下列说法正确的是(2.5分) A.增强CT检查甲状腺结节最有价值
B.MRI检查甲状腺结节最有价值
C.核医学检查甲状腺结节最有价值
D.B超检查甲状腺结节最有价值
E.数字X线摄片(DR)检查甲状腺结节最有价值
D.B超检查甲状腺结节最有价值
6 5 2012 24. 可引起Cushing综合征的是(2.5分) A.肾上腺增生,嗜铬细胞瘤,肾上腺皮质腺瘤
B.肾上腺腺瘤,嗜铬细胞瘤,肾上腺皮质腺瘤
C.肾上腺增生, 肾上腺腺瘤,肾上腺皮质腺瘤
D.肾上腺腺瘤,肾上腺增生,肾上腺髓样脂肪瘤
E.肾上腺腺瘤,转移瘤,囊肿
C.肾上腺增生, 肾上腺腺瘤,肾上腺皮质腺瘤
6 5 2012 25. 女性生殖器官结核首先累及的是 (2.5分) A.子宫体
B.子宫内膜
C.子宫颈
D.子宫腔
E.输卵管
E.输卵管
6 5 2012 26. 有关子宫内膜癌描述错误的是(2.5分) A.联合带的信号改变可作为肌层受侵的标志
B.联合带变薄
C.病变限于子宫内膜时,T1WI和T2WI可为正常
D.联合带于T2WI上的低信号中断
E.联合带于T2WI上的高信号中断
E.联合带于T2WI上的高信号中断
6 5 2012 27. 关于子宫肌瘤的描述,正确的是 (2.5分) A.MRI T1WI上子宫肌瘤的信号高于子宫肌
B.MRI不能检出3mm以下的子宫肌瘤
C.CT增强后子宫肌瘤的密度略高于正常子宫肌的强化
D.MRI T2WI上,典型的子宫肌瘤为明显高信号
E.MRI T2WI上,典型的子宫肌瘤为明显低信号
E.MRI T2WI上,典型的子宫肌瘤为明显低信号
6 5 2012 28. 下列说法正确的是(2.5分) A.大多数前列腺癌发生在中央带
B.大多数前列腺癌发生在周边带
C.大多数前列腺癌发生在移行带
D.大多数前列腺癌发生在尿道周围腺体
E.无一定规律
B.大多数前列腺癌发生在周边带
6 5 2012 29. 下列影像技术对前列腺增生与早期前列腺癌的鉴别诊断价值不大,除了(2.5分) A.CT
B.X线平片
C.超声
D.MRI
E.膀胱造影
D.MRI
6 5 2012 30. 关于前列腺增生的表述,正确的是(2.5分) A.前列腺增生主要发生在周围带
B.前列腺超过耻骨联合上方1cm,可判断为增生
C.T1WI上增大的前列腺呈均一高信号
D.CT上可见前列腺内高密度灶,代表出血
E.以上都错
E.以上都错
6 5 2012 31. 卵巢癌典型的大网膜种植表现为(2.5分) A.呈饼状软组织肿块
B.多发结节状改变
C.网状纤维条索
D.无一定特征
E.以上均不是
A.呈饼状软组织肿块
6 5 2012 32. 下面哪个自身抗体是结缔组织病的标准筛选试验? (2.5分) A.抗核抗体
B.抗Sm抗体
C.抗双链DNA抗体
D.抗SSA抗体
E.抗SSB抗体
A.抗核抗体
6 5 2012 33. 以下哪项不是系统性红斑狼疮光过敏的原因(2.5分) A.紫外线损伤皮肤中的DNA,使DNA变性成为自身抗原
B.紫外线增加抗SSA、SSB和RNP抗体与皮下角蛋白细胞结合
C.紫外线使细胞膜磷脂代谢紊乱,影响炎症
D.紫外线使皮下的角蛋白细胞和朗格汉斯细胞释放炎症介质
E.紫外线使皮下小血管痉挛
E.紫外线使皮下小血管痉挛
6 5 2012 34. 女性病人,28岁。双手掌指关节、近端指间关节疼痛伴月经量增多2年。曾在外院查RF(+),拟类风湿关节炎,治疗效果不佳。查体:面色苍白,浮肿,浅表淋巴结不肿大,心肺(-),肝脾(-)。双手小关节活动无障碍,无畸形,指端可见红斑。查血 WBC3.2×109/L, Hb6.5g/L, PLT54×109/L,网织红细胞1%。尿常规RBC5~8/HP,Pro(++),双手X片未见异常。最可能的诊断是(2.5分) A.风湿性关节炎
B.类风湿关节炎
C.系统性红斑狼疮
D.干燥综合征
E.强直性脊柱炎
C.系统性红斑狼疮
6 5 2012 35. 糖皮质激素通过胎盘时可被灭活,但下面哪种药物除外(2.5分) A.泼尼松
B.甲基泼尼松
C.氢化可的松
D.可的松
E.地塞米松和倍他米松
E.地塞米松和倍他米松
6 5 2012 36. 免疫病理检查几乎所有SLE病人均可出现病变的脏器是(2.5分) A.心脏
B.肾
C.肺
D.肝
E.胰腺
B.肾
6 5 2012 37. 垂体瘤容易出现下列表现,除了(2.5分) A.出血
B.侵犯海绵窦
C.压迫视交叉
D.使垂体柄偏移
E.沿脑脊液种植
E.沿脑脊液种植
6 5 2012 38. 有关宫颈癌的表述,错误的是(2.5分) A.宫颈癌向外侵犯的确切根据是周围脂肪层消失
B.接触性出血是宫颈癌早期的主要症状
C.MRI对肿瘤范围的显示要优于CT
D.宫颈癌早期诊断主要依靠临床检查和液基细胞学检查
E.影像学检查主要用于宫颈癌的定性诊断
E.影像学检查主要用于宫颈癌的定性诊断
6 5 2012 39. 下列哪项不符合原发性生长激素缺乏症(2.5分) A.身材矮小,躯干长、四肢短
B.生长速率<4cm/年
C.智能正常
D.骨化中心发育落后
E.患儿出生时身高和体重均正常
A.身材矮小,躯干长、四肢短
6 5 2012 40. 先天性甲状腺功能减低症的治疗,正确的是(2.5分) A.明确诊断后即应开始治疗
B.药物应大剂量开始
C.持续大剂量冲击治疗
D.应服药5年以上
E.随访5年
A.明确诊断后即应开始治疗
6 5 2014 1. 自身免疫性疾病的特点是 (2.5分) A.体内存在高滴度自身抗体
B.激素和免疫抑制剂治疗有效
C.常有多系统和多器官损害
D.疾病反复,无法治愈
E.以上都是
E.以上都是
6 5 2014 2. 系统性红斑狼疮患者疾病活动时,()抗体特异性高(2.5分) A.抗核抗体
B.抗Sm抗体
C.抗双链DNA抗体
D.抗SSA抗体
E.抗SSB抗体
C.抗双链DNA抗体
6 5 2014 3. 以下()不是系统性红斑狼疮光过敏的原因? (2.5分) A.紫外线损伤皮肤中的DNA,使DNA变性成为自身抗原
B.紫外线增加抗SSA、SSB和RNP抗体与皮下角蛋白细胞结合
C.紫外线使细胞膜磷脂代谢紊乱,影响炎症
D.紫外线使皮下的角蛋白细胞和朗格汉斯细胞释放炎症介质
E.紫外线使皮下小血管痉挛
E.紫外线使皮下小血管痉挛
6 5 2014 4. 女性病人,28岁。双手掌指关节、近端指间关节疼痛伴月经量增多2年。曾在外院查RF(+),拟类风湿关节炎,治疗效果不佳。查体:面色苍白,浮肿,浅表淋巴结不肿大,心肺(-),肝脾(-)。双手小关节活动无障碍,无畸形,指端可见红斑。查血 WBC3.2×109/L, Hb6.5g/L, PLT54×109/L,网织红细胞1%。尿常规RBC5~8/HP,Pro(++),双手X片未见异常。最可能的诊断是 ,(2.5分) A.风湿性关节炎
B.类风湿关节炎
C.系统性红斑狼疮
D.干燥综合征
E.强直性脊柱炎
C.系统性红斑狼疮
6 5 2014 5. 系统性红斑狼疮合并感染发热,与疾病活动所致发热的区别在于(),(2.5分) A.后者ANA滴度较高
B.前者ESR较高
C.后者抗ds-DNA抗体升高
D.后者抗Sm抗体阳性率较高
E.后者尿蛋白(+)
C.后者抗ds-DNA抗体升高
6 5 2014 6. 免疫病理检查几乎所有SLE病人均可出现()病变(2.5分) A.心脏
B.肾
C.肺
D.肝
E.胰腺
B.肾
6 5 2014 7. 妊娠可诱发SLE患者病情疮活动,特别在() 时 (2.5分) A.妊娠早期
B.妊娠中期
C.妊娠晚期
D.妊娠早期和产后6周
E.妊娠晚期和产后6周
D.妊娠早期和产后6周
6 5 2014 8. 系统性红斑狼疮属于() (2.5分) A.Ⅰ型变态反应
B.Ⅱ型变态反应
C.Ⅲ型变态反应
D.Ⅳ型变态反应
E.以上都不是
C.Ⅲ型变态反应
6 5 2014 9. 系统性红斑狼疮可检测到的自身抗体中与习惯性流产有关的是 () ,(2.5分) A.抗RNP抗体
B.抗Sm抗体
C.抗SSA抗体
D.抗ACL抗体
E.抗ds-DNA抗体
D.抗ACL抗体
6 5 2014 10. 目前显示垂体病变最好的影像学检查方法是:(2.5分) A.头颅平片
B.CT
C.核素显像
D.MRI
E.超声
D.MRI
6 5 2014 11. 脑干不包括: (2.5分) A.脑桥
B.延髓
C.中脑
D.四叠体
E.端脑
E.端脑
6 5 2014 12. 关于垂体腺瘤的说法,以下哪种是错误的:(2.5分) A.大腺瘤可突出到鞍上池
B.当腺瘤的直径小于1厘米时,被称为微腺瘤
C.腺瘤可以呈哑铃状
D.大腺瘤不会导致脑积水
E.腺瘤可以侵犯海绵窦
D.大腺瘤不会导致脑积水
6 5 2014 13. 鞍上池内不包括:(2.5分) A.垂体柄
B.视束
C.丘脑
D.颈内动脉
E.视交叉
C.丘脑
6 5 2014 14. 肾上腺皮质最常见的功能性肿瘤是:(2.5分) A.嗜铬细胞瘤
B.髓质脂肪瘤
C.皮质腺瘤
D.神经母细胞瘤
E.肾上腺癌
C.皮质腺瘤
6 5 2014 15. 常累及双侧肾上腺的病变是:(2.5分) A.肾上腺腺瘤
B.肾上腺出血
C.肾上腺结核
D.神经源性肿瘤
E.髓质脂肪瘤
C.肾上腺结核
6 5 2014 16. 关于嗜铬细胞瘤的描述,错误的是:(2.5分) A.多见于14岁以下儿童
B.为圆形或卵圆形肿块
C.即可有实质性肿块,也可有囊性肿块
D.增强扫描后肿瘤强化明显
E.肿瘤大小不能鉴别良恶性
A.多见于14岁以下儿童
6 5 2014 17. 嗜铬细胞瘤被称为10%肿瘤,下列叙述中错误的是:(2.5分) A.10%发生于肾上腺外
B.10%为双侧
C.10%为多发
D.10%为恶性
E.10%为女性
E.10%为女性
6 5 2014 18. 肾上腺转移瘤最常见的原发肿瘤是:(2.5分) A.乳腺癌
B.肝癌
C.胰腺癌
D.胆囊癌
E.肺癌
E.肺癌
6 5 2014 19. 可引起Cushing综合征的是:(2.5分) A.肾上腺增生,嗜铬细胞瘤,肾上腺皮质腺瘤
B.肾上腺腺瘤,嗜铬细胞瘤,肾上腺皮质腺瘤
C.肾上腺增生, 肾上腺腺瘤,肾上腺皮质腺瘤
D.肾上腺腺瘤,肾上腺增生,肾上腺髓样脂肪瘤
E.肾上腺腺瘤,转移瘤,囊肿
C.肾上腺增生, 肾上腺腺瘤,肾上腺皮质腺瘤
6 5 2014 20. 发现和诊断子宫肌瘤最不敏感的影像学检查方法是:(2.5分) A.CT增强
B.X线
C.MRI
D.超声
E.CT平扫
B.X线
6 5 2014 21. 关于子宫内膜异位症的描述,错误的是:(2.5分) A.分外在性和内在性
B.子宫腺肌症是内在性子宫内膜异位
C.子宫内膜异位在卵巢表面,可形成巧克力囊肿
D.子宫内膜可异位在大肠
E.MRI主要表现是T2WI上低信号联合带的变薄
E.MRI主要表现是T2WI上低信号联合带的变薄
6 5 2014 22. 有关宫颈癌的表述,错误的是:(2.5分) A.宫颈癌向外侵犯的确切根据是周围脂肪层消失
B.接触性出血是宫颈癌早期的主要症状
C.MRI对肿瘤范围的显示要优于CT
D.影像学检查主要用于宫颈癌的定性诊断
E.宫颈癌早期诊断主要依靠临床检查和液基细胞学检查
D.影像学检查主要用于宫颈癌的定性诊断
6 5 2014 23. 最常见的子宫肌瘤类型是:(2.5分) A.粘膜下肌瘤
B.肌壁间肌瘤
C.浆膜下肌瘤
D.宫颈肌瘤
E.阔韧带肌瘤
B.肌壁间肌瘤
6 5 2014 24. 关于子宫联合带的描述,正确的是:(2.5分) A.MRI T2WI上为高信号
B.代表子宫肌内层
C.代表子宫肌外层
D.代表子宫内膜
E.代表子宫分泌物
B.代表子宫肌内层
6 5 2014 25. 我国女性生殖系统最常见的恶性肿瘤是:(2.5分) A.子宫颈癌
B.子宫癌
C.盆腔癌
D.卵巢癌
E.阴道癌
A.子宫颈癌
6 5 2014 26. 前列腺癌血行转移最常发生在:(2.5分) A.肺部
B.肝脏
C.骨骼
D.肾脏
E.脑部
C.骨骼
6 5 2014 27. 良性前列腺增生主要发生在:(2.5分) A.移行带
B.中央带
C.周边带
D.前肌纤维质
E.各部位发生率无差别
A.移行带
6 5 2014 28. 关于正常前列腺的中央带与外周带的MRI表述,正确的是:(2.5分) A.T2WI上两部分均为高信号
B.T2WI上两部分均为低信号
C.中央带为等信号,外周带为低信号
D.中央带为高信号,外周带为低信号
E.中央带为低信号,外周带为高信号
E.中央带为低信号,外周带为高信号
6 5 2014 29. 川崎病时皮肤粘膜表现中最具有特征性的改变是:(2.5分) A.皮疹
B.指(趾)端变化
C.眼球结膜充血
D.杨梅舌
E.唇红干燥、皲裂
B.指(趾)端变化
6 5 2014 30. 川崎病最常见的心血管并发症是:(2.5分) A.心包炎
B.冠状动脉损害
C.心律失常
D.心肌炎
E.心内膜炎
B.冠状动脉损害
6 5 2014 31. 下列哪项对川崎病诊断具有特征意义:(2.5分) A.持续高热5天以上
B.颈淋巴结肿大
C.多形性皮疹
D.手足硬肿,指、趾端膜状脱皮
E.双球结膜充血,口唇皲裂,杨梅舌
D.手足硬肿,指、趾端膜状脱皮
6 5 2014 32. 决定川崎病预后的临床表现为:(2.5分) A.心肌炎
B.心包炎
C.关节痛
D.无菌性脑膜炎
E.冠状动脉病变
E.冠状动脉病变
6 5 2014 33. 下列哪一项不是风湿热的主要表现:(2.5分) A.血沉增快
B.心脏炎
C.皮下结节
D.环形红斑
E.舞蹈病
A.血沉增快
6 5 2014 34. 风湿热关节炎的特点:(2.5分) A.最终转为慢性关节炎
B.关节畸形
C.累及小关节为主
D.经治疗后可完全痊愈,不留畸形
E.晨僵
D.经治疗后可完全痊愈,不留畸形
6 5 2014 35. 决定过敏性紫癜预后的临床表现为:(2.5分) A.皮肤紫癜
B.便血
C.关节痛
D.鼻出血
E.肾脏病变
E.肾脏病变
6 5 2014 36. 过敏性紫癜的皮疹特点有:(2.5分) A.全身散在针尖大小紫癜,压之不退色
B.全身出现大块瘀斑,压不退色
C.下肢、臀部出现分布对称红疹,压不退色
D.全身出现风团样皮疹,压之退色
E.大关节出现大块瘀斑,伴功能障碍,压不退色
C.下肢、臀部出现分布对称红疹,压不退色
6 5 2014 37. 可致关节畸形的常见小儿风湿性疾病是:(2.5分) A.过敏性紫癜
B.川崎病
C.风湿热
D.幼年特发性关节炎
E.系统性红斑狼疮
D.幼年特发性关节炎
6 5 2014 38. 引起中枢性甲状腺功能低下的病因是: (2.5分) A.甲状腺不发育或发育不全
B.甲状腺激素合成 途径障碍或母体存在抗甲状腺抗体
C.甲状腺激素合成途径障碍
D.促甲状腺素(TSH)缺乏
E.甲状腺或靶器官反应低下
D.促甲状腺素(TSH)缺乏
6 5 2014 39. 确诊先天性甲状腺功能减低症主要依靠下列哪项检查(2.5分) A.骨骼X线片检查
B.血清T3、 T4 TSH测定
C.血清蛋白结合碘测定
D.染色体检查
E.特殊面容
B.血清T3、 T4 TSH测定
6 5 2014 40. 男,10岁,出生体重3200g,身材比例匀称,智力正常,身高110cm,生长速度3cm/年。最可能的诊断是(2.5分) A.性早熟
B.家族性身矮
C.糖尿病
D.生长激素缺乏
E.甲状腺功能低下
D.生长激素缺乏
6 5 2015 1. 自身免疫性疾病的特点是 (3.0分) A.体内存在高滴度自身抗体
B.激素和免疫抑制剂治疗有效
C.常有多系统和多器官损害
D.疾病反复,无法治愈
E.以上都是
E.以上都是
6 5 2015 2. 系统性红斑狼疮患者疾病活动时, ()抗体特异性高(3.0分) A.抗核抗体
B.抗Sm抗体
C.抗双链DNA抗体
D.抗SSA抗体
E.抗SSB抗体
C.抗双链DNA抗体
6 5 2015 3. 以下()不是系统性红斑狼疮光过敏的原因? (4.0分) A.紫外线损伤皮肤中的DNA,使DNA变性成为自身抗原
B.紫外线增加抗SSA、SSB和RNP抗体与皮下角蛋白细胞结合
C.紫外线使细胞膜磷脂代谢紊乱,影响炎症
D.紫外线使皮下的角蛋白细胞和朗格汉斯细胞释放炎症介质
E.紫外线使皮下小血管痉挛
E.紫外线使皮下小血管痉挛
6 5 2015 4. 女性病人,28岁。双手掌指关节、近端指间关节疼痛伴月经量增多2年。曾在外院查RF(+),拟类风湿关节炎,治疗效果不佳。查体:面色苍白,浮肿,浅表淋巴结不肿大,心肺(-),肝脾(-)。双手小关节活动无障碍,无畸形,指端可见红斑。查血 WBC3.2×109/L, Hb6.5g/L, PLT54×109/L,网织红细胞1%。尿常规RBC5~8/HP,Pro(++),双手X片未见异常。最可能的诊断是(3.0分) A.风湿性关节炎
B.类风湿关节炎
C.系统性红斑狼疮
D.干燥综合征
E.强直性脊柱炎
C.系统性红斑狼疮
6 5 2015 5. 系统性红斑狼疮合并感染发热,与疾病活动所致发热的区别在于,(3.0分) A.后者ANA滴度较高
B.前者ESR较高
C.后者抗ds-DNA抗体升高
D.后者抗Sm抗体阳性率较高
E.后者尿蛋白(+)
C.后者抗ds-DNA抗体升高
6 5 2015 6. 免疫病理检查几乎所有SLE病人均可出现()病变(3.0分) A.心脏
B.肾
C.肺
D.肝
E.胰腺
B.肾
6 5 2015 7. 妊娠可诱发SLE患者病情疮活动,特别在()时, (4.0分) A.妊娠早期
B.妊娠中期
C.妊娠晚期
D.妊娠早期和产后6周
E.妊娠晚期和产后6周
D.妊娠早期和产后6周
6 5 2015 8. 系统性红斑狼疮属于(),(3.0分) A.Ⅰ型变态反应
B.Ⅱ型变态反应
C.Ⅲ型变态反应
D.Ⅳ型变态反应
E.以上都不是
C.Ⅲ型变态反应
6 5 2015 9. 女性病人,30岁,系统性红斑狼疮4年。现自感无异常症状和体征,查血三系正常范围,尿常规:蛋白(-),隐血(-)。ESR10mm/L,ANA1:160(+),抗RNP抗体(+),余抗体阴性。该病人的病情活动度是 (4.0分) A.病情未稳定
B.病情活动
C.轻度活动
D.病情稳定
E.以上都不是
D.病情稳定
6 5 2015 10. 目前显示垂体病变最好的影像学检查方法是(3.0分) A.头颅平片
B.CT
C.核素显像
D.MRI
E.超声
D.MRI
6 5 2015 11. 垂体微腺瘤的MRI直接征象是(4.0分) A.强化的垂体中局限性的低信号灶
B.垂体高度的增加
C.垂体柄偏移
D.鞍底的破坏
E.垂体上缘局部或偏侧膨隆
A.强化的垂体中局限性的低信号灶
6 5 2015 12. 关于垂体腺瘤的说法,以下哪种是错误的:(4.0分) A.大腺瘤可突出到鞍上池
B.当腺瘤的直径小于1厘米时,被称为微腺瘤
C.腺瘤可以呈哑铃状
D.大腺瘤不会导致脑积水
E.腺瘤可以侵犯海绵窦
D.大腺瘤不会导致脑积水
6 5 2015 13. 肾上腺最佳的影像学手段是:(3.0分) A.腹部平片
B.MRI 检查
C.血管造影
D.CT
E.核素显像
D.CT
6 5 2015 14. 肾上腺髓质最常见的功能性肿瘤是(3.0分) A.嗜铬细胞瘤
B.髓质脂肪瘤
C.皮质腺瘤
D.神经母细胞瘤
E.肾上腺瘤肾上腺癌
A.嗜铬细胞瘤
6 5 2015 15. 肾上腺单发肿块,MRI检查在同相位、反相位上信号改变明显,反相位上信号强度明显下降,可能的诊断是(3.0分) A.转移瘤
B.肾上腺腺瘤
C.肾上腺腺癌
D.嗜铭细胞瘤
E.肾上腺腺转移瘤
B.肾上腺腺瘤
6 5 2015 16. Graves 病的叙述不正确的是(4.0分) A.常见的自身免疫性疾病
B.以甲状腺肿大和突眼为特征性表现
C.常有甲状腺功能降低
D.CT增强检查不宜采用
E.MRI检查可显示眼外肌肌腹肥大
C.常有甲状腺功能降低
6 5 2015 17. MRI矢状面显示子宫前壁向外局限性隆起的是(3.0分) A.子宫体癌
B.子宫腺肌症
C.子宫浆膜下肌瘤
D.子宫粘膜下肌瘤
E.子宫先天性畸形
C.子宫浆膜下肌瘤
6 5 2015 18. CT对于诊断前列腺癌的主要意义在于(4.0分) A.早期诊断
B.与良性前列腺增生肥大鉴别
C.确定晚期病变范围
D.确定肿瘤是否穿破包膜
E.判断有无骨转移
C.确定晚期病变范围
6 5 2015 19. 正常子宫输卵管造影表现,哪一项叙述错误?(3.0分) A.造影剂一般用碘剂
B.常用来寻找不孕症的原因
C.子宫呈倒置三角形影
D.输卵管峡部呈细线状,壶腹部及伞端膨大
E.延迟摄片,造影剂不能进入腹腔
E.延迟摄片,造影剂不能进入腹腔
6 5 2015 20. 关于前列腺增生的叙述,哪一项正确?(3.0分) A.前列腺增生又称前列腺癌
B.好发于青年人,常有血尿及尿痛
C.平片可显示肥大的前列腺
D.造影见膀胱底有边缘光整弧形压迹
E.膀胱较小呈圆形,边界光滑
D.造影见膀胱底有边缘光整弧形压迹
6 5 2015 21. 前列腺癌MRI表现,叙述错误的是(4.0分) A.癌结节多位于边缘区
B.T1WI呈低信号,T2WI呈高信号
C.诊断主要靠 T1WI
D.诊断主要靠T2WI
E.增强扫描癌结节轻度强化
C.诊断主要靠 T1WI
6 5 2015 22. 川崎病时皮肤粘膜表现中最具有特征性的改变是(3.0分) A.皮疹
B.指(趾)端变化
C.眼球结膜充血
D.杨梅舌
E.唇红干燥、皲裂
B.指(趾)端变化
6 5 2015 23. 川崎病最常见的心血管并发症是(4.0分) A.心包炎
B.冠状动脉损害
C.心律失常
D.心肌炎
E.心内膜炎
B.冠状动脉损害
6 5 2015 24. 下列哪项对川崎病诊断具有特征意义(3.0分) A.持续高热5天以上
B.颈淋巴结肿大
C.多形性皮疹
D.手足硬肿,指、趾端膜状脱皮
E.双球结膜充血,口唇皲裂,杨梅舌
D.手足硬肿,指、趾端膜状脱皮
6 5 2015 25. 川崎病合并冠状动脉瘤服用阿司匹林的时间为(4.0分) A.血清水杨酸浓度达到治疗水平
B.血沉正常
C.血小板降至正常
D.长期服用直至动脉瘤消退
E.热退后
D.长期服用直至动脉瘤消退
6 5 2015 26. 下列哪一项不是风湿热的主要表现(3.0分) A.血沉增快
B.心脏炎
C.皮下结节
D.环形红斑
E.舞蹈病
A.血沉增快
6 5 2015 27. 风湿热关节炎的特点(3.0分) A.最终转为慢性关节炎
B.关节畸形
C.累及小关节为主
D.经治疗后可完全痊愈,不留畸形
E.晨僵
D.经治疗后可完全痊愈,不留畸形
6 5 2015 28. 决定过敏性紫癜预后的临床表现为(3.0分) A.皮肤紫癜
B.便血
C.关节痛
D.鼻出血
E.肾脏病变
E.肾脏病变
6 5 2015 29. 过敏性紫癜的皮疹特点有(3.0分) A.全身散在针尖大小紫癜,压之不退色
B.全身出现大块瘀斑,压不退色
C.下肢、臀部出现分布对称红疹,压不退色
D.全身出现风团样皮疹,压之退色
E.大关节出现大块瘀斑,伴功能障碍,压不退色
C.下肢、臀部出现分布对称红疹,压不退色
6 5 2015 30. 可致关节畸形的常见小儿风湿性疾病是(3.0分) A.过敏性紫癜
B.川崎病
C.风湿热
D.幼年特发性关节炎
E.系统性红斑狼疮
D.幼年特发性关节炎
6 6 2006 1. 上皮性卵巢癌的特征性肿瘤标志物为(2.0分) A.CA125
B.HCG
C.雌激素
D.AFP
E.HPL
A.CA125
6 6 2006 2. The primary mode of treatment for endometrial carcinoma confined to the uterine corpus is(2.0分) A.Hysterectomy
B.External beam radiation
C.Chemotherapy
D.Intracavitary radium
E.Progestin therapy
A.Hysterectomy
6 6 2006 3. 患者63岁,主诉血性白带,妇检:子宫颈无糜烂宫体稍大,后穹窿分泌物涂片检查找到癌细胞。进一步处理最佳:(2.0分) A.分段诊断性刮宫
B.碘试验
C.阴道镜检查
D.B超检查
E.宫颈荧光检查
A.分段诊断性刮宫
6 6 2006 4. Engagement is best defined as which of the following?(2.0分) A.when the greatest biparietal diameter of the fetal head passes the pelvic inlet
B.when the presenting part goes through the pelvic inlet
C.when the presenting part is level below the ischial spines
D.when the greatest biparietal diameter of the head is level with the ischial spines
E.when the greatest diameter of the fetal presenting part passes through the narrowest and lowest part of the maternal pelvis.
A.when the greatest biparietal diameter of the fetal head passes the pelvic inlet
6 6 2006 5. 子宫内膜癌的主要转移途径是(2.0分) A.淋巴转移
B.直接蔓延
C.血行转移
D.种植转移
E.自发性转移
B.直接蔓延
6 6 2006 6. A patient wishes you to explain the concept of cervical intraepithelial neoplasia (CIN) III, which has been diagnosed from her cervical biopsy after a low-grade squamous intraepithelial lesion (LGSIL) was found on a Pap smear. What can you correctly tell her about CIN III? (2.0分) A.It is due to a bacterial infection.
B.It requires no further treatment.
C.It is an invasive cancer.
D.It includes carcinoma in situ (CIS).
E.It corresponds to the LGSIL shown on her prior Pap smear.
D.It includes carcinoma in situ (CIS).
6 6 2006 7. 梅格斯综合征的特征为(2.0分) A.肿瘤切除后不会自行消失
B.与颗粒细胞瘤相关
C.预后差
D.伴有胸腔积液或腹水
E.为肿瘤转移的表现
D.伴有胸腔积液或腹水
6 6 2006 8. Of the following laboratory studies, which would not be done routinely in a pregnant patient? (2.0分) A.Diabetes screen
B.VDRL (Venereal Disease Research Laboratory)
C.Urinalysis
D.Blood estriol
E.Blood type (ABO and RH)
D.Blood estriol
6 6 2006 9. A pregnant 35-year-old patient is at highest risk for the concurrent development of which of the following malignancies? (2.0分) A.Vagina
B.Cervix
C.Breast
D.Ovary
E.Colon
B.Cervix
6 6 2006 10. CIN的确诊方法:(2.0分) A.阴道经检查
B.宫颈刮片细胞学检查
C.碘试验
D.B超检查
E.宫颈活组织检查
E.宫颈活组织检查
6 6 2006 11. 女性患者,65岁,长期外阴瘙痒,久治不愈,为明确诊断,可行: (2.0分) A.外阴HPV检查
B.阴道镜检查
C.外阴活检
D.碘试验
E.药物试验性治疗
C.外阴活检
6 6 2006 12. I期外阴癌的转移方式主要是: (2.0分) A.淋巴转移
B.直接转移
C.血行转移
D.种植转移
E.直接转移和淋巴转移
E.直接转移和淋巴转移
6 6 2006 13. Most ovarian neoplasms are derived from (2.0分) A.mesenchyma
B.ovarian germ cell
C.ovarian stroma
D.surface epithelium of ovary
E.connective tissue
D.surface epithelium of ovary
6 6 2006 14. An 88 year-old nursing home resident is brought in for evaluation of blood found in her diapers. She has a history lichen sclerosis of the vulva, which was diagnosed fifteen years ago. She quit smoking in her fifties. Examination of the external genitalia reveals an elevated, firm, erythematous, ulcerated lesion arising from the left labia, measuring 2.5 cm in greatest dimension. What is the most likely diagnosis in this patient? (2.0分) A.Squamous cell carcinoma
B.Malignant melanoma
C.Lichen sclerosus
D.Paget's disease
E.Verrucous carcinoma
A.Squamous cell carcinoma
6 6 2006 15. 完全性葡萄胎的染色体核型是(2.0分) A.单倍体
B.三倍体
C.二倍体
D.四倍体
E.五倍体
C.二倍体
6 6 2006 16. 关于人工破膜的描述,下列错误的是:(2.0分) A.破膜应在宫缩期进行
B.破膜时必须检查有无脐带先露
C.宫口扩张3cm或3cm以上可行人工破膜
D.破膜后术者手指应停留在阴道内,经过1~2次宫缩,待胎头入盆后,再将手指取出
E.无头盆不称、胎头已衔接者,可行人工破膜
A.破膜应在宫缩期进行
6 6 2006 17. A 45 year-old G5P5 premenopausal woman was initially seen in your office for work-up and evaluation of a FIGO stage 2 endometrial cancer that was diagnosed by her gynecologist. Which of the following is the most appropriate treatment for this patient? (2.0分) A.Vaginal hysterectomy with bilateral salpingo-oophorectomy
B.Total laparoscopic hysterectomy with bilateral salpingo-oophorectomy
C.Total abdominal hysterectomy, bilateral salpingo-oophorectomy, bilateral pelvic and para-aortic lymphadenectomy, pelvic washings
D.Supracervical abdominal hysterectomy with ovarian preservation
E.Megace
C.Total abdominal hysterectomy, bilateral salpingo-oophorectomy, bilateral pelvic and para-aortic lymphadenectomy, pelvic washings
6 6 2006 18. The LMP was June 30. The expected date of confinement (EDC) is approximately which of the following? (2.0分) A.23-三月
B.7-四月
C.23-四月
D.28-三月
E.7-三月
B.7-四月
6 6 2006 19. Granulosa cell tumor could secrete (2.0分) A.estrogen
B.progesterone
C.HCG
D.FSH
E.testosterone
A.estrogen
6 6 2006 20. The bispinous diameter of a normal pelvis should be at least how many centimeters?(2.0分) A.8cm
B.5 cm
C.10cm
D.11cm
E.12cm
C.10cm
6 6 2006 21. 鉴别侵葡和绒癌的组织学特点(2.0分) A.病灶为单个或多个
B.镜下滋养细胞的增生程度
C.宫腔内有无原发病灶
D.镜下是否能见到绒毛结构或退化的绒毛阴影
E.病灶中是否存在出血坏死
D.镜下是否能见到绒毛结构或退化的绒毛阴影
6 6 2006 22. A 25-year-old G2P1001 woman at 41 weeks' gestation is noted to change her cervix from 6cm to 9cm over 2 hours.Which of the following is the most likely diagnosis? (2.0分) A.Normal labor
B.Prolonged active phase
C.Prolonged latent phase
D.Arrest of active phase
E.Arrest of descent
A.Normal labor
6 6 2006 23. 属于卵巢良性肿瘤的是(2.0分) A.睾丸母细胞瘤
B.颗粒细胞瘤
C.内胚窦瘤
D.纤维瘤
E.无性细胞瘤
D.纤维瘤
6 6 2006 24. 卵巢上皮性恶性肿瘤的治疗原则为(2.0分) A.化疗与放疗有同等重要的地位
B.手术为主,辅以化疗的综合治疗
C.化疗的主要药物为环磷酰胺
D.化疗通常应用3~4疗程
E.化疗为主,手术为辅助手段
B.手术为主,辅以化疗的综合治疗
6 6 2006 25. 以下哪种情况不需手术治疗:(2.0分) A.子宫粘膜下肌瘤1*1cm大小,伴经量增多症状
B.子宫前壁浆膜下肌瘤约5*4cm大小,伴尿频、尿急症状
C.子宫多发肌瘤,子宫增大如孕3月,血红蛋白76g/L
D.丈夫患无精症的不孕患者B超检查发现子宫肌壁间约2*2cm大小肌瘤
E.半年内子宫肌瘤自3cm增大至6cm
D.丈夫患无精症的不孕患者B超检查发现子宫肌壁间约2*2cm大小肌瘤
6 6 2006 26. 正常分娩时最主要的产力是(2.0分) A.腹肌收缩力
B.肛提肌收缩力
C.子宫收缩力
D.膈肌收缩力
E.骨骼肌收缩力
C.子宫收缩力
6 6 2006 27. 完全性葡萄胎的染色体基因组来源于(2.0分) A.父系和母系
B.母系
C.父系
D.母系和子系
E.父系和子系
C.父系
6 6 2006 28. 晚期卵巢癌最适宜的治疗是(2.0分) A.广泛全子宫切除术
B.盆腔淋巴结清扫术
C.子宫次全切
D.肿瘤细胞减灭术辅以化疗
E.放疗
D.肿瘤细胞减灭术辅以化疗
6 6 2006 29. 患者出现一次同房后出血,最不可能的疾病是:(2.0分) A.乳头状瘤
B.输卵管癌
C.宫颈癌
D.宫颈糜烂
E.子宫内膜癌转移到宫颈
B.输卵管癌
6 6 2006 30. Gestational trophoblastic neoplasia is mostly preceded by (2.0分) A.biochemical pregnancy
B.pregnancy at term
C.hydatidiform mole
D.abortion
E.ectopic pregnancy
C.hydatidiform mole
6 6 2006 31. 以下哪项不是子宫内膜异位症好发部位(2.0分) A.子宫直肠陷凹
B.卵巢
C.输卵管
D.子宫骶骨韧带
E.阴道壁
E.阴道壁
6 6 2006 32. 外阴癌的主要治疗是: (2.0分) A.手术治疗
B.激光治疗
C.化学治疗
D.放射治疗
E.物理治疗
A.手术治疗
6 6 2006 33. 卵巢肿瘤最常见的并发症是(2.0分) A.破裂
B.出血
C.蒂扭转
D.感染
E.恶变
C.蒂扭转
6 6 2006 34. 对于妊娠滋养细胞肿瘤治疗结束后随访,正确的是(2.0分) A.每半年随访一次至5年
B.随访期间应严格避孕至少12个月
C.随访期间可选择口服避孕药、宫内节育器或避孕套避孕
D.第一次随访为出院后1周
E.随访内容仅为血β-HCG即可
B.随访期间应严格避孕至少12个月
6 6 2006 35. 外阴癌预后与下列哪项无关:(2.0分) A.瘙痒程度
B.细胞分化程度
C.病变部位
D.有无淋巴结转移
E.病灶大小
A.瘙痒程度
6 6 2006 36. 女性,29岁,无明显诱因下出现持续少量阴道出血1月半。5年前患者有葡萄胎病史,本次行诊刮病理报告结果是:见滋养细胞增生活跃,未见绒毛结构,最可能的诊断是(2.0分) A.侵葡
B.绒癌
C.不全流产
D.重复性葡萄胎
E.滋养细胞内膜炎
B.绒癌
6 6 2006 37. 关于子宫内膜异位症哪项是错误的(2.0分) A.子宫内膜出现和生长在子宫肌层
B.发病与卵巢的周期性变化有关
C.多见于生育年龄妇女,以30-40岁居多
D.具有类似恶性肿瘤的远处转移和种植能力
E.一般极少发生恶变
A.子宫内膜出现和生长在子宫肌层
6 6 2006 38. 女性,25岁,已婚。停经50余天,阴道不规则流血2周,妇科检查:宫颈光,子宫增大如孕3个多月,附件无殊,尿HCG(+),B超呈"落雪状图像"改变。最可能的诊断是(2.0分) A.葡萄胎
B.子宫肌瘤
C.子宫内膜癌
D.子宫内膜增殖症
E.子宫内膜炎
A.葡萄胎
6 6 2006 39. I期子宫内膜癌患者应行:(2.0分) A.筋膜外全子宫切除术及双侧附件切除术
B.放射治疗
C.广泛子宫切除术及双侧盆腔淋巴结及腹主动脉旁淋巴结清扫术
D.孕激素治疗
E.化疗
A.筋膜外全子宫切除术及双侧附件切除术
6 6 2006 40. 子宫肌瘤继发贫血最常见于(2.0分) A.肌瘤囊性变性
B.粘膜下肌瘤
C.浆膜下肌瘤
D.肌瘤红色变性
E.肌壁间子宫肌瘤
B.粘膜下肌瘤
6 6 2006 41. 子宫肌瘤的症状与下述何项关系密切(2.0分) A.肌瘤生长的部位(宫体或宫颈)
B.肌瘤与肌层的关系(粘膜下、浆膜下、肌壁间)
C.肌瘤的大小
D.发生年龄
E.肌瘤的数目
B.肌瘤与肌层的关系(粘膜下、浆膜下、肌壁间)
6 6 2006 42. 宫颈癌中那种癌所占比例最多:(2.0分) A.腺癌
B.鳞状细胞癌
C.宫颈恶性腺癌
D.粘液腺癌
E.鳞腺癌
B.鳞状细胞癌
6 6 2006 43. 子宫肌瘤红色样变常发生在(2.0分) A.妊娠期
B.月经期
C.青春期
D.围绝经期
E.绝经后期
A.妊娠期
6 6 2006 44. 关于子宫肌瘤的手术治疗,下述哪项是不正确的(2.0分) A.近绝经期,子宫如孕2个月大小,症状轻者,可暂不手术
B.突出于阴道内的粘膜下肌瘤应行手术治疗
C.肌瘤症状明显,经保守治疗无效者应考虑手术
D.年轻,未生育过的妇女肌壁间肌瘤不论多发或单发,均应手术
E.疑有恶变者不宜行剔除术
D.年轻,未生育过的妇女肌壁间肌瘤不论多发或单发,均应手术
6 6 2006 45. 畸胎瘤的良恶性的决定因素为(2.0分) A.肿瘤质地
B.患者年龄
C.组织来源
D.组织的分化程度
E.肿瘤的大小
D.组织的分化程度
6 6 2006 46. The most common histological subtype of germ cell tumors is (2.0分) A.embryonal tumors
B.dysgerminoma
C.endodermal sinus tumors
D.teratoma
E.nongestational choriocaecinomas
D.teratoma
6 6 2006 47. 子宫肌瘤短期内迅速增大或伴有阴道流血应考虑(2.0分) A.感染
B.脂肪样变
C.红色变性
D.囊性变
E.肉瘤样变
E.肉瘤样变
6 6 2006 48. 关于子宫内膜异位症的预防,哪项是错误的(2.0分) A.避免手术操作时引起内膜种植
B.月经期应避免不必要的盆腔检查
C.防止经血倒流
D.人工流产时不要突然降低负压
E.输卵管造影术,应在月经前期进行
E.输卵管造影术,应在月经前期进行
6 6 2006 49. 33岁妇女,婚后7年未孕,痛经逐年加重,妇科检查:宫骶韧带处可触及黄豆大小结节2个,触痛明显,右附件可及一5*6cm大小囊性块,活动差,最有效的确诊方案:(2.0分) A.诊断性刮宫
B.腹腔镜手术
C.B超
D.宫腔镜检查
E.血CA125
B.腹腔镜手术
6 6 2006 50. GnRH-a治疗子宫内膜异位症的机理:(2.0分) A.促进垂体促性腺激素分泌,导致卵巢分泌性激素增多
B.抑制垂体促性腺激素,导致卵巢分泌性激素减少
C.抑制垂体促性腺激素,导致卵巢分泌性激素增多
D.促进垂体促性腺激素分泌,导致卵巢分泌性激素减少
E.以上都不是
B.抑制垂体促性腺激素,导致卵巢分泌性激素减少
6 6 2007 1. 子宫肌瘤临床分类标准为(2.0分) A.按临床症状严重程度分类
B.按查体所见子宫肌瘤大小分类
C.按子宫肌瘤的数目分类
D.按子宫肌瘤与子宫内膜的关系分类
E.按肌瘤与子宫肌层的关系分类
E.按肌瘤与子宫肌层的关系分类
6 6 2007 2. 子宫肌瘤的症状与下述何项关系密切(2.0分) A.肌瘤的大小
B.肌瘤生长的部位(宫体或宫颈)
C.发生年龄
D.肌瘤与肌层的关系(粘膜下、浆膜下、肌壁间)
E.肌瘤的数目
D.肌瘤与肌层的关系(粘膜下、浆膜下、肌壁间)
6 6 2007 3. 子宫肌瘤最常见的并发症是(2.0分) A.红色变性
B.继发贫血
C.浆膜下肌瘤蒂扭转
D.肌瘤压迫输尿管引起肾盂积水
E.肌瘤恶变
B.继发贫血
6 6 2007 4. 子宫肌瘤短期内迅速增大或伴有阴道流血应考虑(2.0分) A.感染
B.囊性变
C.红色变性
D.脂肪样变
E.肉瘤样变
E.肉瘤样变
6 6 2007 5. 关于子宫肌瘤的症状下述哪项是错误的 (2.0分) A.肌壁间肌瘤均可引起月经过多
B.黏膜下肌瘤出现症状较早
C.肌瘤一般不引起疼痛
D.不孕的原因之一是由于子宫腔变形,输卵管入口受阻
E.白带增多的原因之一是子宫腔面积增大,腺体分泌过多
A.肌壁间肌瘤均可引起月经过多
6 6 2007 6. 以下哪种情况不需手术治疗:(2.0分) A.丈夫患无精症的不孕患者B超检查发现子宫肌壁间约2*2cm大小肌瘤
B.子宫前壁浆膜下肌瘤约5*4cm大小,伴尿频、尿急症状
C.子宫多发肌瘤,子宫增大如孕3月,血红蛋白76g/L
D.子宫粘膜下肌瘤1*1cm大小,伴经量增多症状
E.半年内子宫肌瘤自3cm增大至6cm
A.丈夫患无精症的不孕患者B超检查发现子宫肌壁间约2*2cm大小肌瘤
6 6 2007 7. 关于子宫内膜异位症哪项是错误的(2.0分) A.多见于生育年龄妇女,以30-40岁居多
B.发病与卵巢的周期性变化有关
C.子宫内膜出现和生长在子宫肌层
D.具有类似恶性肿瘤的远处转移和种植能力
E.一般极少发生恶变
C.子宫内膜出现和生长在子宫肌层
6 6 2007 8. 子宫内膜异位症最主要的临床特点是(2.0分) A.月经失调
B.不孕症发生率高
C.痛经和持续性下腹痛
D.咳血
E.腹痛,腹泻或便秘
C.痛经和持续性下腹痛
6 6 2007 9. 33岁妇女,婚后7年未孕,痛经逐年加重,妇科检查:宫骶韧带处可触及黄豆大小结节2个,触痛明显,右附件可及一5*6cm大小囊性块,活动差,最有效的确诊方案:(2.0分) A.B超
B.诊断性刮宫
C.宫腔镜检查
D.腹腔镜手术
E.血CA125
D.腹腔镜手术
6 6 2007 10. GnRH-a治疗子宫内膜异位症的机理:(2.0分) A.抑制垂体促性腺激素,导致卵巢分泌性激素增多
B.抑制垂体促性腺激素,导致卵巢分泌性激素减少
C.促进垂体促性腺激素分泌,导致卵巢分泌性激素增多
D.促进垂体促性腺激素分泌,导致卵巢分泌性激素减少
E.以上都不是
B.抑制垂体促性腺激素,导致卵巢分泌性激素减少
6 6 2007 11. 经产妇,40岁,近2年出现痛经并逐渐加重,痛经较剧,需服强止痛药,伴经量增多及经期延长。查子宫均匀增大如孕8周,质硬,有压痛,经期压痛明显。痛经逐渐加重的原因,最可能的是(2.0分) A.盆腔炎性疾病
B.子宫腺肌病
C.子宫内膜结核
D.子宫内膜癌
E.子宫粘膜下肌瘤
B.子宫腺肌病
6 6 2007 12. 宫颈癌的癌前期病变是:(2.0分) A.慢性宫颈炎
B.CIN
C.宫颈息肉
D.HPV感染
E.都不是
B.CIN
6 6 2007 13. 宫颈癌好发部位是:(2.0分) A.宫颈阴道部
B.宫颈阴道上部
C.SCJ
D.宫颈组织学内口
E.都不是
C.SCJ
6 6 2007 14. Ia1浸润癌的病理诊断标准是:(2.0分) A.浸润深度≤3 mm,宽度≤ 7mm
B.浸润深度3-5mm,宽度≤ 7mm
C.浸润深度≤5 mm,宽度≤ 7mm
D.浸润深度≤3 mm,宽度≤ 5mm
E.都不是
A.浸润深度≤3 mm,宽度≤ 7mm
6 6 2007 15. CIN及宫颈癌确诊的方法是:(2.0分) A.宫颈细胞学检查
B.HPV检测
C.宫颈活检
D.阴道镜
E.子宫切除
C.宫颈活检
6 6 2007 16. 年轻的早期宫颈癌患者应该首选治疗方法:(2.0分) A.手术治疗
B.放疗
C.化疗
D.随访观察
E.都不是
A.手术治疗
6 6 2007 17. 有关CIN及宫颈癌,下面哪项不正确?(2.0分) A.CIN及宫颈癌的诊断通过"三阶梯"诊断程序,明确诊断需要宫颈活检病理学检查
B.治疗方法选择根据临床分期、患者状况和医院医疗技术水平及设备条件综合考虑
C.手术和放疗二者对子宫颈癌具有不一样的疗效,均为局部治疗,但副作用相同
D.年青患者手术时可保留卵巢功能
E.都不是
C.手术和放疗二者对子宫颈癌具有不一样的疗效,均为局部治疗,但副作用相同
6 6 2007 18. 不是外阴硬化性苔癣临床症状和体征的是:(2.0分) A.瘙痒
B.阴道流血
C.性交痛
D.外阴烧灼感
E.外阴萎缩和皮肤色素改变。
B.阴道流血
6 6 2007 19. 外阴癌中最常见的组织学类型是:(2.0分) A.鳞癌
B.腺癌
C.黑色素瘤
D.基底细胞癌
E.以上都不是
A.鳞癌
6 6 2007 20. 下例哪项与外阴癌病因无关:(2.0分) A.HPV病毒
B.外阴上皮内非瘤变
C.STD疾病
D.HSV病毒
E.性激素缺乏
E.性激素缺乏
6 6 2007 21. 关于外阴癌,下面哪项治疗是正确的?(2.0分) A.以放疗为主
B.以化疗为主
C.以免疫治疗为主
D.以手术治疗为主
E.以放化疗为主
D.以手术治疗为主
6 6 2007 22. 外阴癌放疗主要用在:(2.0分) A.手术前放疗、手术后辅助放疗好复发患者治疗
B.早期患者
C.年轻患者
D.Ⅱ期患者
E.都不是
A.手术前放疗、手术后辅助放疗好复发患者治疗
6 6 2007 23. 子宫内膜癌好发于:(2.0分) A.儿童
B.幼年
C.生育年龄妇女
D.绝经后妇女
E.都不是
D.绝经后妇女
6 6 2007 24. 非雌激素依赖型子宫内膜癌的特点:(2.0分) A.与雌激素无关,比较少见。多见于老年体痩妇女,雌孕激素受体多阴性,预后不良
B.与子宫内膜增生过长有关
C.患者较年轻、肥胖、高血压、糖尿病、不孕或不育与绝经后延
D.雌孕激素受体阳性率高
E.多见于无排卵疾病(功血,PCOS)、功能性卵巢肿瘤
A.与雌激素无关,比较少见。多见于老年体痩妇女,雌孕激素受体多阴性,预后不良
6 6 2007 25. 子宫内膜癌转移途径:(2.0分) A.以直接浸润和淋巴转移为主要转移途径,晚期患者可经血液系统转移
B.单纯直接浸润
C.单纯淋巴转移
D.单纯血液系统转移
E.以直接浸润和血液系统转移为主要转移途径
A.以直接浸润和淋巴转移为主要转移途径,晚期患者可经血液系统转移
6 6 2007 26. I期子宫内膜癌手术方式是:(2.0分) A.广泛子宫切除术及双附件切除术
B.筋膜外全子宫切除术+双侧附件切除术+选择性盆腔及腹主动脉旁淋巴结切除
C.筋膜外全子宫切除术+双侧附件切除术
D.广泛子宫切除术及双附件切除术+盆腔及腹主动脉旁淋巴结切除术
E.均不是
B.筋膜外全子宫切除术+双侧附件切除术+选择性盆腔及腹主动脉旁淋巴结切除
6 6 2007 27. 孕激素治疗不适用于:(2.0分) A.晚期患者
B.复发癌
C.不能手术患者
D.年轻要求生育的早期患者
E.都不是
E.都不是
6 6 2007 28. 属于卵巢良性肿瘤的是(2.0分) A.内胚窦瘤
B.颗粒细胞瘤
C.间质细胞瘤
D.成熟畸胎瘤
E.无性细胞瘤
D.成熟畸胎瘤
6 6 2007 29. 常见的多房性卵巢肿瘤是(2.0分) A.浆液性囊腺瘤
B.无性细胞瘤
C.卵泡膜细胞瘤
D.粘液性囊腺瘤
E.成熟畸胎瘤
D.粘液性囊腺瘤
6 6 2007 30. 目前对女性危害最大的妇科恶性肿瘤是(2.0分) A.卵巢癌
B.外阴癌
C.宫颈癌
D.子宫内膜癌
E.输卵管癌
A.卵巢癌
6 6 2007 31. 卵巢上皮性恶性肿瘤的治疗特点(2.0分) A.手术为主、辅以化疗的综合治疗
B.化疗与放疗有同等重要的地位
C.化疗的主要药物为环鳞酰胺
D.化疗通常应用3-4疗程
E.晚期者首选放疗
A.手术为主、辅以化疗的综合治疗
6 6 2007 32. 梅格斯综合症的特征为:(2.0分) A.胸腔积液或腹水
B.与卵泡膜细胞瘤相关
C.预后差
D.与肿瘤切除无关
E.为肿瘤转移的表现
A.胸腔积液或腹水
6 6 2007 33. 最常用卵巢肿瘤的辅助检查是(2.0分) A.腹部平片
B.CT扫描
C.腹腔镜检查
D.腹部B超
E.细胞学检查
D.腹部B超
6 6 2007 34. 卵巢恶性肿瘤的最主要转移途径是(2.0分) A.直接蔓延及腹腔种植
B.血行转移
C.沿卵巢淋巴管转移到髂内外淋巴结
D.沿卵巢血管走行,从卵巢淋巴管向上至腹主动脉旁淋巴结
E.脑转移
A.直接蔓延及腹腔种植
6 6 2007 35. 下列哪一种卵巢肿瘤不是源自原始体腔上皮(2.0分) A.浆液性囊腺瘤
B.内膜样肿瘤
C.成熟畸胎瘤
D.粘液性囊腺癌
E.浆液性囊腺癌
C.成熟畸胎瘤
6 6 2007 36. 妊娠滋养细胞疾病不包括以下哪种疾病(2.0分) A.葡萄胎
B.原发性卵巢绒癌
C.侵蚀性葡萄胎
D.胎盘部位滋养细胞肿瘤
E.绒毛膜癌
B.原发性卵巢绒癌
6 6 2007 37. 关于完全性葡萄胎巨检的描述错误的是(2.0分) A.纤维样组织串连水泡状物
B.葡萄样水泡大小不一
C.水泡间可充有血液或血凝块
D.水泡间不能发现胎儿及其附属物痕迹
E.水泡间不能发现蜕膜样组织
E.水泡间不能发现蜕膜样组织
6 6 2007 38. 由于下列哪一种激素作用,刺激卵巢卵泡膜细胞形成黄素化囊肿(2.0分) A.FSH
B.LH
C.E
D.T
E.hCG
E.hCG
6 6 2007 39. 化疗对下列哪一类妇科肿瘤效果最佳(2.0分) A.子宫内膜样腺癌
B.宫颈腺癌
C.卵巢上皮性癌
D.胎盘部位滋养细胞肿瘤
E.绒毛膜癌
E.绒毛膜癌
6 6 2007 40. 对于妊娠滋养细胞肿瘤治疗结束后随访,正确的是(2.0分) A.第一次随访为出院后1周
B.每半年随访一次共5年
C.随访期间应严格避孕至少12个月
D.随访期间可选择口服避孕药、宫内节育器或避孕套避孕
E.随访内容仅为h-CG即可
C.随访期间应严格避孕至少12个月
6 6 2007 41. 绒毛膜癌转移的最主要途径是(2.0分) A.淋巴转移
B.盆腔转移
C.血行转移
D.种植转移
E.局部转移
C.血行转移
6 6 2007 42. The best method to safely and reliably diagnose twins is by which of the following? (2.0分) A.ultrasonography
B.Leopold's maneuvers
C.Auscultation
D.X-rays
E.Computed tomography (CT) scan
A.ultrasonography
6 6 2007 43. 关于早期妊娠的诊断,正确的是(2.0分) A.停经6周左右都会有"早孕反应"
B.月经过期未来潮,黄体酮试验阳性,应疑为妊娠
C.子宫增大变软是确定早孕最可靠的依据
D.已婚生育年龄的妇女,平时月经规则,一旦月经过期10日应疑为妊娠
E.哺乳期妇女月经尚未恢复,不会再次妊娠
D.已婚生育年龄的妇女,平时月经规则,一旦月经过期10日应疑为妊娠
6 6 2007 44. A 32-year old and 43-gestational-week pregnant woman(G3P0) is admitted. The obstetrical examinations indicated BP 16/10.7 kPa (120/80mmHg),the height of uterus 35cm, fetal position LOA, FHR 132 beats/min. She is told to perform placental function examinat(2.0分) A.fetal movement ≥ 20 times/12 hours
B.B ultrasound detected maximum amniotic fluid depth ≥3cm
C.blood placental lactogen ≥4u g/L
D.NST is response type
E.OCT is continuously late deceleration
E.OCT is continuously late deceleration
6 6 2007 45. 足月妊娠时,正常胎心率的范围是: (2.0分) A.100-140次/分
B.110-150次/分
C.120-160次/分
D.130-170次/分
E.140-180次/分
C.120-160次/分
6 6 2007 46. The relationship of the long axis of the fetus to the long axis of the mother is called which of the following? (2.0分) A.lie
B.presentation
C.position
D.attitude
E.axis of the conjugate
A.lie
6 6 2007 47. 35岁女性,平素月经规律,现停经54天,黄体酮试验无出血,最可能的诊断是:(2.0分) A.继发闭经
B.早期妊娠
C.卵巢早衰
D.子宫内膜结核
E.垂体性闭经
B.早期妊娠
6 6 2007 48. A 24-year-old pregnancy woman is told her blood AFP value is 6 times than normal mean value during 17 gestational weeks. Which should be the next examination ? (2.0分) A.repeated AFP measurement
B.ultrasound examination
C.amniocentesis
D.amniotic cavity fetus visualization
E.fetoscopy examination
B.ultrasound examination
6 6 2007 49. In assessing risk factors in pregnancy, which of the following would create the most concern?(2.0分) A.maternal age 39
B.maternal age18,with menarche at age 13
C.past history of four normal deliveries
D.history of ovarian dermoid cyst removed 4 years ago
E.a clinically measured pelvic diagonal conjugate of 12cm
A.maternal age 39
6 6 2007 50. During late pregnancy, which of the following implies urinary tract disease?(2.0分) A.decreased serum creatinine
B.failure to excrete concentrated urine after 18 hours without fluids
C.glucosuria
D.dialation of the ureters
E.decreased creatinie clearance
E.decreased creatinie clearance
6 6 2008 1. 属于卵巢良性肿瘤的是(2.0分) A.内胚窦瘤
B.颗粒细胞瘤
C.间质细胞瘤
D.成熟畸胎瘤
E.无性细胞瘤
D.成熟畸胎瘤
6 6 2008 2. 卵巢肿瘤最常见的并发症(2.0分) A.蒂扭转
B.破裂
C.感染
D.出血
E.恶性变
A.蒂扭转
6 6 2008 3. 卵巢上皮性恶性肿瘤的治疗特点(2.0分) A.手术为主、辅以化疗的综合治疗
B.化疗与放疗有同等重要的地位
C.化疗的主要药物为环鳞酰胺
D.化疗通常应用3-4疗程
E.晚期者首选放疗
A.手术为主、辅以化疗的综合治疗
6 6 2008 4. 畸胎瘤良恶性的决定因素为(2.0分) A.组织的成熟程度
B.患者年龄
C.组织来源
D.肿瘤大小及质地
E.肿瘤的成分
A.组织的成熟程度
6 6 2008 5. 最常用卵巢肿瘤的辅助检查是(2.0分) A.腹部平片
B.CT扫描
C.腹腔镜检查
D.腹部B超
E.细胞学检查
D.腹部B超
6 6 2008 6. 卵巢恶性肿瘤的最主要转移途径是(2.0分) A.直接蔓延及腹腔种植
B.血行转移
C.沿卵巢淋巴管转移到髂内外淋巴结
D.沿卵巢血管走行,从卵巢淋巴管向上至腹主动脉旁淋巴结
E.脑转移
A.直接蔓延及腹腔种植
6 6 2008 7. 早孕合并卵巢囊肿适宜的手术时间为(2.0分) A.待至足月
B.尽早手术
C.妊娠4月后
D.妊娠6月后
E.妊娠3月以内
C.妊娠4月后
6 6 2008 8. 颗粒细胞瘤能分泌(2.0分) A.雌激素
B.孕激素
C.FSH
D.hCG
E.睾酮
A.雌激素
6 6 2008 9. 完全性葡萄胎的染色体基因组来源于(2.0分) A.父系
B.母系
C.父系和母系
D.母系和子系
E.父系和子系
A.父系
6 6 2008 10. 关于完全性葡萄胎巨检的描述错误的是(2.0分) A.纤维样组织串连水泡状物
B.葡萄样水泡大小不一
C.水泡间可充有血液或血凝块
D.水泡间不能发现胎儿及其附属物痕迹
E.水泡间不能发现蜕膜样组织
E.水泡间不能发现蜕膜样组织
6 6 2008 11. 由于下列哪一种激素作用,刺激卵巢卵泡膜细胞形成黄素化囊肿(2.0分) A.FSH
B.LH
C.E
D.T
E.hCG
E.hCG
6 6 2008 12. 化疗对下列哪一类妇科肿瘤效果最佳(2.0分) A.子宫内膜样腺癌
B.宫颈腺癌
C.卵巢上皮性癌
D.胎盘部位滋养细胞肿瘤
E.绒毛膜癌
E.绒毛膜癌
6 6 2008 13. 对于诊断为妊娠滋养细胞肿瘤(III:8)的患者,首选化疗方案为(2.0分) A.MTX单药5天方案
B.MTX单药8天方案
C.5-Fu
D.ACTD
E.EMA-CO
E.EMA-CO
6 6 2008 14. 复发和耐药的妊娠滋养细胞肿瘤患者首选的治疗方案是(2.0分) A.手术切除病灶
B.更改其他单药化疗
C.联合化疗
D.化疗+局部腔内放疗
E.化疗+手术切除病灶
C.联合化疗
6 6 2008 15. 绒毛膜癌转移的最主要途径是(2.0分) A.淋巴转移
B.盆腔转移
C.血行转移
D.种植转移
E.局部转移
C.血行转移
6 6 2008 16. 关于人工破膜的描述,下列错误的是(2.0分) A.宫口扩张3cm或3cm以上可行人工破膜
B.破膜时必须检查有无脐带先露
C.破膜应在宫缩期进行
D.破膜后术者手指应停留在阴道内,经过1~2次宫缩,待胎头入盆后,再将手指取出
E.无头盆不称、胎头已衔接者,可行人工破膜
C.破膜应在宫缩期进行
6 6 2008 17. which of the following does the early pregnancy have in common? (2.0分) A.morning sickness
B.breast enlargement
C.abdomen enlargement
D.amenorrhea (cease of mense)
E.urinary frequency
D.amenorrhea (cease of mense)
6 6 2008 18. The LMP was June 30. The expected date of confinement (EDC) is approximately which of the following? (2.0分) A.2013/3/23 0:00:00
B.2013/4/7 0:00:00
C.2013/3/28 0:00:00
D.2013/4/23 0:00:00
E.2013/3/7 0:00:00
B.2013/4/7 0:00:00
6 6 2008 19. 临产后子宫收缩特点,下列哪项不正确(2.0分) A.节律性
B.高张性
C.缩复作用
D.极性
E.对称性
B.高张性
6 6 2008 20. The bispinous diameter of a normal pelvis should be at least how many centimeters?(2.0分) A.5 cm
B.8cm
C. 10cm
D. 11cm
E. 12cm
C. 10cm
6 6 2008 21. Which of the following is characteristic of normal labor?(2.0分) A.Moderate bleeding
B.Low-grade fever (37.6℃) as labor progresses
C.Regular contractions start when active phase has been initiated
D.Lack of fetal movement in the later stages
E.Uterine relaxation between contractions
E.Uterine relaxation between contractions
6 6 2008 22. A contraindication to the use of oxytocin for stimulating labor is which of the following?(2.0分) A.fetal demise
B.hypotonic uterine dysfunction
C.hypertonic uterine dysfunction
D.twin gestation
E.estimated fetal weight less than 3kg
C.hypertonic uterine dysfunction
6 6 2008 23. The best method to safely and reliably diagnose twins is by which of the following? (2.0分) A.ultrasonography
B.Leopold's maneuvers
C.Auscultation
D.X-rays
E.Computed tomography (CT) scan
A.ultrasonography
6 6 2008 24. 关于早期妊娠的诊断,正确的是(2.0分) A.停经6周左右都会有"早孕反应"
B.月经过期未来潮,黄体酮试验阳性,应疑为妊娠
C.子宫增大变软是确定早孕最可靠的依据
D.已婚生育年龄的妇女,平时月经规则,一旦月经过期10日应疑为妊娠
E.哺乳期妇女月经尚未恢复,不会再次妊娠
D.已婚生育年龄的妇女,平时月经规则,一旦月经过期10日应疑为妊娠
6 6 2008 25. A 32-year old and 43-gestational-week pregnant woman(G3P0) is admitted. The obstetrical examinations indicated BP 16/10.7 kPa (120/80mmHg),the height of uterus 35cm, fetal position LOA, FHR 132 beats/min. She is told to perform placental function examination. Which one in the following indicates placenta insufficiency ? (2.0分) A. fetal movement ≥ 20 times/12 hours
B.B ultrasound detected maximum amniotic fluid depth ≥3cm
C.blood placental lactogen ≥4u g/L
D.NST is response type
E.OCT is continuously late deceleration
E.OCT is continuously late deceleration
6 6 2008 26. 足月妊娠时,正常胎心率的范围是:(2.0分) A.100-140次/分
B.110-150次/分
C.120-160次/分
D.130-170次/分
E.140-180次/分
C.120-160次/分
6 6 2008 27. The relationship of the long axis of the fetus to the long axis of the mother is called which of the following? (2.0分) A.lie
B.presentation
C.position
D.attitude
E.axis of the conjugate
A.lie
6 6 2008 28. 35岁女性,平素月经规律,现停经54天,黄体酮试验无出血,最可能的诊断是(2.0分) A.继发闭经
B.早期妊娠
C.卵巢早衰
D.子宫内膜结核
E.垂体性闭经
B.早期妊娠
6 6 2008 29. In assessing risk factors in pregnancy, which of the following would create the most concern?(2.0分) A.maternal age 39
B.maternal age18,with menarche at age 13
C.past history of four normal deliveries
D.history of ovarian dermoid cyst removed 4 years ago
E.a clinically measured pelvic diagonal conjugate of 12cm
A.maternal age 39
6 6 2008 30. During late pregnancy, which of the following implies urinary tract disease?(2.0分) A.decreased serum creatinine
B.failure to excrete concentrated urine after 18 hours without fluids
C.glucosuria
D.dialation of the ureters
E.decreased creatinie clearance
E.decreased creatinie clearance
6 6 2008 31. Advising a 34 yeare old woman at 12 weeks'gestation about the risk of chromosomal defects in the fetus, you can correctly state which of the following.(2.0分) A.there is little worry regarding down syndrome before the age of 35
B.paternal age is very important in the etiology of Down syndrome
C.maternal serum alpha-fetoprotein(MSAFP) is very specific test for down syndrome
D.screening for down syndrome can be improved by checking amniotic fluid and acetylcholinesterase level.
E.efficacy of screening for down syndrome is improved by adding estriol, inhibin A, and hCG concentration to the MSAFP
E.efficacy of screening for down syndrome is improved by adding estriol, inhibin A, and hCG concentration to the MSAFP
6 6 2008 32. 以下哪项不是子宫内膜异位症好发部位(2.0分) A.子宫直肠陷凹
B.卵巢
C.子宫骶骨韧带
D.输卵管
E.阴道壁
E.阴道壁
6 6 2008 33. 卵巢子宫内膜异位囊肿表现为(2.0分) A.囊内液粘稠,咖啡色
B.囊内可见毛发、牙齿
C.囊内液色清,呈淡黄色
D.囊内液粘稠,呈黄色
E.囊内液呈油脂状
A.囊内液粘稠,咖啡色
6 6 2008 34. 关于子宫内膜异位症的预防,哪项是错误的(2.0分) A.防止经血倒流
B.月经期应避免不必要的盆腔检查
C.避免手术操作时引起内膜种植
D.人工流产时不要突然降低负压
E.输卵管造影术,应在月经前期进行
E.输卵管造影术,应在月经前期进行
6 6 2008 35. 中重度内异患者血清可有哪个肿瘤标记物升高(2.0分) A.AFP
B.CA153
C.CA125
D.CEA
E.CA199
C.CA125
6 6 2008 36. 患者40岁,人工流产后经量增多10年,伴痛经进行性加剧,妇科检查:子宫如孕50天大,球形,质硬,活动度欠佳,附件区未及包块,首选诊断:(2.0分) A.子宫肌瘤
B.子宫腺肌病
C.盆腔炎症包块
D.子宫内膜异位症
E.子宫肉瘤
B.子宫腺肌病
6 6 2008 37. 子宫肌瘤临床分类标准为(2.0分) A.按临床症状严重程度分类
B.按查体所见子宫肌瘤大小分类
C.按子宫肌瘤的数目分类
D.按子宫肌瘤与子宫内膜的关系分类
E.按肌瘤与子宫肌层的关系分类
E.按肌瘤与子宫肌层的关系分类
6 6 2008 38. 子宫肌瘤的症状与下述何项关系密切(2.0分) A.肌瘤的大小
B.肌瘤生长的部位(宫体或宫颈)
C.发生年龄
D.肌瘤与肌层的关系(粘膜下、浆膜下、肌壁间)
E.肌瘤的数目
D.肌瘤与肌层的关系(粘膜下、浆膜下、肌壁间)
6 6 2008 39. 子宫肌瘤最常见的并发症是(2.0分) A.红色变性
B.继发贫血
C.浆膜下肌瘤蒂扭转
D.肌瘤压迫输尿管引起肾盂积水
E.肌瘤恶变
B.继发贫血
6 6 2008 40. 子宫肌瘤短期内迅速增大或伴有阴道流血应考虑(2.0分) A.感染
B.囊性变
C.红色变性
D.脂肪样变
E.肉瘤样变
E.肉瘤样变
6 6 2008 41. 关于子宫肌瘤的症状下述哪项是错误的 (2.0分) A.肌壁间肌瘤均可引起月经过多
B.黏膜下肌瘤出现症状较早
C.肌瘤一般不引起疼痛
D.不孕的原因之一是由于子宫腔变形,输卵管入口受阻
E.白带增多的原因之一是子宫腔面积增大,腺体分泌过多
A.肌壁间肌瘤均可引起月经过多
6 6 2008 42. 以下哪种情况不需手术治疗:(2.0分) A.丈夫患无精症的不孕患者B超检查发现子宫肌壁间约2*2cm大小肌瘤
B.子宫前壁浆膜下肌瘤约5*4cm大小,伴尿频、尿急症状
C.子宫多发肌瘤,子宫增大如孕3月,血红蛋白76g/L
D.子宫粘膜下肌瘤1*1cm大小,伴经量增多症状
E.半年内子宫肌瘤自3cm增大至6cm
A.丈夫患无精症的不孕患者B超检查发现子宫肌壁间约2*2cm大小肌瘤
6 6 2008 43. 外阴上皮内非瘤变是一组慢性疾病,下面哪项不是其特征:(2.0分) A.外阴皮肤和粘膜组织发生色素改变;
B.外阴皮肤和粘膜组织发生变性;
C.不典型增生;
D.主要包括外阴硬化性苔癣与外阴鳞状上皮增生;
E.也称慢性外阴营养不良。
C.不典型增生;
6 6 2008 44. 不是外阴鳞状上皮增生治疗手段的是:(2.0分) A.一般治疗。
B.药物治疗
C.物理治疗
D.手术治疗
E.放疗
E.放疗
6 6 2008 45. 下面哪项是正确的(2.0分) A.外阴癌临床相对比较多见
B.外阴癌好发于年轻妇女
C.外阴癌发病高峰年龄为60-80岁
D.外阴癌近年来其发病率有增高趋势
E.以上都不是
C.外阴癌发病高峰年龄为60-80岁
6 6 2008 46. 外阴癌转移途径:(2.0分) A.以直接浸润和淋巴转移为主要转移途径,晚期患者可经血液系统转移
B.单纯直接浸润
C.单纯淋巴转移
D.单纯血液系统转移
E.以直接浸润和血液系统转移为主要转移途径
A.以直接浸润和淋巴转移为主要转移途径,晚期患者可经血液系统转移
6 6 2008 47. 外阴癌的确诊主要依靠:(2.0分) A.临床表现
B.局部组织病理学检查
C.妇科检查
D.影像学检查
E.HPV病毒检测
B.局部组织病理学检查
6 6 2008 48. 子宫内膜癌好发于:(2.0分) A.儿童
B.幼年
C.生育年龄妇女
D.绝经后妇女
E.都不是
D.绝经后妇女
6 6 2008 49. 非雌激素依赖型子宫内膜癌的特点:(2.0分) A.与雌激素无关,比较少见。多见于老年体痩妇女,雌孕激素受体多阴性,预后不良
B.与子宫内膜增生过长有关
C.患者较年轻、肥胖、高血压、糖尿病、不孕或不育与绝经后延
D.雌孕激素受体阳性率高
E.多见于无排卵疾病(功血,PCOS)、功能性卵巢肿瘤
A.与雌激素无关,比较少见。多见于老年体痩妇女,雌孕激素受体多阴性,预后不良
6 6 2008 50. 子宫内膜癌转移途径(2.0分) A.以直接浸润和淋巴转移为主要转移途径,晚期患者可经血液系统转移
B.单纯直接浸润
C.单纯淋巴转移
D.单纯血液系统转移
E.以直接浸润和血液系统转移为主要转移途径
A.以直接浸润和淋巴转移为主要转移途径,晚期患者可经血液系统转移
6 6 2009 1. 正常分娩时最主要的产力是 (2.0分) A.子宫收缩力
B.肛提肌收缩力
C.腹肌收缩力
D.膈肌收缩力
E.骨骼肌收缩力
A.子宫收缩力
6 6 2009 2. which of the following does the early pregnancy have in common? (2.0分) A.morning sickness
B.breast enlargement
C.abdomen enlargement
D.amenorrhea (cease of mense)
E.urinary frequency
D.amenorrhea (cease of mense)
6 6 2009 3. 妊娠35周,少量阴道流血1天入院,无腹痛。查体示:宫底剑突下四横指,LSA臀浮,胎心150次/分,骨盆正常,阴道无活动性出血,无宫缩,宫口未开,一般情况好。恰当的处理是: (2.0分) A.人工破膜
B.期待疗法
C.立即行剖宫产
D.催产素滴注引产
E.臀位牵引术
B.期待疗法
6 6 2009 4. The LMP was June 30. The expected date of confinement (EDC) is approximately which of the following? (2.0分) A.2015/3/23 0:00:00
B.2015/4/7 0:00:00
C.2015/3/28 0:00:00
D.2015/4/23 0:00:00
E.2015/3/7 0:00:00
B.2015/4/7 0:00:00
6 6 2009 5. Of the following laboratory studies, which would not be done routinely in a pregnant patient? (2.0分) A.VDRL (Venereal Disease Research Laboratory)
B.Diabetes screen
C.Urinalysis
D.Blood estriol
E.Blood type (ABO and RH)
D.Blood estriol
6 6 2009 6. The bispinous diameter of a normal pelvis should be at least how many centimeters?(2.0分) A.5 cm
B.8cm
C. 10cm
D. 11cm
E. 12cm
C. 10cm
6 6 2009 7. Engagement is best defined as which of the following? (2.0分) A.when the presenting part goes through the pelvic inlet
B.when the greatest biparietal diameter of the fetal head passes the pelvic inlet
C.when the presenting part is level below the ischial spines
D.when the greatest biparietal diameter of the head is level with the ischial spines
E.when the greatest diameter of the fetal presenting part passes through the narrowest and lowest part of the maternal pelvis.
B.when the greatest biparietal diameter of the fetal head passes the pelvic inlet
6 6 2009 8. 有关产道的描述哪项是错误的? (2.0分) A.产道可分为软产道和骨产道。
B.正常女性骨盆入口平面呈纵椭园型。
C.中骨盆平面是产道中最窄平面。
D.软产道是由子宫下段、宫颈、阴道及盆底软组织构成的弯曲通道。
E.骨盆轴是指连接骨盆各平面中点的假想曲线。
B.正常女性骨盆入口平面呈纵椭园型。
6 6 2009 9. 第三产程的临床经过哪项是错误的? (2.0分) A.协助胎盘娩出;
B.检查胎盘、胎膜完整;
C.检查软产道;
D.预防产后出血;
E.新生儿洗澡;
E.新生儿洗澡;
6 6 2009 10. 第二产程临床经过哪项是错误的?(2.0分) A.等待自然破膜;
B.指导用腹压;
C.密切监测胎心;
D.接产准备;
E.胎头娩出后急于娩出胎肩;
E.胎头娩出后急于娩出胎肩;
6 6 2009 11. 属于卵巢良性肿瘤的是(2.0分) A.内胚窦瘤
B.颗粒细胞瘤
C.间质细胞瘤
D.成熟畸胎瘤
E.无性细胞瘤
D.成熟畸胎瘤
6 6 2009 12. 常见的多房性卵巢肿瘤是(2.0分) A.浆液性囊腺瘤
B.无性细胞瘤
C.卵泡膜细胞瘤
D.粘液性囊腺瘤
E.成熟畸胎瘤
D.粘液性囊腺瘤
6 6 2009 13. 卵巢肿瘤最常见的并发症(2.0分) A.蒂扭转
B.破裂
C.感染
D.出血
E.恶性变
A.蒂扭转
6 6 2009 14. 卵巢上皮性恶性肿瘤的治疗特点(2.0分) A.手术为主、辅以化疗的综合治疗
B.化疗与放疗有同等重要的地位
C.化疗的主要药物为环鳞酰胺
D.化疗通常应用3-4疗程
E.晚期者首选放疗
A.手术为主、辅以化疗的综合治疗
6 6 2009 15. 梅格斯综合症的特征为:(2.0分) A.胸腔积液或腹水
B.与卵泡膜细胞瘤相关
C.预后差
D.与肿瘤切除无关
E.为肿瘤转移的表现
A.胸腔积液或腹水
6 6 2009 16. 最常用卵巢肿瘤的辅助检查是(2.0分) A.腹部平片
B.CT扫描
C.腹腔镜检查
D.腹部B超
E.细胞学检查
D.腹部B超
6 6 2009 17. 最常见于儿童及少女的肿瘤是(2.0分) A.颗粒细胞瘤
B.卵巢内膜样肿瘤
C.粘液性囊腺瘤
D.内胚窦瘤
E.未成熟畸胎瘤
D.内胚窦瘤
6 6 2009 18. 卵巢恶性肿瘤的最主要转移途径是(2.0分) A.直接蔓延及腹腔种植
B.血行转移
C.沿卵巢淋巴管转移到髂内外淋巴结
D.沿卵巢血管走行,从卵巢淋巴管向上至腹主动脉旁淋巴结
E.脑转移
A.直接蔓延及腹腔种植
6 6 2009 19. 早孕合并卵巢囊肿适宜的手术时间为(2.0分) A.待至足月
B.尽早手术
C.妊娠4月后
D.妊娠6月后
E.妊娠3月以内
C.妊娠4月后
6 6 2009 20. 妊娠滋养细胞疾病不包括以下哪种疾病(2.0分) A.葡萄胎
B.原发性卵巢绒癌
C.侵蚀性葡萄胎
D.胎盘部位滋养细胞肿瘤
E.绒毛膜癌
B.原发性卵巢绒癌
6 6 2009 21. 完全性葡萄胎的染色体基因组来源于(2.0分) A.父系
B.母系
C.父系和母系
D.母系和子系
E.父系和子系
A.父系
6 6 2009 22. 关于完全性葡萄胎巨检的描述错误的是(2.0分) A.纤维样组织串连水泡状物
B.葡萄样水泡大小不一
C.水泡间可充有血液或血凝块
D.水泡间不能发现胎儿及其附属物痕迹
E.水泡间不能发现蜕膜样组织
E.水泡间不能发现蜕膜样组织
6 6 2009 23. 葡萄胎清除后,血β-hCG首次降至阴性的平均时间以下列哪一时间正确(2.0分) A.14天
B.42天
C.56天
D.63天
E.70天
D.63天
6 6 2009 24. 化疗对下列哪一类妇科肿瘤效果最佳(2.0分) A.子宫内膜样腺癌
B.宫颈腺癌
C.卵巢上皮性癌
D.胎盘部位滋养细胞肿瘤
E.绒毛膜癌
E.绒毛膜癌
6 6 2009 25. 治疗妊娠滋养细胞肿瘤的主要方法是(2.0分) A.手术切除病灶
B.放疗
C.手术和联合化疗
D.局部腹腔化疗
E.静脉化疗
E.静脉化疗
6 6 2009 26. 对于妊娠滋养细胞肿瘤治疗结束后随访,正确的是(2.0分) A.第一次随访为出院后1周
B.每半年随访一次共5年
C.随访期间应严格避孕至少12个月
D.随访期间可选择口服避孕药、宫内节育器或避孕套避孕
E.随访内容仅为h-CG即可
C.随访期间应严格避孕至少12个月
6 6 2009 27. 复发和耐药的妊娠滋养细胞肿瘤患者首选的治疗方案是(2.0分) A.手术切除病灶
B.更改其他单药化疗
C.联合化疗
D.化疗+局部腔内放疗
E.化疗+手术切除病灶
C.联合化疗
6 6 2009 28. 绒毛膜癌转移的最主要途径是(2.0分) A.淋巴转移
B.盆腔转移
C.血行转移
D.种植转移
E.局部转移
C.血行转移
6 6 2009 29. PSTT确诊依赖(2.0分) A.血β-hCG测定
B.临床表现
C.超声检查
D.组织学依据
E.腹腔镜检查
D.组织学依据
6 6 2009 30. The best method to safely and reliably diagnose twins is by which of the following? ultrasonography(2.0分) A.Leopold's maneuvers
B.Auscultation
C.X-rays
D.Computed tomography (CT) scan
A.Leopold's maneuvers
6 6 2009 31. 关于早期妊娠的诊断,正确的是 (2.0分) A.停经6周左右都会有"早孕反应"
B.月经过期未来潮,黄体酮试验阳性,应疑为妊娠
C.子宫增大变软是确定早孕最可靠的依据
D.已婚生育年龄的妇女,平时月经规则,一旦月经过期10日应疑为妊娠
E.哺乳期妇女月经尚未恢复,不会再次妊娠
D.已婚生育年龄的妇女,平时月经规则,一旦月经过期10日应疑为妊娠
6 6 2009 32. A 32-year old and 43-gestational-week pregnant woman(G3P0) is admitted. The obstetrical examinations indicated BP 16/10.7 kPa (120/80mmHg),the height of uterus 35cm, fetal position LOA, FHR 132 beats/min. She is told to perform placental function examination. Which one in the following indicates placenta insufficiency ?(2.0分) A.fetal movement ≥ 20 times/12 hours
B.B ultrasound detected maximum amniotic fluid depth ≥3cm
C. blood placental lactogen ≥4u g/L
D. NST is response type
E. OCT is continuously late deceleration
E. OCT is continuously late deceleration
6 6 2009 33. 足月妊娠时,正常胎心率的范围是: (2.0分) A.100-140次/分
B.110-150次/分
C.120-160次/分
D.130-170次/分
E.140-180次/分
C.120-160次/分
6 6 2009 34. The relationship of the long axis of the fetus to the long axis of the mother is called which of the following? (2.0分) A.lie
B.presentation
C.position
D.attitude
E.axis of the conjugate
A.lie
6 6 2009 35. 35岁女性,平素月经规律,现停经54天,黄体酮试验无出血,最可能的诊断是:(2.0分) A.继发闭经
B.早期妊娠
C.卵巢早衰
D.子宫内膜结核
E.垂体性闭经
B.早期妊娠
6 6 2009 36. A 24-year-old pregnancy woman is told her blood AFP value is 6 times than normal mean value during 17 gestational weeks. Which should be the next examination ? (2.0分) A.repeated AFP measurement
B.ultrasound examination
C.amniocentesis
D.amniotic cavity fetus visualization
E.fetoscopy examination
B.ultrasound examination
6 6 2009 37. In assessing risk factors in pregnancy, which of the following would create the most concern?(2.0分) A.maternal age 39
B.maternal age18,with menarche at age 13
C.past history of four normal deliveries
D.history of ovarian dermoid cyst removed 4 years ago
E.a clinically measured pelvic diagonal conjugate of 12cm
A.maternal age 39
6 6 2009 38. Advising a 34 yeare old woman at 12 weeks'gestation about the risk of chromosomal defects in the fetus, you can correctly state which of the following.(2.0分) A.there is little worry regarding down syndrome before the age of 35
B.paternal age is very important in the etiology of Down syndrome
C.maternal serum alpha-fetoprotein(MSAFP) is very specific test for down syndrome
D.screening for down syndrome can be improved by checking amniotic fluid and acetylcholinesterase level.
E.efficacy of screening for down syndrome is improved by adding estriol, inhibin A, and hCG concentration to the MSAFP
E.efficacy of screening for down syndrome is improved by adding estriol, inhibin A, and hCG concentration to the MSAFP
6 6 2009 39. 子宫肌瘤临床分类标准为(2.0分) A.按临床症状严重程度分类
B.按查体所见子宫肌瘤大小分类
C.按子宫肌瘤的数目分类
D.按子宫肌瘤与子宫内膜的关系分类
E.按肌瘤与子宫肌层的关系分类
E.按肌瘤与子宫肌层的关系分类
6 6 2009 40. 子宫肌瘤继发贫血最常见于(2.0分) A.浆膜下肌瘤
B.粘膜下肌瘤
C.肌瘤囊性变性
D.肌瘤红色变性
E.肌壁间子宫肌瘤
B.粘膜下肌瘤
6 6 2009 41. 子宫肌瘤合并妊娠时红色变性出现哪些症状:(2.0分) A.腹部包块
B.月经量增多,周期缩短,经期延长
C.尿频、排尿障碍等压迫症状
D.阴道分泌物增多伴不规则阴道流血
E.腹痛、发热
E.腹痛、发热
6 6 2009 42. 子宫肌瘤最常见的并发症是(2.0分) A.红色变性
B.继发贫血
C.浆膜下肌瘤蒂扭转
D.肌瘤压迫输尿管引起肾盂积水
E.肌瘤恶变
B.继发贫血
6 6 2009 43. 子宫肌瘤红色样变常发生在(2.0分) A.青春期
B.月经期
C.妊娠期
D.围绝经期
E.绝经后期
C.妊娠期
6 6 2009 44. 关于子宫肌瘤的症状下述哪项是错误的 (2.0分) A.肌壁间肌瘤均可引起月经过多
B.黏膜下肌瘤出现症状较早
C.肌瘤一般不引起疼痛
D.不孕的原因之一是由于子宫腔变形,输卵管入口受阻
E.白带增多的原因之一是子宫腔面积增大,腺体分泌过多
A.肌壁间肌瘤均可引起月经过多
6 6 2009 45. 关于子宫肌瘤的手术治疗,下述哪项是不正确的(2.0分) A.近绝经期,子宫如孕2个月大小,症状轻者,可暂不手术
B.肌瘤症状明显,经保守治疗无效者应考虑手术
C.突出于阴道内的粘膜下肌瘤应行手术治疗
D.年轻,未生育过的妇女肌壁间肌瘤不论多发或单发,均应手术
E.疑有恶变者不宜行剔除术
D.年轻,未生育过的妇女肌壁间肌瘤不论多发或单发,均应手术
6 6 2009 46. 以下哪种情况建议行子宫切除术:(2.0分) A.患者30岁,已婚未育,子宫多发肌瘤,子宫增大如孕10周
B.患者60岁,15年前发现子宫肌瘤5cm大小,10年前绝经,此后定期复查B超,提示子宫肌瘤逐渐缩小,现肌瘤约2cm大小
C.患者45岁,已婚已育,子宫多发肌瘤,子宫增大如孕2月,无不适症状
D.患者45岁,已婚已育,子宫多发肌瘤,子宫增大如孕50天大,经量增多2年,Hb65g/L
E.患者45岁,已婚已育,子宫粘膜下肌瘤2cm,经量增多2年,Hb65g/L
D.患者45岁,已婚已育,子宫多发肌瘤,子宫增大如孕50天大,经量增多2年,Hb65g/L
6 6 2009 47. 以下哪项不是子宫内膜异位症好发部位(2.0分) A.子宫直肠陷凹
B.卵巢
C.子宫骶骨韧带
D.输卵管
E.阴道壁
E.阴道壁
6 6 2009 48. 关于子宫内膜异位症哪项是错误的(2.0分) A.多见于生育年龄妇女,以30-40岁居多
B.发病与卵巢的周期性变化有关
C.子宫内膜出现和生长在子宫肌层
D.具有类似恶性肿瘤的远处转移和种植能力
E.一般极少发生恶变
C.子宫内膜出现和生长在子宫肌层
6 6 2009 49. 子宫内膜异位症最主要的临床特点是(2.0分) A.月经失调
B.不孕症发生率高
C.痛经和持续性下腹痛
D.咳血
E.腹痛,腹泻或便秘
C.痛经和持续性下腹痛
6 6 2009 50. 33岁妇女,婚后7年未孕,痛经逐年加重,妇科检查:宫骶韧带处可触及黄豆大小结节2个,触痛明显,右附件可及一5*6cm大小囊性块,活动差,最有效的确诊方案:(2.0分) A.B超
B.诊断性刮宫
C.宫腔镜检查
D.腹腔镜手术
E.血CA125
D.腹腔镜手术
6 6 2010 1. 关于人工破膜的描述,下列错误的是(2.5分) A.宫口扩张3cm或3cm以上可行人工破膜
B.破膜时必须检查有无脐带先露
C.破膜应在宫缩期进行
D.破膜后术者手指应停留在阴道内,经过1~2次宫缩,待胎头入盆后,再将手指取出
E.无头盆不称、胎头已衔接者,可行人工破膜
C.破膜应在宫缩期进行
6 6 2010 2. which of the following does the early pregnancy have in common? (2.5分) A.morning sickness
B.breast enlargement
C.abdomen enlargement
D.amenorrhea (cease of mense)
E.urinary frequency
D.amenorrhea (cease of mense)
6 6 2010 3. 妊娠35周,少量阴道流血1天入院,无腹痛。查体示:宫底剑突下四横指,LSA臀浮,胎心150次/分,骨盆正常,阴道无活动性出血,无宫缩,宫口未开,一般情况好。恰当的处理是(2.5分) A.人工破膜
B.期待疗法
C.立即行剖宫产
D.催产素滴注引产
E.臀位牵引术
B.期待疗法
6 6 2010 4. Of the following laboratory studies, which would not be done routinely in a pregnant patient? (2.5分) A.VDRL (Venereal Disease Research Laboratory)
B.Diabetes screen
C.Urinalysis
D.Blood estriol
E.Blood type (ABO and RH)
D.Blood estriol
6 6 2010 5. Engagement is best defined as which of the following?(2.5分) A.when the presenting part goes through the pelvic inlet
B.when the greatest biparietal diameter of the fetal head passes the pelvic inlet
C.when the presenting part is level below the ischial spines
D.when the greatest biparietal diameter of the head is level with the ischial spines
E.when the greatest diameter of the fetal presenting part passes through the narrowest and lowest part of the maternal pelvis.
B.when the greatest biparietal diameter of the fetal head passes the pelvic inlet
6 6 2010 6. 有关产道的描述哪项是错误的? (2.5分) A.产道可分为软产道和骨产道。
B.正常女性骨盆入口平面呈纵椭园型。
C.中骨盆平面是产道中最窄平面。
D.软产道是由子宫下段、宫颈、阴道及盆底软组织构成的弯曲通道。
E.骨盆轴是指连接骨盆各平面中点的假想曲线。
B.正常女性骨盆入口平面呈纵椭园型。
6 6 2010 7. 新生儿阿普加评分法包括下列(2.5分) A.心率、呼吸;
B.皮肤颜色;
C.肌张力;
D.反射;
E.以上全是;
E.以上全是;
6 6 2010 8. 胎盘的组成是(2.5分) A.羊膜、叶状绒毛膜、包蜕膜
B.包蜕膜、叶状绒毛膜、底蜕膜
C.胎膜、叶状绒毛膜、真蜕膜
D.羊膜、叶状绒毛膜、底蜕膜
E.羊膜、胎膜、底蜕膜
D.羊膜、叶状绒毛膜、底蜕膜
6 6 2010 9. 关于胎先露的指示点,下列那项是错误的(2.5分) A.臀先露-骶骨
B.面先露-颏骨
C.枕先露-枕骨
D.枕先露-面骨
E.足先露-骶骨
D.枕先露-面骨
6 6 2010 10. A 32-year old and 43-gestational-week pregnant woman(G3P0) is admitted. The obstetrical examinations indicated BP 16/10.7 kPa (120/80mmHg),the height of uterus 35cm, fetal position LOA, FHR 132 beats/min. She is told to perform placental function examination. Which one in the following indicates placenta insufficiency ? (2.5分) A.fetal movement ≥ 20 times/12 hours
B.ultrasound detected maximum amniotic fluid depth ≥3cm
C.blood placental lactogen ≥4u g/L
D.NST is response type
E.OCT is continuously late deceleration
E.OCT is continuously late deceleration
6 6 2010 11. 下列哪条径线的测量可间接推测骨盆入口前后径长度(2.5分) A.髂棘间径
B.髂嵴间径
C.骶耻外径
D.出口前矢状径
E.出口横径
C.骶耻外径
6 6 2010 12. A 24-year-old pregnancy woman is told her blood AFP value is 6 times than normal mean value during 17 gestational weeks. Which should be the next examination ?(2.5分) A.repeated AFP measurement
B.ultrasound examination
C.amniocentesis
D.amniotic cavity fetus visualization
E.fetoscopy examination
B.ultrasound examination
6 6 2010 13. 下列哪项是提示要考虑遗传性疾病(2.5分) A.孕早期病毒感染
B.孕期接触过有毒物质
C.孕期服用过导致胎儿畸形的药物
D.羊水穿刺细胞核型分析显示染色体异常
E.孕早期拍过X片
D.羊水穿刺细胞核型分析显示染色体异常
6 6 2010 14. During late pregnancy, which of the following implies urinary tract disease?(2.5分) A.decreased serum creatinine
B.failure to excrete concentrated urine after 18 hours without fluids
C.glucosuria
D.dialation of the ureters
E.decreased creatinie clearance
E.decreased creatinie clearance
6 6 2010 15. 胎头衔接是指胎头 (2.5分) A.枕骨进入骨盆入口
B.顶骨进入骨盆入口
C.双顶径进入骨盆入口,颅骨最低点接近或达到坐骨棘水平
D.双顶径到达坐骨棘水平
E.双顶径到达坐骨结节水平
C.双顶径进入骨盆入口,颅骨最低点接近或达到坐骨棘水平
6 6 2010 16. 子宫肌瘤临床分类标准为(2.5分) A.按临床症状严重程度分类
B.按查体所见子宫肌瘤大小分类
C.按子宫肌瘤的数目分类
D.按子宫肌瘤与子宫内膜的关系分类
E.按肌瘤与子宫肌层的关系分类
E.按肌瘤与子宫肌层的关系分类
6 6 2010 17. 子宫肌瘤合并妊娠时红色变性出现哪些症状(2.5分) A.腹部包块
B.月经量增多,周期缩短,经期延长
C.尿频、排尿障碍等压迫症状
D.阴道分泌物增多伴不规则阴道流血
E.腹痛、发热
E.腹痛、发热
6 6 2010 18. 子宫肌瘤短期内迅速增大或伴有阴道流血应考虑(2.5分) A.感染
B.囊性变
C.红色变性
D.脂肪样变
E.肉瘤样变
E.肉瘤样变
6 6 2010 19. 以下哪种情况建议行子宫切除术(2.5分) A.患者30岁,已婚未育,子宫多发肌瘤,子宫增大如孕10周
B.患者60岁,15年前发现子宫肌瘤5cm大小,10年前绝经,此后定期复查B超,提示子宫肌瘤逐渐缩小,现肌瘤约2cm大小
C.患者45岁,已婚已育,子宫多发肌瘤,子宫增大如孕2月,无不适症状
D.患者45岁,已婚已育,子宫多发肌瘤,子宫增大如孕50天大,经量增多2年,Hb65g/L
E.患者45岁,已婚已育,子宫粘膜下肌瘤2cm,经量增多2年,Hb65g/L
D.患者45岁,已婚已育,子宫多发肌瘤,子宫增大如孕50天大,经量增多2年,Hb65g/L
6 6 2010 20. 关于子宫内膜异位症哪项是错误的(2.5分) A.多见于生育年龄妇女,以30-40岁居多
B.发病与卵巢的周期性变化有关
C.子宫内膜出现和生长在子宫肌层
D.具有类似恶性肿瘤的远处转移和种植能力
E.一般极少发生恶变
C.子宫内膜出现和生长在子宫肌层
6 6 2010 21. 子宫内膜异位症最主要的临床特点是(2.5分) A.月经失调
B.不孕症发生率高
C.痛经和持续性下腹痛
D.咳血
E.腹痛,腹泻或便秘
C.痛经和持续性下腹痛
6 6 2010 22. 中重度内异患者血清可有哪个肿瘤标记物升高(2.5分) A.AFP
B.CA153
C.CA125
D.CEA
E.CA199
C.CA125
6 6 2010 23. 宫颈癌的癌前期病变是(2.5分) A.慢性宫颈炎
B.CIN
C.宫颈息肉
D.HPV感染
E.都不是
B.CIN
6 6 2010 24. Ia1浸润癌的病理诊断标准是(2.5分) A.浸润深度≤3 mm,宽度≤ 7mm
B.浸润深度3-5mm,宽度≤ 7mm
C.浸润深度≤5 mm,宽度≤ 7mm
D.浸润深度≤3 mm,宽度≤ 5mm
E.都不是
A.浸润深度≤3 mm,宽度≤ 7mm
6 6 2010 25. CIN及宫颈癌确诊的方法是(2.5分) A.宫颈细胞学检查
B.HPV检测
C.宫颈活检
D.阴道镜
E.子宫切除
C.宫颈活检
6 6 2010 26. 非雌激素依赖型子宫内膜癌的特点(2.5分) A.与雌激素无关,比较少见。多见于老年体痩妇女,雌孕激素受体多阴性,预后不良
B.与子宫内膜增生过长有关
C.患者较年轻、肥胖、高血压、糖尿病、不孕或不育与绝经后延
D.雌孕激素受体阳性率高
E.多见于无排卵疾病(功血,PCOS)、功能性卵巢肿瘤
A.与雌激素无关,比较少见。多见于老年体痩妇女,雌孕激素受体多阴性,预后不良
6 6 2010 27. 子宫内膜癌转移途径(2.5分) A.以直接浸润和淋巴转移为主要转移途径,晚期患者可经血液系统转移
B.单纯直接浸润
C.单纯淋巴转移
D.单纯血液系统转移
E.以直接浸润和血液系统转移为主要转移途径
A.以直接浸润和淋巴转移为主要转移途径,晚期患者可经血液系统转移
6 6 2010 28. 晚期子宫内膜癌治疗应该(2.5分) A.以手术为主、必要时辅助治疗
B.单纯放疗
C.孕激素治疗
D.采用放射、手术、药物综合治疗
E.化疗
D.采用放射、手术、药物综合治疗
6 6 2010 29. 外阴癌中最常见的组织学类型是(2.5分) A.鳞癌
B.腺癌
C.黑色素瘤
D.基底细胞癌
E.以上都不是
A.鳞癌
6 6 2010 30. 外阴癌转移途径(2.5分) A.以直接浸润和淋巴转移为主要转移途径,晚期患者可经血液系统转移
B.单纯直接浸润
C.单纯淋巴转移
D.单纯血液系统转移
E.以直接浸润和血液系统转移为主要转移途径
A.以直接浸润和淋巴转移为主要转移途径,晚期患者可经血液系统转移
6 6 2010 31. 外阴癌放疗主要用在(2.5分) A.手术前放疗、手术后辅助放疗好复发患者治疗
B.早期患者
C.年轻患者
D.Ⅱ期患者
E.都不是
A.手术前放疗、手术后辅助放疗好复发患者治疗
6 6 2010 32. 属于卵巢良性肿瘤的是(2.5分) A.内胚窦瘤
B.颗粒细胞瘤
C.间质细胞瘤
D.成熟畸胎瘤
E.无性细胞瘤
D.成熟畸胎瘤
6 6 2010 33. 目前对女性危害最大的妇科恶性肿瘤是(2.5分) A.卵巢癌
B.外阴癌
C.宫颈癌
D.子宫内膜癌
E.输卵管癌
A.卵巢癌
6 6 2010 34. 畸胎瘤良恶性的决定因素为(2.5分) A.组织的成熟程度
B.患者年龄
C.组织来源
D.肿瘤大小及质地
E.肿瘤的成分
A.组织的成熟程度
6 6 2010 35. 最常见于儿童及少女的肿瘤是(2.5分) A.颗粒细胞瘤
B.卵巢内膜样肿瘤
C.粘液性囊腺瘤
D.内胚窦瘤
E.未成熟畸胎瘤
D.内胚窦瘤
6 6 2010 36. 早孕合并卵巢囊肿适宜的手术时间为(2.5分) A.待至足月
B.尽早手术
C.妊娠4月后
D.妊娠6月后
E.妊娠3月以内
C.妊娠4月后
6 6 2010 37. 完全性葡萄胎的染色体基因组来源于(2.5分) A.父系
B.母系
C.父系和母系
D.母系和子系
E.父系和子系
A.父系
6 6 2010 38. 葡萄胎清除后,血β-hCG首次降至阴性的平均时间以下列哪一时间正确(2.5分) A.14天
B.42天
C.56天
D.63天
E.70天
D.63天
6 6 2010 39. 对于诊断为妊娠滋养细胞肿瘤(III:8)的患者,首选化疗方案为(2.5分) A.MTX单药5天方案
B.MTX单药8天方案
C.5-Fu
D.ACTD
E.EMA-CO
E.EMA-CO
6 6 2010 40. 绒毛膜癌转移的最主要途径是(2.5分) A.淋巴转移
B.盆腔转移
C.血行转移
D.种植转移
E.局部转移
C.血行转移
6 6 2011 1. 属于卵巢良性肿瘤的是(2.5分) A.内胚窦瘤
B.颗粒细胞瘤
C.间质细胞瘤
D.成熟畸胎瘤
E.无性细胞瘤
D.成熟畸胎瘤
6 6 2011 2. 卵巢肿瘤最常见的并发症 (2.5分) A.蒂扭转
B.破裂
C.感染
D.出血
E.恶性变
A.蒂扭转
6 6 2011 3. 卵巢上皮性恶性肿瘤的治疗特点(2.5分) A.手术为主、辅以化疗的综合治疗
B.化疗与放疗有同等重要的地位
C.化疗的主要药物为环鳞酰胺
D.化疗通常应用3-4疗程
E.晚期者首选放疗
A.手术为主、辅以化疗的综合治疗
6 6 2011 4. 畸胎瘤良恶性的决定因素为(2.5分) A.组织的成熟程度
B.患者年龄
C.组织来源
D.肿瘤大小及质地
E.肿瘤的成分
A.组织的成熟程度
6 6 2011 5. 最常用卵巢肿瘤的辅助检查是(2.5分) A.腹部平片
B.CT扫描
C.腹腔镜检查
D.腹部B超
E.细胞学检查
D.腹部B超
6 6 2011 6. 卵巢恶性肿瘤的最主要转移途径是(2.5分) A.直接蔓延及腹腔种植
B.血行转移
C.沿卵巢淋巴管转移到髂内外淋巴结
D.沿卵巢血管走行,从卵巢淋巴管向上至腹主动脉旁淋巴结
E.脑转移
A.直接蔓延及腹腔种植
6 6 2011 7. 早孕合并卵巢囊肿适宜的手术时间为(2.5分) A.待至足月
B.尽早手术
C.妊娠4月后
D.妊娠6月后
E.妊娠3月以内
C.妊娠4月后
6 6 2011 8. 颗粒细胞瘤能分泌(2.5分) A.雌激素
B.孕激素
C.FSH
D.hCG
E.睾酮
A.雌激素
6 6 2011 9. 妊娠滋养细胞疾病不包括以下哪种疾病(2.5分) A.葡萄胎
B.原发性卵巢绒癌
C.侵蚀性葡萄胎
D.胎盘部位滋养细胞肿瘤
E.绒毛膜癌
B.原发性卵巢绒癌
6 6 2011 10. 葡萄胎最重要的病理特点是(2.5分) A.绒毛结构完整
B.绒毛间质水肿
C.滋养细胞增生
D.间质部内胚源性血管消失
E.滋养层间质内陷
C.滋养细胞增生
6 6 2011 11. 葡萄胎清除后,血β-hCG首次降至阴性的平均时间以下列哪一时间正确(2.5分) A.14天
B.42天
C.56天
D.63天
E.70天
D.63天
6 6 2011 12. 关于葡萄胎的处理,正确的一项是(2.5分) A.吸宫术前常规静滴催产素可减少出血
B.葡萄胎吸宫12周后,妊娠试验阳性,排除早孕、黄素化囊肿、宫腔内葡萄胎残留,示恶变可能
C.葡萄胎术后应放置宫内节育器避孕2年
D.葡萄胎排出后,追踪观察到妊娠试验连续3次阴性
E.葡萄胎术后常规化疗以防恶变
B.葡萄胎吸宫12周后,妊娠试验阳性,排除早孕、黄素化囊肿、宫腔内葡萄胎残留,示恶变可能
6 6 2011 13. 治疗妊娠滋养细胞肿瘤的主要方法是(2.5分) A.手术切除病灶
B.放疗
C.手术和联合化疗
D.局部腹腔化疗
E.静脉化疗
E.静脉化疗
6 6 2011 14. 对于妊娠滋养细胞肿瘤治疗结束后随访,正确的是(2.5分) A.第一次随访为出院后1周
B.每半年随访一次共5年
C.随访期间应严格避孕至少12个月
D.随访期间可选择口服避孕药、宫内节育器或避孕套避孕
E.随访内容仅为h-CG即可
C.随访期间应严格避孕至少12个月
6 6 2011 15. 侵蚀性葡萄胎患者突发急性腹痛,需警惕以下可能(2.5分) A.子宫病灶穿破浆膜层
B.肝转移病灶破裂
C.卵巢黄素化囊肿扭转
D.卵巢黄素化囊肿破裂
E.以上都是
E.以上都是
6 6 2011 16. 外阴上皮内非瘤变是一组慢性疾病,下面哪项不是其特征(2.5分) A.外阴皮肤和粘膜组织发生色素改变;
B.外阴皮肤和粘膜组织发生变性;
C.不典型增生;
D.主要包括外阴硬化性苔癣与外阴慢性单纯性苔癣;
E.也称慢性外阴营养不良。
C.不典型增生;
6 6 2011 17. 不是外阴慢性单纯性苔癣治疗手段的是(2.5分) A.一般治疗。
B.药物治疗
C.物理治疗
D.手术治疗
E.放疗
E.放疗
6 6 2011 18. 下面哪项是正确的?(2.5分) A.外阴癌临床相对比较多见
B.外阴癌好发于年轻妇女
C.外阴癌发病高峰年龄为60-80岁
D.外阴癌近年来其发病率有增高趋势
E.以上都不是
C.外阴癌发病高峰年龄为60-80岁
6 6 2011 19. 关于外阴癌,下面哪项治疗是正确的?(2.5分) A.以放疗为主
B.以化疗为主
C.以免疫治疗为主
D.以手术治疗为主
E.以放化疗为主
D.以手术治疗为主
6 6 2011 20. 外阴癌的确诊主要依靠(2.5分) A.临床表现
B.局部组织病理学检查
C.妇科检查
D.影像学检查
E.HPV病毒检测
B.局部组织病理学检查
6 6 2011 21. 子宫内膜癌好发于(2.5分) A.儿童
B.幼年
C.生育年龄妇女
D.绝经后妇女
E.都不是
D.绝经后妇女
6 6 2011 22. 非雌激素依赖型子宫内膜癌的特点(2.5分) A.与雌激素无关,比较少见。多见于老年体痩妇女,雌孕激素受体多阴性,预后不良
B.与子宫内膜增生过长有关
C.患者较年轻、肥胖、高血压、糖尿病、不孕或不育与绝经后延
D.雌孕激素受体阳性率高
E.多见于无排卵疾病(功血,PCOS)、功能性卵巢肿瘤
A.与雌激素无关,比较少见。多见于老年体痩妇女,雌孕激素受体多阴性,预后不良
6 6 2011 23. 确诊子宫内膜癌手段是(2.5分) A.分段诊刮
B.细胞学检查
C.超声检查
D.宫腔镜检查
E.影像学检查
A.分段诊刮
6 6 2011 24. 孕激素治疗不适用于(2.5分) A.晚期患者
B.复发癌
C.不能手术患者
D.年轻要求生育的早期患者
E.都不是
E.都不是
6 6 2011 25. 子宫内膜癌的主要临床表现是(2.5分) A.性生活后出血
B.绝经后阴道流血
C.下腹疼痛
D.阴道排液
E.以上均不是
B.绝经后阴道流血
6 6 2011 26. 宫颈癌的癌前期病变是(2.5分) A.慢性宫颈炎
B.宫颈上皮内病变
C.宫颈息肉
D.HPV感染
E.都不是
B.宫颈上皮内病变
6 6 2011 27. 宫颈癌好发部位是(2.5分) A.宫颈阴道部
B.宫颈阴道上部
C.SCJ
D.宫颈组织学内口
E.都不是
C.SCJ
6 6 2011 28. 年轻的早期宫颈癌患者应该首选治疗方法(2.5分) A.手术治疗
B.放疗
C.化疗
D.随访观察
E.都不是
A.手术治疗
6 6 2011 29. 宫颈上皮内病变的处理一般不采用(2.5分) A.理疗
B.锥切
C.随访
D.子宫切除
E.都不是
D.子宫切除
6 6 2011 30. 宫颈上皮内病变及宫颈癌确诊的方法是(2.5分) A.宫颈细胞学检查
B.HPV检测
C.宫颈活检
D.阴道镜
E.子宫切除
C.宫颈活检
6 6 2011 31. 正常分娩时最主要的产力是(2.5分) A.子宫收缩力
B.肛提肌收缩力
C.腹肌收缩力
D.膈肌收缩力
E.骨骼肌收缩力
A.子宫收缩力
6 6 2011 32. 妊娠35周,少量阴道流血1天入院,无腹痛。查体示:宫底剑突下四横指,LSA臀浮,胎心150次/分,骨盆正常,阴道无活动性出血,无宫缩,宫口未开,一般情况好。恰当的处理是:(2.5分) A.人工破膜
B.期待疗法
C.立即行剖宫产
D.催产素滴注引产
E.臀位牵引术
B.期待疗法
6 6 2011 33. Of the following laboratory studies, which would not be done routinely in a pregnant patient?(2.5分) A.VDRL (Venereal Disease Research Laboratory)
B.Diabetes screen
C.Urinalysis
D.Blood estriol
E.Blood type (ABO and RH)
D.Blood estriol
6 6 2011 34. 临产后子宫收缩特点,下列哪项不正确 (2.5分) A.节律性
B.高张性
C.缩复作用
D.极性
E.对称性
B.高张性
6 6 2011 35. 影响分娩的因素下列哪个不是的? (2.5分) A.产力
B.产道
C.胎儿
D.精神心理因素
E.胎盘
E.胎盘
6 6 2011 36. 第一产程的临床表现哪项是错误的? (2.5分) A.规律宫缩;
B.宫口扩张;
C.胎头下降;
D.胎膜破裂;
E.孕妇必须卧床休息;
E.孕妇必须卧床休息;
6 6 2011 37. The bispinous diameter of a normal pelvis should be at least how many centimeters?(2.5分) A.5 cm
B.8cm
C.10cm
D.11cm
E.12cm
C.10cm
6 6 2011 38. 新生儿阿普加评分法包括下列(2.5分) A.心率、呼吸;
B.皮肤颜色;
C.肌张力;
D.反射;
E.以上全是;
E.以上全是;
6 6 2011 39. A 25-year-old G2P1001 woman at 41 weeks' gestation is noted to change her cervix from 6cm to 9cm over 2 hours.Which of the following is the most likely diagnosis? (2.5分) A.Normal labor
B.Prolonged latent phase
C.Prolonged active phase
D.Arrest of active phase
E.Arrest of descent
A.Normal labor
6 6 2011 40. Although labor is a continuous process, it is divided into(2.5分) A.two functional stages
B.three functional stages
C.four functional stages
D.five functional stages
E.one functional stages
B.three functional stages
6 6 2012 1. 子宫肌瘤临床分类标准为(2.5分) A.按临床症状严重程度分类
B.按查体所见子宫肌瘤大小分类
C.按子宫肌瘤的数目分类
D.按子宫肌瘤与子宫内膜的关系分类
E.按肌瘤与子宫肌层的关系分类
E.按肌瘤与子宫肌层的关系分类
6 6 2012 2. 子宫肌瘤最常见的并发症是(2.5分) A.红色变性
B.继发贫血
C.浆膜下肌瘤蒂扭转
D.肌瘤压迫输尿管引起肾盂积水
E.肌瘤恶变
B.继发贫血
6 6 2012 3. 关于子宫肌瘤,下述哪项是错误的(2.5分) A.粘膜下肌瘤较常发生月经量过多与不规则出血
B.浆膜下肌瘤较少出现月经过多
C.膀胱充盈时,较大肌瘤可升至腹部,病人因此就医
D.较大浆膜下肌瘤可发生蒂扭转
E.浆膜下肌瘤最易引起不孕
E.浆膜下肌瘤最易引起不孕
6 6 2012 4. 关于子宫肌瘤的手术治疗,下述哪项是不正确的(2.5分) A.近绝经期,子宫如孕2个月大小,症状轻者,可暂不手术
B.肌瘤症状明显,经保守治疗无效者应考虑手术
C.突出于阴道内的粘膜下肌瘤应行手术治疗
D.年轻,未生育过的妇女肌壁间肌瘤不论多发或单发,均应手术
E.疑有恶变者不宜行剔除术
D.年轻,未生育过的妇女肌壁间肌瘤不论多发或单发,均应手术
6 6 2012 5. 以下哪种情况建议行子宫切除术:(2.5分) A.患者30岁,已婚未育,子宫多发肌瘤,子宫增大如孕10周
B.患者60岁,15年前发现子宫肌瘤5cm大小,10年前绝经,此后定期复查B超,提示子宫肌瘤逐渐缩小,现肌瘤约2cm大小
C.患者45岁,已婚已育,子宫多发肌瘤,子宫增大如孕2月,无不适症状
D.患者45岁,已婚已育,子宫多发肌瘤,子宫增大如孕50天大,经量增多2年,Hb65g/L
E.患者45岁,已婚已育,子宫粘膜下肌瘤2cm,经量增多2年,Hb65g/L
D.患者45岁,已婚已育,子宫多发肌瘤,子宫增大如孕50天大,经量增多2年,Hb65g/L
6 6 2012 6. 以下哪项不是子宫内膜异位症好发部位(2.5分) A.子宫直肠陷凹
B.卵巢
C.子宫骶骨韧带
D.输卵管
E.阴道壁
E.阴道壁
6 6 2012 7. 子宫内膜异位症最主要的临床特点是(2.5分) A.月经失调
B.不孕症发生率高
C.痛经和持续性下腹痛
D.咳血
E.腹痛,腹泻或便秘
C.痛经和持续性下腹痛
6 6 2012 8. 33岁妇女,婚后7年未孕,痛经逐年加重,妇科检查:宫骶韧带处可触及黄豆大小结节2个,触痛明显,右附件可及一5*6cm大小囊性块,活动差,最有效的确诊方案:(2.5分) A.B超
B.诊断性刮宫
C.宫腔镜检查
D.腹腔镜手术
E.血CA125
D.腹腔镜手术
6 6 2012 9. GnRH-a治疗子宫内膜异位症的机理:(2.5分) A.抑制垂体促性腺激素,导致卵巢分泌性激素增多
B.抑制垂体促性腺激素,导致卵巢分泌性激素减少
C.促进垂体促性腺激素分泌,导致卵巢分泌性激素增多
D.促进垂体促性腺激素分泌,导致卵巢分泌性激素减少
E.以上都不是
B.抑制垂体促性腺激素,导致卵巢分泌性激素减少
6 6 2012 10. 患者40岁,人工流产后经量增多10年,伴痛经进行性加剧,妇科检查:子宫如孕50天大,球形,质硬,活动度欠佳,附件区未及包块,首选诊断:(2.5分) A.子宫肌瘤
B.子宫腺肌病
C.盆腔炎症包块
D.子宫内膜异位症
E.子宫肉瘤
B.子宫腺肌病
6 6 2012 11. 关于胎儿发育,以下说法正确的是:(2.5分) A.16周末,胎儿身长达到16cm,但胎儿仍然没有出现呼吸运动;
B.20周末,胎儿无明显运动,孕妇多数不能感知胎动;
C.28周末,胎儿皮肤呈粉红色,出生后已可存活;
D.32周末,胎儿皮肤呈粉红色,出生后即使积极护理,仍不能存活;
E.36周末,估计体重3000g,出生后不能啼哭和吸吮,生存能力差;
C.28周末,胎儿皮肤呈粉红色,出生后已可存活;
6 6 2012 12. 关于羊水的功能,表述错误的是:(2.5分) A.避免胎儿在子宫内受压;
B.避免胎儿肢体发生黏连;
C.临产时,避免胎儿局部受压引起胎儿窘迫;
D.促进胎肺发育
E.临产后,前羊水囊对宫颈口无压迫扩张作用
E.临产后,前羊水囊对宫颈口无压迫扩张作用
6 6 2012 13. 以下不属于胎儿Manning评分项目的是(2.5分) A.羊水量
B.肌张力
C.胎儿呼吸运动
D.胎心率
E.胎动(30分钟)
D.胎心率
6 6 2012 14. 以下不属于产前诊断适应症范围的是:(2.5分) A.年龄超过35周岁
B.羊水过多
C.孕早期接触过可能导致胎儿先天畸形的物质
D.孕早期保胎病史
E.夫妇有一方患有先天性或遗传性疾病
D.孕早期保胎病史
6 6 2012 15. 关于人工破膜的描述,下列错误的是:(2.5分) A.宫口扩张3cm或3cm以上可行人工破膜
B.破膜时必须检查有无脐带先露
C.破膜应在宫缩期进行
D.破膜后术者手指应停留在阴道内,经过1~2次宫缩,待胎头入盆后,再将手指取出
E.无头盆不称、胎头已衔接者,可行人工破膜
C.破膜应在宫缩期进行
6 6 2012 16. which of the following does the early pregnancy have in common? (2.5分) A.morning sickness
B.breast enlargement
C.abdomen enlargement
D.amenorrhea (cease of mense)
E.urinary frequency
D.amenorrhea (cease of mense)
6 6 2012 17. 妊娠35周,少量阴道流血1天入院,无腹痛。查体示:宫底剑突下四横指,LSA臀浮,胎心150次/分,骨盆正常,阴道无活动性出血,无宫缩,宫口未开,一般情况好。恰当的处理是: (2.5分) A.人工破膜
B.期待疗法
C.立即行剖宫产
D.催产素滴注引产
E.臀位牵引术
B.期待疗法
6 6 2012 18. Of the following laboratory studies, which would not be done routinely in a pregnant patient? (2.5分) A.VDRL (Venereal Disease Research Laboratory)
B.Diabetes screen
C.Urinalysis
D.Blood estriol
E.Blood type (ABO and RH)
D.Blood estriol
6 6 2012 19. A 25-year-old G2P1001 woman at 41 weeks' gestation is noted to change her cervix from 6cm to 9cm over 2 hours.Which of the following is the most likely diagnosis? (2.5分) A.Normal labor
B.Prolonged latent phase
C.Prolonged active phase
D.Arrest of active phase
E.Arrest of descent
A.Normal labor
6 6 2012 20. 第一产程的临床表现哪项是错误的?(2.5分) A.规律宫缩;
B.宫口扩张;
C.胎头下降;
D.胎膜破裂;
E.孕妇必须卧床休息;
E.孕妇必须卧床休息;
6 6 2012 21. 外阴上皮内非瘤变是一组慢性疾病,下面哪项不是其特征:(2.5分) A.外阴皮肤和粘膜组织发生色素改变;
B.外阴皮肤和粘膜组织发生变性;
C.不典型增生;
D.主要包括外阴硬化性苔癣与外阴鳞状上皮增生;
E.也称慢性外阴营养不良。
C.不典型增生;
6 6 2012 22. 外阴癌中最常见的组织学类型是:(2.5分) A.鳞癌
B.腺癌
C.黑色素瘤
D.基底细胞癌
E.以上都不是
A.鳞癌
6 6 2012 23. 下例哪项与外阴癌病因无关:(2.5分) A.HPV病毒
B.外阴上皮内非瘤变
C.STD疾病
D.HSV病毒
E.性激素缺乏
E.性激素缺乏
6 6 2012 24. 外阴癌的确诊主要依靠:(2.5分) A.临床表现
B.局部组织病理学检查
C.妇科检查
D.影像学检查
E.HPV病毒检测
B.局部组织病理学检查
6 6 2012 25. 宫颈癌的癌前期病变是:(2.5分) A.慢性宫颈炎
B.CIN
C.宫颈息肉
D.HPV感染
E.都不是
B.CIN
6 6 2012 26. 宫颈癌好发部位是:(2.5分) A.宫颈阴道部
B.宫颈阴道上部
C.SCJ
D.宫颈组织学内口
E.都不是
C.SCJ
6 6 2012 27. 宫颈癌转移途径:(2.5分) A.以直接浸润和淋巴转移为主要转移途径,晚期患者可经血液系统转移
B.单纯直接浸润
C.单纯淋巴转移
D.单纯血液系统转移
E.以直接浸润和血液系统转移为主要转移途径
A.以直接浸润和淋巴转移为主要转移途径,晚期患者可经血液系统转移
6 6 2012 28. 有关CIN及宫颈癌,下面哪项不正确?(2.5分) A.CIN及宫颈癌的诊断通过"三阶梯"诊断程序,明确诊断需要宫颈活检病理学检查
B.治疗方法选择根据临床分期、患者状况和医院医疗技术水平及设备条件综合考虑
C.手术和放疗二者对子宫颈癌具有不一样的疗效,均为局部治疗,但副作用相同
D.年青患者手术时可保留卵巢功能
E.都不是
C.手术和放疗二者对子宫颈癌具有不一样的疗效,均为局部治疗,但副作用相同
6 6 2012 29. 不是子宫内膜癌发生的高危因素:(2.5分) A.高血压
B.HPV感染
C.肥胖
D.糖尿病
E.不孕或不育
B.HPV感染
6 6 2012 30. I期子宫内膜癌手术方式是:(2.5分) A.广泛子宫切除术及双附件切除术
B.筋膜外全子宫切除术+双侧附件切除术+选择性盆腔及腹主动脉旁淋巴结切除
C.筋膜外全子宫切除术+双侧附件切除术
D.广泛子宫切除术及双附件切除术+盆腔及腹主动脉旁淋巴结切除术
E.均不是
B.筋膜外全子宫切除术+双侧附件切除术+选择性盆腔及腹主动脉旁淋巴结切除
6 6 2012 31. 晚期子宫内膜癌治疗应该:(2.5分) A.以手术为主、必要时辅助治疗
B.单纯放疗
C.孕激素治疗
D.采用放射、手术、药物综合治疗
E.化疗
D.采用放射、手术、药物综合治疗
6 6 2012 32. 属于卵巢良性肿瘤的是(2.5分) A.内胚窦瘤
B.颗粒细胞瘤
C.间质细胞瘤
D.成熟畸胎瘤
E.无性细胞瘤
D.成熟畸胎瘤
6 6 2012 33. 卵巢肿瘤最常见的并发症(2.5分) A.蒂扭转
B.破裂
C.感染
D.出血
E.恶性变
A.蒂扭转
6 6 2012 34. 梅格斯综合症的特征为:(2.5分) A.胸腔积液或腹水
B.与卵泡膜细胞瘤相关
C.预后差
D.与肿瘤切除无关
E.为肿瘤转移的表现
A.胸腔积液或腹水
6 6 2012 35. 最常用卵巢肿瘤的辅助检查是(2.5分) A.腹部平片
B.CT扫描
C.腹腔镜检查
D.腹部B超
E.细胞学检查
D.腹部B超
6 6 2012 36. 葡萄胎最重要的病理特点是(2.5分) A.绒毛结构完整
B.绒毛间质水肿
C.滋养细胞增生
D.间质部内胚源性血管消失
E.滋养层间质内陷
C.滋养细胞增生
6 6 2012 37. 由于下列哪一种激素作用,刺激卵巢卵泡膜细胞形成黄素化囊肿(2.5分) A.FSH
B.LH
C.E
D.T
E.hCG
E.hCG
6 6 2012 38. 对于妊娠滋养细胞肿瘤治疗结束后随访,正确的是(2.5分) A.第一次随访为出院后1周
B.每半年随访一次共5年
C.随访期间应严格避孕至少12个月
D.随访期间可选择口服避孕药、宫内节育器或避孕套避孕
E.随访内容仅为h-CG即可
C.随访期间应严格避孕至少12个月
6 6 2012 39. 绒毛膜癌转移的最主要途径是(2.5分) A.淋巴转移
B.盆腔转移
C.血行转移
D.种植转移
E.局部转移
C.血行转移
6 6 2012 40. PSTT确诊依赖(2.5分) A.血β-hCG测定
B.临床表现
C.超声检查
D.组织学依据
E.腹腔镜检查
D.组织学依据
6 6 2013 1. 子宫肌瘤临床分类标准为(2.5分) A.按临床症状严重程度分类
B.按查体所见子宫肌瘤大小分类
C.按子宫肌瘤的数目分类
D.按子宫肌瘤与子宫内膜的关系分类
E.按肌瘤与子宫肌层的关系分类
E.按肌瘤与子宫肌层的关系分类
6 6 2013 2. 子宫肌瘤最常见的并发症是(2.5分) A.红色变性
B.继发贫血
C.浆膜下肌瘤蒂扭转
D.肌瘤压迫输尿管引起肾盂积水
E.肌瘤恶变
B.继发贫血
6 6 2013 3. 关于子宫肌瘤,下述哪项是错误的(2.5分) A.粘膜下肌瘤较常发生月经量过多与不规则出血
B.浆膜下肌瘤较少出现月经过多
C.膀胱充盈时,较大肌瘤可升至腹部,病人因此就医
D.较大浆膜下肌瘤可发生蒂扭转
E.浆膜下肌瘤最易引起不孕
E.浆膜下肌瘤最易引起不孕
6 6 2013 4. 关于子宫肌瘤的手术治疗,下述哪项是不正确的(2.5分) A.近绝经期,子宫如孕2个月大小,症状轻者,可暂不手术
B.肌瘤症状明显,经保守治疗无效者应考虑手术
C.突出于阴道内的粘膜下肌瘤应行手术治疗
D.年轻,未生育过的妇女肌壁间肌瘤不论多发或单发,均应手术
E.疑有恶变者不宜行剔除术
D.年轻,未生育过的妇女肌壁间肌瘤不论多发或单发,均应手术
6 6 2013 5. 以下哪种情况建议行子宫切除术:(2.5分) A.患者30岁,已婚未育,子宫多发肌瘤,子宫增大如孕10周
B.患者60岁,15年前发现子宫肌瘤5cm大小,10年前绝经,此后定期复查B超,提示子宫肌瘤逐渐缩小,现肌瘤约2cm大小
C.患者45岁,已婚已育,子宫多发肌瘤,子宫增大如孕2月,无不适症状
D.患者45岁,已婚已育,子宫多发肌瘤,子宫增大如孕50天大,经量增多2年,Hb65g/L
E.患者45岁,已婚已育,子宫粘膜下肌瘤2cm,经量增多2年,Hb65g/L
D.患者45岁,已婚已育,子宫多发肌瘤,子宫增大如孕50天大,经量增多2年,Hb65g/L
6 6 2013 6. 以下哪项不是子宫内膜异位症好发部位(2.5分) A.子宫直肠陷凹
B.卵巢
C.子宫骶骨韧带
D.输卵管
E.阴道壁
E.阴道壁
6 6 2013 7. 子宫内膜异位症最主要的临床特点是(2.5分) A.月经失调
B.不孕症发生率高
C.痛经和持续性下腹痛
D.咳血
E.腹痛,腹泻或便秘
C.痛经和持续性下腹痛
6 6 2013 8. 33岁妇女,婚后7年未孕,痛经逐年加重,妇科检查:宫骶韧带处可触及黄豆大小结节2个,触痛明显,右附件可及一5*6cm大小囊性块,活动差,最有效的确诊方案:(2.5分) A.B超
B.诊断性刮宫
C.宫腔镜检查
D.腹腔镜手术
E.血CA125
D.腹腔镜手术
6 6 2013 9. GnRH-a治疗子宫内膜异位症的机理:(2.5分) A.抑制垂体促性腺激素,导致卵巢分泌性激素增多
B.抑制垂体促性腺激素,导致卵巢分泌性激素减少
C.促进垂体促性腺激素分泌,导致卵巢分泌性激素增多
D.促进垂体促性腺激素分泌,导致卵巢分泌性激素减少
E.以上都不是
B.抑制垂体促性腺激素,导致卵巢分泌性激素减少
6 6 2013 10. 患者40岁,人工流产后经量增多10年,伴痛经进行性加剧,妇科检查:子宫如孕50天大,球形,质硬,活动度欠佳,附件区未及包块,首选诊断:(2.5分) A.子宫肌瘤
B.子宫腺肌病
C.盆腔炎症包块
D.子宫内膜异位症
E.子宫肉瘤
B.子宫腺肌病
6 6 2013 11. 关于胎儿发育,以下说法正确的是:(2.5分) A.16周末,胎儿身长达到16cm,但胎儿仍然没有出现呼吸运动;
B.20周末,胎儿无明显运动,孕妇多数不能感知胎动;
C.28周末,胎儿皮肤呈粉红色,出生后已可存活;
D.32周末,胎儿皮肤呈粉红色,出生后即使积极护理,仍不能存活;
E.36周末,估计体重3000g,出生后不能啼哭和吸吮,生存能力差;
C.28周末,胎儿皮肤呈粉红色,出生后已可存活;
6 6 2013 12. 关于羊水的功能,表述错误的是:(2.5分) A.避免胎儿在子宫内受压;
B.避免胎儿肢体发生黏连;
C.临产时,避免胎儿局部受压引起胎儿窘迫;
D.促进胎肺发育
E.临产后,前羊水囊对宫颈口无压迫扩张作用
E.临产后,前羊水囊对宫颈口无压迫扩张作用
6 6 2013 13. 以下不属于胎儿Manning评分项目的是(2.5分) A.羊水量
B.肌张力
C.胎儿呼吸运动
D.胎心率
E.胎动(30分钟)
D.胎心率
6 6 2013 14. 以下不属于产前诊断适应症范围的是:(2.5分) A.年龄超过35周岁
B.羊水过多
C.孕早期接触过可能导致胎儿先天畸形的物质
D.孕早期保胎病史
E.夫妇有一方患有先天性或遗传性疾病
D.孕早期保胎病史
6 6 2013 15. 关于人工破膜的描述,下列错误的是:(2.5分) A.宫口扩张3cm或3cm以上可行人工破膜
B.破膜时必须检查有无脐带先露
C.破膜应在宫缩期进行
D.破膜后术者手指应停留在阴道内,经过1~2次宫缩,待胎头入盆后,再将手指取出
E.无头盆不称、胎头已衔接者,可行人工破膜
C.破膜应在宫缩期进行
6 6 2013 16. which of the following does the early pregnancy have in common? (2.5分) A.morning sickness
B.breast enlargement
C.abdomen enlargement
D.amenorrhea (cease of mense)
E.urinary frequency
D.amenorrhea (cease of mense)
6 6 2013 17. 妊娠35周,少量阴道流血1天入院,无腹痛。查体示:宫底剑突下四横指,LSA臀浮,胎心150次/分,骨盆正常,阴道无活动性出血,无宫缩,宫口未开,一般情况好。恰当的处理是: (2.5分) A.人工破膜
B.期待疗法
C.立即行剖宫产
D.催产素滴注引产
E.臀位牵引术
B.期待疗法
6 6 2013 18. Of the following laboratory studies, which would not be done routinely in a pregnant patient? (2.5分) A.VDRL (Venereal Disease Research Laboratory)
B.Diabetes screen
C.Urinalysis
D.Blood estriol
E.Blood type (ABO and RH)
D.Blood estriol
6 6 2013 19. A 25-year-old G2P1001 woman at 41 weeks' gestation is noted to change her cervix from 6cm to 9cm over 2 hours.Which of the following is the most likely diagnosis? (2.5分) A.Normal labor
B.Prolonged latent phase
C.Prolonged active phase
D.Arrest of active phase
E.Arrest of descent
A.Normal labor
6 6 2013 20. 第一产程的临床表现哪项是错误的?(2.5分) A.规律宫缩;
B.宫口扩张;
C.胎头下降;
D.胎膜破裂;
E.孕妇必须卧床休息;
E.孕妇必须卧床休息;
6 6 2013 21. 外阴上皮内非瘤变是一组慢性疾病,下面哪项不是其特征:(2.5分) A.外阴皮肤和粘膜组织发生色素改变;
B.外阴皮肤和粘膜组织发生变性;
C.不典型增生;
D.主要包括外阴硬化性苔癣与外阴鳞状上皮增生;
E.也称慢性外阴营养不良。
C.不典型增生;
6 6 2013 22. 外阴癌中最常见的组织学类型是:(2.5分) A.鳞癌
B.腺癌
C.黑色素瘤
D.基底细胞癌
E.以上都不是
A.鳞癌
6 6 2013 23. 下例哪项与外阴癌病因无关:(2.5分) A.HPV病毒
B.外阴上皮内非瘤变
C.STD疾病
D.HSV病毒
E.性激素缺乏
E.性激素缺乏
6 6 2013 24. 外阴癌的确诊主要依靠:(2.5分) A.临床表现
B.局部组织病理学检查
C.妇科检查
D.影像学检查
E.HPV病毒检测
B.局部组织病理学检查
6 6 2013 25. 宫颈癌的癌前期病变是:(2.5分) A.慢性宫颈炎
B.CIN
C.宫颈息肉
D.HPV感染
E.都不是
B.CIN
6 6 2013 26. 宫颈癌好发部位是:(2.5分) A.宫颈阴道部
B.宫颈阴道上部
C.SCJ
D.宫颈组织学内口
E.都不是
C.SCJ
6 6 2013 27. 宫颈癌转移途径:(2.5分) A.以直接浸润和淋巴转移为主要转移途径,晚期患者可经血液系统转移
B.单纯直接浸润
C.单纯淋巴转移
D.单纯血液系统转移
E.以直接浸润和血液系统转移为主要转移途径
A.以直接浸润和淋巴转移为主要转移途径,晚期患者可经血液系统转移
6 6 2013 28. 有关CIN及宫颈癌,下面哪项不正确?(2.5分) A.CIN及宫颈癌的诊断通过"三阶梯"诊断程序,明确诊断需要宫颈活检病理学检查
B.治疗方法选择根据临床分期、患者状况和医院医疗技术水平及设备条件综合考虑
C.手术和放疗二者对子宫颈癌具有不一样的疗效,均为局部治疗,但副作用相同
D.年青患者手术时可保留卵巢功能
E.都不是
C.手术和放疗二者对子宫颈癌具有不一样的疗效,均为局部治疗,但副作用相同
6 6 2013 29. 不是子宫内膜癌发生的高危因素:(2.5分) A.高血压
B.HPV感染
C.肥胖
D.糖尿病
E.不孕或不育
B.HPV感染
6 6 2013 30. I期子宫内膜癌手术方式是:(2.5分) A.广泛子宫切除术及双附件切除术
B.筋膜外全子宫切除术+双侧附件切除术+选择性盆腔及腹主动脉旁淋巴结切除
C.筋膜外全子宫切除术+双侧附件切除术
D.广泛子宫切除术及双附件切除术+盆腔及腹主动脉旁淋巴结切除术
E.均不是
B.筋膜外全子宫切除术+双侧附件切除术+选择性盆腔及腹主动脉旁淋巴结切除
6 6 2013 31. 晚期子宫内膜癌治疗应该:(2.5分) A.以手术为主、必要时辅助治疗
B.单纯放疗
C.孕激素治疗
D.采用放射、手术、药物综合治疗
E.化疗
D.采用放射、手术、药物综合治疗
6 6 2013 32. 属于卵巢良性肿瘤的是(2.5分) A.内胚窦瘤
B.颗粒细胞瘤
C.间质细胞瘤
D.成熟畸胎瘤
E.无性细胞瘤
D.成熟畸胎瘤
6 6 2013 33. 卵巢肿瘤最常见的并发症(2.5分) A.蒂扭转
B.破裂
C.感染
D.出血
E.恶性变
A.蒂扭转
6 6 2013 34. 孕妇血容量变化,哪项是正确的(2.5分) A.自妊娠12周血容量开始增加
B.妊娠32-34周达高峰
C.妊娠34周后缓慢增加至足月
D.红细胞增加多于血浆增加
E.孕中期血液处于浓缩状态
B.妊娠32-34周达高峰
6 6 2013 35. 最常用卵巢肿瘤的辅助检查是(2.5分) A.腹部平片
B.CT扫描
C.腹腔镜检查
D.腹部B超
E.细胞学检查
D.腹部B超
6 6 2013 36. 葡萄胎最重要的病理特点是(2.5分) A.绒毛结构完整
B.绒毛间质水肿
C.滋养细胞增生
D.间质部内胚源性血管消失
E.滋养层间质内陷
C.滋养细胞增生
6 6 2013 37. 由于下列哪一种激素作用,刺激卵巢卵泡膜细胞形成黄素化囊肿(2.5分) A.FSH
B.LH
C.E
D.T
E.hCG
E.hCG
6 6 2013 38. 对于妊娠滋养细胞肿瘤治疗结束后随访,正确的是(2.5分) A.第一次随访为出院后1周
B.每半年随访一次共5年
C.随访期间应严格避孕至少12个月
D.随访期间可选择口服避孕药、宫内节育器或避孕套避孕
E.随访内容仅为h-CG即可
C.随访期间应严格避孕至少12个月
6 6 2013 39. 绒毛膜癌转移的最主要途径是(2.5分) A.淋巴转移
B.盆腔转移
C.血行转移
D.种植转移
E.局部转移
C.血行转移
6 6 2013 40. 关于子宫下段,描述不正确的是(2.5分) A.常规剖宫产术中选择子宫下段作为切口
B.子宫下段是梗阻性难产中子宫破裂的好发部位
C.由于子宫肌纤维的缩复作用,随产程进展子宫下段愈来愈厚
D.妊娠晚期子宫峡部逐渐伸展形成子宫下段,其长度可达7-10cm
E.由于子宫上、下段的肌壁厚薄不同,在二者间的子宫内面有一环状隆起,称生理性缩复环
C.由于子宫肌纤维的缩复作用,随产程进展子宫下段愈来愈厚
6 6 2014 1. 子宫肌瘤临床分类标准为(2.5分) A.按临床症状严重程度分类
B.按查体所见子宫肌瘤大小分类
C.按子宫肌瘤的数目分类
D.按子宫肌瘤与子宫内膜的关系分类
E.按肌瘤与子宫肌层的关系分类
E.按肌瘤与子宫肌层的关系分类
6 6 2014 2. 子宫肌瘤最常见的并发症是(2.5分) A.红色变性
B.继发贫血
C.浆膜下肌瘤蒂扭转
D.肌瘤压迫输尿管引起肾盂积水
E.肌瘤恶变
B.继发贫血
6 6 2014 3. 关于子宫肌瘤,下述哪项是错误的(2.5分) A.粘膜下肌瘤较常发生月经量过多与不规则出血
B.浆膜下肌瘤较少出现月经过多
C.膀胱充盈时,较大肌瘤可升至腹部,病人因此就医
D.较大浆膜下肌瘤可发生蒂扭转
E.浆膜下肌瘤最易引起不孕
E.浆膜下肌瘤最易引起不孕
6 6 2014 4. 关于子宫肌瘤的手术治疗,下述哪项是不正确的(2.5分) A.近绝经期,子宫如孕2个月大小,症状轻者,可暂不手术
B.肌瘤症状明显,经保守治疗无效者应考虑手术
C.突出于阴道内的粘膜下肌瘤应行手术治疗
D.年轻,未生育过的妇女肌壁间肌瘤不论多发或单发,均应手术
E.疑有恶变者不宜行剔除术
D.年轻,未生育过的妇女肌壁间肌瘤不论多发或单发,均应手术
6 6 2014 5. 以下哪种情况建议行子宫切除术:(2.5分) A.患者30岁,已婚未育,子宫多发肌瘤,子宫增大如孕10周
B.患者60岁,15年前发现子宫肌瘤5cm大小,10年前绝经,此后定期复查B超,提示子宫肌瘤逐渐缩小,现肌瘤约2cm大小
C.患者45岁,已婚已育,子宫多发肌瘤,子宫增大如孕2月,无不适症状
D.患者45岁,已婚已育,子宫多发肌瘤,子宫增大如孕50天大,经量增多2年,Hb65g/L
E.患者45岁,已婚已育,子宫粘膜下肌瘤2cm,经量增多2年,Hb65g/L
D.患者45岁,已婚已育,子宫多发肌瘤,子宫增大如孕50天大,经量增多2年,Hb65g/L
6 6 2014 6. 以下哪项不是子宫内膜异位症好发部位(2.5分) A.子宫直肠陷凹
B.卵巢
C.子宫骶骨韧带
D.输卵管
E.阴道壁
E.阴道壁
6 6 2014 7. 子宫内膜异位症最主要的临床特点是(2.5分) A.月经失调
B.不孕症发生率高
C.痛经和持续性下腹痛
D.咳血
E.腹痛,腹泻或便秘
C.痛经和持续性下腹痛
6 6 2014 8. 卵巢子宫内膜异位囊肿表现为(2.5分) A.囊内液粘稠,咖啡色
B.囊内可见毛发、牙齿
C.囊内液色清,呈淡黄色
D.囊内液粘稠,呈黄色
E.囊内液呈油脂状
A.囊内液粘稠,咖啡色
6 6 2014 9. GnRH-a治疗子宫内膜异位症的机理:(2.5分) A.抑制垂体促性腺激素,导致卵巢分泌性激素增多
B.抑制垂体促性腺激素,导致卵巢分泌性激素减少
C.促进垂体促性腺激素分泌,导致卵巢分泌性激素增多
D.促进垂体促性腺激素分泌,导致卵巢分泌性激素减少
E.以上都不是
B.抑制垂体促性腺激素,导致卵巢分泌性激素减少
6 6 2014 10. 患者40岁,人工流产后经量增多10年,伴痛经进行性加剧,妇科检查:子宫如孕50天大,球形,质硬,活动度欠佳,附件区未及包块,首选诊断:(2.5分) A.子宫肌瘤
B.子宫腺肌病
C.盆腔炎症包块
D.子宫内膜异位症
E.子宫肉瘤
B.子宫腺肌病
6 6 2014 11. 关于胎儿发育,以下说法正确的是:(2.5分) A.16周末,胎儿身长达到16cm,但胎儿仍然没有出现呼吸运动;
B.20周末,胎儿无明显运动,孕妇多数不能感知胎动;
C.28周末,胎儿皮肤呈粉红色,出生后已可存活;
D.32周末,胎儿皮肤呈粉红色,出生后即使积极护理,仍不能存活;
E.36周末,估计体重3000g,出生后不能啼哭和吸吮,生存能力差;
C.28周末,胎儿皮肤呈粉红色,出生后已可存活;
6 6 2014 12. 关于羊水的功能,表述错误的是:(2.5分) A.避免胎儿在子宫内受压;
B.避免胎儿肢体发生黏连;
C.临产时,避免胎儿局部受压引起胎儿窘迫;
D.促进胎肺发育
E.临产后,前羊水囊对宫颈口无压迫扩张作用
E.临产后,前羊水囊对宫颈口无压迫扩张作用
6 6 2014 13. 以下不属于胎儿Manning评分项目的是(2.5分) A.羊水量
B.肌张力
C.胎儿呼吸运动
D.胎心率
E.胎动(30分钟)
D.胎心率
6 6 2014 14. 以下不属于产前诊断适应症范围的是:(2.5分) A.年龄超过35周岁
B.羊水过多
C.孕早期接触过可能导致胎儿先天畸形的物质
D.孕早期保胎病史
E.夫妇有一方患有先天性或遗传性疾病
D.孕早期保胎病史
6 6 2014 15. 胎盘合成的激素中,哪种激素结构与FSH, LH, TSH类似(2.5分) A.E
B.P
C.HCG
D.HPL
E.上述都不是
C.HCG
6 6 2014 16. 骨盆外测量的各条径线,错误的是:(2.5分) A.髂棘间径正常值为23-26cm
B.坐骨结节间径正常值为8.5-9.5cm
C.出口后矢状径正常值为15cm
D.骶耻外径正常值为18-20cm
E.耻骨弓角度正常值为90度
C.出口后矢状径正常值为15cm
6 6 2014 17. 胎心电子监护胎心率(FHR)变化与子宫收缩完全无关的是(2.5分) A.加速
B.早期减速
C.变异减速
D.晚期减速
E.正弦波形
E.正弦波形
6 6 2014 18. 影响分娩的因素中,不包括:(2.5分) A.孕周
B.胎儿
C.产道
D.精神心理因素
E.产力
A.孕周
6 6 2014 19. 关于妊娠期循环系统的生理改变,错误的是(2.5分) A.心电图常出现电轴左偏
B.妊娠32周时新排血量的增加达高峰
C.多数孕妇心尖区可闻及I-II级柔和吹风样舒张期杂音,产后逐渐消失
D.孕妇下肢、外阴及直肠静脉压增高,容易发生下肢、外阴静脉曲张和痔
E.孕妇若长时间仰卧,能引起回心血量减少,心排出量随之而减少使血压下降,称仰卧位低血压综合征
C.多数孕妇心尖区可闻及I-II级柔和吹风样舒张期杂音,产后逐渐消失
6 6 2014 20. 关于子宫下段,描述不正确的是(2.5分) A.常规剖宫产术中选择子宫下段作为切口
B.子宫下段是梗阻性难产中子宫破裂的好发部位
C.由于子宫肌纤维的缩复作用,随产程进展子宫下段愈来愈厚
D.妊娠晚期子宫峡部逐渐伸展形成子宫下段,其长度可达7-10cm
E.由于子宫上、下段的肌壁厚薄不同,在二者间的子宫内面有一环状隆起,称生理性缩复环
C.由于子宫肌纤维的缩复作用,随产程进展子宫下段愈来愈厚
6 6 2014 21. 外阴上皮内非瘤变是一组慢性疾病,下面哪项不是其特征:(2.5分) A.外阴皮肤和粘膜组织发生色素改变;
B.外阴皮肤和粘膜组织发生变性;
C.不典型增生;
D.主要包括外阴硬化性苔癣与外阴鳞状上皮增生;
E.也称慢性外阴营养不良。
C.不典型增生;
6 6 2014 22. 外阴癌中最常见的组织学类型是:(2.5分) A.鳞癌
B.腺癌
C.黑色素瘤
D.基底细胞癌
E.以上都不是
A.鳞癌
6 6 2014 23. 下例哪项与外阴癌病因无关:(2.5分) A.HPV病毒
B.外阴上皮内非瘤变
C.STD疾病
D.HSV病毒
E.性激素缺乏
E.性激素缺乏
6 6 2014 24. 外阴癌的确诊主要依靠:(2.5分) A.临床表现
B.局部组织病理学检查
C.妇科检查
D.影像学检查
E.HPV病毒检测
B.局部组织病理学检查
6 6 2014 25. 宫颈癌的癌前期病变是:(2.5分) A.慢性宫颈炎
B.CIN
C.宫颈息肉
D.HPV感染
E.都不是
B.CIN
6 6 2014 26. 宫颈癌好发部位是:(2.5分) A.宫颈阴道部
B.宫颈阴道上部
C.SCJ
D.宫颈组织学内口
E.都不是
C.SCJ
6 6 2014 27. 宫颈癌转移途径:(2.5分) A.以直接浸润和淋巴转移为主要转移途径,晚期患者可经血液系统转移
B.单纯直接浸润
C.单纯淋巴转移
D.单纯血液系统转移
E.以直接浸润和血液系统转移为主要转移途径
A.以直接浸润和淋巴转移为主要转移途径,晚期患者可经血液系统转移
6 6 2014 28. 有关CIN及宫颈癌,下面哪项不正确?(2.5分) A.CIN及宫颈癌的诊断通过"三阶梯"诊断程序,明确诊断需要宫颈活检病理学检查
B.治疗方法选择根据临床分期、患者状况和医院医疗技术水平及设备条件综合考虑
C.手术和放疗二者对子宫颈癌具有不一样的疗效,均为局部治疗,但副作用相同
D.年青患者手术时可保留卵巢功能
E.都不是
C.手术和放疗二者对子宫颈癌具有不一样的疗效,均为局部治疗,但副作用相同
6 6 2014 29. 不是子宫内膜癌发生的高危因素:(2.5分) A.高血压
B.HPV感染
C.肥胖
D.糖尿病
E.不孕或不育
B.HPV感染
6 6 2014 30. 最多见的子宫内膜癌组织学类型是:(2.5分) A.浆液性腺癌
B.透明细胞癌
C.粘液性腺癌
D.鳞癌
E.子宫内膜样腺癌
E.子宫内膜样腺癌
6 6 2014 31. 妊娠滋养细胞疾病不包括以下哪种疾病(2.5分) A.葡萄胎
B.原发性卵巢绒癌
C.侵蚀性葡萄胎
D.胎盘部位滋养细胞肿瘤
E.绒毛膜癌
B.原发性卵巢绒癌
6 6 2014 32. 完全性葡萄胎的染色体基因组来源于(2.5分) A.父系
B.母系
C.父系和母系
D.母系和子系
E.父系和子系
A.父系
6 6 2014 33. 关于葡萄胎发病的相关因素,下列说法不正确的是(2.5分) A.病因尚未确切,可能与种族、营养状况、经济因素、年龄、细胞遗传等方面有关
B.不管是完全性葡萄胎或部分性葡萄胎,多余的父源基因物质是造成滋养细胞增生的主要原因
C.前次妊娠为葡萄胎者是葡萄胎发病的高危人群
D.高龄女性是葡萄胎发病的高危人群,年龄越小葡萄胎发病率越低
E.饮食中缺乏维生素A者发生葡萄胎几率高
D.高龄女性是葡萄胎发病的高危人群,年龄越小葡萄胎发病率越低
6 6 2014 34. 对卵巢卵黄囊瘤有特异性诊断价值的肿瘤标志物是:(2.5分) A.CA125
B.HCG
C.AFP
D.雌激素
E.LDH
C.AFP
6 6 2014 35. 女性,45岁,检查提示附件区包块,血CA125 900IU/L,血象正常,诊断首先考虑(2.5分) A.卵巢癌
B.子宫内膜异位囊肿
C.卵巢畸胎瘤
D.盆腔炎性包块
E.卵巢内胚窦瘤
A.卵巢癌
6 6 2014 36. 关于人工破膜的描述,下列错误的是:(2.5分) A.宫口扩张3cm或3cm以上可行人工破膜
B.破膜时必须检查有无脐带先露
C.破膜应在宫缩期进行
D.破膜后术者手指应停留在阴道内,经过1~2次宫缩,待胎头入盆后,再将手指取出
E.无头盆不称、胎头已衔接者,可行人工破膜
C.破膜应在宫缩期进行
6 6 2014 37. 临产后子宫收缩特点,下列哪项不正确(2.5分) A.节律性
B.高张性
C.缩复作用
D.极性
E.对称性
B.高张性
6 6 2014 38. 有关产道的描述哪项是错误的?(2.5分) A.产道可分为软产道和骨产道。
B.正常女性骨盆入口平面呈纵椭园型。
C.中骨盆平面是产道中最窄平面。
D.软产道是由子宫下段、宫颈、阴道及盆底软组织构成的弯曲通道。
B.正常女性骨盆入口平面呈纵椭园型。
6 6 2014 39. Which of the following is characteristic of normal labor?(2.5分) A.Moderate bleeding
B.Low-grade fever (37.6℃) as labor progresses
C.Regular contractions start when active phase has been initiated
D.Lack of fetal movement in the later stages
E.Uterine relaxation between contractions
E.Uterine relaxation between contractions
6 6 2014 40. Engagement is best defined as which of the following? (2.5分) A.when the presenting part goes through the pelvic inlet
B.when the greatest biparietal diameter of the fetal head passes the pelvic inlet
C.when the presenting part is level below the ischial spines
D.when the greatest biparietal diameter of the head is level with the ischial spines
E.when the greatest diameter of the fetal presenting part passes through the narrowest and lowest part of the maternal pelvis.
B.when the greatest biparietal diameter of the fetal head passes the pelvic inlet
6 6 2015 1. 子宫肌瘤临床分类标准为(2.5分) A.按临床症状严重程度分类
B.按查体所见子宫肌瘤大小分类
C.按子宫肌瘤的数目分类
D.按子宫肌瘤与子宫内膜的关系分类
E.按肌瘤与子宫肌层的关系分类
E.按肌瘤与子宫肌层的关系分类
6 6 2015 2. 子宫肌瘤最常见的并发症是(2.5分) A.红色变性
B.继发贫血
C.浆膜下肌瘤蒂扭转
D.肌瘤压迫输尿管引起肾盂积水
E.肌瘤恶变
B.继发贫血
6 6 2015 3. 关于子宫肌瘤,下述哪项是错误的(2.5分) A.粘膜下肌瘤较常发生月经量过多与不规则出血
B.浆膜下肌瘤较少出现月经过多
C.膀胱充盈时,较大肌瘤可升至腹部,病人因此就医
D.较大浆膜下肌瘤可发生蒂扭转
E.浆膜下肌瘤最易引起不孕
E.浆膜下肌瘤最易引起不孕
6 6 2015 4. 子宫肌瘤红色样变常发生在(2.5分) A.青春期
B.月经期
C.妊娠期
D.围绝经期
E.绝经后期
C.妊娠期
6 6 2015 5. 以下哪种情况建议行子宫切除术:(2.5分) A.患者30岁,已婚未育,子宫多发肌瘤,子宫增大如孕10周
B.患者60岁,15年前发现子宫肌瘤5cm大小,10年前绝经,此后定期复查B超,提示子宫肌瘤逐渐缩小,现肌瘤约2cm大小
C.患者45岁,已婚已育,子宫多发肌瘤,子宫增大如孕2月,无不适症状
D.患者45岁,已婚已育,子宫多发肌瘤,子宫增大如孕50天大,经量增多2年,Hb65g/L
E.患者45岁,已婚已育,子宫粘膜下肌瘤2cm,经量增多2年,Hb65g/L
D.患者45岁,已婚已育,子宫多发肌瘤,子宫增大如孕50天大,经量增多2年,Hb65g/L
6 6 2015 6. 以下哪项不是子宫内膜异位症好发部位(2.5分) A.子宫直肠陷凹
B.卵巢
C.子宫骶骨韧带
D.输卵管
E.阴道壁
E.阴道壁
6 6 2015 7. 子宫内膜异位症最主要的临床特点是(2.5分) A.月经失调
B.不孕症发生率高
C.痛经和持续性下腹痛
D.咳血
E.腹痛,腹泻或便秘
C.痛经和持续性下腹痛
6 6 2015 8. 33岁妇女,婚后7年未孕,痛经逐年加重,妇科检查:宫骶韧带处可触及黄豆大小结节2个,触痛明显,右附件可及一5*6cm大小囊性块,活动差,最有效的确诊方案:(2.5分) A.B超
B.诊断性刮宫
C.宫腔镜检查
D.腹腔镜手术
E.血CA125
D.腹腔镜手术
6 6 2015 9. GnRH-a治疗子宫内膜异位症的机理:(2.5分) A.抑制垂体促性腺激素,导致卵巢分泌性激素增多
B.抑制垂体促性腺激素,导致卵巢分泌性激素减少
C.促进垂体促性腺激素分泌,导致卵巢分泌性激素增多
D.促进垂体促性腺激素分泌,导致卵巢分泌性激素减少
E.以上都不是
B.抑制垂体促性腺激素,导致卵巢分泌性激素减少
6 6 2015 10. 关于胎儿发育,以下说法正确的是:(2.5分) A.16周末,胎儿身长达到16cm,但胎儿仍然没有出现呼吸运动;
B.20周末,胎儿无明显运动,孕妇多数不能感知胎动;
C.28周末,胎儿皮肤呈粉红色,出生后已可存活;
D.32周末,胎儿皮肤呈粉红色,出生后即使积极护理,仍不能存活;
E.36周末,估计体重3000g,出生后不能啼哭和吸吮,生存能力差;
C.28周末,胎儿皮肤呈粉红色,出生后已可存活;
6 6 2015 11. 关于羊水的功能,表述错误的是:(2.5分) A.避免胎儿在子宫内受压;
B.避免胎儿肢体发生黏连;
C.临产时,避免胎儿局部受压引起胎儿窘迫;
D.促进胎肺发育
E.临产后,前羊水囊对宫颈口无压迫扩张作用
E.临产后,前羊水囊对宫颈口无压迫扩张作用
6 6 2015 12. 以下不属于胎儿Manning评分项目的是(2.5分) A.羊水量
B.肌张力
C.胎儿呼吸运动
D.胎心率
E.胎动(30分钟)
D.胎心率
6 6 2015 13. 以下不属于产前诊断适应症范围的是:(2.5分) A.年龄超过35周岁
B.羊水过多
C.孕早期接触过可能导致胎儿先天畸形的物质
D.孕早期保胎病史
E.夫妇有一方患有先天性或遗传性疾病
D.孕早期保胎病史
6 6 2015 14. 胎盘合成的激素中,哪种激素结构与FSH, LH, TSH类似(2.5分) A.E
B.P
C.HCG
D.HPL
E.上述都不是
C.HCG
6 6 2015 15. 骨盆外测量的各条径线,错误的是:(2.5分) A.髂棘间径正常值为23-26cm
B.坐骨结节间径正常值为8.5-9.5cm
C.出口后矢状径正常值为15cm
D.骶耻外径正常值为18-20cm
E.耻骨弓角度正常值为90度
C.出口后矢状径正常值为15cm
6 6 2015 16. 胎心电子监护胎心率(FHR)变化与子宫收缩完全无关的是(2.5分) A.加速
B.早期减速
C.变异减速
D.晚期减速
E.正弦波形
E.正弦波形
6 6 2015 17. 影响分娩的因素中,不包括:(2.5分) A.孕周
B.胎儿
C.产道
D.精神心理因素
E.产力
A.孕周
6 6 2015 18. 关于妊娠期循环系统的生理改变,错误的是(2.5分) A.心电图常出现电轴左偏
B.妊娠32周时新排血量的增加达高峰
C.多数孕妇心尖区可闻及I-II级柔和吹风样舒张期杂音,产后逐渐消失
D.孕妇下肢、外阴及直肠静脉压增高,容易发生下肢、外阴静脉曲张和痔
E.孕妇若长时间仰卧,能引起回心血量减少,心排出量随之而减少使血压下降,称仰卧位低血压综合征
C.多数孕妇心尖区可闻及I-II级柔和吹风样舒张期杂音,产后逐渐消失
6 6 2015 19. 关于子宫下段,描述不正确的是(2.5分) A.常规剖宫产术中选择子宫下段作为切口
B.子宫下段是梗阻性难产中子宫破裂的好发部位
C.由于子宫肌纤维的缩复作用,随产程进展子宫下段愈来愈厚
D.妊娠晚期子宫峡部逐渐伸展形成子宫下段,其长度可达7-10cm
E.由于子宫上、下段的肌壁厚薄不同,在二者间的子宫内面有一环状隆起,称生理性缩复环
C.由于子宫肌纤维的缩复作用,随产程进展子宫下段愈来愈厚
6 6 2015 20. 外阴上皮内非瘤变是一组慢性疾病,下面哪项不是其特征:(2.5分) A.外阴皮肤和粘膜组织发生色素改变;
B.外阴皮肤和粘膜组织发生变性;
C.不典型增生;
D.主要包括外阴硬化性苔癣与外阴鳞状上皮增生;
E.也称慢性外阴营养不良。
C.不典型增生;
6 6 2015 21. 外阴癌中最常见的组织学类型是:(2.5分) A.鳞癌
B.腺癌
C.黑色素瘤
D.基底细胞癌
E.以上都不是
A.鳞癌
6 6 2015 22. 下例哪项与外阴癌病因无关:(2.5分) A.HPV病毒
B.外阴上皮内非瘤变
C.STD疾病
D.HSV病毒
E.性激素缺乏
E.性激素缺乏
6 6 2015 23. 外阴癌的确诊主要依靠:(2.5分) A.临床表现
B.局部组织病理学检查
C.妇科检查
D.影像学检查
E.HPV病毒检测
B.局部组织病理学检查
6 6 2015 24. 宫颈癌的癌前期病变是:(2.5分) A.慢性宫颈炎
B.CIN
C.宫颈息肉
D.HPV感染
E.都不是
B.CIN
6 6 2015 25. 宫颈癌好发部位是:(2.5分) A.宫颈阴道部
B.宫颈阴道上部
C.SCJ
D.宫颈组织学内口
E.都不是
C.SCJ
6 6 2015 26. 宫颈癌转移途径:(2.5分) A.以直接浸润和淋巴转移为主要转移途径,晚期患者可经血液系统转移
B.单纯直接浸润
C.单纯淋巴转移
D.单纯血液系统转移
E.以直接浸润和血液系统转移为主要转移途径
A.以直接浸润和淋巴转移为主要转移途径,晚期患者可经血液系统转移
6 6 2015 27. 有关CIN及宫颈癌,下面哪项不正确?(2.5分) A.CIN及宫颈癌的诊断通过"三阶梯"诊断程序,明确诊断需要宫颈活检病理学检查
B.治疗方法选择根据临床分期、患者状况和医院医疗技术水平及设备条件综合考虑
C.手术和放疗二者对子宫颈癌具有不一样的疗效,均为局部治疗,但副作用相同
D.年青患者手术时可保留卵巢功能
E.都不是
C.手术和放疗二者对子宫颈癌具有不一样的疗效,均为局部治疗,但副作用相同
6 6 2015 28. 不是子宫内膜癌发生的高危因素:(2.5分) A.高血压
B.HPV感染
C.肥胖
D.糖尿病
E.不孕或不育
B.HPV感染
6 6 2015 29. 最多见的子宫内膜癌组织学类型是:(2.5分) A.浆液性腺癌
B.透明细胞癌
C.粘液性腺癌
D.鳞癌
E.子宫内膜样腺癌
E.子宫内膜样腺癌
6 6 2015 30. Which of following management fits for choriocarcinoma best? (2.5分) A.hysterectomy
B.Chinese medicine
C.immunotherapy
D.chemotherapy
E.radiation therapy
D.chemotherapy
6 6 2015 31. 完全性葡萄胎的染色体基因组来源于(2.5分) A.父系
B.母系
C.父系和母系
D.母系和子系
E.父系和子系
A.父系
6 6 2015 32. 关于葡萄胎发病的相关因素,下列说法不正确的是(2.5分) A.病因尚未确切,可能与种族、营养状况、经济因素、年龄、细胞遗传等方面有关
B.不管是完全性葡萄胎或部分性葡萄胎,多余的父源基因物质是造成滋养细胞增生的主要原因
C.前次妊娠为葡萄胎者是葡萄胎发病的高危人群
D.高龄女性是葡萄胎发病的高危人群,年龄越小葡萄胎发病率越低
E.饮食中缺乏维生素A者发生葡萄胎几率高
D.高龄女性是葡萄胎发病的高危人群,年龄越小葡萄胎发病率越低
6 6 2015 33. 女性,25岁,已婚。停经50余天,阴道不规则流血2周,妇科检查:宫颈蓝,子宫增大如孕3个多月,附件(-),尿HCG(+),B超呈"落雪状图像"改变。最可能的诊断是(2.5分) A.子宫肌瘤
B.葡萄胎
C.子宫内膜癌
D.难免流产
E.先兆流产
B.葡萄胎
6 6 2015 34. 女性,31岁,已婚,1-0-1-1,发现左附件肿块2年,近两年来肿块大小无明显增减,约5cm左右,未处理,平时月经规律,末次月经1周前,5日净。今晨同房后突觉左下腹疼痛来就诊。诊断首先应考虑为(2.5分) A.黄体破裂
B.卵巢肿瘤破裂
C.急性阑尾炎
D.盆腔炎
E.宫外孕
B.卵巢肿瘤破裂
6 6 2015 35. 女性生殖系统恶性肿瘤中死亡率最高的是(2.5分) A.子宫颈癌
B.外阴癌
C.恶性滋养细胞肿瘤
D.子宫内膜癌
E.卵巢上皮癌
E.卵巢上皮癌
6 6 2015 36. Ovarian choriocarcinoma are identified by secretion of which of the following tumor marker? (2.5分) A.HCG
B.CEA
C.AFP
D.CA-125
E.LDH
A.HCG
6 6 2015 37. which of the following does the early pregnancy have in common? (2.5分) A.morning sickness
B.breast enlargement
C.abdomen enlargement
D.amenorrhea (cease of mense)
E.urinary frequency
D.amenorrhea (cease of mense)
6 6 2015 38. The bispinous diameter of a normal pelvis should be at least how many centimeters?(2.5分) A.5 cm
B.8cm
C.10cm
D.11cm
E.12cm
C.10cm
6 6 2015 39. A contraindication to the use of oxytocin for stimulating labor is which of the following?(2.5分) A.fetal demise
B.hypotonic uterine dysfunction
C.hypertonic uterine dysfunction
D.twin gestation
E.estimated fetal weight less than 3kg
C.hypertonic uterine dysfunction
6 6 2015 40. 新生儿阿普加评分法包括下列(2.5分) A.心率、呼吸;
B.皮肤颜色;
C.肌张力;
D.反射;
E.以上全是;
E.以上全是;
6 7 2006 1. 题干:患者,24岁,1-0-2-1,妊娠43周,家中接生婆接生未成功,临产25小时送至我院,入院时孕妇痛苦异常,子宫呈葫芦状,子宫下端压痛,肉眼血尿。血压120/60mmHg,心率95次/分,胎心未闻及。本例诊断无关的项目为: (3.0分) A.滞产
B.过期妊娠
C.子宫先兆破裂
D.泌尿系感染
E.死产
D.泌尿系感染
6 7 2006 2. 此时应采取的处理措施为: (3.0分) A.麻醉下行内转胎位术
B.断头术或碎胎术
C.外转胎位术
D.剖宫产术
E.上肢还纳术
D.剖宫产术
6 7 2006 3. 采取上述处理措施的理由为: (3.0分) A.宫口开全,可行内倒转术
B.诊为先兆子宫破裂,应行剖宫产术
C.因胎儿已死,应行碎胎术或断头术
D.死胎应争取阴道分娩
E.经产妇肩先露应行上肢还纳术
B.诊为先兆子宫破裂,应行剖宫产术
6 7 2006 4. 题干:初产妇28岁,40周妊娠,凌晨3时临产,当晚8时宫口开全,宫缩20″/2′,至10时10分宫缩持续20″/4′,胎心率140次/分,羊水Ⅰ污染,阴道检查胎头棘下+3cm,胎头矢状缝与母体骨盆前后径一致,后囟在前方。最正确的诊断为 (3.0分) A.胎儿宫内窘迫
B.ROA
C.滞产
D.第二产程延长
E.胎头内旋转受阻
D.第二产程延长
6 7 2006 5. 最恰当的处理为(3.0分) A.继续观察
B.腹部加压助娩
C.会阴侧切+产钳助娩
D.给予缩宫素
E.剖宫产
C.会阴侧切+产钳助娩
6 7 2006 6. A prolonged second stage of labor in the primigravid patient lasts longer than:(3.0分) A.1.0 hours
B.2.0 hours
C.8.0 hours
D.6.0 hours
E.4.0 hours
B.2.0 hours
6 7 2006 7. Which of the following patients are more likely than others to have uterine atony and hemorrhage after delivery? (3.0分) A.Prolonged labor
B.Primigravidas
C.Hypertensive disorders
D.Pudendal anesthesia for delivery
E.Obesity
A.Prolonged labor
6 7 2006 8. A 32-year-old (G3P1) at term is admitted in labor with an initial cervical examination of 6cm dilatation, complete effacement, and the vertex at -1 station. Estimated fetal weight is 3.8kg, and her first pregnancy resulted in an uncomplicated vaginal delivery of a 3.8kg infant. After 2 hours, there is no cervical change. An intrauterine pressure catheter is placed. This shows two contractions in a 10-minute period, each with a strength of 30 mmHg. What is this abnormality of labor termed?(3.0分) A.prolonged latent phase
B.active-phase arrest
C.failure of decent
D.arrest of latent phase
E.protraction of descent
B.active-phase arrest
6 7 2006 9. Which of the following best describes a missed abortion? (3.0分) A.It is the death of the fetus that the patient does not realize has occurred.
B.It is the death of the fetus in which the products of conception are retained after the embryo or fetus has died.
C.It is the death of the fetus in which the products of conception are partially expelled.
D.It is the death of the fetus in which the products of conception cause bleeding and there is an open cervical os.
E.It is the death of the fetus in which the products of conception are no longer recognizable.
B.It is the death of the fetus in which the products of conception are retained after the embryo or fetus has died.
6 7 2006 10. 首先考虑可能诊断为(4.0分) A.先兆流产
B.异位妊娠
C.葡萄胎
D.阑尾脓肿破裂
E.右卵巢囊肿蒂扭转
B.异位妊娠
6 7 2006 11. 妊娠晚期羊水量少于多少被称为羊水过少:(3.0分) A.100ML
B.300ML
C.500ML
D.1000ML
E.2000ML
B.300ML
6 7 2006 12. 孕妇28岁,第一胎,孕38周,定期产前检查无异常,今日门诊检查时作NST为阴性,无宫缩。下列处理哪项最适合?(3.0分) A.可确诊为胎儿窘迫
B.应立即行剖宫产术
C.应取胎儿头皮血协助诊断胎儿窘迫
D.应进一步作OCT
E.正常现象,无需处理
D.应进一步作OCT
6 7 2006 13. 预防胎儿窘迫不包括:(3.0分) A.左侧卧位
B.积极防治妊高征
C.防止胎膜早破
D.做阴道检查
E.合理使用催产素
D.做阴道检查
6 7 2006 14. 胎膜早破定义:(3.0分) A.孕37周前出现胎膜破裂
B.临产后出现胎膜破裂
C.胎儿娩出前出现胎膜破裂
D.临产前出现胎膜破裂
E.见红前出现胎膜破裂
D.临产前出现胎膜破裂
6 7 2006 15. 妊娠达到或超过几周称为过期妊娠:(3.0分) A.28周
B.37周
C.40周
D.41周
E.42周
E.42周
6 7 2006 16. 下面哪项不是多胎妊娠终止妊娠的指征:(3.0分) A.胎儿畸形
B.孕40周,B超提示胎盘Ⅲ级
C.孕妇出现浮肿,血压160/110mmHg,尿蛋白++
D.B超提示羊水指数24cm,孕妇呼吸困难
E.孕妇长期卧床保胎,出现便秘
E.孕妇长期卧床保胎,出现便秘
6 7 2006 17. A 37-gestational-week primipara is complicated with moderate preeclampsia. She felt suddenly severe abdominal pain, dizzy, and a little vaginal bleeding. She is pale and her BP is 11/7kPa(83/53mmHg). The obstetrical examination indicates the uterus is stiff and tenderness, the fetus is head presentation, the fetal heart sound is unclear and the cervical os is unopen. The most proper treatment is:(3.0分) A.B ultrasound examination
B.by artificial rupture of membrane to induce labor
C.blood transfusion, fluid replacement and talking with her relatives
D.immediately cesarean section
E.using fetal monitor for fetal heart surveillance
D.immediately cesarean section
6 7 2006 18. 在妊娠期间可以诊断器质性心脏病的体征主要是 :(3.0分) A.心率达120次/分
B.心尖有II级收缩期吹风样杂音
C.心律失常,有期前收缩
D.心尖部有舒张期雷鸣样杂音
E.心界稍向左扩大
D.心尖部有舒张期雷鸣样杂音
6 7 2006 19. 心脏病孕妇最易发生心衰的时间是:(3.0分) A.妊娠足月
B.妊娠32~34周
C.妊娠32~34周及以后
D.妊娠37~40周
E.妊娠早期
C.妊娠32~34周及以后
6 7 2006 20. 关于心脏病患者早孕终止妊娠的指征下述哪项不当:(3.0分) A.有心衰病史者
B.伴有肺动脉高压表现者
C.心功能II级或以上者
D.严重心肌损害者
E.风湿活动期
C.心功能II级或以上者
6 7 2006 21. 初产妇,25岁,曾因感冒诱发心力衰竭,现妊娠13周就诊。以下哪项为正确的临床处理:(3.0分) A.继续妊娠,无特殊处理
B.继续妊娠,口服地高辛预防心衰
C.终止妊娠,行负压吸宫术
D.终止妊娠,行钳刮术
E.继续妊娠,加强产前监护
E.继续妊娠,加强产前监护
6 7 2006 22. A 34-year-old woman is noted to have significant uterine bleeding after a vaginal delivery complicated by placenta abruption.She is noted to be bleeding from multiple venipuncture sites.Which of the following is the best therapy? (3.0分) A.Immediate hysterectomy
B.Packing of the uterus
C.Hypogastric artery ligation
D.Ligation of utero-ovarian ligament
E.Correction of coagulopathy
E.Correction of coagulopathy
6 7 2006 23. 孕38周,诊断为胎盘早剥行剖宫产术,术中见子宫紫色,质软,胎儿胎盘娩出后,大量出血,注射宫缩剂并按摩子宫,出血不止,血压下降,应立即给予: (3.0分) A.止血药物
B.宫腔填塞
C.按摩子宫加热敷
D.子宫切除
E.大量输血补液等待血压上升
D.子宫切除
6 7 2006 24. A 24-year-old primigravid woman, who is intent on breast-feeding, decides on a home delivery. Immediately after the birth of a 4.1-kg (9-lb) infant, the patient bleeds massively from extensive vaginal and cervical lacerations. She is brought to the nearest hospital in shock. Over 2 hours, 9 units of blood are transfused, and the patient's blood pressure returns to a reasonable level. A hemoglobin value the next day is 7.5 g/dL, and 3 units of packed red blood cells are given. The most likely late sequela to consider in this woman is which of the following? (3.0分) A.Hemochromatosis
B.Stein-Leventhal syndrome
C.Sheehan syndrome
D.Simmonds syndrome
E.Cushing syndrome
C.Sheehan syndrome
6 7 2006 25. 三维超声优势切面是:(3.0分) A.纵切面
B.冠状切面
C.横切面
D.多维切面
B.冠状切面
6 7 2006 26. 胎儿双顶径测量超声最佳切面是:(3.0分) A.小脑平面
B.丘脑透明隔腔平面
C.侧脑室平面
D.枕下前囟径平面
B.丘脑透明隔腔平面
6 7 2006 27. 以下哪个超声表现与卵巢恶性肿瘤的超声模式不相符:(3.0分) A.卵巢囊性包块5cm,内见分隔,膈较纤细,且有血流,阻力指数0.67;
B.卵巢囊性实性包块5cm,实性部分见血流,阻力指数0.38;
C.卵巢囊性包块5cm,内见分隔,膈较粗大,血流较丰富,阻力指数0.40;
D.卵巢囊性包块5cm,包膜较厚,内壁乳头状,见血流。
A.卵巢囊性包块5cm,内见分隔,膈较纤细,且有血流,阻力指数0.67;
6 7 2006 28. 胎儿双顶径切面上主要超声标志物是:(3.0分) A.透明隔腔
B.大脑镰
C.第三脑室
D.丘脑
D.丘脑
6 7 2006 29. 子宫输卵管结核首先累及:(3.0分) A.子宫
B.输卵管
C.卵巢
D.宫颈
B.输卵管
6 7 2006 30. 宫腔粘连的HSG表现:(3.0分) A.宫腔呈雕花样不规则充盈缺损
B.宫腔内圆形或类圆形充盈缺损
C.宫腔内多个大小不等充盈缺损似菊花样
D.宫腔扩大,张力低下
A.宫腔呈雕花样不规则充盈缺损
6 7 2006 31. 卵巢纤维瘤的CT特征:(3.0分) A.盆腔内囊性肿块,无腹水,包膜明显强化。
B.盆腔内实性肿块,伴少量积液,强化明显。
C.盆腔内实性肿块,伴少量胸腹水,无明显强化。
D.盆腔内囊性肿块,伴少量胸腹水,无强化。
C.盆腔内实性肿块,伴少量胸腹水,无明显强化。
6 7 2006 32. 卵巢单纯性囊肿的主要表现:(3.0分) A.薄壁、边界清楚、包膜完整、囊液较清的囊性块。
B.单纯的 清澈的包裹性囊性块。
C.厚壁囊性块。
D.囊液黏稠、包膜完整的囊性块。
A.薄壁、边界清楚、包膜完整、囊液较清的囊性块。
6 7 2006 33. 卵巢畸胎瘤的CT特征:(3.0分) A.盆腔内类圆形囊性肿块
B.盆腔内类圆形实性肿块
C.盆腔内类圆形囊实性肿块
D.盆腔内类圆形含脂肪、牙齿或钙化斑块的混杂性肿块
D.盆腔内类圆形含脂肪、牙齿或钙化斑块的混杂性肿块
6 7 2007 1. 子宫输卵管结核首先累及:(2.5分) A.子宫
B.输卵管
C.卵巢
D.宫颈
B.输卵管
6 7 2007 2. 宫腔粘连的HSG表现:(2.5分) A.宫腔呈雕花样不规则充盈缺损
B.宫腔内圆形或类圆形充盈缺损
C.宫腔内多个大小不等充盈缺损似菊花样
D.宫腔扩大,张力低下
A.宫腔呈雕花样不规则充盈缺损
6 7 2007 3. 卵巢畸胎瘤的CT特征:(2.5分) A.盆腔内类圆形囊性肿块
B.盆腔内类圆形实性肿块
C.盆腔内类圆形囊实性肿块
D.盆腔内类圆形含脂肪、牙齿或钙化斑块的混杂性肿块
D.盆腔内类圆形含脂肪、牙齿或钙化斑块的混杂性肿块
6 7 2007 4. 麦格氏综合征(meigs syndrome)是指(2.5分) A.卵巢纤维瘤伴胸腹水形成。
B.卵巢转移瘤伴胸腹水形成。
C.肿瘤伴胸腹水形成即为麦格氏综合征。
D.恶性肿瘤伴胸腹水形成
A.卵巢纤维瘤伴胸腹水形成。
6 7 2007 5. 卵巢恶性肿瘤多表现为(2.5分) A.盆腔囊实性肿块,CA-125正常
B.盆腔内实性为主肿块,密度不匀,壁厚伴腹水,伴种植转移,肿块强化明显
C.盆腔内囊实性肿块含有少量脂肪
D.盆腔内实性肿块无明显强化
B.盆腔内实性为主肿块,密度不匀,壁厚伴腹水,伴种植转移,肿块强化明显
6 7 2007 6. 卵巢良性肿瘤的主要CT表现:(2.5分) A.盆腔内实性肿块,密度不匀,有低密度坏死区。
B.盆腔内囊性或实性肿块,密度均匀一致,表面光滑,边界清楚。
C.盆腔内囊实性肿块,周边粘连明显。
D.盆腔内与子宫相连的肿块,与子宫同步强化。
B.盆腔内囊性或实性肿块,密度均匀一致,表面光滑,边界清楚。
6 7 2007 7. Each of the following is a risk factor for abruptio placentae EXCEPT:(2.5分) A.Hyptension
B.Cigarette smoking
C.Trauma
D.Cocaine use
E.Primiparity
E.Primiparity
6 7 2007 8. A 24-year-old G2 P1 woman at 39 weeks'gestation presents with painful uterine contractions.She also complains of dark ,vaginal blood mixed with some mucus .Which of the following describes the most likely etiology of her bleeding? (2.5分) A.Placenta previa
B.Placenta abruption
C.Bloody show
D.Vasa previa
E.Cervical laceration
C.Bloody show
6 7 2007 9. In which of the following conditions would consumptive coagulopathy most likely be seen? (2.5分) A.Placental abruption
B.Placenta previa
C.Gestational diabetes
D.Multifetal gestation
E.Gestational trophoblastic disease
A.Placental abruption
6 7 2007 10. 孕38周,诊断为胎盘早剥行剖宫产术,术中见子宫紫色,质软,胎儿胎盘娩出后,大量出血,注射宫缩剂并按摩子宫,出血不止,血压下降,应立即给予: (2.5分) A.止血药物
B.宫腔填塞
C.按摩子宫加热敷
D.子宫切除
E.大量输血补液等待血压上升
D.子宫切除
6 7 2007 11. 中央性前置胎盘大出血的治疗方法哪项是正确的(2.5分) A.如为臀位,行足牵引
B.催产素引产
C.胎头吸引助产
D.剖宫产
E.人工破膜,头皮钳牵引压迫止血
D.剖宫产
6 7 2007 12. 在耻骨联合上方听到胎盘杂音时,胎盘应附着在(2.5分) A.子宫底部
B.子宫上段前壁
C.子宫上段后壁
D.子宫下段前壁
E.子宫下段后壁
D.子宫下段前壁
6 7 2007 13. 下列哪项不符合前置胎盘的表现(2.5分) A.先露下降受阻
B.无痛性阴道流血
C.子宫下段闻及胎盘血流音
D.子宫张力较高,胎心音不易闻及
E.宫底高度与孕周相符
D.子宫张力较高,胎心音不易闻及
6 7 2007 14. 胎盘早剥哪项是正确的(2.5分) A.先期待疗法后考虑终止妊娠
B.多无诱因,往往在半夜阴道出血
C.易发生产后出血,为软产道损伤所致
D.贫血程度与阴道出血量成正比
E.子宫坚硬压痛,宫底逐渐升高胎心不清
E.子宫坚硬压痛,宫底逐渐升高胎心不清
6 7 2007 15. Which of the following is the most common cause of postpartum hemorrhage after a cesarean delivery ?(2.5分) A.Uterine atony
B.Uterine laceration
C.Coagulopathy
D.Uterine inversion
E.Retained placenta
A.Uterine atony
6 7 2007 16. Which of the following diseases rarely leads to postpartum homarrhage?(2.5分) A.twin pregnancy
B.polyhydramnios
C.premature rupture of membrane
D.placenta previa
E.severe preeclampsia
C.premature rupture of membrane
6 7 2007 17. 女,25岁。孕40周,初孕,规律宫缩2小时来院,当时宫口扩张4cm,因宫缩强,半小时后宫口开全,第二产程仅15分钟即顺利娩出一男婴,胎儿娩出后即有鲜红血流出,5分钟后胎盘自然娩出。此后出血量仍较多,有血块。此时分析其出血原因最可能为(2.5分) A.子宫收缩乏力
B.胎膜残留
C.胎盘残留
D.宫颈裂伤
E.凝血功能障碍
D.宫颈裂伤
6 7 2007 18. 初孕妇,因第二产程延长,行左侧会阴切开+低位产钳助产,娩出4000g一活婴,产后2小时伤口疼痛,肛门坠胀并有便意,大便常规检验正常。体检:贫血貌,血压12/8kPa(90/60mmHg),左会阴稍肿,阴道出血不多。下列哪项诊断可能性最大(2.5分) A.菌痢
B.宫颈撕裂
C.阴道壁血肿
D.会阴Ⅲ度破裂
E.肠胃炎
C.阴道壁血肿
6 7 2007 19. 胎盘未剥离时过早挤揉子宫可造成(2.5分) A.胎盘残留
B.胎盘粘连
C.胎盘剥离不全
D.胎盘嵌顿
E.胎盘植入
C.胎盘剥离不全
6 7 2007 20. 28岁初产妇,临产后静脉滴注催产素,破膜后不久出现烦躁不安、呛咳、呼吸困难、紫绀,10分钟后死亡。本例最可能的诊断是(2.5分) A.低纤维蛋白原血症
B.子痫
C.羊水栓塞
D.重型胎盘早剥
E.子宫破裂
C.羊水栓塞
6 7 2007 21. 可引起产科休克的疾病不包括 (2.5分) A.产后出血
B.胎盘早剥
C.羊水栓塞
D.早产
E.子宫破裂
D.早产
6 7 2007 22. 经产妇36岁,孕40周,晨3时突然大量阴道出血,急诊来院,体检:血压16/10kPa(120/75mmHg),尿蛋白(-),腹部检查:子宫高35cm,胎头高浮,子宫前壁无压痛。阴道检查:阴道内有手拳大的凝血块,宫颈软,宫口开大一指,先露部未及胎盘组织。该产妇分娩后5分钟突然发生烦躁不安,寒战,呕吐,咳嗽,呼吸困难,发绀,血压10.7/5.3kPa (80/40mmHg),脉细弱,首先应考虑下列何种情况 (2.5分) A.失血性休克
B.脑血管意外
C.羊水栓塞
D.感染与休克
E.子宫破裂
C.羊水栓塞
6 7 2007 23. 羊水栓塞最早出现的症状是(2.5分) A.急性左心衰竭
B.急性肝功能衰竭
C.急性肾功能衰竭
D.急性呼吸衰竭
E.急性DIC
D.急性呼吸衰竭
6 7 2007 24. A 38-year-old G3P3 begins to breast-feed her 5-day-old infant. The baby latches on appropriately and begins to suckle. In the mother, which of the following is a response to suckling?(2.5分) A.Decrease of oxytocin
B.Increase of prolactin-inhibiting factor
C.Increase of hypothalamic dopamine
D.Increase of hypothalamic prolactin
E.Increase of luteinizing hormone-releasing factor
D.Increase of hypothalamic prolactin
6 7 2007 25. A 21-year-old G2P2 calls her physician 7 days postpartum because she is concerned that she is still bleeding from the vagina. She describes the bleeding as light pink to bright red and less heavy than the first few days postdelivery. She denies fever or any cramping pain. On examination she is afebrile and has an appropriately sized, nontender uterus. The vagina contains about 10 cc of old, dark blood. The cervix is closed. Which of the following is the most appropriate treatment ?(2.5分) A.Antibiotics for endometritis
B.High-dose oral estrogen for placental subinvolution
C.Oxytocin for uterine atony
D.Suction dilation and curettage for retained placenta
E.Reassurance
E.Reassurance
6 7 2007 26. 下列哪项是正常产褥期的表现(2.5分) A.产后24小时体温超过38°
B.产后第一天宫底达脐平
C.产后脉搏一般偏快
D.产后24小时白细胞应恢复至正常范围
E.产后二周恶露开始转为浆液性
B.产后第一天宫底达脐平
6 7 2007 27. 产褥病率的定义是(2.5分) A.产后24小时应每4小时测体温一次,体温两次达到或超过38℃者
B.产后10天每4小时测体温一次,体温两次达到或超过38℃者
C.产后每4小时测体温一次,体温两次达到38℃者
D.产后24小时至10天内,用口表每日测量体温4次,有两次≥38℃者
E.产褥期内两次体温达到或超过38℃者
D.产后24小时至10天内,用口表每日测量体温4次,有两次≥38℃者
6 7 2007 28. Which of the following best describes a missed abortion? (2.5分) A.It is the death of the fetus that the patient does not realize has occurred.
B.It is the death of the fetus in which the products of conception are retained after the embryo or fetus has died.
C.It is the death of the fetus in which the products of conception are partially expelled.
D.It is the death of the fetus in which the products of conception cause bleeding and there is an open cervical os.
E.It is the death of the fetus in which the products of conception are no longer recognizable.
B.It is the death of the fetus in which the products of conception are retained after the embryo or fetus has died.
6 7 2007 29. The majority of ectopic gestations were located in: (2.5分) A.ovary
B.cervix
C.the ampullary portion of the fallopian tube
D.the isthmus portion of the fallopian tube
E.the fimbrial region of the fallopian tube
C.the ampullary portion of the fallopian tube
6 7 2007 30. A 33-year-old woman at 10 weeks' gestation complains of vaginal bleeding and passage of a whitish substance along with something "meatlike." She continues to have cramping, and her cervix is 2 cm dilated, Which of the following is best treatment?(2.5分) A.Laparoscopy
B.Follow up hCG level in 48 hours
C.Cervical cerclage
D.Dilation and curettage of uterus
E.Expectant management
D.Dilation and curettage of uterus
6 7 2007 31. Among those listed below, what is the best therapy for the patient?(2.5分) A.phenothiazines
B.hyonosis
C.IV hydration
D.psychiatric referral
E.outpatient antiemetic therapy
C.IV hydration
6 7 2007 32. 下列哪项不是输卵管妊娠的结局:(2.5分) A.输卵管妊娠流产
B.输卵管妊娠破裂
C.妊娠囊经宫腔由阴道排出
D.持续性异位妊娠
E.继发腹腔妊娠
C.妊娠囊经宫腔由阴道排出
6 7 2007 33. 女,25岁,G5P0,停经45天,阴道少量流血1周,右下腹剧痛1天。体检:面色苍白,P120次/分,血压80/50mmHg,腹部压痛(+),反跳痛(+),移动性浊音(+)。妇科检查:外阴少量暗红色分泌物,宫颈光,举痛,子宫前位,略大,右附件区可及4×4×5cm3包块,触痛明显,左侧无殊。下列哪项检查对诊断及评估病情不是必需?(2.5分) A.尿HCG
B.盆腔B超
C.经阴道后穹窿穿刺
D.盆腔CT
E.血常规
D.盆腔CT
6 7 2007 34. 关于羊水来源下列哪项是错误的?(2.5分) A.妊娠早期.羊水是母体血清经胎膜入羊膜腔透析液
B.经脐带的华通胶
C.胎盘表面羊膜
D.妊娠早期还可通过胎儿尿液
E.通过未角化的胎儿皮肤漏出
D.妊娠早期还可通过胎儿尿液
6 7 2007 35. 下列那种情况不是胎儿窘迫的临床征象:(2.5分) A.频繁的变异减速
B.晚期减速
C.羊水Ⅲ度
D.胎动减少明显
E.早期减速
E.早期减速
6 7 2007 36. 孕妇28岁,第一胎,孕38周,定期产前检查无异常,今日门诊检查时作NST为阴性,无宫缩。下列处理哪项最适合?(2.5分) A.可确诊为胎儿窘迫
B.应立即行剖宫产术
C.应取胎儿头皮血协助诊断胎儿窘迫
D.应进一步作OCT
E.正常现象,无需处理
D.应进一步作OCT
6 7 2007 37. 检查胎儿头皮血PH,诊断胎儿窘迫的指标:(2.5分) A.PH<7.00
B.PH<7.10
C.PH<7.20
D.PH<7.30
E.PH<7.35
C.PH<7.20
6 7 2007 38. 下列哪种情况是早产最常见原因: (2.5分) A.胎膜早破
B.妊娠合并中度贫血
C.双胎
D.羊水过多
E.ABO血型不合
A.胎膜早破
6 7 2007 39. 胎膜早破的患者终止妊娠的指征: (2.5分) A.出现宫内感染
B.临产
C.胎儿未成熟
D.羊水过多
E.阴道少量出血
A.出现宫内感染
6 7 2007 40. 胎膜早破定义: (2.5分) A.孕37周前出现胎膜破裂
B.临产后出现胎膜破裂
C.胎儿娩出前出现胎膜破裂
D.临产前出现胎膜破裂
E.见红前出现胎膜破裂
D.临产前出现胎膜破裂
6 7 2008 1. A 37-gestational-week primipara is complicated with moderate preeclampsia. She felt suddenly severe abdominal pain, dizzy, and a little vaginal bleeding. She is pale and her BP is 11/7kPa(83/53mmHg). The obstetrical examination indicates the uterus is stiff and tenderness, the fetus is head presentation, the fetal heart sound is unclear and the cervical os is unopen. The most proper treatment is:(2.0分) A.B ultrasound examination
B.by artificial rupture of membrane to induce labor
C. blood transfusion, fluid replacement and talking with her relatives
D. immediately cesarean section
E. using fetal monitor for fetal heart surveillance
D. immediately cesarean section
6 7 2008 2. Which one doesn't belong to patients with preeclampsia in the following symptoms? (2.0分) A.Dizziness
B.Emesis
C.chest distress
D.blurred vision
E.headache
C.chest distress
6 7 2008 3. Which of the following symptoms is NOT due to magnesium sulfate toxicity? (2.0分) A.Respiratory depression
B.Central nervous system depression
C.Cardiac depression
D.Loss of deep tendon reflexes
E.Edema
E.Edema
6 7 2008 4. 硫酸镁治疗妊娠期高血压疾病剂量过大时,最先出现的毒性反应是:(2.0分) A.头晕、血压过低
B.呼吸减慢
C.心率减慢
D.膝反射减退
E.尿量过少
D.膝反射减退
6 7 2008 5. 妊娠期高血压疾病的基本病理变化:(2.0分) A.过度水钠储溜
B.全身小动脉痉挛
C.血液浓缩
D.凝血功能障碍,慢性DIC
E.患者对血管紧张素Ⅱ的敏感性增高
B.全身小动脉痉挛
6 7 2008 6. 不是胎儿宫内生长发育受限的因素:(2.0分) A.胎儿自身发育缺陷
B.孕母妊娠合并症
C.遗传因素
D.营养因素
E.生活习惯
E.生活习惯
6 7 2008 7. 巨大儿阴道分娩的并发症不包括:(2.0分) A.软产道裂伤
B.尾骨骨折
C.肩难产
D.羊水栓塞
E.新生儿产伤
D.羊水栓塞
6 7 2008 8. 下列关于巨大儿肩难产的处理,错误的是:(2.0分) A.抑制宫缩
B.屈大腿
C.压前肩
D.牵后臂
E.做充分大的会阴后一斜切开
A.抑制宫缩
6 7 2008 9. 关于死胎,下列哪项是不正确的:(2.0分) A.死胎在宫腔内停留过节,能引起母体凝血功能障碍
B.胎儿死亡后的80%在2-3周内自然娩出
C.B超发现胎心和胎动消失是诊断死胎的可靠依据
D.发现死胎后,需等待2-3周后,再行处理
E.死胎一确诊,应予引产
D.发现死胎后,需等待2-3周后,再行处理
6 7 2008 10. 在妊娠期间可以诊断器质性心脏病的体征主要是(2.0分) A.心率达120次/分
B.心尖有II级收缩期吹风样杂音
C.心律失常,有期前收缩
D.心尖部有舒张期雷鸣样杂音
E.心界稍向左扩大
D.心尖部有舒张期雷鸣样杂音
6 7 2008 11. 妊娠合并心脏病对胎儿的影响,下列那项是正确的:(2.0分) A.心脏代偿功能良好的孕妇,也易引起死胎
B.二尖瓣狭窄手术后已恢复工作的孕妇,易发生早产
C.心功能III级以上的孕妇,胎儿窘迫发生率高
D.妊娠合并心脏病孕妇和其胎儿的预后均很差
E.单有房间隔缺损的孕妇易发生胎儿宫内发育迟缓
C.心功能III级以上的孕妇,胎儿窘迫发生率高
6 7 2008 12. 心脏病患者,现妊娠8周,出现急性心力衰竭,最好的处理方法是:(2.0分) A.即刻终止妊娠
B.控制心力衰竭后继续妊娠
C.控制心力衰竭后,吸宫终止妊娠
D.控制心力衰竭后,剖宫取胎终止妊娠
E.边控制心衰,边终止妊娠
C.控制心力衰竭后,吸宫终止妊娠
6 7 2008 13. 促使心脏病孕妇死亡的主要因素是:(2.0分) A.心脏病病程长
B.产程中用力过度致心衰
C.孕妇年龄过大
D.心衰与感染
E.产后哺乳致心衰
D.心衰与感染
6 7 2008 14. 对于妊娠合并肝炎,下述处理正确的是:(2.0分) A.妊娠早期保胎
B.妊娠中期需终止妊娠
C.妊娠晚期及早终止妊娠
D.终止妊娠前应用维生素K
E.应以剖宫产终止妊娠
D.终止妊娠前应用维生素K
6 7 2008 15. 乙型肝炎孕妇分娩的新生儿注射乙肝疫苗时间为出生后(2.0分) A.0、1、3个月
B.1、2、6个月
C.1、3、6个月
D.0、2、6个月
E.0、1、6个月
E.0、1、6个月
6 7 2008 16. 下列哪项不属于乙型病毒性肝炎的母婴传播方式?(2.0分) A.粪-口传播
B.娩出时接触母亲的产道分泌液或血液
C.乳汁传染
D. 母婴垂直传播
E.密切生活接触传染
A.粪-口传播
6 7 2008 17. 糖筛查时间常在何时进行:(2.0分) A.16-20周
B.20-24周
C.24-28周
D.28-30周
E.32-34周
C.24-28周
6 7 2008 18. 三维超声优势切面是:(2.0分) A.纵切面;
B.冠状切面;
C.横切面;
D.多维切面
B.冠状切面;
6 7 2008 19. 胎儿双顶径测量超声最佳切面是:(2.0分) A.小脑平面;
B.丘脑透明隔腔平面;
C.侧脑室平面;
D.枕下前囟径平面
B.丘脑透明隔腔平面;
6 7 2008 20. 以下哪个超声表现与卵巢恶性肿瘤的超声模式不相符:(2.0分) A.卵巢囊性包块5cm,内见分隔,膈较纤细,且有血流,阻力指数0.67;
B.卵巢囊性实性包块5cm,实性部分见血流,阻力指数0.38;
C.卵巢囊性包块5cm,内见分隔,膈较粗大,血流较丰富,阻力指数0.40;
D.卵巢囊性包块5cm,包膜较厚,内壁乳头状,见血流。
A.卵巢囊性包块5cm,内见分隔,膈较纤细,且有血流,阻力指数0.67;
6 7 2008 21. 胎儿双顶径切面上主要超声标志物是:(2.0分) A.透明隔腔;
B.大脑镰;
C.第三脑室;
D.丘脑
C.第三脑室;
6 7 2008 22. 患者,24岁,1-0-2-1,妊娠43周,家中接生婆接生未成功,临产25小时送至我院,入院时孕妇痛苦异常,子宫呈葫芦状,子宫下端压痛,肉眼血尿。血压120/60mmHg,心率95次/分,胎心未闻及。本例诊断无关的项目为: (2.0分) A.滞产
B.过期妊娠
C.子宫先兆破裂
D.泌尿系感染
E.死产
D.泌尿系感染
6 7 2008 23. 上题,此时应采取的处理措施为: (2.0分) A.麻醉下行内转胎位术
B.断头术或碎胎术
C.外转胎位术
D.剖宫产术
E.上肢还纳术
D.剖宫产术
6 7 2008 24. 上题,采取上述处理措施的理由为: (2.0分) A.宫口开全,可行内倒转术
B.诊为先兆子宫破裂,应行剖宫产术
C.因胎儿已死,应行碎胎术或断头术
D.死胎应争取阴道分娩
E.经产妇肩先露应行上肢还纳术
B.诊为先兆子宫破裂,应行剖宫产术
6 7 2008 25. A prolonged second stage of labor in the primigravid patient lasts longer than :(2.0分) A.1.0 hours
B.2.0 hours
C.8.0 hours
D.6.0 hours
E.4.0 hours
B.2.0 hours
6 7 2008 26. 某孕妇,39周妊娠,GP,先露头,胎心正常,胎膜未破,骨盆外测量,骶耻外径17.5cm,余正常,估计胎儿3300g左右。下列哪项处理是错误的 (2.0分) A.可待宫口开全后产钳助产
B.可给予试产机会
C.必须剖宫产
D.宫口开全后可行胎头吸引
E.NST监护及B超检查
C.必须剖宫产
6 7 2008 27. 上题,过10小时后,宫缩规律为35″~40″/2′~3′,先露棘上2cm,宫口开大3cm,胎膜未破,下列哪项处理最恰当(2.0分) A.剖宫产
B.吸氧
C.肌注安定
D.行人工破膜后再试产2~4小时
E.左侧卧位并行胎儿监护
D.行人工破膜后再试产2~4小时
6 7 2008 28. 上题,上述处理后4小时,宫口仍开大3cm,先露棘上2cm,,胎心率正常,问采取下列哪项处理最恰当(2.0分) A.CST监护
B.继续试产
C.剖宫产
D.吸氧
E.肌注哌替啶
C.剖宫产
6 7 2008 29. Each of the following is a risk factor for abruptio placentae EXCEPT: (2.0分) A.Hyptension
B.Cigarette smoking
C.Trauma
D.Cocaine use
E.Primiparity
E.Primiparity
6 7 2008 30. A 25-year-old woman at 34 weeks' gestation is noted to have a placenta previa, after she presented with vaginal bleeding and has undergone sonography. At 37 weeks, she has a scheduled cesarean. Upon cesarean section, bluish tissue densely adherent between the uterus and maternal ladder is noted. Which of the following is the most likely diagnosis? (2.0分) A.Placenta accreta
B.Placenta melanoma
C.Placenta percreta
D.Placental polyp
E.Placenta increta
C.Placenta percreta
6 7 2008 31. 孕38周,诊断为胎盘早剥行剖宫产术,术中见子宫紫色,质软,胎儿胎盘娩出后,大量出血,注射宫缩剂并按摩子宫,出血不止,血压下降,应立即给予: (2.0分) A.止血药物
B.宫腔填塞
C.按摩子宫加热敷
D.子宫切除
E.大量输血补液等待血压上升
D.子宫切除
6 7 2008 32. Which of the followings is not the complication of severe placenta abruption? (2.0分) A.acute renal failure
B.coagulation dysfunction
C.postpartum hemorrhage
D.hypertension
E.DIC
D.hypertension
6 7 2008 33. 在耻骨联合上方听到胎盘杂音时,胎盘应附着在(2.0分) A.子宫底部
B.子宫上段前壁
C.子宫上段后壁
D.子宫下段前壁
E.子宫下段后壁
D.子宫下段前壁
6 7 2008 34. 完全性前置胎盘不可能发生在下述何种情况 (2.0分) A.胎位LST,单臀,先露未入盆
B.胎位RSCP
C.胎位LOP,胎头未入盆
D.胎位ROA ,先露已衔接
E.胎位LOT ,先露高浮
D.胎位ROA ,先露已衔接
6 7 2008 35. 下列哪项不符合前置胎盘的表现(2.0分) A.先露下降受阻
B.无痛性阴道流血
C.子宫下段闻及胎盘血流音
D.子宫张力较高,胎心音不易闻及
E.宫底高度与孕周相符
D.子宫张力较高,胎心音不易闻及
6 7 2008 36. 胎盘早剥哪项是正确的(2.0分) A.先期待疗法后考虑终止妊娠
B.多无诱因,往往在半夜阴道出血
C.易发生产后出血,为软产道损伤所致
D.贫血程度与阴道出血量成正比
E.子宫坚硬压痛,宫底逐渐升高胎心不清
E.子宫坚硬压痛,宫底逐渐升高胎心不清
6 7 2008 37. 下列哪种情况不是重度胎盘早剥的临床表现(2.0分) A.剧烈腹痛后阴道流血
B.阴道出血量与全身症状不成正比
C.子宫底升高
D.子宫板状
E.胎位,胎心清晰
E.胎位,胎心清晰
6 7 2008 38. A 24-year-old primigravid woman, who is intent on breast-feeding, decides on a home delivery. Immediately after the birth of a 4.1-kg (9-lb) infant, the patient bleeds massively from extensive vaginal and cervical lacerations. She is brought to the nearest hospital in shock. Over 2 hours, 9 units of blood are transfused, and the patient's blood pressure returns to a reasonable level. A hemoglobin value the next day is 7.5 g/dL, and 3 units of packed red blood cells are given. The most likely late sequela to consider in this woman is which of the following?(2.0分) A.Hemochromatosis
B.Stein-Leventhal syndrome
C.Sheehan syndrome
D.Simmonds syndrome
E.Cushing syndrome
C.Sheehan syndrome
6 7 2008 39. Which of the following diseases rarely leads to postpartum homarrhage? (2.0分) A.twin pregnancy
B.polyhydramnios
C.premature rupture of membrane
D.placenta previa
E.severe preeclampsia
C.premature rupture of membrane
6 7 2008 40. 女,25岁。孕40周,初孕,规律宫缩2小时来院,当时宫口扩张4cm,因宫缩强,半小时后宫口开全,第二产程仅15分钟即顺利娩出一男婴,胎儿娩出后即有鲜红血流出,5分钟后胎盘自然娩出。此后出血量仍较多,有血块。此时分析其出血原因最可能为 (2.0分) A.子宫收缩乏力
B.胎膜残留
C.胎盘残留
D.宫颈裂伤
E.凝血功能障碍
D.宫颈裂伤
6 7 2008 41. 初孕妇,因第二产程延长,行左侧会阴切开+低位产钳助产,娩出4000g一活婴,产后2小时伤口疼痛,肛门坠胀并有便意,大便常规检验正常。体检:贫血貌,血压12/8kPa(90/60mmHg),左会阴稍肿,阴道出血不多。下列哪项诊断可能性最大 (2.0分) A.菌痢
B.宫颈撕裂
C.阴道壁血肿
D.会阴Ⅲ度破裂
E.肠胃炎
C.阴道壁血肿
6 7 2008 42. 胎盘未剥离时过早挤揉子宫可造成(2.0分) A.胎盘残留
B.胎盘粘连
C.胎盘剥离不全
D.胎盘嵌顿
E.胎盘植入
C.胎盘剥离不全
6 7 2008 43. 28岁初产妇,临产后静脉滴注催产素,破膜后不久出现烦躁不安、呛咳、呼吸困难、紫绀,10分钟后死亡。本例最可能的诊断是(2.0分) A.低纤维蛋白原血症
B.子痫
C.羊水栓塞
D.重型胎盘早剥
E.子宫破裂
C.羊水栓塞
6 7 2008 44. 下述哪项不是抢救羊水栓塞的措施 (2.0分) A.抗循环衰竭
B.抗呼吸衰竭
C.纠正D IC 及继发纤溶
D.在第一产程者应加强催产素应用,促使其尽早分娩
E.在第二产程发生者可根据情况经阴道助产
D.在第一产程者应加强催产素应用,促使其尽早分娩
6 7 2008 45. 羊水栓塞早期相符的症状是(2.0分) A.有妊高征病史
B.突然剧烈腹痛
C.阴道出血与休克成反比
D.呼吸困难,咳嗽,发绀等呼吸衰竭 状态
E.子宫呈板状,胎心消失
D.呼吸困难,咳嗽,发绀等呼吸衰竭 状态
6 7 2008 46. A 38-year-old G3P3 begins to breast-feed her 5-day-old infant. The baby latches on appropriately and begins to suckle. In the mother, which of the following is a response to suckling? (2.0分) A.Decrease of oxytocin
B.Increase of prolactin-inhibiting factor
C.Increase of hypothalamic dopamine
D.Increase of hypothalamic prolactin
E.Increase of luteinizing hormone-releasing factor
D.Increase of hypothalamic prolactin
6 7 2008 47. A 34-year-old G1P1 who delivered her first baby 5 weeks ago calls your office and asks to speak with you. She tells you that she is feeling very overwhelmed and anxious. She feels that she cannot do anything right and feels sad throughout the day. She tells you that she finds herself crying all the time and is unable to sleep at night. Which of the following is the most likely diagnosis?(2.0分) A.Postpartum depression
B.Maternity blues
C.Postpartum psychosis
D.Bipolar disease
E.Postpartum blues
A.Postpartum depression
6 7 2008 48. 产褥病率的定义是(2.0分) A.产后24小时应每4小时测体温一次,体温两次达到或超过38℃者
B.产后10天每4小时测体温一次,体温两次达到或超过38℃者
C.产后每4小时测体温一次,体温两次达到38℃者
D.产后24小时至10天内,用口表每日测量体温4次,有两次≥38℃者
E.产褥期内两次体温达到或超过38℃者
D.产后24小时至10天内,用口表每日测量体温4次,有两次≥38℃者
6 7 2008 49. 子宫输卵管结核首先累及:(2.0分) A.子宫
B.输卵管
C.卵巢
D.宫颈
B.输卵管
6 7 2008 50. 卵巢纤维瘤的CT特征:(2.0分) A.盆腔内囊性肿块,无腹水,包膜明显强化。
B.盆腔内实性肿块,伴少量积液,强化明显。
C.盆腔内实性肿块,伴少量胸腹水,无明显强化。
D.盆腔内囊性肿块,伴少量胸腹水,无强化。
C.盆腔内实性肿块,伴少量胸腹水,无明显强化。
6 7 2009 1. 患者,24岁,1-0-2-1,妊娠43周,家中接生婆接生未成功,临产25小时送至我院,入院时孕妇痛苦异常,子宫呈葫芦状,子宫下端压痛,肉眼血尿。血压120/60mmHg,心率95次/分,胎心未闻及。本例诊断无关的项目为(2.5分) A.滞产
B.过期妊娠
C.子宫先兆破裂
D.泌尿系感染
E.死产
D.泌尿系感染
6 7 2009 2. 初产妇28岁,40周妊娠,凌晨3时临产,当晚8时宫口开全,宫缩20″/2′,至10时10分宫缩持续20″/4′,胎心率140次/分,羊水Ⅰ污染,阴道检查胎头棘下+3cm,胎头矢状缝与母体骨盆前后径一致,后囟在前方。最正确的诊断为 (2.5分) A.胎儿宫内窘迫
B.ROA
C.滞产
D.第二产程延长
E.胎头内旋转受阻
D.第二产程延长
6 7 2009 3. A prolonged second stage of labor in the primigravid patient lasts longer than :(2.5分) A.1.0 hours
B.2.0 hours
C.8.0 hours
D.6.0 hours
E.4.0 hours
B.2.0 hours
6 7 2009 4. 某孕妇,39周妊娠,GP,先露头,胎心正常,胎膜未破,骨盆外测量,骶耻外径17.5cm,余正常,估计胎儿3300g左右。下列哪项处理是错误的 (2.5分) A.可待宫口开全后产钳助产
B.可给予试产机会
C.必须剖宫产
D.宫口开全后可行胎头吸引
E.NST监护及B超检查
C.必须剖宫产
6 7 2009 5. A 32-year-old (G3P1) at term is admitted in labor with an initial cervical examination of 6cm dilatation, complete effacement, and the vertex at -1 station. Estimated fetal weight is 3.8kg, and her first pregnancy resulted in an uncomplicated vaginal delivery of a 3.8kg infant. After 2 hours, there is no cervical change. An intrauterine pressure catheter is placed. This shows two contractions in a 10-minute period, each with a strength of 30 mmHg. What is this abnormality of labor termed? (2.5分) A.prolonged latent phase
B.active-phase arrest
C.failure of decent
D.arrest of latent phase
E.protraction of descent
B.active-phase arrest
6 7 2009 6. The following auxiliary examinations are performed on preeclampsia patients, which one can indicate severity of the disease?(2.5分) A. WBC elevated in the peripheral blood
B. blood Ca2+ decreased
C.24 hours ' urine protein quantitation
D.urine WBC counts
E. plasma albumin
C.24 hours ' urine protein quantitation
6 7 2009 7. 硫酸镁治疗妊娠期高血压疾病剂量过大时,最先出现的毒性反应是:(2.5分) A.头晕、血压过低
B.呼吸减慢
C.心率减慢
D.膝反射减退
E.尿量过少
D.膝反射减退
6 7 2009 8. Which type of the IUGR fetus is due to chromosomal anomalies or structural malformations? (2.5分) A.Normal small fetuses
B.Abnormal small fetuses
C.Growth restricted fetuses
D.Hydrocephalus
E.Macrosomia
B.Abnormal small fetuses
6 7 2009 9. 关于肩难产的描述,以下哪项正确:(2.5分) A.肩难产必然发生于肩先露
B.头先露胎儿过大不会造成肩难产
C.估计胎儿过大肩娩出困难者应及时剖宫产
D.一旦发生肩难产应立即剖宫产
E.一旦发生肩难产应立即产钳助产
C.估计胎儿过大肩娩出困难者应及时剖宫产
6 7 2009 10. 在妊娠期间可以诊断器质性心脏病的体征主要是 :(2.5分) A.心率达120次/分
B.心尖有II级收缩期吹风样杂音
C.心律失常,有期前收缩
D.心尖部有舒张期雷鸣样杂音
E.心界稍向左扩大
D.心尖部有舒张期雷鸣样杂音
6 7 2009 11. 心脏病患者,现妊娠8周,出现急性心力衰竭,最好的处理方法是:(2.5分) A.即刻终止妊娠
B.控制心力衰竭后继续妊娠
C.控制心力衰竭后,吸宫终止妊娠
D.控制心力衰竭后,剖宫取胎终止妊娠
E.边控制心衰,边终止妊娠
C.控制心力衰竭后,吸宫终止妊娠
6 7 2009 12. 妊娠合并心脏病对胎儿的影响,下列那项是正确的:(2.5分) A.心脏代偿功能良好的孕妇,也易引起死胎
B.二尖瓣狭窄手术后已恢复工作的孕妇,易发生早产
C.心功能III级以上的孕妇,胎儿窘迫发生率高
D.妊娠合并心脏病孕妇和其胎儿的预后均很差
E.单有房间隔缺损的孕妇易发生胎儿宫内发育迟缓
C.心功能III级以上的孕妇,胎儿窘迫发生率高
6 7 2009 13. 乙型肝炎孕妇分娩的新生儿注射乙肝疫苗时间为出生后(2.5分) A.0、1、3个月
B.1、2、6个月
C.1、3、6个月
D.0、2、6个月
E.0、1、6个月
E.0、1、6个月
6 7 2009 14. 26岁孕妇,G1P0 , 孕28周来院产前检查。转氨酶值正常,HBsAg(+),HBeAg(+)。其母婴传播的情况可能是: (2.5分) A.乙肝病毒不通过胎盘传递给胎儿
B.其婴儿多半受感染
C.分娩时通过母血传播,但其唾液不传播给婴儿
D.其婴儿将不会成为病毒携带者
E.按程序注射乙肝疫苗可阻断母婴传播
B.其婴儿多半受感染
6 7 2009 15. 对于妊娠合并肝炎,下述处理正确的是:(2.5分) A.妊娠早期保胎
B.妊娠中期需终止妊娠
C.妊娠晚期及早终止妊娠
D.终止妊娠前应用维生素K
E.应以剖宫产终止妊娠
D.终止妊娠前应用维生素K
6 7 2009 16. 糖尿病对妊娠的影响,下列哪项错误:(2.5分) A.孕期宫内死胎的发生率高
B.易发生巨大儿
C.合并妊高征的几率增高
D.胎儿成熟较晚,故一般应待妊娠38周后终止妊娠
E.易发生胎儿畸形
C.合并妊高征的几率增高
6 7 2009 17. 促使心脏病孕妇死亡的主要因素是:(2.5分) A.心脏病病程长
B.产程中用力过度致心衰
C.孕妇年龄过大
D.心衰与感染
E.产后哺乳致心衰
D.心衰与感染
6 7 2009 18. 三维超声优势切面是: (2.5分) A.纵切面;
B.冠状切面;
C.横切面;
D.多维切面
B.冠状切面;
6 7 2009 19. 子宫输卵管结核首先累及:(2.5分) A.子宫
B.输卵管
C.卵巢
D.宫颈
B.输卵管
6 7 2009 20. 宫腔粘连的HSG表现:(2.5分) A.宫腔呈雕花样不规则充盈缺损
B. 宫腔内圆形或类圆形充盈缺损
C. 宫腔内多个大小不等充盈缺损似菊花样
D.宫腔扩大,张力低下
A.宫腔呈雕花样不规则充盈缺损
6 7 2009 21. 绝经后患者Ca-125升高要考虑:(2.5分) A.子宫肌瘤
B. 子宫内膜癌
C. 卵巢良性肿瘤
D.卵巢癌
D.卵巢癌
6 7 2009 22. 卵巢恶性肿瘤多表现为:(2.5分) A.盆腔囊实性肿块,CA-125正常
B.盆腔内实性为主肿块,密度不匀,壁厚伴腹水,伴种植转移,肿块强化明显
C.盆腔内囊实性肿块含有少量脂肪
D.盆腔内实性肿块无明显强化
B.盆腔内实性为主肿块,密度不匀,壁厚伴腹水,伴种植转移,肿块强化明显
6 7 2009 23. 卵巢良性肿瘤的主要CT表现:(2.5分) A.盆腔内实性肿块,密度不匀,有低密度坏死区。
B. 盆腔内囊性或实性肿块,密度均匀一致,表面光滑,边界清楚。
C.盆腔内囊实性肿块,周边粘连明显。
D.盆腔内与子宫相连的肿块,与子宫同步强化。
B. 盆腔内囊性或实性肿块,密度均匀一致,表面光滑,边界清楚。
6 7 2009 24. A 34-year-old woman is noted to have significant uterine bleeding after a vaginal delivery complicated by placenta abruption.She is noted to be bleeding from multiple venipuncture sites.Which of the following is the best therapy? (2.5分) A.Immediate hysterectomy
B.Packing of the uterus
C.Hypogastric artery ligation
D.Ligation of utero-ovarian ligament
E.Correction of coagulopathy
E.Correction of coagulopathy
6 7 2009 25. 孕38周,诊断为胎盘早剥行剖宫产术,术中见子宫紫色,质软,胎儿胎盘娩出后,大量出血,注射宫缩剂并按摩子宫,出血不止,血压下降,应立即给予: (2.5分) A.止血药物
B.宫腔填塞
C.按摩子宫加热敷
D.子宫切除
E.大量输血补液等待血压上升
D.子宫切除
6 7 2009 26. In which of the following conditions would consumptive coagulopathy most likely be seen? (2.5分) A.Placental abruption
B.Placenta previa
C.Gestational diabetes
D.Multifetal gestation
E.Gestational trophoblastic disease
A.Placental abruption
6 7 2009 27. 关于胎盘早剥时的处理,正确的是(2.5分) A.纠正休克,大量补液
B.确诊为轻型者,可行期待疗法
C.经阴道分娩者不宜破膜
D.产妇情况恶化,不论胎儿是否存活,均应及时行剖宫产术
E.应用肝素治疗凝血功能障碍
D.产妇情况恶化,不论胎儿是否存活,均应及时行剖宫产术
6 7 2009 28. 胎盘早剥哪项是正确的(2.5分) A.先期待疗法后考虑终止妊娠
B.多无诱因,往往在半夜阴道出血
C.易发生产后出血,为软产道损伤所致
D.贫血程度与阴道出血量成正比
E.子宫坚硬压痛,宫底逐渐升高胎心不清
E.子宫坚硬压痛,宫底逐渐升高胎心不清
6 7 2009 29. A 24-year-old primigravid woman, who is intent on breast-feeding, decides on a home delivery. Immediately after the birth of a 4.1-kg (9-lb) infant, the patient bleeds massively from extensive vaginal and cervical lacerations. She is brought to the nearest hospital in shock. Over 2 hours, 9 units of blood are transfused, and the patient's blood pressure returns to a reasonable level. A hemoglobin value the next day is 7.5 g/dL, and 3 units of packed red blood cells are given. The most likely late sequela to consider in this woman is which of the following? (2.5分) A.Hemochromatosis
B.Stein-Leventhal syndrome
C.Sheehan syndrome
D.Simmonds syndrome
E.Cushing syndrome
C.Sheehan syndrome
6 7 2009 30. The most common cause of maternal death in China is:(2.5分) A.Abortion
B.Anesthesia
C.Embolism
D.Infection
E.Hemorrhage
E.Hemorrhage
6 7 2009 31. 女,25岁。孕40周,初孕,规律宫缩2小时来院,当时宫口扩张4cm,因宫缩强,半小时后宫口开全,第二产程仅15分钟即顺利娩出一男婴,胎儿娩出后即有鲜红血流出,5分钟后胎盘自然娩出。此后出血量仍较多,有血块。此时分析其出血原因最可能为 (2.5分) A.子宫收缩乏力
B.胎膜残留
C.胎盘残留
D.宫颈裂伤
E.凝血功能障碍
D.宫颈裂伤
6 7 2009 32. 初孕妇,因第二产程延长,行左侧会阴切开+低位产钳助产,娩出4000g一活婴,产后2小时伤口疼痛,肛门坠胀并有便意,大便常规检验正常。体检:贫血貌,血压12/8kPa(90/60mmHg),左会阴稍肿,阴道出血不多。下列哪项诊断可能性最大 (2.5分) A.菌痢
B.宫颈撕裂
C.阴道壁血肿
D.会阴Ⅲ度破裂
E.肠胃炎
C.阴道壁血肿
6 7 2009 33. 女,32岁。孕40周,妊娠合并子宫肌瘤,阴道分娩胎盘娩出后阴道出血量多,暗红色检查:宫底高,子宫软,产道无裂伤, 血自宫腔流出,有血块,检查胎盘完整,血压14.6/12kPa(110/90mmHg)。除给宫缩剂外首先处理的是(2.5分) A.止血剂
B.抗炎
C.迅速按摩子宫止血
D.子宫腔内纱布填塞
E.髂内动脉结扎
C.迅速按摩子宫止血
6 7 2009 34. 产妇,26岁。孕38周时胎膜早破入院,48小时后因持续性枕横位以产钳术助娩一活男婴3300g,术后3天发热达39℃,检查发现咽部轻度充血,乳房胀满疼痛,局部皮肤不红,按之无波动感。宫底脐下一横指,宫体有压痛,下腹壁无反跳痛,恶露混浊,稍有异味。该病人最可能的诊断是(2.5分) A.急性子宫内膜炎
B.急性子宫内膜及肌炎
C.上呼吸道感染
D.乳腺炎
E.盆腔腹膜炎
B.急性子宫内膜及肌炎
6 7 2009 35. 女,25岁,G5P0,停经45天,阴道少量流血1周,右下腹剧痛1天。体检:面色苍白,P120次/分,血压80/50mmHg,腹部压痛(+),反跳痛(+),移动性浊音(+)。妇科检查:外阴少量暗红色分泌物,宫颈光,举痛,子宫前位,略大,右附件区可及4×4×5cm3包块,触痛明显,左侧无殊。 下列哪项检查对诊断及评估病情不是必需?(2.5分) A.尿HCG
B.盆腔B超
C.经阴道后穹窿穿刺
D.盆腔CT
E.血常规
D.盆腔CT
6 7 2009 36. A 22-year-old woman, who is pregnant at 5 weeks' gestation, complains of severe lower abdominal pain. On examination, she is noted to have a blood pressure of 86/44 mm Hg and heart rate of 120 bpm. Her abdomen is tender. The pelvic examination is difficult to perform due to guarding. The hCG level is 500 mIU/mL and the transvaginal sonogram reveals no intrauterine gestational sac and no adnexal masses. There is some free fluid in the abdomenal cavity. Which of the following is the best management for this patient?(2.5分) A.Repeat hCG level in 48 hours to assess for a rise of 66%.
B.Check the serum progesterone level.
C.Immediate surgery.
D.Intramuscular methotrexate.
E.Repeat sonography in 48 hours.
C.Immediate surgery.
6 7 2009 37. A 20-year-old G2Ab1 woman at 9 weeks' gestation by LMP underwent an induced abortion. After the abortion, the histological examination of the uterine curetting revealed no chorionic villi. Her pregnancy test was positive. What is the most likely diagnosis? (2.5分) A.ectopic pregnancy
B.PID
C.endometriosis
D.appendicitis
E.ruptured corpus luteum cyst of the ovary
A.ectopic pregnancy
6 7 2009 38. 前置胎盘病例,仅适用于阴道分娩的是 (2.5分) A.部分性前置胎盘而胎儿为头位
B.低置胎盘而胎儿为头位
C.部分性前置胎盘而胎儿为臀位
D.低置胎盘而胎儿为臀位
E.以上都不是
B.低置胎盘而胎儿为头位
6 7 2009 39. 28岁初产妇,临产后静脉滴注催产素,破膜后不久出现烦躁不安、呛咳、呼吸困难、紫绀,10分钟后死亡。本例最可能的诊断是:(2.5分) A.低纤维蛋白原血症
B.子痫
C.羊水栓塞
D.重型胎盘早剥
E.子宫破裂
C.羊水栓塞
6 7 2009 40. 可引起产科休克的疾病不包括 (2.5分) A.产后出血
B.胎盘早剥
C.羊水栓塞
D.早产
E.子宫破裂
D.早产
6 7 2010 1. The most common obstetric complication of hypertensive disorders complicating pregnancy is(3.0分) A.placental abruption
B.acute renal failure
C.heart failure
D.retinal detachment
E.HELLP syndrome
A.placental abruption
6 7 2010 2. 引起子痫抽搐的主要病理原因是 (3.0分) A.颅内脑组织散在出血点
B.颅内小动脉痉挛,脑水肿
C.血钙低下
D.代谢性酸中毒
E.胎盘毒素
B.颅内小动脉痉挛,脑水肿
6 7 2010 3. 下述何项不属乙型病毒性肝炎母婴传播途径 (3.0分) A.粪-口传染
B.娩出时接触母亲产道分泌液或血污染
C.母婴垂直传染
D.乳汁传染
E.密切生活接触传染
A.粪-口传染
6 7 2010 4. 硫酸镁治疗妊高征剂量过大时,最先出现的毒性反应是 (3.0分) A.头晕,血压过低
B.呼吸减慢
C.心率减慢
D.膝反射减退或消失
E.尿量过少
D.膝反射减退或消失
6 7 2010 5. 女,31岁,妊娠26周。发现尿糖(+),口服葡萄糖耐量试验结果:空腹血糖5.6mmol/L,1小时血糖8.9mmol/L,2小时血糖7.6mmol/L,既往无糖尿病史。最可能的诊断是 (3.0分) A.肾性糖尿
B.糖尿病合并妊娠
C.妊娠期糖尿病
D.继发性糖尿病
E.妊娠期生理性糖尿
C.妊娠期糖尿病
6 7 2010 6. 子宫输卵管结核首先累及(3.0分) A.子宫
B.输卵管
C.卵巢
D.宫颈
B.输卵管
6 7 2010 7. 卵巢纤维瘤的CT特征(3.0分) A.盆腔内囊性肿块,无腹水,包膜明显强化。
B.盆腔内实性肿块,伴少量积液,强化明显。
C.盆腔内实性肿块,伴少量胸腹水,无明显强化。
D.盆腔内囊性肿块,伴少量胸腹水,无强化。
C.盆腔内实性肿块,伴少量胸腹水,无明显强化。
6 7 2010 8. 卵巢畸胎瘤的CT特征(2.0分) A.盆腔内类圆形囊性肿块
B.盆腔内类圆形实性肿块
C.盆腔内类圆形囊实性肿块
D.盆腔内类圆形含脂肪、牙齿或钙化斑块的混杂性肿块
D.盆腔内类圆形含脂肪、牙齿或钙化斑块的混杂性肿块
6 7 2010 9. 库肯勃氏瘤(krukenberg/s tumor)是特指那一类肿瘤?(3.0分) A.肝转移瘤
B.卵巢恶性肿瘤
C.卵巢的胃肠道转移瘤
D.卵巢的乳腺转移瘤
C.卵巢的胃肠道转移瘤
6 7 2010 10. 卵巢恶性肿瘤多表现为(3.0分) A.盆腔囊实性肿块,CA-125正常
B.盆腔内实性为主肿块,密度不匀,壁厚伴腹水,伴种植转移,肿块强化明显
C.盆腔内囊实性肿块含有少量脂肪
D.盆腔内实性肿块无明显强化
B.盆腔内实性为主肿块,密度不匀,壁厚伴腹水,伴种植转移,肿块强化明显
6 7 2010 11. A 24-year-old G2 P1 woman at 39 weeks'gestation presents with painful uterine contractions.She also complains of dark ,vaginal blood mixed with some mucus .Which of the following describes the most likely etiology of her bleeding?(3.0分) A.Placenta previa
B.Placenta abruption
C.Bloody show
D.Vasa previa
E.Cervical laceration
C.Bloody show
6 7 2010 12. A 34-year-old woman is noted to have significant uterine bleeding after a vaginal delivery complicated by placenta abruption.She is noted to be bleeding from multiple venipuncture sites.Which of the following is the best therapy?(3.0分) A.Immediate hysterectomy
B.Packing of the uterus
C.Hypogastric artery ligation
D.Ligation of utero-ovarian ligament
E.Correction of coagulopathy
E.Correction of coagulopathy
6 7 2010 13. 前置胎盘病例,仅适用于阴道分娩的是(3.0分) A.中央性前置胎盘而胎儿为头位
B.边缘性胎盘而胎儿为头位
C.部分性前置胎盘而胎儿为臀位
D.边缘性胎盘而胎儿为臀位
E.以上都不是
B.边缘性胎盘而胎儿为头位
6 7 2010 14. The most common cause of maternal death in China is:(3.0分) A.Abortion
B.Anesthesia
C.Embolism
D.Infection
E.Hemorrhage
E.Hemorrhage
6 7 2010 15. 初产妇,急产娩一男婴,体重3900g,胎盘娩出后半小时内有较多量间歇性阴道出血,色红,宫底,宫颈及肌肉已注催产素10单位,再次查看胎盘完整,胎膜有一处见血管中断于胎膜边缘,究竟其出血原因最可能是(3.0分) A.胎盘剥离不全
B.软产道损伤
C.胎盘残留
D.产后宫缩乏力
E.凝血功能障碍
C.胎盘残留
6 7 2010 16. 28岁初产妇,临产后静脉滴注催产素,破膜后不久出现烦躁不安、呛咳、呼吸困难、紫绀,10分钟后死亡。本例最可能的诊断是(3.0分) A.纤维蛋白原血症
B.子痫
C.羊水栓塞
D.重型胎盘早剥
E.子宫破裂
C.羊水栓塞
6 7 2010 17. 经产妇36岁,孕40周,晨3时突然大量阴道出血,急诊来院,体检:血压16/10kPa(120/75mmHg),尿蛋白(-),腹部检查:子宫高35cm,胎头高浮,子宫前壁无压痛。阴道检查:阴道内有手拳大的凝血块,宫颈软,宫口开大一指,先露部未及胎盘组织。该产妇分娩后5分钟突然发生烦躁不安,寒战,呕吐,咳嗽,呼吸困难,发绀,血压10.7/5.3kPa (80/40mmHg),脉细弱,首先应考虑下列何种情况(2.0分) A.失血性休克
B.脑血管意外
C.羊水栓塞
D.感染与休克
E.子宫破裂
C.羊水栓塞
6 7 2010 18. A 38-year-old G3P3 begins to breast-feed her 5-day-old infant. The baby latches on appropriately and begins to suckle. In the mother, which of the following is a response to suckling?(3.0分) A.Decrease of oxytocin
B.Increase of prolactin-inhibiting factor
C.Increase of hypothalamic dopamine
D.Increase of hypothalamic prolactin
E.Increase of luteinizing hormone-releasing factor
D.Increase of hypothalamic prolactin
6 7 2010 19. .产妇,26岁。孕38周时胎膜早破入院,48小时后因持续性枕横位以产钳术助娩一活男婴3300g,术后3天发热达39℃,检查发现咽部轻度充血,乳房胀满疼痛,局部皮肤不红,按之无波动感。宫底脐下一横指,宫体有压痛,下腹壁无反跳痛,恶露混浊,稍有异味。该病人最可能的诊断是(3.0分) A.急性子宫内膜炎
B.急性子宫内膜及肌炎
C.上呼吸道感染
D.乳腺炎
E.盆腔腹膜炎
B.急性子宫内膜及肌炎
6 7 2010 20. 中央性前置胎盘大出血的治疗方法哪项是正确的(3.0分) A.如为臀位,行足牵引
B.催产素引产
C.胎头吸引助产
D.剖宫产
E.人工破膜,头皮钳牵引压迫止血
D.剖宫产
6 7 2010 21. 下列哪项不符合前置胎盘的表现(3.0分) A.先露下降受阻
B.无痛性阴道流血
C.子宫下段闻及胎盘血流音
D.子宫张力较高,胎心音不易闻及
E.宫底高度与孕周相符
D.子宫张力较高,胎心音不易闻及
6 7 2010 22. 胎盘早剥哪项是正确的(3.0分) A.先期待疗法后考虑终止妊娠
B.多无诱因,往往在半夜阴道出血
C.易发生产后出血,为软产道损伤所致
D.贫血程度与阴道出血量成正比
E.子宫坚硬压痛,宫底逐渐升高胎心不清
E.子宫坚硬压痛,宫底逐渐升高胎心不清
6 7 2010 23. 有关先兆子宫破裂的诊断依据,下列哪项是错误的 (3.0分) A.先露下降受阻,产程延长
B.血红蛋白迅速下降
C.可见病理性缩复环
D.可导致明显血尿
E.下腹剧痛,拒按
B.血红蛋白迅速下降
6 7 2010 24. 女,32岁。孕40周,妊娠合并子宫肌瘤,阴道分娩胎盘娩出后阴道出血量多,暗红色检查:宫底高,子宫软,产道无裂伤, 血自宫腔流出,有血块,检查胎盘完整,血压14.6/12kPa(110/90mmHg)。除给宫缩剂外首先处理的是(3.0分) A.止血剂
B.抗炎
C.迅速按摩子宫止血
D.子宫腔内纱布填塞
E.髂内动脉结扎
C.迅速按摩子宫止血
6 7 2010 25. A 21-year-old G2P2 calls her physician 7 days postpartum because she is concerned that she is still bleeding from the vagina. She describes the bleeding as light pink to bright red and less heavy than the first few days postdelivery. She denies fever or any cramping pain. On examination she is afebrile and has an appropriately sized, nontender uterus. The vagina contains about 10 cc of old, dark blood. The cervix is closed. Which of the following is the most appropriate treatment? (3.0分) A.Antibiotics for endometritis
B.High-dose oral estrogen for placental subinvolution
C.Oxytocin for uterine atony
D.Suction dilation and curettage for retained placenta
E.Reassurance
E.Reassurance
6 7 2010 26. Which of the following best describes a missed abortion?(3.0分) A.It is the death of the fetus that the patient does not realize has occurred.
B.It is the death of the fetus in which the products of conception are retained after the embryo or fetus has died.
C.It is the death of the fetus in which the products of conception are partially expelled.
D.It is the death of the fetus in which the products of conception cause bleeding and there is an open cervical os.
E.It is the death of the fetus in which the products of conception are no longer recognizable.
B.It is the death of the fetus in which the products of conception are retained after the embryo or fetus has died.
6 7 2010 27. The majority of ectopic gestations were located in:(2.0分) A.ovary
B.cervix
C.the ampullary portion of the fallopian tube
D.the isthmus portion of the fallopian tube
E.the fimbrial region of the fallopian tube
C.the ampullary portion of the fallopian tube
6 7 2010 28. A 23-year-old(gravida 1)at about 12 weeks' gestation develops persistent nausea and vomiting that progresses from an occasional episode to a constant retching. She has no fever or diarrhea but loses 5 lb in 1 week and appears dehydrated. What is your disgnosis? (3.0分) A.anorexia nervosa
B.morning sichness
C.ptyalism
D.hyperemesis gravidarum
E.gastroenteritis
D.hyperemesis gravidarum
6 7 2010 29. Which of the following factors is NOT measured as part of a biophysical prol-ile (BPP)?(3.0分) A.amniotic fluid volume
B.a contraction stress test (CST)
C.a nonstress test (NST)
D.fetal breathing motion
E.fetal limb movements
B.a contraction stress test (CST)
6 7 2010 30. 下列那种情况不是胎儿窘迫的临床征象(2.0分) A.频繁的变异减速
B.晚期减速
C.羊水Ⅲ度
D.胎动减少明显
E.早期减速
E.早期减速
6 7 2010 31. 孕妇,33岁,孕1产0,孕41周,产前检查均正常。自觉胎动减少1天收入院。查体:血压110/75mmHg,宫高35cm,腹围100cm,胎位LOA,胎头先露,胎心率120次/分。以下哪项并不提示胎儿窘迫? (3.0分) A.胎儿头皮血PH7.2
B.NST有反应型
C.OCT重度变异减速
D.OCT频发晚期减速
E.胎心率基线120~110次/分平直型
B.NST有反应型
6 7 2010 32. A pregnant woman had regular menstruation(3-5/30) and normal prenatal examination since 12 gestational weeks. Now she was 42 gestational weeks and felt decreased fetal movement. Which examination isn't essential for the diagnosis of fetus state? (3.0分) A.NST test
B.OCT test
C.blood AFP determination
D.blood HPL determination
E.E3 determination in 24 hour's urine
C.blood AFP determination
6 7 2010 33. 妊娠达到或超过几周称为过期妊娠(3.0分) A.28周
B.37周
C.40周
D.41周
E.42周
E.42周
6 7 2010 34. 胎膜早破的患者终止妊娠的指征(2.0分) A.出现宫内感染
B.临产
C.胎儿未成熟
D.羊水过多
E.阴道少量出血
A.出现宫内感染
6 7 2010 35. 23岁初孕妇,39周妊娠,近两周来胎动时常感腹痛。入院查体:宫高28cm,腹围85cm,子宫敏感性高,胎位LSA,胎心140次/分,B超检查:胎儿发育正常,羊水指数4cm。首选的处理方案是(3.0分) A.尽快破膜引产
B.期待疗法
C.羊膜腔输液
D.立即剖宫产
E.饮水疗法
D.立即剖宫产
6 7 2011 1. The most common obstetric complication of hypertensive disorders complicating pregnancy is(2.5分) A.placental abruption
B.acute renal failure
C.heart failure
D.retinal detachment
E.HELLP syndrome
A.placental abruption
6 7 2011 2. 妊娠合并心脏病者,下列哪项不是早期心衰体征 (2.5分) A.轻微活动后有胸闷气息及心悸感
B.休息时心率>110次/分
C.休息时呼吸>20次/分
D.肺底有湿啰音,咳嗽后消失
E.阵发性夜间呼吸困难
D.肺底有湿啰音,咳嗽后消失
6 7 2011 3. 硫酸镁治疗妊高征剂量过大时,最先出现的毒性反应是(2.5分) A.头晕,血压过低
B.呼吸减慢
C.心率减慢
D.膝反射减退或消失
E.尿量过少
D.膝反射减退或消失
6 7 2011 4. 关于巨大儿,下列说法错误的是(2.5分) A.估计胎儿体重≥4000g且合并糖尿病者,建议剖宫产终止妊娠
B.巨大儿的高危因素包括孕妇肥胖、妊娠合并糖尿病、过期妊娠、经产妇、父母身材高大、高龄产妇、有巨大儿分娩史、种族因素
C.妊娠期发现巨大儿可疑者,建议预防性引产
D.巨大儿经阴道分娩过程中易发生肩难产
E.新生儿需预防低血糖,在出生后30分钟监测血糖
C.妊娠期发现巨大儿可疑者,建议预防性引产
6 7 2011 5. 胎儿双顶径切面上主要超声标志物是:(2.5分) A.透明隔腔;
B.大脑镰;
C.第三脑室;
D.丘脑
E.枕下前囟径平面
D.丘脑
6 7 2011 6. 患者,24岁,1-0-2-1,妊娠43周,家中接生婆接生未成功,临产25小时送至我院,入院时孕妇痛苦异常,子宫呈葫芦状,子宫下端压痛,肉眼血尿。血压120/60mmHg,心率95次/分,胎心未闻及。本例诊断无关的项目为: (2.5分) A.滞产
B.过期妊娠
C.子宫先兆破裂
D.泌尿系感染
E.死产
D.泌尿系感染
6 7 2011 7. 患者,24岁,1-0-2-1,妊娠43周,家中接生婆接生未成功,临产25小时送至我院,入院时孕妇痛苦异常,子宫呈葫芦状,子宫下端压痛,肉眼血尿。血压120/60mmHg,心率95次/分,胎心未闻及。此时应采取的处理措施为: (2.5分) A.麻醉下行内转胎位术
B.断头术或碎胎术
C.外转胎位术
D.剖宫产术
E.上肢还纳术
D.剖宫产术
6 7 2011 8. 患者,24岁,1-0-2-1,妊娠43周,家中接生婆接生未成功,临产25小时送至我院,入院时孕妇痛苦异常,子宫呈葫芦状,子宫下端压痛,肉眼血尿。血压120/60mmHg,心率95次/分,胎心未闻及。采取上述处理措施的理由为:(2.5分) A.宫口开全,可行内倒转术
B.诊为先兆子宫破裂,应行剖宫产术
C.因胎儿已死,应行碎胎术或断头术
D.死胎应争取阴道分娩
E.经产妇肩先露应行上肢还纳术
B.诊为先兆子宫破裂,应行剖宫产术
6 7 2011 9. 女性生殖器结核最常见的是(2.5分) A.输卵管结核
B.子宫内膜结核
C.子宫颈结核
D.卵巢结核
E.结核性子宫内膜炎合并卵巢结核
A.输卵管结核
6 7 2011 10. 下列哪种检查方法常兼有治疗作用(2.5分) A.子宫输卵管碘油造影
B.盆腔充气造影
C.盆腔动脉造影
D.盆腔静脉造影
E.腹部平片
A.子宫输卵管碘油造影
6 7 2011 11. 诊断异位妊娠最简单的方法是: (2.5分) A.尿妊娠试验
B.B超
C.诊断性刮宫
D.后穹窿穿刺
E.妇科检查
B.B超
6 7 2011 12. 乳腺钼靶摄影最佳时间(2.5分) A.月经干净后一周内
B.月经期
C.月经前期
D.月经中期
E.与经期无关
A.月经干净后一周内
6 7 2011 13. 子宫腺肌病MRI最常表现为累及(2.5分) A.子宫浆膜层
B.子宫内膜
C.结合带
D.双侧卵巢
E.两侧输卵管
C.结合带
6 7 2011 14. 以下哪种肿瘤与AFP高度相关为: (2.5分) A.卵巢癌
B.卵巢纤维瘤
C.卵黄囊瘤
D.颗粒细胞瘤
E.子宫内膜癌
C.卵黄囊瘤
6 7 2011 15. 子宫内膜癌侵犯至宫外,但范围局限于真盆腔为( )期(2.5分) A.Ⅰ
B.Ⅱ
C.Ⅲ
D.Ⅳ
E.Ⅴ
B.Ⅱ
6 7 2011 16. 以下为恶性肿瘤的是(2.5分) A.卵巢囊肿
B.子宫肌瘤
C.卵巢囊腺瘤
D.子宫内膜不典型增生
E.无性细胞瘤
E.无性细胞瘤
6 7 2011 17. 下面哪项不是多胎妊娠终止妊娠的指征: (2.5分) A.胎儿畸形
B.孕40周,B超提示胎盘Ⅲ级
C.孕妇出现浮肿,血压160/110mmHg,尿蛋白++
D.B超提示羊水指数24cm,孕妇呼吸困难
E.孕妇长期卧床保胎,出现便秘
E.孕妇长期卧床保胎,出现便秘
6 7 2011 18. 胎膜早破的患者终止妊娠的指征:(2.5分) A.出现宫内感染
B.临产
C.胎儿未成熟
D.羊水过多
E.阴道少量出血
A.出现宫内感染
6 7 2011 19. 检查胎儿头皮血PH,诊断胎儿窘迫的指标: (2.5分) A.PH<7.00
B.PH<7.10
C.PH<7.20
D.PH<7.30
E.PH<7.35
C.PH<7.20
6 7 2011 20. 下列那种情况不是胎儿窘迫的临床征象(2.5分) A.频繁的变异减速
B.晚期减速
C.羊水Ⅲ度
D.胎动减少明显
E.早期减速
E.早期减速
6 7 2011 21. 女,25岁,G5P0,停经45天,阴道少量流血1周,右下腹剧痛1天。体检:面色苍白,P120次/分,血压80/50mmHg,腹部压痛(+),反跳痛(+),移动性浊音(+)。妇科检查:外阴少量暗红色分泌物,宫颈光,举痛,子宫前位,略大,右附件区可及4×4×5cm3包块,触痛明显,左侧无殊。下列哪项检查对诊断及评估病情不是必需? (2.5分) A.尿HCG
B.盆腔B超
C.经阴道后穹窿穿刺
D.盆腔CT
E.血常规
D.盆腔CT
6 7 2011 22. 女,25岁,G5P0,停经45天,阴道少量流血1周,右下腹剧痛1天。体检:面色苍白,P120次/分,血压80/50mmHg,腹部压痛(+),反跳痛(+),移动性浊音(+)。妇科检查:外阴少量暗红色分泌物,宫颈光,举痛,子宫前位,略大,右附件区可及4×4×5cm3包块,触痛明显,左侧无殊。首先考虑可能诊断为(2.5分) A.先兆流产
B.异位妊娠
C.葡萄胎
D.阑尾脓肿破裂
E.右卵巢囊肿蒂扭转
B.异位妊娠
6 7 2011 23. A 20-year-old G2Ab1 woman at 9 weeks' gestation by LMP underwent an induced abortion. After the abortion, the histological examination of the uterine curetting revealed no chorionic villi. Her pregnancy test was positive. What is the most likely diagnosis? (2.5分) A.ectopic pregnancy
B.PID
C.Endometriosis
D.appendicitis
E.ruptured corpus luteum cyst of the ovary
A.ectopic pregnancy
6 7 2011 24. 预防胎儿窘迫不包括(2.5分) A.左侧卧位
B.积极防治妊高征
C.防止胎膜早破
D.做阴道检查
E.合理使用催产素
D.做阴道检查
6 7 2011 25. Each of the following is a risk factor for abruptio placentae EXCEPT: (2.5分) A.Hyptension
B.Cigarette smoking
C.Trauma
D.Cocaine use
E.Primiparity
E.Primiparity
6 7 2011 26. A 34-year-old woman is noted to have significant uterine bleeding after a vaginal delivery complicated by placenta abruption.She is noted to be bleeding from multiple venipuncture sites.Which of the following is the best therapy? (2.5分) A.Immediate hysterectomy
B.Packing of the uterus
C.Hypogastric artery ligation
D.Ligation of utero-ovarian ligament
E.Correction of coagulopathy
E.Correction of coagulopathy
6 7 2011 27. 中央性前置胎盘大出血的治疗方法哪项是正确的 (2.5分) A.如为臀位,行足牵引
B.催产素引产
C.胎头吸引助产
D.剖宫产
E.人工破膜,头皮钳牵引压迫止血
D.剖宫产
6 7 2011 28. 完全性前置胎盘不可能发生在下述何种情况 (2.5分) A.胎位LST,单臀,先露未入盆
B.胎位RSCP
C.胎位LOP,胎头未入盆
D.胎位ROA ,先露已衔接
E.胎位LOT ,先露高浮
D.胎位ROA ,先露已衔接
6 7 2011 29. hich of the following is the most common cause of postpartum hemorrhage after a cesarean delivery ?(2.5分) A.Uterine atony
B.Uterine laceration
C.Coagulopathy
D.Uterine inversion
E.Retained placenta
A.Uterine atony
6 7 2011 30. Which of the following diseases rarely leads to postpartum homarrhage? (2.5分) A.twin pregnancy
B.polyhydramnios
C.premature rupture of membrane
D.placenta previa
E.severe preeclampsia
C.premature rupture of membrane
6 7 2011 31. 有关先兆子宫破裂的诊断依据,下列哪项是错误的 (2.5分) A.先露下降受阻,产程延长
B.血红蛋白迅速下降
C.可见病理性缩复环
D.可导致明显血尿
E.下腹剧痛,拒按
B.血红蛋白迅速下降
6 7 2011 32. 初孕妇,因第二产程延长,行左侧会阴切开+低位产钳助产,娩出4000g一活婴,产后2小时伤口疼痛,肛门坠胀并有便意,大便常规检验正常。体检:贫血貌,血压12/8kPa(90/60mmHg),左会阴稍肿,阴道出血不多。下列哪项诊断可能性最大 (2.5分) A.菌痢
B.宫颈撕裂
C.阴道壁血肿
D.会阴Ⅲ度破裂
E.肠胃炎
C.阴道壁血肿
6 7 2011 33. 可引起产科休克的疾病不包括(2.5分) A.产后出血
B.胎盘早剥
C.羊水栓塞
D.早产
E.子宫破裂
D.早产
6 7 2011 34. 下列哪项是正常产褥期的表现: (2.5分) A.产后24小时体温超过38°
B.产后第一天宫底达脐平
C.产后脉搏一般偏快
D.产后24小时白细胞应恢复至正常范围
E.产后二周恶露开始转为浆液性
B.产后第一天宫底达脐平
6 7 2011 35. A 34-year-old G1P1 who delivered her first baby 5 weeks ago calls your office and asks to speak with you. She tells you that she is feeling very overwhelmed and anxious. She feels that she cannot do anything right and feels sad throughout the day. She tells you that she finds herself crying all the time and is unable to sleep at night. Which of the following is the most likely diagnosis? (2.5分) A.Postpartum depression
B.Maternity blues
C.Postpartum psychosis
D.Bipolar disease
E.Postpartum blues
A.Postpartum depression
6 7 2011 36. 28岁初产妇,临产后静脉滴注催产素,破膜后不久出现烦躁不安、呛咳、呼吸困难、紫绀,10分钟后死亡。本例最可能的诊断是(2.5分) A.低纤维蛋白原血症
B.子痫
C.羊水栓塞
D.重型胎盘早剥
E.子宫破裂
C.羊水栓塞
6 7 2011 37. 胎盘早剥哪项是正确的(2.5分) A.先期待疗法后考虑终止妊娠
B.多无诱因,往往在半夜阴道出血
C.易发生产后出血,为软产道损伤所致
D.贫血程度与阴道出血量成正比
E.子宫坚硬压痛,宫底逐渐升高胎心不清
E.子宫坚硬压痛,宫底逐渐升高胎心不清
6 7 2011 38. 下列哪项不符合中央性(完全性)前置胎盘的常见临床表现(2.5分) A.胎位不正的发生率高
B.先露高浮
C.第一次出血多在孕36周以后
D.易发生出血性休克
E.较其他类型前置胎盘易发生胎儿宫内窘迫
C.第一次出血多在孕36周以后
6 7 2011 39. 下列哪种情况不是重度胎盘早剥的临床表现 (2.5分) A.剧烈腹痛后阴道流血
B.阴道出血量与全身症状不成正比
C.子宫底升高
D.子宫板状
E.胎位,胎心清晰
E.胎位,胎心清晰
6 7 2011 40. 产褥病率的定义是(2.5分) A.产后24小时应每4小时测体温一次,体温两次达到或超过38℃者
B.产后10天每4小时测体温一次,体温两次达到或超过38℃者
C.产后每4小时测体温一次,体温两次达到38℃者
D.产后24小时至10天内,用口表每日测量体温4次,有两次≥38℃者
E.产褥期内两次体温达到或超过38℃者
D.产后24小时至10天内,用口表每日测量体温4次,有两次≥38℃者
6 7 2012 1. A 32-year-old (G3P1) at term is admitted in labor with an initial cervical examination of 6cm dilatation, complete effacement, and the vertex at -1 station. Estimated fetal weight is 3.8kg, and her first pregnancy resulted in an uncomplicated vaginal delivery of a 3.8kg infant. After 2 hours, there is no cervical change. An intrauterine pressure catheter is placed. This shows two contractions in a 10-minute period, each with a strength of 30 mmHg. What is this abnormality of labor termed? (2.5分) A.prolonged latent phase
B.active-phase arrest
C.failure of decent
D.arrest of latent phase
E.protraction of descent
B.active-phase arrest
6 7 2012 2. A prolonged second stage of labor in the primigravid patient lasts longer than:(2.5分) A.1.0 hours
B.2.0 hours
C.8.0 hours
D.6.0 hours
E.4.0 hours
B.2.0 hours
6 7 2012 3. 患者,24岁,1-0-2-1,妊娠43周,家中接生婆接生未成功,临产25小时送至我院,入院时孕妇痛苦异常,子宫呈葫芦状,子宫下端压痛,肉眼血尿。血压120/60mmHg,心率95次/分,胎心未闻及。本例诊断无关的项目为(3.5分) A.滞产
B.过期妊娠
C.子宫先兆破裂
D.泌尿系感染
E.死产
D.泌尿系感染
6 7 2012 4. 妊娠期高血压疾病最常见的产科并发症是(2.5分) A.胎盘早期剥离
B.急性肾功能衰竭
C.妊娠期高血压疾病性心脏病
D.视网膜剥离
E.HE LLP综合征
A.胎盘早期剥离
6 7 2012 5. 硫酸镁治疗妊娠期高血压疾病剂量过大时,最先出现的毒性反应是(2.5分) A.头晕,血压过低
B.呼吸减慢
C.心率减慢
D.膝反射减退或消失
E.尿量过少
D.膝反射减退或消失
6 7 2012 6. 引起子痫抽搐的主要病理原因是(2.5分) A.颅内脑组织散在出血点
B.颅内小动脉痉挛,脑水肿
C.血钙低下
D.代谢性酸中毒
E.胎盘毒素
B.颅内小动脉痉挛,脑水肿
6 7 2012 7. 妊娠近足月患子痫前期的孕妇,恰当的处理应是(2.5分) A.积极治疗,等待产程发动
B.积极治疗24~48小时,症状无明显改善时应终止妊娠
C.积极治疗一周后,予以引产
D.立即引产
E.立即行剖宫术
B.积极治疗24~48小时,症状无明显改善时应终止妊娠
6 7 2012 8. 初孕妇孕36周,早孕时血压90/60mmHg,近一周来头痛,眼花伴视物不清1天,突然全身抽搐1次,急诊入院。查神志尚清,血压170/105mmHg),LOA ,胎心148次/分,下述处理哪项正确(2.5分) A.应用硫酸镁和地西泮控制抽搐
B.引产
C.积极治疗,24小时后行剖宫产
D.积极控制抽搐,病情控制后6~12小时终止妊娠
E.控制抽搐,稳定病情,至孕37周终止妊娠
A.应用硫酸镁和地西泮控制抽搐
6 7 2012 9. 第一胎,停经38周,一月前血压正常,近一周出现下肢水肿,伴头昏,眼花视力模糊,血压160/100mmHg,尿蛋白(+),尿雌三醇10mg/24h,胎心好,产科处理应是(2.5分) A.立即剖腹取胎
B.积极治疗一周,考虑终止妊娠
C.积极治疗24~48小时,考虑终止妊娠
D.积极治疗等待自然分娩
E.立即破膜及静滴催产素引产
C.积极治疗24~48小时,考虑终止妊娠
6 7 2012 10. 妊娠合并风湿性心脏病,下列哪个体征是早期心衰的可靠诊断依据(3.5分) A.心界扩大
B.心尖部闻及Ⅱ级收缩期杂音
C.肺底部持续性湿 音
D.休息时心率>110次.分
E.下肢凹陷性水肿Ⅰ度
D.休息时心率>110次.分
6 7 2012 11. 下述何项不属乙型病毒性肝炎母婴传播途径 (2.5分) A.粪-口传染
B.娩出时接触母亲产道分泌液或血污染
C.母婴垂直传染
D.乳汁传染
E.密切生活接触传染
A.粪-口传染
6 7 2012 12. 病毒性肝炎对妊娠的影响,下述哪项是错误的(2.5分) A.妊娠早期患肝炎致畸发生率高
B.妊娠早期患肝炎易发展为急性、亚急性肝炎
C.妊娠晚期发病易并发子痫前期
D.妊娠中晚期发病易诱发D IC
E.妊娠期发生病毒性肝炎致围生(产)儿死亡率高
D.妊娠中晚期发病易诱发D IC
6 7 2012 13. Which period of pregnancy is most likely to occur heart failure in pregnant women suffering from heat disease:(2.5分) A.20-22 weeks of pregnancy
B.24-26 weeks of pregnancy
C.28-30 weeks of pregnancy
D.32-34 weeks of pregnancy
E.36-38 weeks of pregnancy
D.32-34 weeks of pregnancy
6 7 2012 14. 女,31岁,妊娠26周。发现尿糖(+),口服葡萄糖耐量试验结果:空腹血糖5.6mmol/L,1小时血糖8.9mmol/L,2小时血糖7.6mmol/L,既往无糖尿病史。最可能的诊断是 (3.5分) A.肾性糖尿
B.糖尿病合并妊娠
C.妊娠期糖尿病
D.继发性糖尿病
E.妊娠期生理性糖尿
C.妊娠期糖尿病
6 7 2012 15. 关于巨大儿,下列说法错误的是(2.5分) A.估计胎儿体重≥4000g且合并糖尿病者,建议剖宫产终止妊娠
B.巨大儿的高危因素包括孕妇肥胖、妊娠合并糖尿病、过期妊娠、经产妇、父母身材高大、高龄产妇、有巨大儿分娩史、种族因素
C.妊娠期发现巨大儿可疑者,建议预防性引产
D.巨大儿经阴道分娩过程中易发生肩难产
E.新生儿需预防低血糖,在出生后30分钟内监测血糖
C.妊娠期发现巨大儿可疑者,建议预防性引产
6 7 2012 16. Each of the following is a risk factor for abruptio placentae EXCEPT(2.5分) A.Hyptension
B.Cigarette smoking
C.Trauma
D.Cocaine use
E.Primiparity
E.Primiparity
6 7 2012 17. A 25-year-old woman at 34 weeks' gestation is noted to have a placenta previa, after she presented with vaginal bleeding and has undergone sonography. At 37 weeks, she has a scheduled cesarean. Upon cesarean section, bluish tissue densely adherent between the uterus and maternal bladder is noted. Which of the following is the most likely diagnosis? (2.5分) A.Placenta accreta
B.Placenta melanoma
C.Placenta percreta
D.Placental polyp
E.Placenta increta
C.Placenta percreta
6 7 2012 18. In which of the following conditions would consumptive coagulopathy most likely be seen? (2.5分) A.Placental abruption
B.Placenta previa
C.Gestational diabetes
D.Multifetal gestation
E.Gestational trophoblastic disease
A.Placental abruption
6 7 2012 19. 在耻骨联合上方听到胎盘杂音时,胎盘应附着在(2.5分) A.子宫底部
B.子宫上段前壁
C.子宫上段后壁
D.子宫下段前壁
E.子宫下段后壁
D.子宫下段前壁
6 7 2012 20. 下列前置胎盘病例,可选择阴道分娩的是(3.5分) A.中央性前置胎盘而胎儿为头位
B.边缘性胎盘而胎儿为头位
C.部分性前置胎盘而胎儿为臀位
D.边缘性胎盘而胎儿为臀位
E.以上都不是
B.边缘性胎盘而胎儿为头位
6 7 2012 21. 下列关于胎盘早剥的处理,正确的是(2.5分) A.大量补液,静滴催产素
B.确诊为重型者,可短期期待观察
C.经阴道分娩者不宜破膜
D.产妇情况恶化,不论胎儿是否存活,均应及时行剖宫产术
E.应用肝素治疗凝血功能障碍
D.产妇情况恶化,不论胎儿是否存活,均应及时行剖宫产术
6 7 2012 22. Which of the following is the most common cause of postpartum hemorrhage after a cesarean delivery ? (2.5分) A.Uterine atony
B.Uterine laceration
C.Coagulopathy
D.Uterine inversion
E.Retained placenta
A.Uterine atony
6 7 2012 23. A 24-year-old primigravid woman,on a home delivery. Immediately after the birth of a 4.1-kg infant, the patient bleeds massively from extensive vaginal and cervical lacerations. She is brought to the nearest hospital in shock. Over 2 hours, 9 units of blood are transfused, and the patient's blood pressure returns to a reasonable level. A hemoglobin value the next day is 6.5 g/dL, and 3 units of packed red blood cells are given. The most likely late sequela to consider in this woman is which of the following? (2.5分) A.Hemochromatosis
B.Stein-Leventhal syndrome
C.Sheehan syndrome
D.Simmonds syndrome
E.Cushing syndrome
C.Sheehan syndrome
6 7 2012 24. 初产妇,急产娩一男婴,体重3900g,胎盘娩出后半小时内有较多量间歇性阴道出血,色红,已肌肉注射催产素20单位,再次查看胎盘完整,检查胎膜有一处见血管中断于胎膜边缘,究竟其出血原因最可能是(2.5分) A.胎盘剥离不全
B.软产道损伤
C.副胎盘残留
D.产后宫缩乏力
E.凝血功能障碍
C.副胎盘残留
6 7 2012 25. 28岁初产妇,临产后静脉滴注催产素,破膜后不久出现烦躁不安、呛咳、呼吸困难、紫绀,10分钟后死亡。本例最可能的诊断是(2.5分) A.低纤维蛋白原血症
B.子痫
C.羊水栓塞
D.重型胎盘早剥
E.子宫破裂
C.羊水栓塞
6 7 2012 26. A 21-year-old G2P2 calls her physician 7 days postpartum because she is concerned that she is still bleeding from the vagina. She describes the bleeding as light pink to bright red and less heavy than the first few days postdelivery. She denies fever or any cramping pain. On examination she is afebrile and has an appropriately sized, nontender uterus. The vagina contains about 10 cc of old, dark blood. The cervix is closed. Which of the following is the most appropriate treatment? (2.5分) A.Antibiotics for endometritis
B.High-dose oral estrogen for placental subinvolution
C.Oxytocin for uterine atony
D.Suction dilation and curettage for retained placenta
E.Reassurance
E.Reassurance
6 7 2012 27. 产妇,26岁。孕38周时胎膜早破入院,48小时后因持续性枕横位以产钳术助娩一活男婴3300g,术后3天发热达39℃,检查发现咽部轻度充血,乳房胀满略疼痛,局部皮肤不红,按之无波动感。宫底脐下一横指,宫体压痛明显,下腹无反跳痛,恶露混浊,稍有臭味。该病人最可能的诊断是(2.5分) A.急性子宫内膜炎
B.急性子宫内膜炎及子宫肌炎
C.上呼吸道感染
D.乳腺炎
E.盆腔腹膜炎
B.急性子宫内膜炎及子宫肌炎
6 7 2012 28. 胎儿双顶径测量超声最佳切面是(2.5分) A.小脑平面;
B.丘脑透明隔腔平面;
C.侧脑室平面;
D.枕下前囟径平面
B.丘脑透明隔腔平面;
6 7 2012 29. 三维超声优势切面是(2.5分) A.纵切面;
B.冠状切面;
C.横切面;
D.多维切面
B.冠状切面;
6 7 2012 30. 女性生殖系统肿瘤影像学表现,下列说法错误的是(2.5分) A.卵巢成熟型畸胎瘤含脂肪.钙化,CT和MR诊断准确率高
B.浆液性囊腺瘤多为单囊.密度(信号)单一,粘液性囊腺瘤多囊.密度(信号)多变
C.卵巢粘液性癌发病率高于浆液性癌。
D.MR对子宫肌瘤的显示优于CT
E.MR对子宫内膜癌的显示优于CT
C.卵巢粘液性癌发病率高于浆液性癌。
6 7 2012 31. 乳腺钼靶摄影最佳时间(3.5分) A.月经干净后一周内
B.月经期
C.月经前期
D.月经中期
E.与经期无关
A.月经干净后一周内
6 7 2012 32. 子宫内膜异位症最常见于(2.5分) A.子宫肌层
B.输卵管
C.卵巢
D.子宫直肠陷凹
E.宫旁韧带
C.卵巢
6 7 2012 33. 子宫内膜癌的最佳检查方法为(2.5分) A.X线
B.CT
C.MRI
D.HSG
E.B超
C.MRI
6 7 2012 34. 诊断异位妊娠最简单的方法是(2.5分) A.尿妊娠试验
B.B超
C.诊断性刮宫
D.后穹窿穿刺
E.妇科检查
B.B超
6 7 2012 35. A 22-year-old woman, who is pregnant at 5 weeks' gestation, complains of severe lower abdominal pain. On examination, she is noted to have a blood pressure of 86/44 mm Hg and heart rate of 120 bpm. Her abdomen is tender. The pelvic examination is difficult to perform due to guarding. The hCG level is 500 mIU/mL and the transvaginal sonogram reveals no intrauterine gestational sac and no adnexal masses. There is some free fluid in the abdomenal cavity. Which of the following is the best management for this patient? (2.5分) A.Repeat hCG level in 48 hours to assess for a rise of 66%.
B.Check the serum progesterone level.
C.Immediate surgery.
D.Intramuscular methotrexate.
E.Repeat sonography in 48 hours.
C.Immediate surgery.
6 7 2012 36. 下列哪项不是输卵管妊娠的结局(2.5分) A.输卵管妊娠流产
B.输卵管妊娠破裂
C.妊娠囊经宫腔由阴道排出
D.持续性异位妊娠
E.继发腹腔妊娠
C.妊娠囊经宫腔由阴道排出
6 7 2012 37. 下列哪项是正常产褥期的表现(2.5分) A.产后24小时体温超过38°
B.产后第一天宫底达脐平
C.产后心动过速
D.产后24小时白细胞应恢复至正常范围
E.产后二周恶露开始转为浆液性
B.产后第一天宫底达脐平
6 7 2012 38. 下述哪项不是抢救羊水栓塞的措施(2.5分) A.抗循环衰竭
B.抗呼吸衰竭
C.纠正D IC 及继发纤溶
D.在第一产程者应加强催产素应用,促使其尽早分娩
E.在第二产程发生者可根据情况经阴道助产
D.在第一产程者应加强催产素应用,促使其尽早分娩
6 7 2013 1. A 32-year-old (G3P1) at term is admitted in labor with an initial cervical examination of 6cm dilatation, complete effacement, and the vertex at -1 station. Estimated fetal weight is 3.8kg, and her first pregnancy resulted in an uncomplicated vaginal delivery of a 3.8kg infant. After 2 hours, there is no cervical change. An intrauterine pressure catheter is placed. This shows two contractions in a 10-minute period, each with a strength of 30 mmHg. What is this abnormality of labor termed? (2.5分) A.prolonged latent phase
B.active-phase arrest
C.failure of decent
D.arrest of latent phase
E.protraction of descent
B.active-phase arrest
6 7 2013 2. A prolonged second stage of labor in the primigravid patient lasts longer than:(2.5分) A.1.0 hours
B.2.0 hours
C.8.0 hours
D.6.0 hours
E.4.0 hours
B.2.0 hours
6 7 2013 3. 子宫动脉发自哪条动脉(2.5分) A.腹主动脉
B.左肾动脉
C.骼总动脉
D.骼外动脉
E.骼内动脉
E.骼内动脉
6 7 2013 4. 下列子宫肌瘤声像图表现,错误的是(2.5分) A.子宫增大或出现局限性隆起
B.肌瘤一般呈圆形低回声区
C.子宫内膜移位与变形
D.宫颈内见无回声液性暗区
E.膀胧产生压迹与变形
D.宫颈内见无回声液性暗区
6 7 2013 5. 下列哪一项临床或声像图表现与子宫发育畸形无关(2.5分) A.原发性闭经
B.不孕
C.习惯性流产
D.难产
E.子宫内膜增厚
E.子宫内膜增厚
6 7 2013 6. 妊娠期高血压疾病最常见的产科并发症是(2.5分) A.胎盘早期剥离
B.急性肾功能衰竭
C.妊娠期高血压疾病性心脏病
D.视网膜剥离
E.HE LLP综合征
A.胎盘早期剥离
6 7 2013 7. 硫酸镁治疗妊娠期高血压疾病剂量过大时,最先出现的毒性反应是(2.5分) A.头晕,血压过低
B.呼吸减慢
C.心率减慢
D.膝反射减退或消失
E.尿量过少
D.膝反射减退或消失
6 7 2013 8. 引起子痫抽搐的主要病理原因是(2.5分) A.颅内脑组织散在出血点
B.颅内小动脉痉挛,脑水肿
C.血钙低下
D.代谢性酸中毒
E.胎盘毒素
B.颅内小动脉痉挛,脑水肿
6 7 2013 9. 妊娠近足月患子痫前期的孕妇,恰当的处理应是(2.5分) A.积极治疗,等待产程发动
B.积极治疗24~48小时,症状无明显改善时应终止妊娠
C.积极治疗一周后,予以引产
D.立即引产
E.立即行剖宫术
B.积极治疗24~48小时,症状无明显改善时应终止妊娠
6 7 2013 10. 初孕妇孕36周,早孕时血压90/60mmHg,近一周来头痛,眼花伴视物不清1天,突然全身抽搐1次,急诊入院。查神志尚清,血压170/105mmHg),LOA ,胎心148次/分,下述处理哪项正确(2.5分) A.应用硫酸镁和地西泮控制抽搐
B.引产
C.积极治疗,24小时后行剖宫产
D.积极控制抽搐,病情控制后6~12小时终止妊娠
E.控制抽搐,稳定病情,至孕37周终止妊娠
A.应用硫酸镁和地西泮控制抽搐
6 7 2013 11. 第一胎,停经38周,一月前血压正常,近一周出现下肢水肿,伴头昏,眼花视力模糊,血压160/100mmHg,尿蛋白(+),尿雌三醇10mg/24h,胎心好,产科处理应是(2.5分) A.立即剖腹取胎
B.积极治疗一周,考虑终止妊娠
C.积极治疗24~48小时,考虑终止妊娠
D.积极治疗等待自然分娩
E.立即破膜及静滴催产素引产
C.积极治疗24~48小时,考虑终止妊娠
6 7 2013 12. 妊娠合并风湿性心脏病,下列哪个体征是早期心衰的可靠诊断依据(2.5分) A.心界扩大
B.心尖部闻及Ⅱ级收缩期杂音
C.肺底部持续性湿 音
D.休息时心率>110次.分
E.下肢凹陷性水肿Ⅰ度
D.休息时心率>110次.分
6 7 2013 13. 下述何项不属乙型病毒性肝炎母婴传播途径 (2.5分) A.粪-口传染
B.娩出时接触母亲产道分泌液或血污染
C.母婴垂直传染
D.乳汁传染
E.密切生活接触传染
A.粪-口传染
6 7 2013 14. 病毒性肝炎对妊娠的影响,下述哪项是错误的(2.5分) A.妊娠早期患肝炎致畸发生率高
B.妊娠早期患肝炎易发展为急性、亚急性肝炎
C.妊娠晚期发病易并发子痫前期
D.妊娠中晚期发病易诱发D IC
E.妊娠期发生病毒性肝炎致围生(产)儿死亡率高
D.妊娠中晚期发病易诱发D IC
6 7 2013 15. Which period of pregnancy is most likely to occur heart failure in pregnant women suffering from heat disease:(2.5分) A.20-22 weeks of pregnancy
B.24-26 weeks of pregnancy
C.28-30 weeks of pregnancy
D.32-34 weeks of pregnancy
E.36-38 weeks of pregnancy
D.32-34 weeks of pregnancy
6 7 2013 16. 女,31岁,妊娠26周。发现尿糖(+),口服葡萄糖耐量试验结果:空腹血糖5.6mmol/L,1小时血糖8.9mmol/L,2小时血糖7.6mmol/L,既往无糖尿病史。最可能的诊断是 (2.5分) A.肾性糖尿
B.糖尿病合并妊娠
C.妊娠期糖尿病
D.继发性糖尿病
E.妊娠期生理性糖尿
C.妊娠期糖尿病
6 7 2013 17. 关于巨大儿,下列说法错误的是(2.5分) A.估计胎儿体重≥4000g且合并糖尿病者,建议剖宫产终止妊娠
B.巨大儿的高危因素包括孕妇肥胖、妊娠合并糖尿病、过期妊娠、经产妇、父母身材高大、高龄产妇、有巨大儿分娩史、种族因素
C.妊娠期发现巨大儿可疑者,建议预防性引产
D.巨大儿经阴道分娩过程中易发生肩难产
E.新生儿需预防低血糖,在出生后30分钟内监测血糖
C.妊娠期发现巨大儿可疑者,建议预防性引产
6 7 2013 18. Each of the following is a risk factor for abruptio placentae EXCEPT: (2.5分) A.Hyptension
B.Cigarette smoking
C.Trauma
D.Cocaine use
E.Primiparity
E.Primiparity
6 7 2013 19. A 25-year-old woman at 34 weeks' gestation is noted to have a placenta previa, after she presented with vaginal bleeding and has undergone sonography. At 37 weeks, she has a scheduled cesarean. Upon cesarean section, bluish tissue densely adherent between the uterus and maternal bladder is noted. Which of the following is the most likely diagnosis? (2.5分) A.Placenta accreta
B.Placenta melanoma
C.Placenta percreta
D.Placental polyp
E.Placenta increta
C.Placenta percreta
6 7 2013 20. In which of the following conditions would consumptive coagulopathy most likely be seen? (2.5分) A.Placental abruption
B.Placenta previa
C.Gestational diabetes
D.Multifetal gestation
E.Gestational trophoblastic disease
A.Placental abruption
6 7 2013 21. 在耻骨联合上方听到胎盘杂音时,胎盘应附着在(2.5分) A.子宫底部
B.子宫上段前壁
C.子宫上段后壁
D.子宫下段前壁
E.子宫下段后壁
D.子宫下段前壁
6 7 2013 22. 下列前置胎盘病例,可选择阴道分娩的是 (2.5分) A.中央性前置胎盘而胎儿为头位
B.边缘性胎盘而胎儿为头位
C.部分性前置胎盘而胎儿为臀位
D.边缘性胎盘而胎儿为臀位
E.以上都不是
B.边缘性胎盘而胎儿为头位
6 7 2013 23. 下列关于胎盘早剥的处理,正确的是 (2.5分) A.大量补液,静滴催产素
B.确诊为重型者,可短期期待观察
C.经阴道分娩者不宜破膜
D.产妇情况恶化,不论胎儿是否存活,均应及时行剖宫产术
E.应用肝素治疗凝血功能障碍
D.产妇情况恶化,不论胎儿是否存活,均应及时行剖宫产术
6 7 2013 24. Which of the following is the most common cause of postpartum hemorrhage after a cesarean delivery ? (2.5分) A.Uterine atony
B.Uterine laceration
C.Coagulopathy
D.Uterine inversion
E.Retained placenta
A.Uterine atony
6 7 2013 25. A 24-year-old primigravid woman,on a home delivery. Immediately after the birth of a 4.1-kg infant, the patient bleeds massively from extensive vaginal and cervical lacerations. She is brought to the nearest hospital in shock. Over 2 hours, 9 units of blood are transfused, and the patient's blood pressure returns to a reasonable level. A hemoglobin value the next day is 6.5 g/dL, and 3 units of packed red blood cells are given. The most likely late sequela to consider in this woman is which of the following? (2.5分) A.Hemochromatosis
B.Stein-Leventhal syndrome
C.Sheehan syndrome
D.Simmonds syndrome
E.Cushing syndrome
C.Sheehan syndrome
6 7 2013 26. 初产妇,急产娩一男婴,体重3900g,胎盘娩出后半小时内有较多量间歇性阴道出血,色红,已肌肉注射催产素20单位,再次查看胎盘完整,检查胎膜有一处见血管中断于胎膜边缘,究竟其出血原因最可能是 (2.5分) A.胎盘剥离不全
B.软产道损伤
C.副胎盘残留
D.产后宫缩乏力
E.凝血功能障碍
C.副胎盘残留
6 7 2013 27. 28岁初产妇,临产后静脉滴注催产素,破膜后不久出现烦躁不安、呛咳、呼吸困难、紫绀,10分钟后死亡。本例最可能的诊断是:(2.5分) A.低纤维蛋白原血症
B.子痫
C.羊水栓塞
D.重型胎盘早剥
E.子宫破裂
C.羊水栓塞
6 7 2013 28. A 21-year-old G2P2 calls her physician 7 days postpartum because she is concerned that she is still bleeding from the vagina. She describes the bleeding as light pink to bright red and less heavy than the first few days postdelivery. She denies fever or any cramping pain. On examination she is afebrile and has an appropriately sized, nontender uterus. The vagina contains about 10 cc of old, dark blood. The cervix is closed. Which of the following is the most appropriate treatment? (2.5分) A.Antibiotics for endometritis
B.High-dose oral estrogen for placental subinvolution
C.Oxytocin for uterine atony
D.Suction dilation and curettage for retained placenta
E.Reassurance
E.Reassurance
6 7 2013 29. 产妇,26岁。孕38周时胎膜早破入院,48小时后因持续性枕横位以产钳术助娩一活男婴3300g,术后3天发热达39℃,检查发现咽部轻度充血,乳房胀满略疼痛,局部皮肤不红,按之无波动感。宫底脐下一横指,宫体压痛明显,下腹无反跳痛,恶露混浊,稍有臭味。该病人最可能的诊断是(2.5分) A.急性子宫内膜炎
B.急性子宫内膜炎及子宫肌炎
C.上呼吸道感染
D.乳腺炎
E.盆腔腹膜炎
B.急性子宫内膜炎及子宫肌炎
6 7 2013 30. 胎儿双顶径测量超声最佳切面是:(2.5分) A.小脑平面;
B.丘脑透明隔腔平面;
C.侧脑室平面;
D.枕下前囟径平面
B.丘脑透明隔腔平面;
6 7 2013 31. 三维超声优势切面是:(2.5分) A.纵切面;
B.冠状切面;
C.横切面;
D.多维切面
B.冠状切面;
6 7 2013 32. 女性生殖系统肿瘤影像学表现,下列说法错误的是(2.5分) A.卵巢成熟型畸胎瘤含脂肪.钙化,CT和MR诊断准确率高
B.浆液性囊腺瘤多为单囊.密度(信号)单一,粘液性囊腺瘤多囊.密度(信号)多变
C.卵巢粘液性癌发病率高于浆液性癌。
D.MR对子宫肌瘤的显示优于CT
E.MR对子宫内膜癌的显示优于CT
C.卵巢粘液性癌发病率高于浆液性癌。
6 7 2013 33. 乳腺钼靶摄影最佳时间(2.5分) A.月经干净后一周内
B.月经期
C.月经前期
D.月经中期
E.与经期无关
A.月经干净后一周内
6 7 2013 34. 子宫内膜异位症最常见于(2.5分) A.子宫肌层
B.输卵管
C.卵巢
D.子宫直肠陷凹
E.宫旁韧带
C.卵巢
6 7 2013 35. 子宫内膜癌的最佳检查方法为(2.5分) A.X线
B.CT
C.MRI
D.HSG
E.B超
C.MRI
6 7 2013 36. 诊断异位妊娠最简单的方法是(2.5分) A.尿妊娠试验
B.B超
C.诊断性刮宫
D.后穹窿穿刺
E.妇科检查
B.B超
6 7 2013 37. A 22-year-old woman, who is pregnant at 5 weeks' gestation, complains of severe lower abdominal pain. On examination, she is noted to have a blood pressure of 86/44 mm Hg and heart rate of 120 bpm. Her abdomen is tender. The pelvic examination is difficult to perform due to guarding. The hCG level is 500 mIU/mL and the transvaginal sonogram reveals no intrauterine gestational sac and no adnexal masses. There is some free fluid in the abdomenal cavity. Which of the following is the best management for this patient? (2.5分) A.Repeat hCG level in 48 hours to assess for a rise of 66%.
B.Check the serum progesterone level.
C.Immediate surgery.
D.Intramuscular methotrexate.
E.Repeat sonography in 48 hours.
C.Immediate surgery.
6 7 2013 38. 下列哪项不是输卵管妊娠的结局:(2.5分) A.输卵管妊娠流产
B.输卵管妊娠破裂
C.妊娠囊经宫腔由阴道排出
D.持续性异位妊娠
E.继发腹腔妊娠
C.妊娠囊经宫腔由阴道排出
6 7 2013 39. 下列哪项是正常产褥期的表现(2.5分) A.产后24小时体温超过38°
B.产后第一天宫底达脐平
C.产后心动过速
D.产后24小时白细胞应恢复至正常范围
E.产后二周恶露开始转为浆液性
B.产后第一天宫底达脐平
6 7 2013 40. 下述哪项不是抢救羊水栓塞的措施 (2.5分) A.抗循环衰竭
B.抗呼吸衰竭
C.纠正D IC 及继发纤溶
D.在第一产程者应加强催产素应用,促使其尽早分娩
E.在第二产程发生者可根据情况经阴道助产
D.在第一产程者应加强催产素应用,促使其尽早分娩
6 7 2014 1. Which of the following patients are more likely than others to have uterine atony and hemorrhage after delivery? (2.5分) A.Prolonged labor
B.Primigravidas
C.Hypertensive disorders
D.Pudendal anesthesia for delivery
E.Obesity
A.Prolonged labor
6 7 2014 2. A prolonged second stage of labor in the primigravid patient lasts longer than:(2.5分) A.1.0 hours
B.2.0 hours
C.8.0 hours
D.6.0 hours
E.4.0 hours
B.2.0 hours
6 7 2014 3. 正常成年女性的子宫大小,下列哪组数据是正确的:(2.5分) A.纵径:7.0~8.0cm、前后径:3.0~4.0cm、横径:4.0~5.0cm
B.纵径:5.0~6.0cm、前后径4.0~5.0cm、横径:3.0-4.0cm
C.纵径:7.0~8.0cm、前后径:4.0~5.0cm 、横径:3.0~4.0cm
D.纵径:5.0~6.0cm、前后径:3.0~4.0cm、横径:4.0~5.0cm
E.纵径:6.0~8.0cm、前后径:3.0~4.0cm、横径:3.0~5.0cm
A.纵径:7.0~8.0cm、前后径:3.0~4.0cm、横径:4.0~5.0cm
6 7 2014 4. 下面哪一选项不是卵巢内膜异位囊肿的超声特征:(2.5分) A.囊内液稠厚
B.囊壁粗糙
C.囊内无实性回声
D.常为多房囊性
E.囊壁上血流信号不明显
D.常为多房囊性
6 7 2014 5. 胎儿中孕期超声检查头颅扫查,哪一选项不是超声检查常规要求的?(2.5分) A.透明隔腔
B.脑中线
C.侧脑室
D.脑沟回
E.后颅窝
D.脑沟回
6 7 2014 6. 24岁初孕妇,经检查确诊为妊娠33周轻度子痫前期。为防止发展为重度子痫前期,下列处置不适当的项目是:(2.5分) A.适当减轻工作,保证睡眠10小时
B.休息和睡眠时取左侧卧位
C.适当服用镇静药物
D.严格限制食盐摄入量
E.适当增加产前检查次数
D.严格限制食盐摄入量
6 7 2014 7. 胎儿生长受限最常见的病因是:(2.5分) A.臀先露
B.高龄初产
C.妊娠期高血压疾病
D.胎盘早剥
E.宫内感染
C.妊娠期高血压疾病
6 7 2014 8. 关于死胎正确的是:(2.5分) A.胚胎死于子宫内
B.胎儿死亡后立即释放促凝血物质入血
C.胎儿分娩过程中死亡称死产,也是死胎的一种
D.孕妇自觉胎动停止,子宫停止生长即可诊断死胎
E.胎儿死亡不久,B型超声即可见到颅板塌陷,颅骨重叠,呈袋状变形
E.胎儿死亡不久,B型超声即可见到颅板塌陷,颅骨重叠,呈袋状变形
6 7 2014 9. 重度子痫前期的产科处理,下述哪项是错误的:(2.5分) A.孕<34周,经治疗病情好转而稳定,可继续妊娠
B.孕>34周,估计胎儿可成活,应考虑终止妊娠
C.经积极治疗24~48小时病情继续恶化,应继续积极治疗至病情稳定后终止妊娠
D.子痫患者应积极治疗,控制抽搐6~12小时终止妊娠
E.引产失败应行剖宫术
D.子痫患者应积极治疗,控制抽搐6~12小时终止妊娠
6 7 2014 10. 34岁,初孕妇,妊娠32周,口服75克葡萄糖耐量试验(OGTT) 3项血糖值均异常,诊断为妊娠期糖尿病,经饮食控制后尿糖+。首选的处理措施是:(2.5分) A.继续控制饮食,监测三餐前后血糖
B.胰岛素治疗
C.人工破膜术终止妊娠
D.缩宫素静滴引产
E.立即行剖宫产术
A.继续控制饮食,监测三餐前后血糖
6 7 2014 11. 第一胎,停经37周,直至一周前血压均正常。近一周下肢浮肿,有头昏,视力模糊症状。血压21/13.3kPa(154/100 mmHg),尿蛋白(+),听胎心音正常,下列处理哪项最合适:(2.5分) A.积极治疗,38周时终止妊娠
B.积极治疗,等待自然分娩
C.积极治疗24~48小时,终止妊娠
D.立即人工破膜加静脉滴注催产素引产
E.立即剖宫产
C.积极治疗24~48小时,终止妊娠
6 7 2014 12. 孕34周,血压160/110mmHg,尿蛋白5.3g/24h,上腹痛,头晕眼花,应诊断为:(2.5分) A.妊娠期高血压病
B.轻度子痫前期
C.重度子痫前期
D.妊娠合并肝炎
E.妊娠合并慢性肾炎
C.重度子痫前期
6 7 2014 13. 孕早期心脏病患者,决定是否能继续妊娠的最重要依据是:(2.5分) A.心脏病种类
B.心脏病变部位
C.心功能分级
D.症状严重程度
E.有否以往生育史
C.心功能分级
6 7 2014 14. 妊娠合并心脏病中最常见的类型是:(2.5分) A.风湿性心脏病
B.妊娠期高血压疾病性心脏病
C.先天性心脏病
D.贫血性心脏病
E.围生期心肌病
C.先天性心脏病
6 7 2014 15. 下述何项不属乙型病毒性肝炎母婴传播途径:(2.5分) A.粪-口传染
B.娩出时接触母亲产道分泌液或血污染
C.母婴垂直传染
D.乳汁传染
E.密切生活接触传染
A.粪-口传染
6 7 2014 16. 病毒性肝炎对妊娠的影响,下述哪项是错误的:(2.5分) A.妊娠早期患肝炎致畸发生率高
B.妊娠早期患肝炎易发展为急性、亚急性肝炎
C.妊娠晚期发病易并发子痫前期
D.妊娠中晚期发病易诱发D IC
E.妊娠期发生病毒性肝炎致围生(产)儿死亡率高
D.妊娠中晚期发病易诱发D IC
6 7 2014 17. 女,31岁,妊娠26周。发现尿糖(+),口服葡萄糖耐量试验结果:空腹血糖5.6mmol/L,1小时血糖8.9mmol/L,2小时血糖7.6mmol/L,既往无糖尿病史。最可能的诊断是:(2.5分) A.肾性糖尿
B.糖尿病合并妊娠
C.妊娠期糖尿病
D.继发性糖尿病
E.妊娠期生理性糖尿
C.妊娠期糖尿病
6 7 2014 18. Each of the following is a risk factor for abruptio placentae EXCEPT: (2.5分) A.Hyptension
B.Cigarette smoking
C.Trauma
D.Cocaine use
E.Primiparity
E.Primiparity
6 7 2014 19. A 25-year-old woman at 34 weeks' gestation is noted to have a placenta previa, after she presented with vaginal bleeding and has undergone sonography. At 37 weeks, she has a scheduled cesarean. Upon cesarean section, bluish tissue densely adherent between the uterus and maternal bladder is noted. Which of the following is the most likely diagnosis? (2.5分) A.Placenta accreta
B.Placenta melanoma
C.Placenta percreta
D.Placental polyp
E.Placenta increta
C.Placenta percreta
6 7 2014 20. In which of the following conditions would consumptive coagulopathy most likely be seen? (2.5分) A.Placental abruption
B.Placenta previa
C.Gestational diabetes
D.Multifetal gestation
E.Gestational trophoblastic disease
A.Placental abruption
6 7 2014 21. 在耻骨联合上方听到胎盘杂音时,胎盘应附着在(2.5分) A.子宫底部
B.子宫上段前壁
C.子宫上段后壁
D.子宫下段前壁
E.子宫下段后壁
D.子宫下段前壁
6 7 2014 22. 下列前置胎盘病例,可选择阴道分娩的是:(2.5分) A.中央性前置胎盘而胎儿为头位
B.边缘性胎盘而胎儿为头位
C.部分性前置胎盘而胎儿为臀位
D.边缘性胎盘而胎儿为臀位
E.以上都不是
B.边缘性胎盘而胎儿为头位
6 7 2014 23. 下列关于胎盘早剥的处理,正确的是(2.5分) A.大量补液,静滴催产素
B.确诊为重型者,可短期期待观察
C.经阴道分娩者不宜破膜
D.产妇情况恶化,不论胎儿是否存活,均应及时行剖宫产术
E.应用肝素治疗凝血功能障碍
D.产妇情况恶化,不论胎儿是否存活,均应及时行剖宫产术
6 7 2014 24. Which of the following is the most common cause of postpartum hemorrhage after a cesarean delivery ? (2.5分) A.Uterine atony
B.Uterine laceration
C.Coagulopathy
D.Uterine inversion
E.Retained placenta
A.Uterine atony
6 7 2014 25. A 24-year-old primigravid woman,on a home delivery. Immediately after the birth of a 4.1-kg infant, the patient bleeds massively from extensive vaginal and cervical lacerations. She is brought to the nearest hospital in shock. Over 2 hours, 9 units of blood are transfused, and the patient's blood pressure returns to a reasonable level. A hemoglobin value the next day is 6.5 g/dL, and 3 units of packed red blood cells are given. The most likely late sequela to consider in this woman is which of the following? (2.5分) A.Hemochromatosis
B.Stein-Leventhal syndrome
C.Sheehan syndrome
D.Simmonds syndrome
E.Cushing syndrome
C.Sheehan syndrome
6 7 2014 26. 初产妇,急产娩一男婴,体重3900g,胎盘娩出后半小时内有较多量间歇性阴道出血,色红,已肌肉注射催产素20单位,再次查看胎盘完整,检查胎膜有一处见血管中断于胎膜边缘,究竟其出血原因最可能是:(2.5分) A.胎盘剥离不全
B.软产道损伤
C.副胎盘残留
D.产后宫缩乏力
E.凝血功能障碍
C.副胎盘残留
6 7 2014 27. 28岁初产妇,临产后静脉滴注催产素,破膜后不久出现烦躁不安、呛咳、呼吸困难、紫绀,10分钟后死亡。本例最可能的诊断是:(2.5分) A.低纤维蛋白原血症
B.子痫
C.羊水栓塞
D.重型胎盘早剥
E.子宫破裂
C.羊水栓塞
6 7 2014 28. A 21-year-old G2P2 calls her physician 7 days postpartum because she is concerned that she is still bleeding from the vagina. She describes the bleeding as light pink to bright red and less heavy than the first few days postdelivery. She denies fever or any cramping pain. On examination she is afebrile and has an appropriately sized, nontender uterus. The vagina contains about 10 cc of old, dark blood. The cervix is closed. Which of the following is the most appropriate treatment? (2.5分) A.Antibiotics for endometritis
B.High-dose oral estrogen for placental subinvolution
C.Oxytocin for uterine atony
D.Suction dilation and curettage for retained placenta
E.Reassurance
E.Reassurance
6 7 2014 29. 产妇,26岁。孕38周时胎膜早破入院,48小时后因持续性枕横位以产钳术助娩一活男婴3300g,术后3天发热达39℃,检查发现咽部轻度充血,乳房胀满略疼痛,局部皮肤不红,按之无波动感。宫底脐下一横指,宫体压痛明显,下腹无反跳痛,恶露混浊,稍有臭味。该病人最可能的诊断是(2.5分) A.急性子宫内膜炎
B.急性子宫内膜炎及子宫肌炎
C.上呼吸道感染
D.乳腺炎
E.盆腔腹膜炎
B.急性子宫内膜炎及子宫肌炎
6 7 2014 30. 女性生殖系统肿瘤影像学表现,下列说法错误的是(2.5分) A.卵巢成熟型畸胎瘤含脂肪.钙化,CT和MR诊断准确率高
B.浆液性囊腺瘤多为单囊.密度(信号)单一,粘液性囊腺瘤多囊.密度(信号)多变
C.卵巢粘液性癌发病率高于浆液性癌。
D.MR对子宫肌瘤的显示优于CT
E.MR对子宫内膜癌的显示优于CT
C.卵巢粘液性癌发病率高于浆液性癌。
6 7 2014 31. 乳腺钼靶摄影常规位置:(2.5分) A.轴位+侧位
B.轴位+斜位
C.轴位+放大摄影
D.侧位+斜位
E.侧位+放大摄影
B.轴位+斜位
6 7 2014 32. 子宫腺肌病MRI最常表现为累及(2.5分) A.子宫浆膜层
B.子宫内膜
C.结合带
D.双侧卵巢
E.两侧输卵管
C.结合带
6 7 2014 33. A 20-year-old G2P1 woman at 9 weeks gestation underwent an spontaneous abortion. But her pathology of the uterine curetting revealed no chorionic villi. Her pregnancy test was still positive. Which of the following is the most likely diagnosis?(2.5分) A.Ectopic pregnancy
B.PID
C.Endometriosis
D.Appendicitis
E.Ruptured corpus luteum cyst of the ovary
A.Ectopic pregnancy
6 7 2014 34. A 23-year-old woman(G1P1001)at about 12 weeks' gestation develops persistent nausea and vomiting that progresses from an occasional episode to a constant retching. She has no fever or diarrhea but loses 5 Kg in 1 week and appears dehydrated. What is her most likely diagnosis? (2.5分) A.Anorexia nervosa
B.Morning sichness
C.Ptyalism
D.Hyperemesis gravidarum
E.Gastroenteritis
D.Hyperemesis gravidarum
6 7 2014 35. 下列那种情况不是胎儿窘迫的临床征象(2.5分) A.频繁的变异减速
B.晚期减速
C.羊水Ⅲ度
D.胎动减少明显
E.早期减速
E.早期减速
6 7 2014 36. 妊娠晚期羊水量少于多少被称为羊水过少(2.5分) A.100ML
B.300ML
C.500ML
D.1000ML
E.2000ML
B.300ML
6 7 2014 37. A 22-year-old woman, who is pregnant at 5 weeks' gestation, complains of severe lower abdominal pain. On examination, she is noted to have a blood pressure of 86/44 mm Hg and heart rate of 120 bpm. Her abdomen is tender. The pelvic examination is difficult to perform due to guarding. The hCG level is 500 mIU/mL and the transvaginal sonogram reveals no intrauterine gestational sac and no adnexal masses. There is some free fluid in the abdomenal cavity. Which of the following is the best management for this patient? (2.5分) A.Repeat hCG level in 48 hours to assess for a rise of 66%.
B.Check the serum progesterone level.
C.Immediate surgery.
D.Intramuscular methotrexate.
E.Repeat sonography in 48 hours.
C.Immediate surgery.
6 7 2014 38. You are counseling a 28-year-old woman who is a G2P1 at 42 weeks gestation. She reports that her period always is regular (3-5/30) and she had routine OB visits since 12 weeks gestation. She is disappointed and anxiety because there is no any signs of labor. And she tell you fetal movement decreased 12 hours ago. Which of the following laboratory tests is necessary for the diagnosis of fetus state, except: (2.5分) A.NST test
B.OCT test
C.Blood AFP determination
D.Blood HPL determination
E.E3 determination in 24 hour's urine
C.Blood AFP determination
6 7 2014 39. 孕32周时出现胎膜早破,予期待疗法,下列何种药物不适合:(2.5分) A.硫酸镁
B.安定
C.舒喘灵
D.氨苄青
E.利托君
B.安定
6 7 2014 40. 患者孕34周,因诊断先兆早产保胎治疗。一日静脉滴注硫酸镁时发现膝反射消失,呼吸15次/分,正确的处理是:(2.5分) A.停用硫酸镁、静滴硫化钾
B.停用硫酸镁、静注钙剂
C.停用硫酸镁、静滴氯化钠
D.停用硫酸镁、静滴葡萄糖
E.停用硫酸镁、静滴维生素C
B.停用硫酸镁、静注钙剂
6 7 2015 1. 患者,24岁,1-0-2-1,妊娠43周,家中接生婆接生未成功,临产25小时送至我院,入院时孕妇痛苦异常,子宫呈葫芦状,子宫下端压痛,肉眼血尿。血压120/60mmHg,心率95次/分,胎心未闻及。本例诊断无关的项目为:(2.5分) A.滞产
B.过期妊娠
C.子宫先兆破裂
D.泌尿系感染
E.死产
D.泌尿系感染
6 7 2015 2. 初产妇28岁,40周妊娠,凌晨3时临产,当晚8时宫口开全,宫缩20″/2′,至10时10分宫缩持续20″/4′,胎心率140次/分,羊水Ⅰ污染,阴道检查胎头棘下+3cm,胎头矢状缝与母体骨盆前后径一致,后囟在前方。最正确的诊断为(2.5分) A.胎儿宫内窘迫
B.ROA
C.滞产
D.第二产程延长
E.胎头内旋转受阻
D.第二产程延长
6 7 2015 3. A 32-year-old (G3P1) at term is admitted in labor with an initial cervical examination of 6cm dilatation, complete effacement, and the vertex at -1 station. Estimated fetal weight is 3.8kg, and her first pregnancy resulted in an uncomplicated vaginal delivery of a 3.8kg infant. After 2 hours, there is no cervical change. An intrauterine pressure catheter is placed. This shows two contractions in a 10-minute period, each with a strength of 30 mmHg. What is this abnormality of labor termed?(2.5分) A.prolonged latent phase
B.active-phase arrest
C.failure of decent
D.arrest of latent phase
E.protraction of descent
B.active-phase arrest
6 7 2015 4. 5岁妊娠38周初孕妇,晨起突然剧烈头痛伴喷射性呕吐,查血压160/110mmHg。尿蛋白(+++),下列治疗措施错误的是:(2.5分) A.静注硫酸镁4g后,继续静滴1g/h
B.静注地西泮10mg
C.口服硝苯地平10mg
D.快速静滴20%甘露醇250ml
E.静注地塞米松20mg
E.静注地塞米松20mg
6 7 2015 5. 关于胎儿生长受限正确的是:(2.5分) A.足月胎儿出生体重<2000g
B.胎儿体重低于同孕龄平均体重的3个标准差
C.胎儿体重低于同孕龄体重的第5百分位数
D.胎儿发育指数正常值为>+3
E.宫高、腹围连续3周测量在第10百分位数以下为筛选FGR的指标
E.宫高、腹围连续3周测量在第10百分位数以下为筛选FGR的指标
6 7 2015 6. 关于死胎正确的是(2.5分) A.胚胎死于子宫内
B.胎儿死亡后立即释放促凝血物质入血
C.胎儿分娩过程中死亡称死产,也是死胎的一种
D.孕妇自觉胎动停止,子宫停止生长即可诊断死胎
E.胎儿死亡不久,B型超声即可见到颅板塌陷,颅骨重叠,呈袋状变形
C.胎儿分娩过程中死亡称死产,也是死胎的一种
6 7 2015 7. 重度子痫前期的产科处理,下述哪项是错误的(2.5分) A.孕<34周,经治疗病情好转而稳定,可继续妊娠
B.孕>34周,估计胎儿可成活,应考虑终止妊娠
C.经积极治疗24~48小时病情继续恶化,应继续积极治疗至病情稳定后终止妊娠
D.子痫患者应积极治疗,控制抽搐6~12小时终止妊娠
E.引产失败应行剖宫术
D.子痫患者应积极治疗,控制抽搐6~12小时终止妊娠
6 7 2015 8. 29岁,初孕妇,妊娠37周,妊娠期糖尿病,胎心率140次/分,产前监护NST无反应型,首先采用的处理措施是: (2.5分) A.监测孕妇血糖
B.B型超声生物物理评分
C.人工破膜了解羊水性状
D.立即剖宫产
E.3日后复查NST
B.B型超声生物物理评分
6 7 2015 9. 初孕妇孕36周,早孕时血压90/60mmHg,近一周来头痛,眼花伴视物不清1天,突然全身抽搐1次,急诊入院。查神志尚清,血压170/105mmHg),LOA ,胎心148次/分,下述处理哪项正确(2.5分) A.应用硫酸镁和地西泮控制抽搐
B.引产
C.积极治疗,24小时后行剖宫产
D.积极控制抽搐,病情控制后6~12小时终止妊娠
E.控制抽搐,稳定病情,至孕37周终止妊娠
A.应用硫酸镁和地西泮控制抽搐
6 7 2015 10. 初孕妇,妊娠36周,血压190/110mmHg,尿蛋白(++),下腹及颜面浮肿,今晨起头痛剧烈,伴眼花,呕吐,胎位胎心正常,无宫缩,本例最恰当的紧急处理是(2.5分) A.立即剖腹产
B.催产素静滴引产
C.胎头吸引术
D.人工破膜
E.解痉,镇静,利尿,降低颅内压,短期内终止妊娠
E.解痉,镇静,利尿,降低颅内压,短期内终止妊娠
6 7 2015 11. 孕早期心脏病患者,决定是否能继续妊娠的最重要依据是(2.5分) A.心脏病种类
B.心脏病变部位
C.心功能分级
D.症状严重程度
E.有否以往生育史
C.心功能分级
6 7 2015 12. 妊娠合并心脏病早期心衰的表现是(2.5分) A.踝部凹陷性水肿
B.休息时心率>110次/分
C.心脏浊音界扩大
D.心尖部闻及期前收缩
E.颈静脉怒张
B.休息时心率>110次/分
6 7 2015 13. 妊娠合并心脏病孕妇分娩期血流动力学变化,下述哪项是错误的:(2.5分) A.第一产程,规律宫缩使周围循环阻力增加,回心血量减少
B.第二产程,宫缩加上腹压,周围阻力增大
C.第二产程,腹压增高使内脏血流涌向心脏,回心血量增加
D.第三产程,子宫迅速缩小,腹压减低,大量血液流回内脏血管,回心血管减少
E.第三产程,胎儿娩出后子宫胎盘循环停止,大量血液进入体循环,回心血量增加
D.第三产程,子宫迅速缩小,腹压减低,大量血液流回内脏血管,回心血管减少
6 7 2015 14. 关于风湿性心脏病孕妇的分娩期处理,正确的是(2.5分) A.除有产科指征外,不需作剖宫产术
B.宫口开全要防止产妇用力屏气
C.忌用吗啡
D.无感染征象,不需使用抗生素
E.肌注麦角新硷,预防产后出血
B.宫口开全要防止产妇用力屏气
6 7 2015 15. 有关妊娠合并心脏病,下述哪项是错误的(2.5分) A.宫口开全后应防止产妇用力屏气
B.胎儿娩出后给予镇静剂如杜冷丁或吗啡
C.胎儿娩出后,若子宫收缩不佳,可肌注麦角新碱
D.如有产后出血,可以输血,但需要注意输血速度
E.产前,产时有心力衰竭的产妇产后继续用强心药
C.胎儿娩出后,若子宫收缩不佳,可肌注麦角新碱
6 7 2015 16. 对于妊娠合并病毒性肝炎的并发症,哪项是错误的(2.5分) A.易出现早产、流产
B.可出现胎儿窘迫
C.易发生重型肝炎
D.易发生羊水过多
E.子痫前期发生率增加
D.易发生羊水过多
6 7 2015 17. 28岁,女性,风湿性心脏病、二尖瓣狭窄病史5年。平时不用药,登三楼无明显不适。孕5月起活动时常有轻度心慌、气促。现孕38周,因心悸,咳嗽,夜间不能平卧,心功能Ⅲ级而急诊入院。在制定治疗计划时,最佳的方案是:(2.5分) A.积极控制心衰后终止妊娠
B.积极控制心衰,同时行剖宫产术
C.积极控制心衰,同时行引产术
D.适量应用抗生素后继续妊娠
E.纠正心功能,等待自然临产
A.积极控制心衰后终止妊娠
6 7 2015 18. 下列不是胎盘早剥严重并发症的是(2.5分) A.急性肾功能衰竭
B.凝血功能异常
C.产后出血
D.高血压
E.重度贫血
D.高血压
6 7 2015 19. 34岁初产妇。阴道分娩后出现大量阴道流血,检查发现胎盘子体面约1/5血凝块压迹,考虑部分性胎盘早剥。出现头晕面色苍白,静脉穿刺点多处渗血.以下治疗首选的是 (2.5分) A.子宫切除
B.宫腔填塞
C.髂内动脉结扎术
D.子宫卵巢动脉结扎术
E.纠正凝血功能
E.纠正凝血功能
6 7 2015 20. 孕38周,诊断为胎盘早剥行剖宫产术,术中见子宫紫色,质软,胎儿胎盘娩出后,大量出血,注射宫缩剂并按摩子宫,出血不止,重度休克,凝血功能严重异常。此时首选的处理措施是 (2.5分) A.止血药物
B.宫腔填塞
C.按摩子宫加热敷
D.子宫切除
E.大量输血补液等待血压上升
D.子宫切除
6 7 2015 21. 在耻骨联合上方听到胎盘杂音时,胎盘应附着在(2.5分) A.子宫底部
B.子宫上段前壁
C.子宫上段后壁
D.子宫下段前壁
E.子宫下段后壁
D.子宫下段前壁
6 7 2015 22. 下列哪项不符合中央性(完全性)前置胎盘的常见临床表现(2.5分) A.胎位不正的发生率高
B.先露高浮
C.第一次出血多在孕36周以后
D.易发生出血性休克
E.较其他类型前置胎盘易发生胎儿宫内窘迫
C.第一次出血多在孕36周以后
6 7 2015 23. 剖宫产后产后出血最常见的原因是(2.5分) A.宫缩发力
B.子宫裂伤
C.凝血功能异常
D.子宫内翻
E.胎盘残留
A.宫缩发力
6 7 2015 24. 24岁初产妇,家庭分娩4.1Kg男婴。新生儿娩出后出现大量阴道流血,转运到最近医院,出现意识模糊及血压低。2小时内输注9u红细胞血压恢复正常.次日血Hg 6.5 g/dL,再输3.5U红细胞。最可能发生的晚期并发症是(2.5分) A.贫血
B.Stein-Leventhal综合征
C.希恩综合征
D.西蒙兹综合征
E.库欣综合征
C.希恩综合征
6 7 2015 25. 有关先兆子宫破裂的诊断依据,下列哪项是错误的 (2.5分) A.先露下降受阻,产程延长
B.血红蛋白迅速下降
C.可见病理性缩复环
D.可导致明显血尿
E.下腹剧痛,拒按
B.血红蛋白迅速下降
6 7 2015 26. 胎盘未剥离时过早挤揉子宫可造成 (2.5分) A.胎盘残留
B.胎盘粘连
C.胎盘剥离不全
D.胎盘嵌顿
E.胎盘植入
C.胎盘剥离不全
6 7 2015 27. 28岁初产妇,临产后静脉滴注催产素,破膜后不久出现烦躁不安、呛咳、呼吸困难、紫绀,10分钟后死亡。本例最可能的诊断是(2.5分) A.低纤维蛋白原血症
B.子痫
C.羊水栓塞
D.重型胎盘早剥
E.子宫破裂
C.羊水栓塞
6 7 2015 28. 羊水栓塞最早出现的症状是(2.5分) A.急性左心衰竭
B.急性肝功能衰竭
C.急性肾功能衰竭
D.急性呼吸衰竭
E.急性DIC
D.急性呼吸衰竭
6 7 2015 29. 下列是正常产褥期表现的是(2.5分) A.产后24小时体温超过38°
B.产后第一天宫底达脐平
C.产后心动过速
D.产后24小时白细胞应恢复至正常范围
E.产后二周恶露开始转为浆液性
B.产后第一天宫底达脐平
6 7 2015 30. 26岁,初产妇。孕38周时胎膜早破入院,48小时后因持续性枕横位以产钳术助娩一活男婴3300g,术后3天发热达39℃,检查发现咽部轻度充血,乳房胀满略疼痛,局部皮肤不红,按之无波动感。宫底脐下一横指,宫体压痛明显,下腹无反跳痛,恶露混浊,稍有臭味。该病人最可能的诊断是(2.5分) A.急性子宫内膜炎
B.急性子宫内膜炎及子宫肌炎
C.上呼吸道感染
D.乳腺炎
E.盆腔腹膜炎
B.急性子宫内膜炎及子宫肌炎
6 7 2015 31. A 20-year-old G2P1 woman at 9 weeks gestation underwent an spontaneous abortion. But her pathology of the uterine curetting revealed no chorionic villi. Her pregnancy test was still positive. Which of the following is the most likely diagnosis? (2.5分) A.Ectopic pregnancy
B.PID
C.Endometriosis
D.Appendicitis
E.Ruptured corpus luteum cyst of the ovary
A.Ectopic pregnancy
6 7 2015 32. Which of the following cases can be diagnosed as a missed abortion? (2.5分) A.Uterine bleeding at 12 weeks gestation accompanied by cervical dilation without passage of tissue.
B.Passage of some but not all placental tissue through the cervix at 9 weeks gestation.
C.Fetal death at 15 weeks gestation without expulsion of any fetal or maternal tissue for at least 8 weeks.
D.Expulsion of all fetal and placental tissue from the uterine cavity at10 weeks gestation.
E.Uterine bleeding at 7 weeks gestation without any cervical dilation.
C.Fetal death at 15 weeks gestation without expulsion of any fetal or maternal tissue for at least 8 weeks.
6 7 2015 33. A 33-year-old woman at 10 weeks gestation complains of vaginal bleeding and passage of a whitish substance along with something "meatlike." She continues to have cramping, and her cervix is 2 cm dilated, Which of the following is the best treatment? (2.5分) A.Laparoscopy
B.Follow up hCG level in 48 hours
C.Cervical cerclage
D.Dilation and curettage of uterus
E.Expectant management
D.Dilation and curettage of uterus
6 7 2015 34. 关于羊水来源下列哪项是错误的? (2.5分) A.妊娠早期.羊水是母体血清经胎膜入羊膜腔透析液
B.经脐带的华通胶
C.胎盘表面羊膜
D.妊娠早期还可通过胎儿尿液
E.通过未角化的胎儿皮肤漏出
D.妊娠早期还可通过胎儿尿液
6 7 2015 35. 下列那种情况不是胎儿窘迫的临床征象(2.5分) A.频繁的变异减速
B.晚期减速
C.羊水Ⅲ度
D.胎动减少明显
E.早期减速
E.早期减速
6 7 2015 36. 下种何种情况需急诊剖宫产终止妊娠(2.5分) A.孕36周,胎心出现早期减速
B.宫口开大3cm,发现羊水Ⅰ°
C.胎心出现频繁晚期减速
D.宫口已开全,先露头,+3,羊水Ⅲ°
E.胎心持续偏快,基线率在170次/分,吸氧,左侧卧位后恢复正常
C.胎心出现频繁晚期减速
6 7 2015 37. 孕32周时出现胎膜早破,予期待疗法,下列何种药物不适合(2.5分) A.硫酸镁
B.安定
C.舒喘灵
D.氨苄青
E.利托君
B.安定
6 7 2015 38. 下列子宫肌瘤声像图表现,错误的是(2.5分) A.子宫增大或出现局限性隆起
B.肌瘤一般呈圆形低回声区
C.子宫内膜移位与变形
D.宫颈内见无回声液性暗区
E.膀胧产生压迹与变形
D.宫颈内见无回声液性暗区
6 7 2015 39. 卵巢肿瘤影像学表现,下列说法错误的是(2.5分) A.卵巢成熟型畸胎瘤含脂肪、钙化,CT和MR诊断准确率高
B.浆液性囊腺瘤多为单囊,密度/信号单一,粘液性囊腺瘤多囊,密度/信号多变
C.卵巢性索间质肿瘤的发病率高于上皮性肿瘤。
D.MR对子宫肌瘤的显示优于CT
E.MR对子宫内膜癌的显示优于CT
C.卵巢性索间质肿瘤的发病率高于上皮性肿瘤。
6 7 2015 40. 下列表述错误的是(2.5分) A.胎儿MRI受母亲情况影响小,磁共振不受胎儿骨骼及羊水量影响
B.胎儿MRI在孕妇肥胖、羊水过少、双胎时显示明显优于超声
C.胎儿MRI在孕18周以前检查可以比超声获得更多的信息
D.胎儿MRI成像视野大,可观察胎儿全貌
E.胎儿MRI对软组织分辨率高于超声
C.胎儿MRI在孕18周以前检查可以比超声获得更多的信息
6 8 2006 1. 头部不适宜用 (3.0分) A.酊剂
B.洗剂
C.乳剂
D.溶液
E.乳剂
B.洗剂
6 8 2006 2. 红斑狼疮中SCLE约占:(3.0分) A. 5%一10%
B. 15%一20%
C. 10%一15%
D. 20%一25%
C. 10%一15%
6 8 2006 3. 皮肤的颜色与下列哪个因素无关 (3.0分) A.营养
B.身高
C.性别
D.种族
E.部位
B.身高
6 8 2006 4. 湿敷适用于下列哪种情况 (3.0分) A.无皮疹仅瘙痒时
B.急性湿疹伴大量渗出时
C.红斑丘疹时
D.慢性皮炎时
E.荨麻疹的风团
B.急性湿疹伴大量渗出时
6 8 2006 5. 冷冻治疗的原理 (3.0分) A.细胞膜类脂蛋白复合物变性
B.细胞中毒
C.机械损伤
D.血液淤滞
E.以上都是
A.细胞膜类脂蛋白复合物变性
6 8 2006 6. 腋臭是由于哪种腺体在青春期分泌的乳状液经细菌分解后产生臭味引起的 (3.0分) A.皮脂腺
B.小汗腺
C.乳腺
D.顶泌汗腺
E.唾液腺
D.顶泌汗腺
6 8 2006 7. 下列那种细胞不是真皮中的成分(3.0分) A.巨噬细胞
B.成纤维细胞
C.肥大细胞
D.梅克尔细胞
E.树突状细胞
D.梅克尔细胞
6 8 2006 8. 异位性素质(atopy)的含义不包括(3.0分) A.血清中IgE增高
B.常对异种蛋白过敏
C.外周血嗜酸粒细胞增多
D.常有易患哮喘、过敏性鼻炎、湿疹的家族性倾向
E.自身免疫病倾向
E.自身免疫病倾向
6 8 2006 9. 下列哪种皮疹不是原发疹(3.0分) A.水疱
B.脓疱
C.斑疹
D.脓痂
E.结节
D.脓痂
6 8 2006 10. 以下疾病中不属于结缔组织病的是:(3.0分) A. 硬皮病
B. 皮肌炎
C. 红斑狼疮
D. 结节性红斑
D. 结节性红斑
6 8 2006 11. 皮肤划痕试验适用于检测(3.0分) A.湿疹
B.接触性皮炎
C.荨麻疹
D.脂溢性皮炎
E.银屑病
C.荨麻疹
6 8 2006 12. 下列哪项不属于麻疹型药疹的特点(3.0分) A.Koplik斑
B.皮疹对称分布
C.起病突然,发展较快
D.全身中毒症状较轻
E.自觉瘙痒
A.Koplik斑
6 8 2006 13. 下列关于急性荨麻疹的叙述错误的是 (3.0分) A.严重者皮损处可出现橘皮样外观
B.多有瘙痒
C.无皮肤外的任何器官受累
D.起病急
E.抗组胺药治疗有效
C.无皮肤外的任何器官受累
6 8 2006 14. 硬皮病临床上可分为:(3.0分) A. 局限性硬皮病与系统性硬皮病
B. 局限性硬皮病与多发性斑状硬皮病
C. 肢端硬化型与系统硬化型
D. 硬斑病与线状硬皮病
A. 局限性硬皮病与系统性硬皮病
6 8 2006 15. 引起急性荨麻疹的变态反应主要是(3.0分) A.Ⅳ型变态反应
B.Ⅲ型变态反应
C.I型变态反应
D.II型变态反应
E.两种以上变态反应
C.I型变态反应
6 8 2006 16. 皮肤划痕试验适用于检测的疾病是(3.0分) A.脂溢性皮炎
B.接触性皮炎
C.湿疹
D.荨麻疹
E.银屑病
D.荨麻疹
6 8 2006 17. 深在型红斑狼疮指的是:(3.0分) A. 新生儿红斑狼疮
B. 药物性红斑狼疮
C. 红斑狼疮性脂膜炎
D. 亚急性皮肤型红斑狼疮
C. 红斑狼疮性脂膜炎
6 8 2006 18. 特应性皮炎各年龄期皮损特点不包括(3.0分) A.常发生在屈侧
B.婴儿期渗出性皮损多见
C.常发生在伸侧
D.成人期皮疹类似于神经性皮炎
E.皮损可缓解、发作交替
C.常发生在伸侧
6 8 2006 19. 播散性DLE指的是皮损超过(3.0分) A. 头面部
B. 面颊部
C. 躯干上1/3
D. 整个躯干
A. 头面部
6 8 2006 20. 下列关于慢性湿疹描述不正确的是(3.0分) A.往往病因单一,与感染密切相关
B.常由急性演变而来
C.多发于面、手足、四肢及外阴部
D.反复发作、有渗出史
E.男性发病高于女性
A.往往病因单一,与感染密切相关
6 8 2006 21. 血管性水肿特点包括下述各点,除外 (3.0分) A.局限性水肿
B.痒感不明显
C.凹陷性水肿
D.好发于眼睑、包皮
E.遗传性引起是由于血液和组织中C1酯酶抑制物水平低或无活性所致
C.凹陷性水肿
6 8 2006 22. 下列哪项不是激素的作用(3.0分) A.抗炎
B.抗病毒
C.免疫抑制
D.抗休克
E.抗增生
B.抗病毒
6 8 2006 23. 下列关于急性湿疹的描述正确的是(3.0分) A.皮肤苔藓化
B.皮损特点是原发性多形性疹,境界不清
C.发病部位常局限于接触部位
D.常有明显的致病外因
E.皮疹多无症状
B.皮损特点是原发性多形性疹,境界不清
6 8 2006 24. 关于丘疹,下列哪项是错误的(4.0分) A.丘疹病变常位于表皮及真皮浅层
B.丘疹为局限性隆起的皮疹
C.丘疹的直径>1cm
D.丘疹表面可为扁平、乳头状、脐凹
E.丘疹可有多种颜色
C.丘疹的直径>1cm
6 8 2006 25. 皮肌炎最先累及的肌群是(3.0分) A. 腹部肌群
B. 四肢近端肌群
C. 颈部和咽喉肌群
D. 肩胛间肌群
B. 四肢近端肌群
6 8 2006 26. 起病急骤,红斑迅速弥漫全身,尼氏征阳性的药疹多考虑为 (3.0分) A.光变态反应性药疹
B.固定型药疹
C.大疱性表皮松解型药疹
D.剥脱性皮炎型药疹
E.猩红热型药疹
C.大疱性表皮松解型药疹
6 8 2006 27. 治疗SLE的首选药物是:(3.0分) A. 免疫球蛋白静脉滴注
B. 糖皮质激素
C. 免疫抑制剂
D. 非街体抗炎药物
B. 糖皮质激素
6 8 2006 28. 皮肤发挥屏障作用的主要是(3.0分) A.颗粒层
B.棘层
C.基底层
D.透明层
E.角质层
E.角质层
6 8 2006 29. 关于固定型药疹,不正确的是(3.0分) A.皮疹好发于皮肤黏膜交界处
B.皮疹常为1个
C.典型皮疹为圆形或类圆形的水肿性暗紫红色斑疹
D.皮疹愈合后留下灰黑色色素斑,约1周消退
E.黏膜皱褶处糜烂时常感疼痛
D.皮疹愈合后留下灰黑色色素斑,约1周消退
6 8 2006 30. 结缔组织病自身抗体产生的可能原因是:(3.0分) A. 辅助性T细胞功能受损
B. B细胞功能受损
C. 抑制性T细胞功能受损
D. 紫外线诱发DNA抗原改变
C. 抑制性T细胞功能受损
6 8 2006 31. 钱币状湿疹的临床特点不包括(3.0分) A.可有渗出
B.好发于儿童
C.皮疹的形状与钱币相似
D.瘙痒明显
E.好发于四肢
B.好发于儿童
6 8 2006 32. 引起过敏性休克的发病机制主要是哪型变态反应(3.0分) A.Ⅱ型
B.非变态反应
C.Ⅳ型
D.Ⅰ型
E.Ⅲ型
D.Ⅰ型
6 8 2006 33. 局限性硬皮病皮损特点是:(3.0分) A. 肿胀、硬化
B. 肿胀、萎缩
C. 肿胀、硬化、萎缩
D. 硬化
C. 肿胀、硬化、萎缩
6 8 2007 1. 皮肤的颜色与下列哪个因素无关(2.0分) A.种族
B.身高
C.性别
D.营养
E.部位
B.身高
6 8 2007 2. 起病急骤,红斑迅速弥漫全身,尼氏征阳性的药疹多考虑为(2.0分) A.大疱性表皮松解型药疹
B.固定型药疹
C.光变态反应性药疹
D.剥脱性皮炎型药疹
E.猩红热型药疹
A.大疱性表皮松解型药疹
6 8 2007 3. 表皮层细胞主要靠哪种结构彼此连接 (2.0分) A.半桥粒
B.透明层
C.张力细丝
D.细胞间质
E.桥
E.桥
6 8 2007 4. 正常的表皮通过时间为( )天(2.0分) A.2
B.28
C.48
D.56
E.以上都不是
B.28
6 8 2007 5. 冷冻治疗的原理(2.0分) A.机械损伤
B.细胞中毒
C.细胞膜类脂蛋白复合物变性
D.血液淤滞
E.以上都是
C.细胞膜类脂蛋白复合物变性
6 8 2007 6. 表皮的主要细胞是(2.0分) A.黑素细胞
B.角质形成细胞
C.朗格汉斯细胞
D.梅克尔细胞
E.成纤维细胞
B.角质形成细胞
6 8 2007 7. 真皮的主要成分是(2.0分) A.细胞
B.基质
C.纤维
D.血管
E.神经
C.纤维
6 8 2007 8. 皮肤发挥屏障作用的主要是(2.0分) A.基底层
B.棘层
C.颗粒层
D.透明层
E.角质层
E.角质层
6 8 2007 9. 下列哪种皮疹不是继发疹(2.0分) A.风团
B.糜烂
C.苔藓样变
D.瘢痕
E.鳞屑
A.风团
6 8 2007 10. 关于斑块,下列哪项是错误的(2.0分) A.局限性隆起的皮疹
B.皮疹的直径>1cm
C.斑块多由丘疹融合而成
D.斑块可由斑片扩大而成
E.斑块常见于银屑病
D.斑块可由斑片扩大而成
6 8 2007 11. 有关风团的叙述错误的是(2.0分) A.风团是水肿性隆起的损害
B.伴有剧烈瘙痒
C.消退后不留痕迹
D.真皮浅层血管扩张渗出所致
E.风团又称风疹
E.风团又称风疹
6 8 2007 12. 关于结节,下述哪项是错误的(2.0分) A.结节是局限、实性、深在性损害
B.结节由真皮或皮下炎性浸润所致
C.结节可高出皮面
D.结节可不高出皮面
E.结节破溃后不形成溃疡
E.结节破溃后不形成溃疡
6 8 2007 13. 下列哪项不是激素的作用 (2.0分) A.抗炎
B.免疫抑制
C.抗病毒
D.抗休克
E.抗增生
C.抗病毒
6 8 2007 14. 以下哪项不是糖皮质激素的副作用(2.0分) A.钠水储留
B.血糖升高
C.电解质紊乱
D.肝功能损伤
E.精神兴奋
D.肝功能损伤
6 8 2007 15. 下列哪种疾病并非人乳头瘤病毒引起?(2.0分) A.寻常疣
B.尖锐湿疣
C.传染性软疣
D.跖疣
E.扁平疣
C.传染性软疣
6 8 2007 16. 扁平疣经搔抓可沿抓痕呈串珠状排列,称之为 (2.0分) A.皮肤划痕征
B.尼氏征
C.同形反应
D.Auspitz征
E.Gottron征
C.同形反应
6 8 2007 17. 人乳头瘤病菌主要存在于(2.0分) A.角质层
B.透明层
C.棘层
D.基底层
E.真皮浅层
C.棘层
6 8 2007 18. 带状疱疹治疗首选下列哪种药物?(2.0分) A.青霉素
B.氯霉素
C.红霉素
D.阿昔洛韦
E.灰黄霉素
D.阿昔洛韦
6 8 2007 19. 患者女,50岁,右足跖前部出现数个黄豆大淡黄色角质性丘疹3个月,除去表面角质层可见呈白色软刺状,表面有小黑点,有挤压痛最,最可能的诊断是?(2.0分) A.肢端疼痛症
B.胼胝
C.皲裂
D.鸡眼
E.跖疣
E.跖疣
6 8 2007 20. 下列哪种疾病不属于脓皮病?(2.0分) A.臁疮
B.脓癣
C.丹毒
D.蜂窝织炎
E.脓疱疮
B.脓癣
6 8 2007 21. 多个相邻毛囊及毛囊周围炎相互融合形成皮肤深部感染称之为 (2.0分) A.毛囊炎
B.疖
C.痈
D.丹毒
E.蜂窝织炎
C.痈
6 8 2007 22. 大疱性脓疱疮通常由下列哪种病原菌引起?(2.0分) A.链球菌
B.金葡菌
C.支原体
D.人乳头瘤病毒
E.表皮葡萄球菌
B.金葡菌
6 8 2007 23. 关于寻常型脓疱疮下列哪条叙述是错误的?(2.0分) A.致病菌为微细棒状杆菌
B.常伴皮肤瘙痒
C.好发于炎热季节
D.儿童多见
E.外用莫匹罗星软膏治疗有效
A.致病菌为微细棒状杆菌
6 8 2007 24. 下列哪种说法是错误的?(2.0分) A.细菌性毛囊炎系整个毛囊细菌感染发生化脓性炎症
B.秃发性毛囊炎的病原菌主要是金黄色葡萄球菌
C.项部疤痕疙瘩性毛囊炎指病程缓慢,发生于颈项部的愈后形成瘢痕硬结者
D.头部脓肿性穿掘性毛囊炎是一种少见的头顶部慢性非化脓性皮肤病
E.在疤痕形成期的头部脓肿性穿掘性毛囊炎可选用青霉素联合异维A酸口服治疗
D.头部脓肿性穿掘性毛囊炎是一种少见的头顶部慢性非化脓性皮肤病
6 8 2007 25. 关于黄癣,下列哪条叙述是错误的?(2.0分) A.外用克霉唑治疗
B.愈后留有永久性秃发
C.皮损呈碟状
D.皮损通常轻度瘙痒
E.Wood灯下皮损处不显示荧光
E.Wood灯下皮损处不显示荧光
6 8 2007 26. 花斑糠疹的致病菌主要存在于(2.0分) A.角质层
B.颗粒层
C.棘层
D.基底层
E.真皮浅层
A.角质层
6 8 2007 27. Wood灯(滤过紫外线灯)检查白癣病发呈(2.0分) A.不显示荧光
B.显示亮绿色荧光
C.显示暗绿色荧光
D.显示珊瑚红色荧光
E.以上全不对
B.显示亮绿色荧光
6 8 2007 28. 下列哪种菌不属于皮肤癣菌(2.0分) A.红色毛癣菌
B.许兰毛癣菌
C.念珠菌
D.须癣毛癣菌
E.犬小孢子菌
C.念珠菌
6 8 2007 29. 下列哪条是错误的?(2.0分) A.癣菌疹通常真菌检查阳性
B.鹅口疮的致病菌是念珠菌
C.红色毛癣菌是最常见的浅部真菌病的致病菌
D.黄癣的致病菌是许兰毛癣菌
E.白癣愈后不留瘢痕
A.癣菌疹通常真菌检查阳性
6 8 2007 30. 对于浅部真菌病下列哪条是错误的?(2.0分) A.真菌镜检培养具有诊断价值
B.主要累及角质层
C.可侵犯甲板
D.可侵犯毛发
E.主要累及真皮
E.主要累及真皮
6 8 2007 31. DLE的好发部位(2.0分) A.耳廓
B.两颊及鼻背
C.头皮
D.唇
E.手臂
B.两颊及鼻背
6 8 2007 32. ScLE的首选治疗药物是(2.0分) A.免疫抑制剂
B.抗生素
C.抗疟药羟基氯喹
D.维生素
E.非甾体消炎药
C.抗疟药羟基氯喹
6 8 2007 33. 关于DLE,错误的是(2.0分) A.可致局限性永久性脱发
B.晚期可继发鳞状细胞癌
C.好发于男性,一般不会发展成为SLE
D.可伴有雷诺现象、冻疮病史
E.愈合遗留色素减退的萎缩性瘢痕
C.好发于男性,一般不会发展成为SLE
6 8 2007 34. 自身免疫性疾病患者易出现纤维化的脏器是(2.0分) A.心
B.肺
C.胃
D.肝
E.胰
B.肺
6 8 2007 35. 40岁以上的皮肌炎病人要警惕合并哪种疾病的可能性(2.0分) A.糖尿病
B.冠心病
C.高脂血症
D.高血压
E.内脏恶性肿瘤
E.内脏恶性肿瘤
6 8 2007 36. 系统性硬皮病的标志抗体是(2.0分) A.抗Scl-70抗体
B.抗Jo-1抗体
C.抗Sm抗体
D.ANA
E..抗dsDNA抗体
A.抗Scl-70抗体
6 8 2007 37. 皮肌炎中血清肌酶常升高,下列哪项叙述是正确的(2.0分) A.血清肌酶的增减不能反映疾病的活动性
B.血清肌酶一般在肌力改善前5-6周降低
C.血清肌酶升高即可说明皮肌炎的存在
D.CK和ALD特异性升高
E.LDH和CK特异性升高
D.CK和ALD特异性升高
6 8 2007 38. 关于皮肌炎,下列哪项是错误的(2.0分) A.面部特别是眼睑部水肿性紫红斑
B.肌无力
C.与内脏恶性肿瘤有关
D.尿肌酸排出减少
E.肌肉自觉痛和压痛
D.尿肌酸排出减少
6 8 2007 39. 皮肌炎的特异性抗体是: (2.0分) A.ANA
B.SM抗体
C.Scl-70抗体
D.抗JO-1抗体
E.抗着丝点抗体
D.抗JO-1抗体
6 8 2007 40. 红斑狼疮的皮肤组织直接免疫荧光检查常可见补体沉积在(2.0分) A.表皮细胞内
B.表皮细胞间隙
C.表皮真皮交界处
D.真皮网状层
E.真皮乳头层
C.表皮真皮交界处
6 8 2007 41. 皮肌炎患者血清中多种肌酶值:(2.0分) A.升高
B.降低
C.正常
D.无临床意义
E.与疾病活动性无关
A.升高
6 8 2007 42. 皮肤异色症样皮疹的表现不包括(2.0分) A.毛细血管扩张
B.色素脱失
C.水疱
D.色素增加
E.萎缩
C.水疱
6 8 2007 43. 硬皮病临床上可分为: (2.0分) A.肢端硬化型与系统硬化型
B.局限性硬皮病与多发性斑状硬皮病
C.局限性硬皮病与系统性硬皮病
D.硬斑病与线状硬皮病
E.弥漫性硬皮病和硬化症
C.局限性硬皮病与系统性硬皮病
6 8 2007 44. Morphea occurs usually in (2.0分) A.Forehead
B.trunk
C.limbs
D.hand
E.lip
B.trunk
6 8 2007 45. 150mg prednisone is equivalent to methylprednisolone( )mg (2.0分) A.120
B.180
C.100
D.187.5
E.150
A.120
6 8 2007 46. Which of the following are the risk factors of contact dermatitis caused by primary stimuli?(2.0分) A.Is the first contact or contacts again
B.Whether the contacts are antigenic
C.The Irritant or toxic property of the contacts
D.Whether the body is allergic to the material
E.The resistance of the body
E.The resistance of the body
6 8 2007 47. 下列有关湿疹的治疗叙述错误的是(2.0分) A.内服药的目的主要是抗炎止痒
B.合并感染者,可加用抗生素
C.慢性湿疹迁延不愈者,需口服糖皮质激素
D.根据皮疹形态特点,选用适当的剂型和药物
E.消除体内慢性病灶及其他全身性疾病
C.慢性湿疹迁延不愈者,需口服糖皮质激素
6 8 2007 48. Which of the following diseases are not suitable for external use of glucocorticoid treatment?(2.0分) A.Coin-like eczema
B.fixed drug eruption
C.atopic dermatitis
D.tinea corporis and tinea cruris
E.psoriasis
D.tinea corporis and tinea cruris
6 8 2007 49. 下列哪项不是接触性皮炎的临床特点?(2.0分) A.接触致敏物后发病
B.皮疹单一形态,境界清楚
C.好发于暴露部位,尤其接触部位
D.起病迅速,无潜伏期
E.可有系统症状发生
D.起病迅速,无潜伏期
6 8 2007 50. The features of photoallergic reaction of drug eruption does not include (2.0分) A.occurring in a small number of people
B.having a definite stage of latency
C.only occurring at the light exposure portion
D.the skin eruption appears as eczema
E.with a long course of disease
D.the skin eruption appears as eczema
6 8 2008 1. 关于传染性软疣下列哪条是错的?(2.0分) A.由HPV病毒感染引起,且多见于老年人
B.多见于儿童及青少年
C.呈脐凹状丘疹
D.由痘病毒引起
E.可挤出白色乳酪样物质
A.由HPV病毒感染引起,且多见于老年人
6 8 2008 2. 扁平疣经搔抓后可沿抓痕出现新发条状排列的皮损,称之为(2.0分) A.皮肤划痕征
B.Auspitz征
C.尼氏征
D.Gottron征
E.同形反应
E.同形反应
6 8 2008 3. 关于跖疣,下列哪条是错的?(2.0分) A.是由痘病毒引起
B.皮疹可以一个或多个
C.皮疹表面粗糙不平,质地硬
D.挤压时明显疼痛
E.病程慢性,可自然消退
A.是由痘病毒引起
6 8 2008 4. 带状疱疹治疗首选下列哪种药物?(2.0分) A.青霉素
B.阿昔洛韦
C.红霉素
D.氯霉素
E.灰黄霉素
B.阿昔洛韦
6 8 2008 5. 患者男,30岁,运动员,右足跖前部出现数个黄豆大淡黄色角质性丘疹3个月,除去表面角质层可见呈白色软刺状,表面有小黑点,有挤压痛,最可能的诊断是?(2.0分) A.肢端疼痛症
B.胼胝
C.皲裂
D.跖疣
E.鸡眼
D.跖疣
6 8 2008 6. 下列哪种疾病不属于脓皮病?(2.0分) A.脓癣
B.臁疮
C.丹毒
D.蜂窝织炎
E.脓疱疮
A.脓癣
6 8 2008 7. 由凝固酶阳性金葡菌引起多个相邻毛囊及毛囊周围化脓性炎症称之为 (2.0分) A.毛囊炎
B.疖
C.痈
D.丹毒
E.蜂窝织炎
B.疖
6 8 2008 8. 大疱性脓疱疮通常由下列哪种病原菌引起?(2.0分) A.链球菌
B.金葡菌
C.支原体
D.人乳头瘤病毒
E.表皮葡萄球菌
B.金葡菌
6 8 2008 9. 关于寻常型脓疱疮下列哪条叙述是错误的?(2.0分) A.常继发于瘙痒性皮肤病
B.致病菌为微细棒状杆菌
C.好发于炎热季节
D.儿童多见
E.外用莫匹罗星软膏治疗有效
B.致病菌为微细棒状杆菌
6 8 2008 10. 下列哪种说法是错误的?(2.0分) A.细菌性毛囊炎系整个毛囊细菌感染发生化脓性炎症
B.秃发性毛囊炎的病原菌主要是金黄色葡萄球菌
C.项部疤痕疙瘩性毛囊炎指病程缓慢,发生于颈项部的愈后形成瘢痕硬结者
D.头部脓肿性穿掘性毛囊炎是一种少见的头顶部慢性非化脓性皮肤病
E.在疤痕形成期的头部脓肿性穿掘性毛囊炎可选用青霉素联合异维A酸口服治疗
D.头部脓肿性穿掘性毛囊炎是一种少见的头顶部慢性非化脓性皮肤病
6 8 2008 11. 关于黄癣,下列哪条叙述是错误的?(2.0分) A.外用克霉唑治疗
B.愈后留有永久性秃发
C.皮损呈碟状
D.Wood灯下皮损处不显示荧光
E.皮损通常轻度瘙痒
D.Wood灯下皮损处不显示荧光
6 8 2008 12. 花斑糠疹的致病菌主要存在于(2.0分) A.透明层
B.角质层
C.棘层
D.基底层
E.真皮浅层
B.角质层
6 8 2008 13. Wood灯(滤过紫外线灯)检查白癣病发呈(2.0分) A.不显示荧光
B.显示亮绿色荧光
C.显示暗绿色荧光
D.显示珊瑚红色荧光
E.以上全不对
B.显示亮绿色荧光
6 8 2008 14. 下列哪种菌不属于皮肤癣菌(2.0分) A.念珠菌
B.红色毛癣菌
C.许兰毛癣菌
D.须癣毛癣菌
E.犬小孢子菌
A.念珠菌
6 8 2008 15. 下列哪条是错误的?(2.0分) A.癣菌疹通常真菌检查阳性
B.黄癣的致病菌是许兰毛癣菌
C.红色毛癣菌是最常见的浅部真菌病的致病菌
D.鹅口疮的致病菌是念珠菌
E.白癣愈后不留瘢痕
A.癣菌疹通常真菌检查阳性
6 8 2008 16. 下列关于急性湿疹的描述正确的是(2.0分) A.常有明显的致病外因
B.皮损特点是原发性多形性疹,境界不清
C.发病部位常局限于接触部位
D.皮肤苔藓化
E.皮疹多无症状
B.皮损特点是原发性多形性疹,境界不清
6 8 2008 17. 引起急性荨麻疹的变态反应主要是(2.0分) A.I型变态反应
B.II型变态反应
C.Ⅲ型变态反应
D.Ⅳ型变态反应
E.两种以上变态反应
A.I型变态反应
6 8 2008 18. 关于药疹下列叙述不正确的是(2.0分) A.有明确的服药史
B.有一定的潜伏期
C.皮疹可广泛发生
D.瘙痒明显
E.不发生于粘膜
E.不发生于粘膜
6 8 2008 19. 患者,男性,因反复出现左手腕部皮疹伴痒数周就诊,检查时发现左腕带表处有条带状分布的红斑丘疹,表面有细小鳞屑,皮损单一,边界较清。本患者最有可能的诊断是(2.0分) A.体癣
B.湿疹
C.神经性皮炎
D.接触性皮炎
E.荨麻疹
D.接触性皮炎
6 8 2008 20. 第7题病例患者的外用药治疗的注意事项是(2.0分) A.用皮质类固醇制剂
B.用水疗
C.选用温和、有保护作用的药物和制剂
D.无论有无皮疹,须全身使用
E.每个病人需用抗生素外用药
C.选用温和、有保护作用的药物和制剂
6 8 2008 21. 皮肤划痕试验适用于检测的疾病是(2.0分) A.湿疹
B.接触性皮炎
C.脂溢性皮炎
D.荨麻疹
E.银屑病
D.荨麻疹
6 8 2008 22. 下列哪项不是变应性接触性皮炎的临床特点(2.0分) A.接触致敏物后发病
B.皮疹单一形态,境界清楚
C.好发于暴露部位,尤其接触部位
D.起病迅速,无潜伏期
E.严重者可有系统症状发生
D.起病迅速,无潜伏期
6 8 2008 23. 关于接触性皮炎与急性湿疹的鉴别,下列哪项是错误的(2.0分) A.皮疹是否境界清楚
B.皮疹是否对称发生
C.皮疹是否单一形态
D.皮疹是否瘙痒
E.病因是否明确
D.皮疹是否瘙痒
6 8 2008 24. 引起过敏性休克的发病机制主要是哪型变态反应(2.0分) A.Ⅱ型
B.非变态反应
C.Ⅰ型
D.Ⅳ型
E.Ⅲ型
C.Ⅰ型
6 8 2008 25. 药疹初次发病的潜伏期一般为(2.0分) A.1~5天
B.4~20天
C.3~7天
D.7~8天
E.4~10天
B.4~20天
6 8 2008 26. 关于变态反应性药疹的特点,下列哪项叙述错误(2.0分) A.只发生于少数对药物过敏者
B.病情轻重与药物的剂量相关
C.一种药物可引起多种形态的皮疹,不同药物可引起同一种皮疹
D.初次用药一般需4~20天后才出现药疹
E.病情轻重与药物的药理、毒理作用无相关性
B.病情轻重与药物的剂量相关
6 8 2008 27. 下列哪项不属于麻疹型药疹的特点(2.0分) A.Koplik斑
B.皮疹对称分布
C.全身中毒症状较轻
D.起病突然,发展较快
E.自觉瘙痒
A.Koplik斑
6 8 2008 28. 下列哪种药物最易引起痤疮样药疹(2.0分) A.青霉素类
B.头孢菌素类
C.扑热息痛
D.糖皮质激素
E.苯巴比妥
D.糖皮质激素
6 8 2008 29. 下列关于急性荨麻疹的叙述错误的是(2.0分) A.起病急
B.多有瘙痒
C.无皮肤外的任何器官受累
D.严重者皮损处可出现橘皮样外观
E.抗组胺药治疗有效
C.无皮肤外的任何器官受累
6 8 2008 30. 钱币状湿疹的临床特点不包括(2.0分) A.瘙痒明显
B.皮疹的形状与钱币相似
C.可有渗出
D.好发于儿童
E.好发于四肢
D.好发于儿童
6 8 2008 31. 头部不适宜:用(2.0分) A.溶液
B.洗剂
C.乳剂
D.酊剂
E.乳剂
B.洗剂
6 8 2008 32. 表皮主要是由角质形成细胞和( )细胞组成(2.0分) A.树枝状
B. 扁平状
C.立方形
D. 圆柱形
E. 以上都是
A.树枝状
6 8 2008 33. 腋臭是由于哪种腺体在青春期分泌的乳状液经细菌分解后产生臭味引起的 (2.0分) A.小汗腺
B.顶泌汗腺
C.皮脂腺
D.乳腺
E.唾液腺
B.顶泌汗腺
6 8 2008 34. 下列哪种皮疹不是继发疹(2.0分) A.风团
B.糜烂
C.苔藓样变
D.瘢痕
E.鳞屑
A.风团
6 8 2008 35. 关于斑块,下列哪项是错误的(2.0分) A.局限性隆起的皮疹
B.皮疹的直径>1cm
C.斑块多由丘疹融合而成
D.斑块可由斑片扩大而成
E.斑块常见于银屑病
D.斑块可由斑片扩大而成
6 8 2008 36. 有关风团的叙述错误的是(2.0分) A.风团是水肿性隆起的损害
B.伴有剧烈瘙痒
C.消退后不留痕迹
D.真皮浅层血管扩张渗出所致
E.风团又称风疹
E.风团又称风疹
6 8 2008 37. 关于结节,下述哪项是错误的(2.0分) A.结节是局限、实性、深在性损害
B.结节由真皮或皮下炎性浸润所致
C.结节可高出皮面
D.结节可不高出皮面
E.结节破溃后不形成溃疡
E.结节破溃后不形成溃疡
6 8 2008 38. 下列哪项不是激素的作用(2.0分) A.抗炎
B.免疫抑制
C.抗病毒
D.抗休克
E.抗增生
C.抗病毒
6 8 2008 39. 以下哪项不是糖皮质激素的副作用(2.0分) A.钠水储留
B.血糖升高
C.电解质紊乱
D.肝功能损伤
E.精神兴奋
D.肝功能损伤
6 8 2008 40. Which of the following are the risk factors of contact dermatitis caused by primary stimuli?(2.0分) A.Is the first contact or contacts again
B.Whether the contacts are antigenic
C.The Irritant or toxic property of the contacts
D.Whether the body is allergic to the material
E.he resistance of the body
E.he resistance of the body
6 8 2008 41. 下列有关湿疹的治疗叙述错误的是(2.0分) A.内服药的目的主要是抗炎止痒
B.合并感染者,可加用抗生素
C.慢性湿疹迁延不愈者,需口服糖皮质激素
D.根据皮疹形态特点,选用适当的剂型和药物
E.消除体内慢性病灶及其他全身性疾病
C.慢性湿疹迁延不愈者,需口服糖皮质激素
6 8 2008 42. Which of the following diseases are not suitable for external use of glucocorticoid treatment?(2.0分) A.Coin-like eczema
B.fixed drug eruption
C.atopic dermatitis
D.tineacorporis and tineacruris
E.psoriasis
D.tineacorporis and tineacruris
6 8 2008 43. 下列哪项不是接触性皮炎的临床特点?(2.0分) A.接触致敏物后发病
B.皮疹单一形态,境界清楚
C.好发于暴露部位,尤其接触部位
D.起病迅速,无潜伏期
E.可有系统症状发生
D.起病迅速,无潜伏期
6 8 2008 44. The features of photoallergic reaction of drug eruption does not include(2.0分) A.occurring in a small number of people
B.having a definite stage of latency
C.only occurring at the light exposure portion
D.he skin eruption appears as eczema
E.with a long course of disease
D.he skin eruption appears as eczema
6 8 2008 45. 药疹的治疗哪项是错误的(2.0分) A.可用钙剂
B.给抗过敏药
C.所有药疹都必须内用糖皮质激素
D.只有重症药疹才能内用糖皮质激素
E.可给维生素
C.所有药疹都必须内用糖皮质激素
6 8 2008 46. 变态反应性接触性皮炎的说法哪项不正确(2.0分) A.刺激物本身无毒性
B.大多数接触后不致病
C.刺激物多具有抗原性
D.皮炎的轻重与个体易感性有关
E.刺激物分子量多较低
C.刺激物多具有抗原性
6 8 2008 47. 以下哪组是错误的(2.0分) A.急性皮炎伴大量渗液--冷湿敷
B.急性或亚急性皮炎无渗液--粉剂
C.急性皮炎--软膏
D.慢性局限性浸润肥厚性皮肤病--硬膏
E.瘙痒病--酊剂或醑剂
C.急性皮炎--软膏
6 8 2008 48. 下列哪项与色素性荨麻疹关系密切(2.0分) A.嗜碱性粒细胞
B.肥大细胞
C.食物过敏
D.寒冷性刺激
E.淋巴细胞
B.肥大细胞
6 8 2008 49. 乳房湿疹的特点是(2.0分) A.多见于老年人
B.皮损呈棕红色
C.以苔藓化的皮损为主
D.瘙痒不明显
E.停止哺乳后皮损加重
B.皮损呈棕红色
6 8 2008 50. 粉剂适用于以下哪种皮肤病(2.0分) A.脓疱病
B.脓癣
C.寻常性银屑病
D.无渗出的褶烂
E.有渗出的急性皮炎
D.无渗出的褶烂
6 8 2009 1. 下列关于急性湿疹的描述正确的是(2.0分) A.常有明显的致病外因
B.皮损特点是原发性多形性疹,境界不清
C.发病部位常局限于接触部位
D.皮肤苔藓化
E.皮疹多无症状
B.皮损特点是原发性多形性疹,境界不清
6 8 2009 2. 下列关于慢性湿疹描述不正确的是(2.0分) A.多发于面、手足、四肢及外阴部
B.反复发作、有渗出史
C.常由急性演变而来
D.往往病因单一,与感染密切相关
E.男性发病高于女性
D.往往病因单一,与感染密切相关
6 8 2009 3. 引起急性荨麻疹的变态反应主要是(2.0分) A.I型变态反应
B.II型变态反应
C.Ⅲ型变态反应
D.Ⅳ型变态反应
E.两种以上变态反应
A.I型变态反应
6 8 2009 4. 下列不属于重症药疹的是(2.0分) A.猩红热样药疹
B.重症多形红斑型药疹
C.剥脱性皮炎型药疹
D.大疱性表皮松解型药疹
E.药物超敏反应综合征
A.猩红热样药疹
6 8 2009 5. 关于药疹下列叙述不正确的是(2.0分) A.有明确的服药史
B.有一定的潜伏期
C.皮疹可广泛发生
D.瘙痒明显
E.不发生于粘膜
E.不发生于粘膜
6 8 2009 6. 治疗药疹的首要措施是(2.0分) A.使用足量的糖皮质激素
B.使用抗生素治疗
C.抗过敏治疗
D.停用一切可疑的致敏药物
E.尽早使用外用药
D.停用一切可疑的致敏药物
6 8 2009 7. 下列哪项不是变应性接触性皮炎的临床特点(2.0分) A.接触致敏物后发病
B.皮疹单一形态,境界清楚
C.好发于暴露部位,尤其接触部位
D.起病迅速,无潜伏期
E.严重者可有系统症状发生
D.起病迅速,无潜伏期
6 8 2009 8. 下列哪项不属于变应性接触性皮炎(2.0分) A.尿布皮炎
B.外用药使用后局部的皮炎
C.化妆品皮炎
D.滴眼药水引起的过敏
E.栓剂使用后的过敏
E.栓剂使用后的过敏
6 8 2009 9. 关于接触性皮炎与急性湿疹的鉴别,下列哪项是错误的(2.0分) A.皮疹是否境界清楚
B.皮疹是否对称发生
C.皮疹是否单一形态
D.皮疹是否瘙痒
E.病因是否明确
D.皮疹是否瘙痒
6 8 2009 10. 下列哪项不属于麻疹型药疹的特点(2.0分) A.Koplik斑
B.皮疹对称分布
C.全身中毒症状较轻
D.起病突然,发展较快
E.自觉瘙痒
A.Koplik斑
6 8 2009 11. 大疱性表皮松解型药疹的临床特点不包括(2.0分) A.松弛性水疱或大疱
B.尼氏征阴性
C.大面积表皮松解
D.大量渗出
E.广泛黏膜受累
B.尼氏征阴性
6 8 2009 12. 下列哪种药物最易引起痤疮样药疹 (2.0分) A.青霉素类
B.头孢菌素类
C.扑热息痛
D.糖皮质激素
E.苯巴比妥
D.糖皮质激素
6 8 2009 13. 原发性刺激性接触性皮炎的发生是由于 (2.0分) A.IV型变态反应
B.I型变态反应
C.暴露于化学物质
D.Th细胞的激活
E.固定性药疹
C.暴露于化学物质
6 8 2009 14. 下列关于急性荨麻疹的叙述错误的是 (2.0分) A.起病急
B.多有瘙痒
C.无皮肤外的任何器官受累
D.严重者皮损处可出现橘皮样外观
E.抗组胺药治疗有效
C.无皮肤外的任何器官受累
6 8 2009 15. 异位性素质(atopy)的含义不包括(2.0分) A.常对异种蛋白过敏
B.血清中IgE增高
C.外周血嗜酸粒细胞增多
D.常有易患哮喘、过敏性鼻炎、湿疹的家族性倾向
E.自身免疫病倾向
E.自身免疫病倾向
6 8 2009 16. 钱币状湿疹的临床特点不包括(2.0分) A.瘙痒明显
B.皮疹的形状与钱币相似
C.可有渗出
D.好发于儿童
E.好发于四肢
D.好发于儿童
6 8 2009 17. 关于传染性软疣下列哪条是错的?(2.0分) A.由HPV病毒感染引起,且多见于老年人
B.多见于儿童及青少年
C.呈脐凹状丘疹
D.由痘病毒引起
E.可挤出白色乳酪样物质
A.由HPV病毒感染引起,且多见于老年人
6 8 2009 18. 下列哪种疾病的病原菌并非HPV病毒引起?(2.0分) A.寻常疣
B.尖锐湿疣
C.跖疣
D.传染性软疣
E.扁平疣
D.传染性软疣
6 8 2009 19. 扁平疣经搔抓后可沿抓痕出现新发条状排列的皮损,称之为 (2.0分) A.皮肤划痕征
B.Auspitz征
C.尼氏征
D.Gottron征
E.同形反应
E.同形反应
6 8 2009 20. 下列哪项不是传染性软疣的好发部位?(2.0分) A.面部
B.手背
C.头皮
D.躯干
E.外生殖器
C.头皮
6 8 2009 21. 带状疱疹治疗首选下列哪种药物?(2.0分) A.青霉素
B.阿昔洛韦
C.红霉素
D.氯霉素
E.灰黄霉素
B.阿昔洛韦
6 8 2009 22. 患者男,30岁,运动员,右足跖前部出现数个黄豆大淡黄色角质性丘疹3个月,除去表面角质层可见呈白色软刺状,表面有小黑点,有挤压痛,最可能的诊断是?(2.0分) A.肢端疼痛症
B.胼胝
C.皲裂
D.跖疣
E.鸡眼
D.跖疣
6 8 2009 23. 由凝固酶阳性金葡菌引起多个相邻毛囊及毛囊周围化脓性炎症称之为 (2.0分) A.毛囊炎
B.疖
C.痈
D.丹毒
E.蜂窝织炎
B.疖
6 8 2009 24. 葡萄球菌烫伤样皮肤综合征的治疗应首选 (2.0分) A.红霉素
B.头孢类
C.喹诺酮类
D.氯霉素
E.链霉素
B.头孢类
6 8 2009 25. 大疱性脓疱疮通常由下列哪种病原菌引起? (2.0分) A.链球菌
B.金葡菌
C.支原体
D.人乳头瘤病毒
E.表皮葡萄球菌
B.金葡菌
6 8 2009 26. 患者男,50岁。有2型糖尿病病史多年,血糖控制欠佳。近半月后颈部出现两个结节并迅速增大,疼痛明显。3天前出现发热,最高体温38.8度。伴畏寒、乏力。查体:颈部颌下触及多个肿大淋巴结,触痛明显。颈部两处暗红色斑块,直径分别为7cm及5cm,已自行破溃,有大量脓血性分泌物流出。最可能的引起该患者皮肤疾病的病原体是 (2.0分) A.金黄色葡萄球菌
B.溶血性链球菌
C.非结核分枝杆菌
D.巴尔通体
E.产气荚膜杆菌
A.金黄色葡萄球菌
6 8 2009 27. 关于花斑糠疹,下列哪条是正确的?(2.0分) A.剧烈瘙痒
B.皮损为红斑、斑块
C.多发于手足
D.冬季易发
E.愈后可留有色素减退斑
E.愈后可留有色素减退斑
6 8 2009 28. 花斑糠疹的致病菌主要存在于(2.0分) A.透明层
B.角质层
C.棘层
D.基底层
E.真皮浅层
B.角质层
6 8 2009 29. 下列哪种菌不属于皮肤癣菌(2.0分) A.念珠菌
B.红色毛癣菌
C.许兰毛癣菌
D.须癣毛癣菌
E.犬小孢子菌
A.念珠菌
6 8 2009 30. 手足癣不应选择下列哪种药物?(2.0分) A.克霉唑乳膏
B.红霉素软膏
C.联苯苄唑乳膏
D.酮康唑乳膏
E.特比萘芬软膏
B.红霉素软膏
6 8 2009 31. 下列哪条是错误的?(2.0分) A.癣菌疹通常真菌检查阳性
B.黄癣的致病菌是许兰毛癣菌
C.红色毛癣菌是最常见的浅部真菌病的致病菌
D.鹅口疮的致病菌是念珠菌
E.白癣愈后不留瘢痕
A.癣菌疹通常真菌检查阳性
6 8 2009 32. 对于浅部真菌病下列哪条是错误的?(2.0分) A.真菌镜检培养具有诊断价值
B.主要累及角质层
C.可侵犯甲板
D.主要累及真皮
E.可侵犯毛发
D.主要累及真皮
6 8 2009 33. Which of the following are the risk factors of contact dermatitis caused by primary stimuli?(2.0分) A.Is the first contact or contacts again
B. Whether the contacts are antigenic
C. The Irritant or toxic property of the contacts
D.Whether the body is allergic to the material
E.The resistance of the body
E.The resistance of the body
6 8 2009 34. 下列有关湿疹的治疗叙述错误的是(2.0分) A.内服药的目的主要是抗炎止痒
B.合并感染者,可加用抗生素
C.慢性湿疹迁延不愈者,需口服糖皮质激素
D.根据皮疹形态特点,选用适当的剂型和药物
E.消除体内慢性病灶及其他全身性疾病
C.慢性湿疹迁延不愈者,需口服糖皮质激素
6 8 2009 35. 下列哪项不是接触性皮炎的临床特点?(2.0分) A.接触致敏物后发病
B.皮疹单一形态,境界清楚
C.好发于暴露部位,尤其接触部位
D.起病迅速,无潜伏期
E.可有系统症状发生
D.起病迅速,无潜伏期
6 8 2009 36. In the condition of acute dermatitis representing erythema,papule, papulovesicle, with exudation,which one of the following is suitable (2.0分) A.tincture
B.plaster
C.solution
D.spiritus
E.ointment
C.solution
6 8 2009 37. 变态反应性接触性皮炎的说法哪项不正确(2.0分) A.刺激物本身无毒性
B.大多数接触后不致病
C.刺激物多具有抗原性
D.皮炎的轻重与个体易感性有关
E.刺激物分子量多较低
C.刺激物多具有抗原性
6 8 2009 38. 一妇女染发几小时后,面部出现红斑,肿胀明显,应考虑(2.0分) A.变态反应性接触性皮炎
B.原发刺激性接触性皮炎
C.过敏
D.特应性皮炎
E.急性湿疹
A.变态反应性接触性皮炎
6 8 2009 39. 以下哪组是错误的(2.0分) A.急性皮炎伴大量渗液--冷湿敷
B.急性或亚急性皮炎无渗液--粉剂
C.急性皮炎--软膏
D.慢性局限性浸润肥厚性皮肤病--硬膏
E.瘙痒病--酊剂或醑剂
C.急性皮炎--软膏
6 8 2009 40. 原发刺激性接触性皮炎的特点哪项不正确(2.0分) A.刺激物本身有强烈的刺激性
B.皮炎的轻重与刺激物的性质有关
C.弱刺激物引起慢性皮炎
D.与年龄无关
E.累积性原发刺激性皮炎见于家庭妇女
D.与年龄无关
6 8 2009 41. 湿疹的水疱位于(2.0分) A.表皮内
B.表皮下
C.真皮内
D.真皮下
E.均有
A.表皮内
6 8 2009 42. 湿敷适用于下列哪种情况 (2.0分) A.慢性皮炎时
B.急性湿疹伴大量渗出时
C.红斑丘疹时
D.无皮疹仅瘙痒时
E.荨麻疹的风团
B.急性湿疹伴大量渗出时
6 8 2009 43. 腋臭是由于哪种腺体在青春期分泌的乳状液经细菌分解后产生臭味引起的 (2.0分) A.小汗腺
B.顶泌汗腺
C.皮脂腺
D.乳腺
E.唾液腺
B.顶泌汗腺
6 8 2009 44. 表皮的主要细胞是(2.0分) A.黑素细胞
B.角质形成细胞
C.朗格汉斯细胞
D.梅克尔细胞
E.成纤维细胞
B.角质形成细胞
6 8 2009 45. 关于斑块,下列哪项是错误的 (2.0分) A.局限性隆起的皮疹
B.皮疹的直径>1cm
C.斑块多由丘疹融合而成
D.斑块可由斑片扩大而成
E.斑块常见于银屑病
D.斑块可由斑片扩大而成
6 8 2009 46. 一般认为皮肤大疱的直径大于(2.0分) A.0.5cm
B.1cm
C.3cm
D.2cm
E.1.5cm
B.1cm
6 8 2009 47. 有关风团的叙述错误的是(2.0分) A.风团是水肿性隆起的损害
B.伴有剧烈瘙痒
C.消退后不留痕迹
D.真皮浅层血管扩张渗出所致
E.风团又称风疹
E.风团又称风疹
6 8 2009 48. 关于丘疹,下列哪项是错误的(2.0分) A.丘疹为局限性隆起的皮疹
B.丘疹病变常位于表皮及真皮浅层
C.丘疹的直径>1cm
D.丘疹表面可为扁平、乳头状、脐凹
E.丘疹可有多种颜色
C.丘疹的直径>1cm
6 8 2009 49. 以下哪项不是糖皮质激素的副作用(2.0分) A.钠水储留
B.血糖升高
C.电解质紊乱
D.肝功能损伤
E.精神兴奋
D.肝功能损伤
6 8 2009 50. 粉剂适用于以下哪种皮肤病(2.0分) A.脓疱病
B.脓癣
C.寻常性银屑病
D.无渗出的褶烂
E.有渗出的急性皮炎
D.无渗出的褶烂
6 8 2010 1. 皮肤的颜色与下列哪个因素无关(2.5分) A.种族
B.身高
C.性别
D.营养
E.部位
B.身高
6 8 2010 2. 下列属于第一代抗组胺药的是(2.5分) A.阿司咪唑
B.特非那定
C.塞庚定
D.氯雷他定
E.西替利嗪
C.塞庚定
6 8 2010 3. 湿敷适用于下列哪种情况(2.5分) A.慢性皮炎时
B.急性湿疹伴大量渗出时
C.红斑丘疹时
D.无皮疹仅瘙痒时
E.荨麻疹的风团
B.急性湿疹伴大量渗出时
6 8 2010 4. 表皮的主要细胞是(2.5分) A.黑素细胞
B.角质形成细胞
C.朗格汉斯细胞
D.梅克尔细胞
E.成纤维细胞
B.角质形成细胞
6 8 2010 5. 下列哪种皮疹不是原发疹 (2.5分) A.脓疱
B.水疱
C.斑疹
D.脓痂
E.结节
D.脓痂
6 8 2010 6. 关于斑块,下列哪项是错误的(2.5分) A.局限性隆起的皮疹
B.皮疹的直径>1cm
C.斑块多由丘疹融合而成
D.斑块可由斑片扩大而成
E.斑块常见于银屑病
D.斑块可由斑片扩大而成
6 8 2010 7. 一般认为皮肤大疱的直径大于(2.5分) A.0.5cm
B.1cm
C.3cm
D.2cm
E.1.5cm
B.1cm
6 8 2010 8. 皮肤划痕试验适用于检测(2.5分) A.湿疹
B.接触性皮炎
C.脂溢性皮炎
D.荨麻疹
E.银屑病
D.荨麻疹
6 8 2010 9. 下列哪项不是激素的作用(2.5分) A.抗炎
B.免疫抑制
C.抗病毒
D.抗休克
E.抗增生
C.抗病毒
6 8 2010 10. 抗组胺药的作用机理(2.5分) A.与组胺竞争受体
B.直接破坏细胞壁
C.诱导细胞产生抗炎性介质的物质
D.直接抗炎性介质
A.与组胺竞争受体
6 8 2010 11. Gottron's papules or sign is seen in (2.5分) A.Dermatomyositis
B.DLE
C.SCLE
D.Scleroderma
E.Psoriasis
A.Dermatomyositis
6 8 2010 12. Morphea occurs usually in ____(2.5分) A.Forehead
B.Stemum
C.Limbs
D.Back
E.Ear
A.Forehead
6 8 2010 13. Which one is most valuable for the diagnosis of dermatomyositis?(2.5分) A.ANA
B.dsDNA
C.lupus cell
D.CK
E.none of the above
D.CK
6 8 2010 14. Acantholytic vesicles are formed due to (2.5分) A.the death of epidermal cells
B.the rupture of epidermal cells
C.the impairment of hemidesmosomes
D.the breakdown of desmosomes
E.all of the above
D.the breakdown of desmosomes
6 8 2010 15. Which of the following is not the clinical characteristic of bullous pemphigoid?(2.5分) A.usually in elderly
B.often accompanied by mucosal involvement
C.favourable prognosis
D.responsive to corticosteroids
E.may be complicated by malignant tumors
B.often accompanied by mucosal involvement
6 8 2010 16. 下列关于急性湿疹的描述正确的是(2.5分) A.常有明显的致病外因
B.皮损特点是原发性多形性疹,境界不清
C.发病部位常局限于接触部位
D.皮肤苔藓化
E.皮疹多无症状
B.皮损特点是原发性多形性疹,境界不清
6 8 2010 17. 引起急性荨麻疹的变态反应主要是(2.5分) A.I型变态反应
B.II型变态反应
C.Ⅲ型变态反应
D.Ⅳ型变态反应
E.两种以上变态反应
A.I型变态反应
6 8 2010 18. 关于药疹下列叙述不正确的是(2.5分) A.有明确的服药史
B.有一定的潜伏期
C.皮疹可广泛发生
D.瘙痒明显
E.不发生于粘膜
E.不发生于粘膜
6 8 2010 19. 患者,男性,因反复出现左手腕部皮疹伴痒数周就诊,检查时发现左腕带表处有条带状分布的红斑丘疹,表面有细小鳞屑,皮损单一,边界较清。本患者最有可能的诊断是 (2.5分) A.体癣
B.湿疹
C.神经性皮炎
D.接触性皮炎
E.荨麻疹
D.接触性皮炎
6 8 2010 20. 对第19题病例的诊断最简单可靠的检测方法是(2.5分) A.点刺试验
B.皮肤划痕试验
C.皮内试验
D.斑贴试验
E.血液特异性IgE检测
D.斑贴试验
6 8 2010 21. 第19题病例患者的外用药治疗的注意事项是(2.5分) A.用皮质类固醇制剂
B.用水疗
C.选用温和、有保护作用的药物和制剂
D.无论有无皮疹,须全身使用
E.每个病人需用抗生素外用药
C.选用温和、有保护作用的药物和制剂
6 8 2010 22. 第19题病例所诊断疾病的主要可能发病机制是(2.5分) A.I型变态反应
B.II型变态反应
C.III型变态反应
D.Ⅳ型变态反应
E.两种以上变态反应
D.Ⅳ型变态反应
6 8 2010 23. 下列哪项不是变应性接触性皮炎的临床特点(2.5分) A.接触致敏物后发病
B.皮疹单一形态,境界清楚
C.好发于暴露部位,尤其接触部位
D.起病迅速,无潜伏期
E.严重者可有系统症状发生
D.起病迅速,无潜伏期
6 8 2010 24. 关于接触性皮炎与急性湿疹的鉴别,下列哪项是错误的(2.5分) A.皮疹是否境界清楚
B.皮疹是否对称发生
C.皮疹是否单一形态
D.皮疹是否瘙痒
E.病因是否明确
D.皮疹是否瘙痒
6 8 2010 25. 引起过敏性休克的发病机制主要是哪型变态反应(2.5分) A.Ⅱ型
B.非变态反应
C.Ⅰ型
D.Ⅳ型
E.Ⅲ型
C.Ⅰ型
6 8 2010 26. 药疹初次发病的潜伏期一般为(2.5分) A.1~5天
B.4~20天
C.3~7天
D.7~8天
E.4~10天
B.4~20天
6 8 2010 27. 关于固定型药疹,不正确的是(2.5分) A.皮疹愈合后留下灰黑色色素斑,约1周消退
B.皮疹好发于皮肤黏膜交界处
C.皮疹常为1个
D.典型皮疹为圆形或类圆形的水肿性暗紫红色斑疹
E.黏膜皱褶处糜烂时常感疼痛
A.皮疹愈合后留下灰黑色色素斑,约1周消退
6 8 2010 28. 下列关于急性荨麻疹的叙述错误的是(2.5分) A.起病急
B.多有瘙痒
C.无皮肤外的任何器官受累
D.严重者皮损处可出现橘皮样外观
E.抗组胺药治疗有效
C.无皮肤外的任何器官受累
6 8 2010 29. 异位性素质(即遗传过敏性素质atopy)的含义不包括(2.5分) A.常对异种蛋白过敏
B.血清中IgE增高
C.外周血嗜酸粒细胞增多
D.常有易患哮喘、过敏性鼻炎、湿疹的家族性倾向
E.自身免疫病倾向
E.自身免疫病倾向
6 8 2010 30. 荨麻疹的治疗(2.5分) A.瘙痒明显病因治疗
B.对症治疗
C.抗组胺治疗
D.免疫抑制剂
E.以上都正确
E.以上都正确
6 8 2010 31. 下列哪种疾病的并非HPV病毒感染所致?(2.5分) A.扁平疣
B.尖锐湿疣
C.跖疣
D.传染性软疣
E.寻常疣
D.传染性软疣
6 8 2010 32. 列哪种药物对带状疱疹治疗无效?(2.5分) A.泛昔洛韦
B.阿昔洛韦
C.喷昔洛韦
D.灰黄霉素
E.膦甲酸钠
D.灰黄霉素
6 8 2010 33. 患者女,20岁,左足底多发绿豆至黄豆大角质性丘疹3个月,皮疹渐增大增多,除去角质层可见呈白色软刺状,表面有小黑点,有挤压痛,你认为最可能的诊断是?(2.5分) A.胼胝
B.肢端疼痛症
C.跖疣
D.皲裂
E.鸡眼
C.跖疣
6 8 2010 34. 葡萄球菌烫伤样皮肤综合征的治疗应首选(2.5分) A.头孢类抗生素
B.氯霉素
C.灰黄霉素
D.喹诺酮类
E.链霉素
A.头孢类抗生素
6 8 2010 35. 花斑糠疹的致病菌主要存在表皮的那一层?(2.5分) A.角质层
B.透明层
C.棘层
D.基底层
E.真皮浅层
A.角质层
6 8 2010 36. 关于黄癣,下列哪条叙述是错误的(2.5分) A.外用克霉唑治疗
B.愈后留有永久性秃发
C.Wood灯照射皮损处不显示荧光
D.皮损呈碟状
E.皮损通常轻度瘙痒
C.Wood灯照射皮损处不显示荧光
6 8 2010 37. 下列哪条不对?(2.5分) A.鹅口疮的致病菌是念珠菌
B.癣菌疹通常真菌检查阳性
C.黄癣的致病菌是许兰毛癣菌
D.红色毛癣菌是最常见的浅部真菌病的致病菌
E.白癣愈后不留瘢痕
B.癣菌疹通常真菌检查阳性
6 8 2010 38. 对于浅部真菌病下列哪条是错误的?(2.5分) A.主要累及角质层
B.真菌镜检培养具有诊断价值
C.可侵犯甲板
D.主要累及真皮
E.可侵犯毛发
D.主要累及真皮
6 8 2010 39. 大疱性脓疱疮最常见病原菌是?(2.5分) A.链球菌
B.支原体
C.金葡菌
D.人乳头瘤病毒
E.表皮葡萄球菌
C.金葡菌
6 8 2010 40. 人乳头瘤病原菌主要存在于(2.5分) A.角质层
B.基底层
C.棘层
D.皮下脂肪层
E.真皮浅层
C.棘层
6 8 2011 1. 皮肌炎的特征性皮损是(2.5分) A.面部蝶形红斑
B.双上眼睑紫红斑
C.双手背虹膜状红斑
D.面部环状红斑
E.以上都是
B.双上眼睑紫红斑
6 8 2011 2. 大疱性类天疱疮特点是(2.5分) A.以青壮年居多
B.厚壁、张力性大疱
C.疱壁破裂后糜烂面难以自愈
D.尼氏征阳性
E.以上都是
B.厚壁、张力性大疱
6 8 2011 3. 下列哪项是紫外线治疗的禁忌证?(2.5分) A.玫瑰糠疹
B.银屑病
C.红斑狼疮
D.白癜风
E.以上都是
C.红斑狼疮
6 8 2011 4. 治疗SLE的首选药物为(2.5分) A.环磷酰胺
B.糖皮质激素
C.氯喹
D.环孢素
E.以上都是
B.糖皮质激素
6 8 2011 5. 关于系统性硬皮病的描述,哪项是错误的(2.5分) A.雷诺现象是该病的特征性表现之一
B.关节症状主要为小关节的非对称性疼痛
C.肺部病变是系统性硬皮病的主要死因
D.CREST综合征属该病的一种亚型
E.以上都是
B.关节症状主要为小关节的非对称性疼痛
6 8 2011 6. 皮肤的颜色与下列哪个因素无关 (2.5分) A.种族
B.身高
C.性别
D.营养
E.部位
B.身高
6 8 2011 7. 下列属于第一代抗组胺药的是 (2.5分) A.阿司咪唑
B.特非那定
C.塞庚定
D.氯雷他定
E.西替利嗪
C.塞庚定
6 8 2011 8. 湿敷适用于下列哪种情况 (2.5分) A.慢性皮炎时
B.急性湿疹伴大量渗出时
C.红斑丘疹时
D.无皮疹仅瘙痒时
E.荨麻疹的风团
B.急性湿疹伴大量渗出时
6 8 2011 9. 表皮的主要细胞是 (2.5分) A.黑素细胞
B.角质形成细胞
C.朗格汉斯细胞
D.梅克尔细胞
E.成纤维细胞
B.角质形成细胞
6 8 2011 10. 下列哪种皮疹不是继发疹 (2.5分) A.风团
B.糜烂
C.苔藓样变
D.瘢痕
E.鳞屑
A.风团
6 8 2011 11. 关于斑块,下列哪项是错误的 (2.5分) A.局限性隆起的皮疹
B.皮疹的直径>1cm
C.斑块多由丘疹融合而成
D.斑块可由斑片扩大而成
E.斑块常见于银屑病
D.斑块可由斑片扩大而成
6 8 2011 12. 有关风团的叙述错误的是(2.5分) A.风团是水肿性隆起的损害
B.伴有剧烈瘙痒
C.消退后不留痕迹
D.真皮浅层血管扩张渗出所致
E.风团又称风疹
E.风团又称风疹
6 8 2011 13. 关于结节,下述哪项是错误的 (2.5分) A.结节是局限、实性、深在性损害
B.结节由真皮或皮下炎性浸润所致
C.结节可高出皮面
D.结节可不高出皮面
E.结节破溃后不形成溃疡
E.结节破溃后不形成溃疡
6 8 2011 14. 下列哪项不是激素的作用 (2.5分) A.抗炎
B.免疫抑制
C.抗病毒
D.抗休克
E.抗增生
C.抗病毒
6 8 2011 15. 腋臭是由于哪种腺体在青春期分泌的乳状液经细菌分解后产生臭味引起的 (2.5分) A.小汗腺
B.顶泌汗腺
C.皮脂腺
D.乳腺
E. 唾液腺
B.顶泌汗腺
6 8 2011 16. 以下哪项不是糖皮质激素的副作用((2.5分) A.钠水储留
B.血糖升高
C.电解质紊乱
D.肝功能损伤
E.精神兴奋
D.肝功能损伤
6 8 2011 17. 表皮主要是由角质形成细胞和( )细胞组成(2.5分) A.树枝状
B.扁平状
C.立方形
D.圆柱形
E.以上都是
A.树枝状
6 8 2011 18. Gottron's papules or sign is seen in (2.5分) A.Dermatomyositis
B.Multiple myeloma
C.Acute myeloid leukemia
D.Psoriasis
E.All above are true
A.Dermatomyositis
6 8 2011 19. Morphea occurs usually in (2.5分) A.Forehead
B.Stenum
C.Limbs
D.Back
E.All above are true
A.Forehead
6 8 2011 20. Nail involvement is not a feature of (2.5分) A.Dermatophytosis
B.lichen planus
C.DLE
D.Psoriasis
E.All above are true
C.DLE
6 8 2011 21. Epidermal cells of normal tissues are replaced completely (2.5分) A.4 days
B.12 days
C.28 days
D.38 days
E.All above are wrong
C.28 days
6 8 2011 22. Do you think the HPV virus is mainly located at which layer of the epidermis? (2.5分) A.stratum corneum
B.the subcutaneous fat layer
C.stratum spinosum
D.dermis layer
E.stratum basale
C.stratum spinosum
6 8 2011 23. 下列何种疾病其病原菌并非HPV病毒引起?(2.5分) A.尖锐湿疣
B.扁平疣
C.跖疣
D.传染性软疣
E.寻常疣
D.传染性软疣
6 8 2011 24. 扁平疣经搔抓后可沿抓痕出现新发条状排列的皮损,这种现象称之为 (2.5分) A.Auspitz征
B.皮肤划痕征
C.尼氏征
D.Gottron征
E.同形反应
E.同形反应
6 8 2011 25. 你认为下列哪个部位并非传染性软疣的好发部位?(2.5分) A.手背
B.面部
C.头皮
D.躯干
E.外生殖器
C.头皮
6 8 2011 26. 下列哪条不符合带状疱疹的特点?(2.5分) A.单侧分布
B.区域神经痛
C.呈簇的粟米大水疱
D.由单纯疱疹病毒感染引起
E.条带状分布
D.由单纯疱疹病毒感染引起
6 8 2011 27. 有关传染性软疣你认为下列哪条不对?(2.5分) A.由HPV病毒感染引起,且多见于老年人
B.呈脐凹状丘疹
C.由痘病毒引起
D.多见于儿童及青少年
E.可挤出白色乳酪样物质
A.由HPV病毒感染引起,且多见于老年人
6 8 2011 28. 关于跖疣,下列哪条是错的?(2.5分) A.皮疹可以多发
B.病程慢性,可自然消退
C.是由痘病毒引起
D.皮疹表面粗糙不平,质地硬
E.挤压时有明显疼痛
C.是由痘病毒引起
6 8 2011 29. 疖病的诱发因素不包括下列哪一条?(2.5分) A.老年患者
B.糖尿病
C.皮肤瘙痒
D.不良卫生习惯
E.全身性慢性疾病
A.老年患者
6 8 2011 30. 葡萄球菌烫伤样皮肤综合征的治疗应首选(2.5分) A.灰黄霉素
B.喹诺酮类
C.氯霉素
D.头孢类抗生素
E.链霉素
D.头孢类抗生素
6 8 2011 31. 由凝固酶阳性金葡菌引起多个相邻毛囊及毛囊周围化脓性炎症称之为(2.5分) A.毛囊炎
B.痈
C.蜂窝织炎
D.疖
E.丹毒
B.痈
6 8 2011 32. 丹毒的治疗应首先考虑使用下列哪种药物?(2.5分) A.氯霉素
B.青霉素
C.红霉素
D.灰黄霉素
E.氧氟沙星
B.青霉素
6 8 2011 33. 关于寻常型脓疱疮下列哪条叙述是错误的?(2.5分) A.常继发于瘙痒性皮肤病
B.致病菌为微细棒状杆菌
C.好发于炎热季节
D.儿童多见
E.外用莫匹罗星软膏治疗有效
B.致病菌为微细棒状杆菌
6 8 2011 34. 大疱性脓疱疮最常见病原菌是?(2.5分) A.金葡菌
B.链球菌
C.支原体
D.表皮葡萄球菌
E.人乳头瘤病毒
A.金葡菌
6 8 2011 35. 关于黄癣,下列哪条叙述是错误的(2.5分) A.Wood灯照射皮损处不显示荧光
B.外用克霉唑治疗
C.愈后留有永久性秃发
D.皮损呈碟状
E.皮损通常轻度瘙痒
A.Wood灯照射皮损处不显示荧光
6 8 2011 36. 花斑糠疹的致病菌主要存在表皮的那一层? (2.5分) A.透明层
B.角质层
C.棘层
D.基底层
E.真皮浅层
B.角质层
6 8 2011 37. 有关花斑糠疹,下列哪条正确?(2.5分) A.剧烈瘙痒
B.皮损为红斑、斑块
C.愈后可留有暂时性色素减退斑
D.多发于手足
E.冬季易发
C.愈后可留有暂时性色素减退斑
6 8 2011 38. 手足癣的治疗不应选择下列哪种药物?(2.5分) A.克霉唑乳膏
B.联苯苄唑乳膏
C.酮康唑乳膏
D.特比萘芬软膏
E.红霉素软膏
E.红霉素软膏
6 8 2011 39. Which of the following is not a kind of superficial fungi ? (2.5分) A.T. rubrum
B.T. Mentagrophytes
C.M. Canis
D.E . Floccosum
E.candida
E.candida
6 8 2011 40. Wood灯(滤过紫外线灯)检查白癣病发呈 (2.5分) A.暗绿色荧光
B.无荧光
C.亮绿色荧光
D.珊瑚红色荧光
E.以上均不对
C.亮绿色荧光
6 8 2012 1. 表皮的主要细胞是(2.5分) A.黑素细胞
B.角质形成细胞
C.朗格汉斯细胞
D.梅克尔细胞
E.成纤维细胞
B.角质形成细胞
6 8 2012 2. 下列哪种皮疹不是原发疹(2.5分) A.脓疱
B.水疱
C.斑疹
D.脓痂
E.结节
D.脓痂
6 8 2012 3. 关于丘疹,下列哪项是错误的(2.5分) A.丘疹为局限性隆起的皮疹
B.丘疹病变常位于表皮及真皮浅层
C.丘疹的直径>1cm
D.丘疹表面可为扁平、乳头状、脐凹
E.丘疹可有多种颜色
C.丘疹的直径>1cm
6 8 2012 4. 抗组胺药的作用机理(2.5分) A.与组胺竞争受体
B.直接破坏细胞壁
C.直接抗炎性介质
D.诱导细胞产生抗炎性介质的物质
E.以上都不对
A.与组胺竞争受体
6 8 2012 5. 以下哪项不是糖皮质激素的副作用(2.5分) A.钠水储留
B.血糖升高
C.电解质紊乱
D.肝功能损伤
E.精神兴奋
D.肝功能损伤
6 8 2012 6. 有关传染性软疣下列哪条不对? (2.5分) A.由痘病毒引起
B.呈脐凹状丘疹
C.由HPV 病毒感染引起,且多见于老年人
D.有软疣小体
E.多见于儿童及青少年
C.由HPV 病毒感染引起,且多见于老年人
6 8 2012 7. Do you think the HPV virus is mainly located at which layer of the epidermis? (2.5分) A.stratum corneum
B.the subcutaneous fat layer
C.dermis layer
D.stratum spinosum
E.stratum basale
D.stratum spinosum
6 8 2012 8. 关于跖疣,下列哪条是错的? (2.5分) A.皮疹可以多发
B.是由痘病毒引起
C.病程慢性,可自然消退
D.皮疹表面粗糙不平,质地坚硬
E.挤压时有明显疼痛
B.是由痘病毒引起
6 8 2012 9. 下列哪种药物对带状疱疹治疗无效? (2.5分) A.阿昔洛韦
B.泛昔洛韦
C.灰黄霉素
D.膦甲酸钠
E.喷昔洛韦
C.灰黄霉素
6 8 2012 10. 患者男,20 岁,左足底多发粟米至黄豆大角质性丘疹3 个月,皮疹渐增大增多,有明显挤压痛,除去表面角质层可见呈白色软刺状,表面有小黑点,你认为最可能的诊断是? (2.5分) A.跖疣
B.肢端疼痛症
C.胼胝
D.鸡眼
E.扁平疣
A.跖疣
6 8 2012 11. Which of the following disease does not belong to pyoderma ?(2.5分) A.ecthyma
B.erysipelas
C.cellulitis
D.kerion
E.Impetigo
D.kerion
6 8 2012 12. 葡萄球菌烫伤样皮肤综合征的治疗应首选(2.5分) A.灰黄霉素
B.喹诺酮类
C.新型四环素
D.链霉素
E.头孢类抗生素
E.头孢类抗生素
6 8 2012 13. 关于寻常型脓疱疮下列哪条叙述是错误的? (2.5分) A.好发于炎热季节
B.常继发于瘙痒性皮肤病
C.外用莫匹罗星软膏治疗有效
D.致病菌为微细棒状杆菌
E.儿童多见
D.致病菌为微细棒状杆菌
6 8 2012 14. 下列哪种说法你认为是不对的? (2.5分) A.细菌性毛囊炎系整个毛囊细菌感染发生化脓性炎症
B.项部疤痕疙瘩性毛囊炎指病程缓慢,发生于颈项部的愈后形成瘢痕硬结者
C.秃发性毛囊炎的病原菌主要是金黄色葡萄球菌
D.在疤痕形成期的头部脓肿性穿掘性毛囊炎可选用青霉素联合异维A 酸口服治疗
E.头部脓肿性穿掘性毛囊炎是一种少见的头顶部慢性非化脓性皮肤病
E.头部脓肿性穿掘性毛囊炎是一种少见的头顶部慢性非化脓性皮肤病
6 8 2012 15. 关于黄癣,下列哪条叙述是错误的? (2.5分) A.外用克霉唑治疗有效
B.Wood 灯照射皮损处不显示荧光
C.皮损通常轻度瘙痒
D.愈后留有永久性秃发
E.皮损呈碟状
B.Wood 灯照射皮损处不显示荧光
6 8 2012 16. 下列哪条不对? (2.5分) A.鹅口疮的致病菌是念珠菌
B.黄癣的致病菌是许兰毛癣菌
C.癣菌疹通常真菌检查阳性
D.红色毛癣菌是最常见的浅部真菌病的致病菌
E.白癣愈后不留瘢痕
C.癣菌疹通常真菌检查阳性
6 8 2012 17. 手足癣的治疗不应选择下列哪种药物? (2.5分) A.红霉素软膏
B.联苯苄唑乳膏
C.酮康唑乳膏
D.特比萘芬软膏
E.克霉唑乳膏
A.红霉素软膏
6 8 2012 18. 下列关于慢性湿疹描述不正确的是(2.5分) A.多发于面、手足、四肢及外阴部
B.反复发作、有渗出史
C.常由急性演变而来
D.往往病因单一,与感染密切相关
E.男性发病高于女性
D.往往病因单一,与感染密切相关
6 8 2012 19. 下列不属于重症药疹的是(2.5分) A.猩红热样药疹
B.重症多形红斑型药疹
C.剥脱性皮炎型药疹
D.大疱性表皮松解型药疹
E.药物超敏反应综合征
A.猩红热样药疹
6 8 2012 20. 治疗药疹的首要措施是(2.5分) A.使用足量的糖皮质激素
B.使用抗生素治疗
C.抗过敏治疗
D.停用一切可疑的致敏药物
E.尽早使用外用药
D.停用一切可疑的致敏药物
6 8 2012 21. 皮肤划痕试验适用于检测的疾病是(2.5分) A.湿疹
B.接触性皮炎
C.脂溢性皮炎
D.荨麻疹
E.银屑病
D.荨麻疹
6 8 2012 22. 下列哪项不是变应性接触性皮炎的临床特点(2.5分) A.接触致敏物后发病
B.皮疹单一形态,境界清楚
C.好发于暴露部位,尤其接触部位
D.起病迅速,无潜伏期
E.严重者可有系统症状发生
D.起病迅速,无潜伏期
6 8 2012 23. 关于接触性皮炎与急性湿疹的鉴别,下列哪项是错误的(2.5分) A.皮疹是否境界清楚
B.皮疹是否对称发生
C.皮疹是否单一形态
D.皮疹是否瘙痒
E.病因是否明确
D.皮疹是否瘙痒
6 8 2012 24. 引起过敏性休克的发病机制主要是哪型变态反应(2.5分) A.Ⅱ型
B.非变态反应
C.Ⅰ型
D.Ⅳ型
E.Ⅲ型
C.Ⅰ型
6 8 2012 25. 药疹初次发病的潜伏期一般为(2.5分) A.1~5天
B.4~20天
C.3~7天
D.7~8天
E.4~10天
B.4~20天
6 8 2012 26. 关于变态反应性药疹的特点,下列哪项叙述错误(2.5分) A.只发生于少数对药物过敏者
B.病情轻重与药物的剂量相关
C.一种药物可引起多种形态的皮疹,不同药物可引起同一种皮疹
D.初次用药一般需4~20天后才出现药疹
E.病情轻重与药物的药理、毒理作用无相关性
B.病情轻重与药物的剂量相关
6 8 2012 27. 下列哪项不属于麻疹型药疹的特点(2.5分) A.Koplik斑
B.皮疹对称分布
C.全身中毒症状较轻
D.起病突然,发展较快
E.自觉瘙痒
A.Koplik斑
6 8 2012 28. 下列关于急性荨麻疹的叙述错误的是(2.5分) A.起病急
B.多有瘙痒
C.无皮肤外的任何器官受累
D.严重者皮损处可出现橘皮样外观
E.抗组胺药治疗有效
C.无皮肤外的任何器官受累
6 8 2012 29. 扁平疣经搔抓后可沿抓痕出现新发条状排列的皮损,这种现象称之为(2.5分) A.尼氏征
B.Auspitz 征
C.皮肤划痕征
D.同形反应
E.Gottron 征
D.同形反应
6 8 2012 30. 疖病的诱发因素不包括下列哪一条?(2.5分) A.糖尿病
B.老年患者
C.皮肤瘙痒
D.不良卫生习惯
E.全身性慢性疾病
B.老年患者
6 8 2012 31. 患者男,56 岁。患2 型糖尿病病史3 年,血糖一直控制不佳。最近半月颈部出现弥漫性浸润硬块,紫红色紧张发亮,周围红肿,皮损中央区坏死可见多个脓头溢出黄白色脓液,伴发热,最高体温39.6 度。伴畏寒、乏力。该患者最可能的诊断是(A)(2.5分) A.痈
B.毛囊炎
C.脓癣
D.SSSS
E.疖
A.痈
6 8 2012 32. 关于黄癣,下列哪条叙述是错误的? (2.5分) A.外用克霉唑治疗有效
B.Wood 灯照射皮损处不显示荧光
C.皮损通常轻度瘙痒
D.愈后留有永久性秃发
E.皮损呈碟状
B.Wood 灯照射皮损处不显示荧光
6 8 2012 33. 皮肌炎的特征性皮损是 (2.5分) A.面部蝶形红斑
B.双上眼睑紫红斑
C.双手背虹膜状红斑
D.面部环状红斑
B.双上眼睑紫红斑
6 8 2012 34. 下列哪项是紫外线治疗的禁忌证?(2.5分) A.玫瑰糠疹
B.银屑病
C.红斑狼疮
D.白癜风
C.红斑狼疮
6 8 2012 35. 治疗SLE的首选药物为 (2.5分) A.环磷酰胺
B.糖皮质激素
C.氯喹
D.环孢素
B.糖皮质激素
6 8 2012 36. Gottron's papules or sign is seen in (2.5分) A.Dermatomyositis
B.Multiple myeloma
C.Acute myeloid leukemia
D.Psoriasis
A.Dermatomyositis
6 8 2012 37. Which of the following are false about pemphigus ? (2.5分) A.Corticosteroid is the preferred treatment.
B.Acantholysis is the basic pathological character.
C.Blisters locate at subepidermis.
D.The anti-pemphigus antibody titer is coincident with the severity of the disease.
D.The anti-pemphigus antibody titer is coincident with the severity of the disease.
6 8 2012 38. 剥脱性皮炎型药疹的特点哪项叙述不正确(2.5分) A.可伴药物性肝炎
B.大量脱屑
C.初次发病潜伏期约1周
D.病程反复长达数月
E.可累及黏膜
C.初次发病潜伏期约1周
6 8 2012 39. Nail involvement is not a feature of (2.5分) A.Dermatophytosis
B.lichen planus
C.DLE
D.Psoriasis
C.DLE
6 8 2012 40. Wood灯(滤过紫外线灯)检查白癣病发呈(2.5分) A.暗绿色荧光
B.无荧光
C.亮绿色荧光
D.珊瑚红色荧光
E.以上均不对
C.亮绿色荧光
6 8 2013 1. 湿敷适用于下列哪种情况(2.5分) A.慢性皮炎时
B.急性湿疹伴大量渗出时
C.红斑丘疹时
D.无皮疹仅瘙痒时
E.荨麻疹的风团
B.急性湿疹伴大量渗出时
6 8 2013 2. 下列哪种皮疹不是原发疹(2.5分) A.脓疱
B.水疱
C.斑疹
D.脓痂
E.结节
D.脓痂
6 8 2013 3. 关于丘疹,下列哪项是错误的(2.5分) A.丘疹为局限性隆起的皮疹
B.丘疹病变常位于表皮及真皮浅层
C.丘疹的直径>1cm
D.丘疹表面可为扁平、乳头状、脐凹
E.丘疹可有多种颜色
C.丘疹的直径>1cm
6 8 2013 4. 下列哪项不属于变应性接触性皮炎(2.5分) A.尿布皮炎
B.外用药使用后局部的皮炎
C.化妆品皮炎
D.滴眼药水引起的过敏
E.栓剂使用后的过敏
E.栓剂使用后的过敏
6 8 2013 5. 以下哪项不是糖皮质激素的副作用(2.5分) A.钠水储留
B.血糖升高
C.电解质紊乱
D.肝功能损伤
E.精神兴奋
D.肝功能损伤
6 8 2013 6. 有关传染性软疣下列哪条不对? (2.5分) A.由痘病毒引起
B.呈脐凹状丘疹
C.由HPV 病毒感染引起,且多见于老年人
D.有软疣小体
E.多见于儿童及青少年
C.由HPV 病毒感染引起,且多见于老年人
6 8 2013 7. Do you think the HPV virus is mainly located at which layer of the epidermis? (2.5分) A.stratum corneum
B.the subcutaneous fat layer
C.dermis layer
D.stratum spinosum
E.stratum basale
D.stratum spinosum
6 8 2013 8. 关于跖疣,下列哪条是错的? (2.5分) A.皮疹可以多发
B.是由痘病毒引起
C.病程慢性,可自然消退
D.皮疹表面粗糙不平,质地坚硬
E.挤压时有明显疼痛
B.是由痘病毒引起
6 8 2013 9. 下列哪种药物对带状疱疹治疗无效? (2.5分) A.阿昔洛韦
B.泛昔洛韦
C.灰黄霉素
D.膦甲酸钠
E.喷昔洛韦
C.灰黄霉素
6 8 2013 10. 关于斑块,下列哪项是错误的(2.5分) A.局限性隆起的皮疹
B.皮疹的直径>1cm
C.斑块多由丘疹融合而成
D.斑块可由斑片扩大而成
E.斑块常见于银屑病
D.斑块可由斑片扩大而成
6 8 2013 11. Which of the following disease does not belong to pyoderma ?(2.5分) A.ecthyma
B.erysipelas
C.cellulitis
D.kerion
E.Impetigo
D.kerion
6 8 2013 12. 葡萄球菌烫伤样皮肤综合征的治疗应首选(2.5分) A.灰黄霉素
B.喹诺酮类
C.新型四环素
D.链霉素
E.头孢类抗生素
E.头孢类抗生素
6 8 2013 13. 关于寻常型脓疱疮下列哪条叙述是错误的? (2.5分) A.好发于炎热季节
B.常继发于瘙痒性皮肤病
C.外用莫匹罗星软膏治疗有效
D.致病菌为微细棒状杆菌
E.儿童多见
D.致病菌为微细棒状杆菌
6 8 2013 14. 下列哪种说法你认为是不对的? (2.5分) A.细菌性毛囊炎系整个毛囊细菌感染发生化脓性炎症
B.项部疤痕疙瘩性毛囊炎指病程缓慢,发生于颈项部的愈后形成瘢痕硬结者
C.秃发性毛囊炎的病原菌主要是金黄色葡萄球菌
D.在疤痕形成期的头部脓肿性穿掘性毛囊炎可选用青霉素联合异维A 酸口服治疗
E.头部脓肿性穿掘性毛囊炎是一种少见的头顶部慢性非化脓性皮肤病
E.头部脓肿性穿掘性毛囊炎是一种少见的头顶部慢性非化脓性皮肤病
6 8 2013 15. 关于黄癣,下列哪条叙述是错误的? (2.5分) A.外用克霉唑治疗有效
B.Wood 灯照射皮损处不显示荧光
C.皮损通常轻度瘙痒
D.愈后留有永久性秃发
E.皮损呈碟状
B.Wood 灯照射皮损处不显示荧光
6 8 2013 16. 下列哪条不对? (2.5分) A.鹅口疮的致病菌是念珠菌
B.黄癣的致病菌是许兰毛癣菌
C.癣菌疹通常真菌检查阳性
D.红色毛癣菌是最常见的浅部真菌病的致病菌
E.白癣愈后不留瘢痕
C.癣菌疹通常真菌检查阳性
6 8 2013 17. 手足癣的治疗不应选择下列哪种药物? (2.5分) A.红霉素软膏
B.联苯苄唑乳膏
C.酮康唑乳膏
D.特比萘芬软膏
E.克霉唑乳膏
A.红霉素软膏
6 8 2013 18. 下列关于慢性湿疹描述不正确的是(2.5分) A.多发于面、手足、四肢及外阴部
B.反复发作、有渗出史
C.常由急性演变而来
D.往往病因单一,与感染密切相关
E.男性发病高于女性
D.往往病因单一,与感染密切相关
6 8 2013 19. 下列不属于重症药疹的是(2.5分) A.猩红热样药疹
B.重症多形红斑型药疹
C.剥脱性皮炎型药疹
D.大疱性表皮松解型药疹
E.药物超敏反应综合征
A.猩红热样药疹
6 8 2013 20. 治疗药疹的首要措施是(2.5分) A.使用足量的糖皮质激素
B.使用抗生素治疗
C.抗过敏治疗
D.停用一切可疑的致敏药物
E.尽早使用外用药
D.停用一切可疑的致敏药物
6 8 2013 21. 皮肤划痕试验适用于检测的疾病是(2.5分) A.湿疹
B.接触性皮炎
C.脂溢性皮炎
D.荨麻疹
E.银屑病
D.荨麻疹
6 8 2013 22. 下列哪项不是变应性接触性皮炎的临床特点(2.5分) A.接触致敏物后发病
B.皮疹单一形态,境界清楚
C.好发于暴露部位,尤其接触部位
D.起病迅速,无潜伏期
E.严重者可有系统症状发生
D.起病迅速,无潜伏期
6 8 2013 23. 关于接触性皮炎与急性湿疹的鉴别,下列哪项是错误的(2.5分) A.皮疹是否境界清楚
B.皮疹是否对称发生
C.皮疹是否单一形态
D.皮疹是否瘙痒
E.病因是否明确
D.皮疹是否瘙痒
6 8 2013 24. 引起过敏性休克的发病机制主要是哪型变态反应(2.5分) A.Ⅱ型
B.非变态反应
C.Ⅰ型
D.Ⅳ型
E.Ⅲ型
C.Ⅰ型
6 8 2013 25. 关于结节,下述哪项是错误的(2.5分) A.结节是局限、实性、深在性损害
B.结节由真皮或皮下炎性浸润所致
C.结节可高出皮面
D.结节可不高出皮面
E.结节破溃后不形成溃疡
E.结节破溃后不形成溃疡
6 8 2013 26. 关于变态反应性药疹的特点,下列哪项叙述错误(2.5分) A.只发生于少数对药物过敏者
B.病情轻重与药物的剂量相关
C.一种药物可引起多种形态的皮疹,不同药物可引起同一种皮疹
D.初次用药一般需4~20天后才出现药疹
E.病情轻重与药物的药理、毒理作用无相关性
B.病情轻重与药物的剂量相关
6 8 2013 27. 下列哪项不属于麻疹型药疹的特点(2.5分) A.Koplik斑
B.皮疹对称分布
C.全身中毒症状较轻
D.起病突然,发展较快
E.自觉瘙痒
A.Koplik斑
6 8 2013 28. 下列关于急性荨麻疹的叙述错误的是(2.5分) A.起病急
B.多有瘙痒
C.无皮肤外的任何器官受累
D.严重者皮损处可出现橘皮样外观
E.抗组胺药治疗有效
C.无皮肤外的任何器官受累
6 8 2013 29. 扁平疣经搔抓后可沿抓痕出现新发条状排列的皮损,这种现象称之为(2.5分) A.尼氏征
B.Auspitz 征
C.皮肤划痕征
D.同形反应
E.Gottron 征
D.同形反应
6 8 2013 30. 疖病的诱发因素不包括下列哪一条?(2.5分) A.糖尿病
B.老年患者
C.皮肤瘙痒
D.不良卫生习惯
E.全身性慢性疾病
B.老年患者
6 8 2013 31. 患者男,56 岁。患2 型糖尿病病史3 年,血糖一直控制不佳。最近半月颈部出现弥漫性浸润硬块,紫红色紧张发亮,周围红肿,皮损中央区坏死可见多个脓头溢出黄白色脓液,伴发热,最高体温39.6 度。伴畏寒、乏力。该患者最可能的诊断是(A)(2.5分) A.痈
B.毛囊炎
C.脓癣
D.SSSS
E.疖
A.痈
6 8 2013 32. 关于黄癣,下列哪条叙述是错误的? (2.5分) A.外用克霉唑治疗有效
B.Wood 灯照射皮损处不显示荧光
C.皮损通常轻度瘙痒
D.愈后留有永久性秃发
E.皮损呈碟状
B.Wood 灯照射皮损处不显示荧光
6 8 2013 33. 皮肌炎的特征性皮损是 (2.5分) A.面部蝶形红斑
B.双上眼睑紫红斑
C.双手背虹膜状红斑
D.面部环状红斑
B.双上眼睑紫红斑
6 8 2013 34. 下列哪项是紫外线治疗的禁忌证?(2.5分) A.玫瑰糠疹
B.银屑病
C.红斑狼疮
D.白癜风
C.红斑狼疮
6 8 2013 35. 治疗SLE的首选药物为 (2.5分) A.环磷酰胺
B.糖皮质激素
C.氯喹
D.环孢素
B.糖皮质激素
6 8 2013 36. Gottron's papules or sign is seen in (2.5分) A.Dermatomyositis
B.Multiple myeloma
C.Acute myeloid leukemia
D.Psoriasis
A.Dermatomyositis
6 8 2013 37. Which of the following are false about pemphigus ? (2.5分) A.Corticosteroid is the preferred treatment.
B.Acantholysis is the basic pathological character.
C.Blisters locate at subepidermis.
D.The anti-pemphigus antibody titer is coincident with the severity of the disease.
D.The anti-pemphigus antibody titer is coincident with the severity of the disease.
6 8 2013 38. 剥脱性皮炎型药疹的特点哪项叙述不正确(2.5分) A.可伴药物性肝炎
B.大量脱屑
C.初次发病潜伏期约1周
D.病程反复长达数月
E.可累及黏膜
C.初次发病潜伏期约1周
6 8 2013 39. Nail involvement is not a feature of(2.5分) A.Dermatophytosis
B.lichen planus
C.DLE
D.Psoriasis
C.DLE
6 8 2013 40. Wood灯(滤过紫外线灯)检查白癣病发呈 (2.5分) A.暗绿色荧光
B.无荧光
C.亮绿色荧光
D.珊瑚红色荧光
E.以上均不对
C.亮绿色荧光
6 8 2014 1. 表皮主要是由角质形成细胞和()细胞组成(2.5分) A.树枝状
B.扁平状
C.立方形
D.圆柱形
E.以上都是
A.树枝状
6 8 2014 2. 腋臭是由于哪种腺体在青春期分泌的乳状液经细菌分解后产生臭味引起的(2.5分) A.小汗腺
B.顶泌汗腺
C.皮脂腺
D.乳腺
E.唾液腺
B.顶泌汗腺
6 8 2014 3. 关于丘疹,下列哪项是错误的(2.5分) A.丘疹为局限性隆起的皮疹
B.丘疹病变常位于表皮及真皮浅层
C.丘疹的直径>1cm
D.丘疹表面可为扁平、乳头状、脐凹
E.丘疹可有多种颜色
C.丘疹的直径>1cm
6 8 2014 4. 下列哪种皮疹不是继发疹(2.5分) A.风团
B.糜烂
C.苔藓样变
D.瘢痕
E.鳞屑
A.风团
6 8 2014 5. 以下哪项不是糖皮质激素的副作用(2.5分) A.钠水储留
B.血糖升高
C.电解质紊乱
D.肝功能损伤
E.精神兴奋
D.肝功能损伤
6 8 2014 6. 有关传染性软疣下列哪条不对?(2.5分) A.由痘病毒引起
B.呈脐凹状丘疹
C.由HPV病毒感染引起,且多见于老年人
D.有软疣小体
E.多见于儿童及青少年
C.由HPV病毒感染引起,且多见于老年人
6 8 2014 7. 下列哪条不符合带状疱疹的特点?(2.5分) A.单侧分布
B.发病年龄越小其后遗神经痛越明显
C.区域淋巴结肿大
D.呈簇的粟米大水疱
E.条带状分布
B.发病年龄越小其后遗神经痛越明显
6 8 2014 8. 关于跖疣,下列哪条是错的?(2.5分) A.皮疹可以多发
B.是由痘病毒引起
C.病程慢性,可自然消退
D.皮疹表面粗糙不平,质地坚硬
E.挤压时有明显疼痛
B.是由痘病毒引起
6 8 2014 9. 下列哪种药物对带状疱疹治疗无效?(2.5分) A.阿昔洛韦
B.泛昔洛韦
C.灰黄霉素
D.膦甲酸钠
E.喷昔洛韦
C.灰黄霉素
6 8 2014 10. 关于丹毒下列哪条是正确的?(2.5分) A.好发于面部及小腿
B.为皮肤的浅表感染
C.好发部位为躯干
D.不伴有全身症状
E.皮损中可分离出杆菌
A.好发于面部及小腿
6 8 2014 11. Which of the following disease does not belong to pyoderma ?(2.5分) A.ecthyma
B.erysipelas
C.cellulitis
D.kerion
E.Impetigo
D.kerion
6 8 2014 12. 葡萄球菌烫伤样皮肤综合征的治疗应首选(2.5分) A.灰黄霉素
B.喹诺酮类
C.新型四环素
D.链霉素
E.头孢类抗生素
E.头孢类抗生素
6 8 2014 13. 大疱性脓疱疮最常见病原菌是?(2.5分) A.人乳头瘤病毒
B.金黄色葡萄球菌
C.支原体
D.表皮葡萄球菌
E.溶血性链球菌
B.金黄色葡萄球菌
6 8 2014 14. 下列哪种说法你认为是不对的?(2.5分) A.细菌性毛囊炎系整个毛囊细菌感染发生化脓性炎症
B.项部疤痕疙瘩性毛囊炎指病程缓慢,发生于颈项部的愈后形成瘢痕硬结者
C.秃发性毛囊炎的病原菌主要是金黄色葡萄球菌
D.在疤痕形成期的头部脓肿性穿掘性毛囊炎可选用青霉素联合异维A 酸口服治疗
E.头部脓肿性穿掘性毛囊炎是一种少见的头顶部慢性非化脓性皮肤病
E.头部脓肿性穿掘性毛囊炎是一种少见的头顶部慢性非化脓性皮肤病
6 8 2014 15. 关于黄癣,下列哪条叙述是错误的?(2.5分) A.外用克霉唑治疗有效
B.Wood 灯照射皮损处不显示荧光
C.皮损通常轻度瘙痒
D.愈后留有永久性秃发
E.皮损呈碟状
B.Wood 灯照射皮损处不显示荧光
6 8 2014 16. 下列哪条不对?(2.5分) A.鹅口疮的致病菌是念珠菌
B.黄癣的致病菌是许兰毛癣菌
C.癣菌疹通常真菌检查阳性
D.红色毛癣菌是最常见的浅部真菌病的致病菌
E.白癣愈后不留瘢痕
C.癣菌疹通常真菌检查阳性
6 8 2014 17. 手足癣的治疗不应选择下列哪种药物?(2.5分) A.联苯苄唑乳膏
B.酮康唑乳膏
C.特比萘芬软膏
D.金霉素软膏
E.克霉唑乳膏
D.金霉素软膏
6 8 2014 18. 下列关于慢性湿疹描述不正确的是(2.5分) A.多发于面、手足、四肢及外阴部
B.反复发作、有渗出史
C.常由急性演变而来
D.往往病因单一,与感染密切相关
E.男性发病高于女性
D.往往病因单一,与感染密切相关
6 8 2014 19. 下列不属于重症药疹的是(2.5分) A.猩红热样药疹
B.重症多形红斑型药疹
C.剥脱性皮炎型药疹
D.大疱性表皮松解型药疹
E.药物超敏反应综合征
A.猩红热样药疹
6 8 2014 20. 治疗药疹的首要措施是(2.5分) A.使用足量的糖皮质激素
B.使用抗生素治疗
C.抗过敏治疗
D.停用一切可疑的致敏药物
E.尽早使用外用药
D.停用一切可疑的致敏药物
6 8 2014 21. 引起急性荨麻疹的变态反应主要是(2.5分) A.Ⅲ型变态反应
B.II型变态反应
C.I型变态反应
D.Ⅳ型变态反应
E.两种以上变态反应
C.I型变态反应
6 8 2014 22. 下列哪项不是变应性接触性皮炎的临床特点(2.5分) A.接触致敏物后发病
B.皮疹单一形态,境界清楚
C.好发于暴露部位,尤其接触部位
D.起病迅速,无潜伏期
E.严重者可有系统症状发生
D.起病迅速,无潜伏期
6 8 2014 23. 关于接触性皮炎与急性湿疹的鉴别,下列哪项是错误的(2.5分) A.皮疹是否境界清楚
B.皮疹是否对称发生
C.皮疹是否单一形态
D.皮疹是否瘙痒
E.病因是否明确
D.皮疹是否瘙痒
6 8 2014 24. 引起过敏性休克的发病机制主要是哪型变态反应(2.5分) A.Ⅱ型
B.非变态反应
C.Ⅰ型
D.Ⅳ型
E.Ⅲ型
C.Ⅰ型
6 8 2014 25. 下列哪项不属于麻疹型药疹的特点(2.5分) A.Koplik斑
B.皮疹对称分布
C.全身中毒症状较轻
D.起病突然,发展较快
E.自觉瘙痒
A.Koplik斑
6 8 2014 26. 药疹初次发病的潜伏期一般为(2.5分) A.1~5天
B.4~20天
C.3~7天
D.7~8天
E.4~10天
B.4~20天
6 8 2014 27. 抗组胺药的作用机理(2.5分) A.与组胺竞争受体
B.直接破坏细胞壁
C.直接抗炎性介质
D.诱导细胞产生抗炎性介质的物质
E.以上都不对
A.与组胺竞争受体
6 8 2014 28. 下列关于急性荨麻疹的叙述错误的是(2.5分) A.起病急
B.多有瘙痒
C.无皮肤外的任何器官受累
D.严重者皮损处可出现橘皮样外观
E.抗组胺药治疗有效
C.无皮肤外的任何器官受累
6 8 2014 29. 扁平疣经搔抓后可沿抓痕出现新发条状排列的皮损,这种现象称之为(2.5分) A.尼氏征
B.Auspitz 征
C.皮肤划痕征
D.同形反应
E.Gottron 征
D.同形反应
6 8 2014 30. 疖病的诱发因素不包括下列哪一条?(2.5分) A.糖尿病
B.老年患者
C.皮肤瘙痒
D.不良卫生习惯
E.全身性慢性疾病
B.老年患者
6 8 2014 31. 患者男,56 岁。患2 型糖尿病病史3 年,血糖一直控制不佳。最近半月颈部出现弥漫性浸润硬块,紫红色紧张发亮,周围红肿,皮损中央区坏死可见多个脓头溢出黄白色脓液,伴发热,最高体温39.6 度。伴畏寒、乏力。该患者最可能的诊断是(2.5分) A.痈
B.毛囊炎
C.脓癣
D.SSSS
E.疖
A.痈
6 8 2014 32. 大疱性表皮松解型药疹的临床特点不包括(2.5分) A.松弛性水疱或大疱
B.大量渗出
C.大面积表皮松解
D.尼氏征阴性
E.广泛黏膜受累
D.尼氏征阴性
6 8 2014 33. 皮肌炎的特征性皮损是(2.5分) A.面部蝶形红斑
B.多形红斑
C.双手背虹膜状红斑
D.面部环状红斑
E.双上眼睑紫红斑
B.多形红斑
6 8 2014 34. 下列哪项是紫外线治疗的禁忌证?(2.5分) A.玫瑰糠疹
B.银屑病
C.红斑狼疮
D.白癜风
E.湿疹
C.红斑狼疮
6 8 2014 35. 治疗SLE的首选药物为(2.5分) A.环磷酰胺
B.糖皮质激素
C.氯喹
D.环孢素
E.硫唑嘌呤
B.糖皮质激素
6 8 2014 36. 剥脱性皮炎型药疹的特点哪项叙述不正确(2.5分) A.可伴药物性肝炎
B.大量脱屑
C.初次发病潜伏期约1周
D.病程反复长达数月
E.可累及黏膜
C.初次发病潜伏期约1周
6 8 2014 37. Wood灯(滤过紫外线灯)检查白癣病发呈(2.5分) A.暗绿色荧光
B.无荧光
C.亮绿色荧光
D.珊瑚红色荧光
E.以上均不对
C.亮绿色荧光
6 8 2014 38. Gottron's papules or sign is seen in (2.5分) A.Dermatomyositis
B.Multiple myeloma
C.Acute myeloid leukemia
D.Psoriasis
A.Dermatomyositis
6 8 2014 39. Which of the following are false about pemphigus ? (2.5分) A.Corticosteroid is the preferred treatment.
B.Acantholysis is the basic pathological character.
C.Blisters locate at subepidermis.
D.The anti-pemphigus antibody titer is coincident with the severity of the disease.
D.The anti-pemphigus antibody titer is coincident with the severity of the disease.
6 8 2014 40. Nail involvement is not a feature of(2.5分) A.Dermatophytosis
B.lichen planus
C.DLE
D.Psoriasis
C.DLE
6 8 2015 1. 表皮主要是由角质形成细胞和( )细胞组成(2.5分) A.树枝状
B.扁平状
C.立方形
D.圆柱形
E.以上都是
A.树枝状
6 8 2015 2. 腋臭是由于哪种腺体在青春期分泌的乳状液经细菌分解后产生臭味引起的 (2.5分) A.小汗腺
B.顶泌汗腺
C.皮脂腺
D.乳腺
E.唾液腺
B.顶泌汗腺
6 8 2015 3. 真皮的主要成分是(2.5分) A.细胞
B.基质
C.纤维
D.血管
E.神经
C.纤维
6 8 2015 4. 下列哪种皮疹不是继发疹(2.5分) A.风团
B.糜烂
C.苔藓样变
D.瘢痕
E.鳞屑
A.风团
6 8 2015 5. 关于斑块,下列哪项是错误的(2.5分) A.局限性隆起的皮疹
B.皮疹的直径>1cm
C.斑块多由丘疹融合而成
D.斑块可由斑片扩大而成
E.斑块常见于银屑病
D.斑块可由斑片扩大而成
6 8 2015 6. 关于结节,下述哪项是错误(2.5分) A.结节是局限、实性、深在性损害
B.结节由真皮或皮下炎性浸润所致
C.结节可高出皮面
D.结节可不高出皮面
E.结节破溃后不形成溃疡
E.结节破溃后不形成溃疡
6 8 2015 7. 有关风团的叙述错误的是(2.5分) A.风团是水肿性隆起的损害
B.伴有剧烈瘙痒
C.消退后不留痕迹
D.真皮浅层血管扩张渗出所致
E.风团又称风疹
E.风团又称风疹
6 8 2015 8. 抗组胺药的作用机理(2.5分) A.与组胺竞争受体
B.直接破坏细胞壁
C.直接抗炎性介质
D.诱导细胞产生抗炎性介质的物质
E.以上都不对
A.与组胺竞争受体
6 8 2015 9. 下列属于第一代抗组胺药的是(2.5分) A.阿司咪唑
B.特非那定
C.酮替芬
D.氯雷他定
E.西替利嗪
C.酮替芬
6 8 2015 10. 关于初发发作的单纯疱疹下列哪条说法不正确? (2.5分) A.主要由 HSV-1 感染引起
B.多数患者感染后有临床症状
C.10%的患者有系统症状,如发热、乏力等
D.可表现为疱疹性咽峡炎,齿龈炎等
E.初发者症状比复发者严重
B.多数患者感染后有临床症状
6 8 2015 11. 带状疱疹的临床特点不包括下列哪一条? (2.5分) A.水痘-带状疱疹病毒引起的局部神经和皮肤感染
B.皮疹单侧分布
C.皮疹为群集、带状分布的水疱
D.皮疹所在区域瘙痒剧烈
E.区域淋巴结肿大
E.区域淋巴结肿大
6 8 2015 12. 丘疹中央发生脐凹的疾病有(2.5分) A.丘疹性梅毒疹
B.传染性软疣
C.神经性皮炎
D.银屑病
E.寻常疣
B.传染性软疣
6 8 2015 13. 带状疱疹皮疹最显著的特征表现是(2.5分) A.皮疹多形性,无一定好发部位
B.簇集性水疱,沿单侧神经走向分布
C.皮疹为簇集性水疱,对称分布
D.在潮红基础上出现脓疱
E.皮疹反复发生在同一部位
B.簇集性水疱,沿单侧神经走向分布
6 8 2015 14. 下述哪种疾病不是由病毒引起的 (2.5分) A.传染性软疣
B.水痘
C.手、足、口病
D.带状疱疹
E.花斑癣
E.花斑癣
6 8 2015 15. 下列哪种疾病并非由细菌感染所致 (2.5分) A.脓疱疮
B.脓癣
C.毛囊炎
D.疖
E.痈
B.脓癣
6 8 2015 16. 臁疮的主要致病菌是(2.5分) A.金黄色葡萄球菌
B.表皮葡萄球菌
C.绿脓杆菌
D.溶血性链球菌
E.变形杆菌
D.溶血性链球菌
6 8 2015 17. 丹毒和蜂窝织炎的常见致病菌是 (2.5分) A.金黄色葡萄球菌
B.溶血性链球菌
C.结核杆菌
D.凝固酶阳性金葡菌
E.麻风分枝杆菌
B.溶血性链球菌
6 8 2015 18. 下列关于葡萄球菌性烫伤样皮肤综合征的描述不正确的是 (2.5分) A.多累及出生3个月内的婴儿
B.起病前常伴有化脓性感染病灶
C.尼氏征阴性
D.手足皮肤可呈手套、袜套样剥脱
E.易并发败血症、肺炎而危及生命
C.尼氏征阴性
6 8 2015 19. 对脓疱疮的糜烂面治疗首选 (2.5分) A.干棉球吸取渗液
B.溶液湿敷
C.包扎
D.暴露
E.霜剂
B.溶液湿敷
6 8 2015 20. 手足癣的临床类型不包括 (2.5分) A.水疱型
B.浸渍糜烂型
C.鳞屑角化型
D.间擦型
E.弥漫型
E.弥漫型
6 8 2015 21. 男性,40 岁。臀部皮肤发生多片环形损害,境界清楚,边有丘疹、丘疱疹、鳞屑,中心消退,痒。最可能的诊断是(2.5分) A.体癣
B.带状疱疹
C.玫瑰糠疹
D.脂溢性皮炎
E.麻风
A.体癣
6 8 2015 22. 女,48 岁。保育员,双手指间皱襞处发生略带椭圆形的红斑,边界清楚,其上表皮浸渍发白,部分为红色糜烂面。考虑可能诊断为(2.5分) A.湿疹
B.皮肤念珠菌病
C.多形性红斑
D.接触性皮炎
E.药疹
B.皮肤念珠菌病
6 8 2015 23. 小儿头皮发生一约胡桃大小的隆起性肿块,边界清楚、质软、表面有多数蜂窝状排脓小 孔,发根松,易拔除。自觉症状不明显。应考虑(2.5分) A.黄癣
B.黑点癣
C.股癣
D.脓癣
E.花斑癣
D.脓癣
6 8 2015 24. 下列哪种皮肤病无痒感 (2.5分) A.疥疮
B.慢性单纯性苔藓
C.过敏性皮炎
D.二期梅毒疹
E.丘疹性荨麻疹
D.二期梅毒疹
6 8 2015 25. 疥疮的病原微生物为 (2.5分) A.禽螨
B.人型疥螨
C.牛型疥螨
D.沙螨
E.尘螨
B.人型疥螨
6 8 2015 26. 疥疮皮损好发于 (2.5分) A.头部、面部和颈部
B.胸背部及腰部
C.四肢的伸侧
D.臀部及双下肢、手掌及足背
E.指缝、腕部屈侧、下腹部、股内侧
E.指缝、腕部屈侧、下腹部、股内侧
6 8 2015 27. 下列关于急性湿疹的描述正确的是(2.5分) A.常有明显的致病外因
B.皮损特点是原发性多形性疹,境界常不清
C.发病部位常局限于接触部位
D.皮损表现为苔藓样变
E.皮疹多无症状
B.皮损特点是原发性多形性疹,境界常不清
6 8 2015 28. 患者,女性,因反复出现左手腕部皮疹伴痒数周就诊,检查时发现左腕带表处有条带状分布的红斑丘疹,小水疱,表面细小鳞屑,皮损单一,边界较清楚。本患者最有可能的诊断是(2.5分) A.体癣
B.湿疹
C.神经性皮炎
D.荨麻疹
E.接触性皮炎
E.接触性皮炎
6 8 2015 29. 治疗药疹的首要措施是(2.5分) A.使用足量的糖皮质激素
B.使用抗生素治疗
C.抗过敏治疗
D.停用一切可疑的致敏药物
E.尽早使用外用药
D.停用一切可疑的致敏药物
6 8 2015 30. 下列哪项不是变应性接触性皮炎的临床特点(2.5分) A.接触致敏物后发病
B.皮疹单一形态,境界清楚
C.好发于暴露部位,尤其接触部位
D.起病迅速,无潜伏期
E.严重者可有系统症状发生
D.起病迅速,无潜伏期
6 8 2015 31. 下列特应性皮炎各年龄期皮损特点是错的(2.5分) A.常发生在屈侧
B.皮损可缓解、发作交替
C.婴儿期渗出性皮损多见
D.成人期皮疹类似于神经性皮炎
E.常发生在伸侧
E.常发生在伸侧
6 8 2015 32. 参与剥脱性皮炎型药疹的发病机制主要是哪型变态反应(2.5分) A.Ⅱ型
B.Ⅰ型
C.Ⅳ型
D.非变态反应
E.Ⅲ型
C.Ⅳ型
6 8 2015 33. 药疹初次发病的潜伏期通常为(2.5分) A.1~5天
B.7~8天
C.3~7天
D.4~20天
E.4~10天
D.4~20天
6 8 2015 34. 关于固定型药疹,不正确的是(2.5分) A.皮疹常为1个到数个
B.皮疹好发于皮肤黏膜交界处
C.皮疹愈合后留下灰黑色色素斑,约1周消退
D.典型皮疹为圆形或类圆形的水肿性暗紫红色斑疹
E.黏膜皱褶处糜烂时常感疼痛
C.皮疹愈合后留下灰黑色色素斑,约1周消退
6 8 2015 35. 大疱性表皮松解型药疹的临床特点不包括(2.5分) A.松弛性水疱或大疱
B.皮肤表面大量渗出
C.大面积表皮松解脱落
D.尼氏征阴性
E.广泛腔口黏膜受累
D.尼氏征阴性
6 8 2015 36. 钱币状湿疹的临床特点不包括(2.5分) A.瘙痒明显
B.皮疹的形状与钱币相似
C.可有渗出
D.好发于儿童
E.好发于下肢
D.好发于儿童
6 8 2015 37. 下列不属于重症药疹的是(2.5分) A.猩红热样药疹
B.重症多形红斑型药疹
C.剥脱性皮炎型药疹
D.大疱性表皮松解型药疹
E.药物超敏反应综合征
A.猩红热样药疹
6 8 2015 38. 下列关于急性荨麻疹的叙述错误的是(2.5分) A.起病急
B.多有瘙痒
C.不发生皮肤外的任何器官受累
D.严重者皮损处可出现橘皮样外观
E.抗组胺药治疗有效
C.不发生皮肤外的任何器官受累
6 8 2015 39. 皮肌炎的特征性皮损是 (2.5分) A.面部蝶形红斑
B.多形红斑
C.双手背虹膜状红斑
D.面部环状红斑
E.双上眼睑紫红斑
E.双上眼睑紫红斑
6 8 2015 40. 下列哪项是紫外线治疗的禁忌证?(2.5分) A.玫瑰糠疹
B.银屑病
C.红斑狼疮
D.白癜风
E.湿疹
C.红斑狼疮